Patho Questions

Réussis tes devoirs et examens dès maintenant avec Quizwiz!

A nurse notices a patient's chart that she has a deficiency in von Willebrand factor. Which of the following may the nurse expect the patient to display? Select all that apply a. Increased bruising b. Increased menstrual blood loss c. Increased clot formation d. Increased platelet activity e. Decreased activated partial thromboplastin time (aPTT) time

(a, b, e) Von Willebrand factor is needed for platelet functioning. Without this factor, excessive bleeding may occur. Clot formation and platelet activity is reduced with von Willebrand factor

A patient has severe metabolic alkalosis. Which of the following compensations will the nurse expect to observe? a. Increased H+ retention by the kidneys b. Increased H+ excretion by the kidneys c. Increased H+ secretion by the kidneys d. Increased respirations

(a) Increased hydrogen retention will reduce plasma pH. Excretion of H+, increased H+ secretion and increased respiration will increase pH of plasma further.

During thyroid surgery, a patient's parathyroid glands were accidentally damaged. Which of the following signs may the nurse observe? a. Increased blood glucose b. Trousseau's sign c. Elevated potassium levels d. Muscle weakness

(b) Parathyroid hormone helps to regulate calcium levels. Without this hormone, calcium levels in the serum may fall. Trousseau sign is common in hypocalcemia. The parathyroid glands do NOT regulate blood glucose and alter potassium levels. Muscle weakness occurs with hypercalcemia

A young patient has an Escherichia coli infection from eating improperly prepared hamburgers. Which of the following is the clinician most worried about the patient developing? a. Immune thrombocytopenic purpura b. Disseminated intravascular coagulation c. Primary thrombocytosis d. Hemolytic uremic syndrome

(d) Although NOT the only cause of hemolytic uremic syndrome, infections of E coli, especially in children, can lead to its development. An elevation in platelets is unrelated to this infection. In children, immune thrombocytopenic purpura usually follows a viral infection

A patient with gallbladder inflammation reports that he always feels sick after he eats his nightly dessert of ice cream and a small piece of cake. Which explanation by the nurse is appropriate? a. With all that sugar, your blood sugar is likely elevating, causing you to feel ill b. Because of your gallbladder inflammation, you can no longer digest fats c. You are likely lacking digestive enzymes for fats because of your gallbladder issue d. When you eat fats, your gallbladder, which is inflame, has to release a substance to help you digest this fat

(d) An inflamed gallbladder can be irritated by high fat diets. The gallbladder stores fats. Other tissues, such as the pancreas, produce digestive enzymes

A patient states that he heard that he should take an aspirin each day. He wonders whether there is a benefit to doing this and wants the aspirin is supposed to do. Which of the following is an appropriate response by the nurse? a. Aspirin dilates your blood vessels b. Aspirin is a "clot buster" c. Aspirin dilutes your blood, making it "thinner" d. Aspirin decreases the adherence of factors in the blood that can lead to a clot

(d) Aspiring works by decreasing platelet adherence. Aspirin does NOT destroy clots nor dilute the blood

Which of the following laboratory values may indicate to a nurse the presence of a bleeding ulcer in a patient with a history of ulcers? a. Absence of melena b. Normal serum ferritin c. Hemoglobin of 15 g/dL d. Hematocrit of 30%

(d) The hematocrit level of 30% is low which indicates blood loss. Melena is blood in the stool, normal serum ferritin indicates normal iron levels in the blood, and hemoglobin level of 15 g/dL is normal and does NOT indicate bleeding

A patient is given a medication that blocks the effects of angiotensin II. Which of the following may result from the medication? Select all that apply a. Decreased blood pressure (BP) b. Increased vessel dilation c. Increased aldosterone levels d. Increased vasoconstriction e. Decreased fluid retention

Blocking angiotensin II will cause decreased blood pressure, vasodilation, and decreased fluid retention (aldosterone increases fluid retention)

A patient with acute pancreatitis has developed bruising around the flank area. What is the correct interpretation by the nurse? a. The patient is displaying Cullen's sign b. The acute pancreatitis is destroying clotting factors c. The pancreas is ruptured d. The patient is displaying Grey Turner's sign

Grey Turners sign is a sign of pancreatitis and causes bruising in the flank area. Cullen sign is bruising around the umbilical area. Clotting factors are NOT destroyed in pancreatitis and bruising in the flank area does NOT indicate pancreatic rupture

A nursing student is learning about leukemias. Which information would be correct to share about acute myelogenous leukemia (AML)? Select all that apply a. Acute myelogenous leukemia is a form of lymphoma b. Acute myelogenous leukemia is more common in children c. Acute myelogenous leukemia cause proliferation of undifferentiated myeloid blast cells d. Acute myelogenous leukemia is the most common form of leukemia in the United States e. Acute myelogenous leukemia development is associated with radiation exposure

High levels of undifferentiated myeloid cells are a hallmark of acute myelogenous leukemia. Radiation exposure has been associated with increased risk of developing AML (c, e)

A nurse is explaining how uncontrolled diabetes mellitus (DM) may lead to dehydration. Please place the following steps in the proper sequence to explain this development - Increased plasma blood glucose - Fluid loss in urine - Transport maximum of kidney exceeded - Glucose enters urine - Osmotic diuresis by glucose in urine

In diabetes mellitus, blood glucose levels are elevated. If the transport maximum for glucose in the kidney is exceeded, glucose appears in the urine. By osmotic diuresis, glucose pulls fluid with it into the urine, increasing fluid loss in the urine (Increased plasma blood glucose - Transport maximum of kidney exceeded - Glucose enters urine - Osmotic diuresis by glucose in urine - Fluid loss in urine)

Please place the following sequence of events in the proper order to show how diabetic ketoacidosis alters potassium levels - K+ loss in urine occurs - Acidic ketones are formed - H+ ions move from extracellular fluid (ECF) to intracellular fluid (ICF) - Metabolic acidosis develops - K+ ions move from ICF to ECF

In diabetic ketoacidosis, acidic ketones are formed, which causes metabolic acidosis. In metabolic acidosis, H+ move into the cell, while K+ ions move into the plasma. K+ ions can then be lost in the urine ( Acidic ketones are formed - Metabolic acidosis develops - H+ ions move from extracellular fluid (ECF) to intracellular fluid (ICF) - K+ ions move from ICF to ECF - K+ loss in urine occurs )

A nurse is reviewing laboratory values and vital signs for a patient with congestive heart failure. Which of the following associations will be correct for the nurse to make? a. In heart failure, blood pressure increases, so aldosterone levels will be low b. Angiotensin levels will be low due to increased kidney perfusion c. Brian natriuretic peptide (BNP) will be increased due to blood volume overload d. Blood pressure will be elevated, while heart rate is lower than normal

In heart failure, fluid is retained. Brian natriuretic peptide (BNP) is released in response to this fluid overload. Heart failure results in lower blood pressure and decreased kidney perfusion

For a male patient, age 50, recent bloodwork reveals the incidental finding of iron deficiency anemia. Which test should next be considered? a. Erythropoietin (EPO) assessment b. Computed tomography (CT) scan of abdomen c. Intrinsic factor assessment d. Fecal-occult blood test

In males, loss of blood in the GI tract must be assessed when anemia is present, The first thing to rule out with anemia in a male patient is GI blood loss

Place the following sequence of events in order leading to orthostatic hypotension - Increased in heart rate (HR) - Decreased blood pressure - Movement from supine to standing - Baroreceptors send signal to medulla oblongata - Decreased venous return

In orthostatic hypotension, a person moves from supine to standing, which reduces venous return to the heart, causing a decrease in blood pressure detected by the baroreceptors. The HR is increased to compensate for pressure decrease ( Movement from supine to standing - Decreased venous return - Decreased blood pressure - Baroreceptors send signal to medulla oblongata - Increased in heart rate (HR))

Diagnosing pelvic inflammatory disease (PID) can be difficult. In addition to abdominal pain and pelvic tenderness, which signs or symptoms may be present? Select all that apply a. Decreased erythrocyte sedimentation rate (ESR) b. Absence of fever c. Presence of gonorrheal infection d. Presence of white blood cells (WBCs) in vaginal secretions e. Abnormal vaginal discharge

In pelvic inflammatory disease (PID), a gonorrhea test may reveal positive results, WBCs may be present in vaginal secretions, and vaginal discharge may be present (c, d, e)

An individual who was male at birth is transitioning to female. Which of the following statements reflects this status? a. Sex and gender are both female b. Sex is female, while gender is male c. Sex and gender are male d. Sex is male, while gender is female

The sex is male and the gender is female

Upon reviewing the charts for the day, a nurse sees that a patient with trigeminal neuralgia has an appointment. When the nurse meets the patient, what findings may the nurse expect? a. Information about the patient's prior varicella zoster infection b. Information about the specific tender points on the patient c. History of diabetes mellitus with poor wound healing d. A primary complaint of unilateral severe facial pain

Trigeminal neuralgia involves increased sensitivity of the trigeminal nerve, which innervates the facial area. Varicella zoster is associated with shingles and specific tender points is common in fibromyalgia. Diabetes may slow wound healing but is NOT an aspect of trigeminal neuralgia

A patient has experienced a vertebral-basilar insufficiency. Which of the following will nurse expect the patient to experience? a. Loss of language expression b. Loss of language reception c. Loss of coordination d. Loss of hearing

Vertigo and loss of coordination often occur when blood flow is disrupted in this area

On the Virchow's triad, there is Vessel injury and Hypercoagulability, what is the third term that is missing? a. Pulmonary embolism b. Venous stasis c. Deep vein thrombosis d. Venous ulcer

Virchow's triad also includes venous stasis

A nursing educator is instructing students to look for characteristics of Virchow's triad when assessing for deep vein thrombosis (DVT). Which of the following items are to be included? Select all that apply a. Vessel injury b. Anticoagulant usage c. Hypercoagulability d. Venous stasis e. Reduced blood pressure

Virchow's triad includes vessel injury, hypercoagulability, and venous stasis (a, c, d)

A woman reports that she had an abortion at age 16. She is now pregnant with her second child and is 26 weeks pregnant. Which of the following is correctly noted? a. Gravida 3, para 2 b. Gravida 2, para 1 c. Gravida 1, para 1 d. Gravida 2, para 3

The woman has been pregnant two times, and one pregnancy has reached the gestational age of 24 weeks (b)

A nursing student want to remember which vital signs may indicate fluid volume depletion. Which of the following would be best to remember ? a. Increased heart rate and decreased respiration b. Increased heart rate and decreased blood pressure c. Increased blood pressure and increased respiration d. Increased respiration and decreased heart rate

With fluid volume depletion, blood pressure decreases and heart rate increases to compensate. Changes in respiration are NOT a primary sign of the fluid volume status

A patient has a severe heart murmur that a nurse can hear with a stethoscope off the chest wall. What grade would she assign the murmur? a. Grade 1 b. Grade 10 c. Grade 3b d. Grade 6

Grade 6 murmurs can be heard with the stethoscope off the chest wall

A patient has been diagnosed with hepatitis A and wants to known how he may have contracted this disease? What is the primary route of hepatitis A transmission? a. Blood transfusion b. Needle stick c. Sexual activity d. Fecal-oral

Hepatitis A is transmitted by the fecal oral route

A nurse is evaluating a patient in relatively poor health. The greatest risk of developing left ventricular failure arises from which of the following? a. Blood pressure 160/100 mm Hg b. Pulmonary trunk stenosis c. Myocardial infarction in the left ventricle d. Mild rheumatic heart disease

Hypertension is the number one cause of heart failure

A male patient has decided to get a vasectomy. Which of the following statements are correct regarding this procedure? Select all that apply a. The vas deferens in cut b. Sperm development is inhibited c. A vasectomy is easily reversible d. Transport of sperm from the testes to ejaculate is prevented e. A vasectomy causes significant changes in the ejaculate

In a vasectomy, the vas deferens is cut and it prevents sperm from being in the ejaculate. Sperm development occurs normally. A vasectomy is NOT easily reversed nor alters the ejaculate (a, d)

A patient is diagnosed with nephrotic syndrome. Which of the following does the nurse expect to observe? a. Decreased serum blood urea nitrogen (BUN) b. Fatty casts in urine c. Absence of edema d. Patient in severe pain due to stone formation

In nephrotic syndrome, casts may appear in the urine due to nephron damage. Elevated BUn and edema may develop as kidney function is compromised. Nephrotic syndrome does NOT lead to stone formation

In infective endocarditis, which portion of the heart is directly infected? a. Coronary arteries b. Myocardial cells c. Aorta d. Inner lining of the heart

Infective endocarditis would imply that the inner lining of the heart is infected

A nurse states that a patient has type 2 diabetes mellitus rather than type 1 diabetes mellitus (DM). Which laboratory value led her to this conclusion? a. Elevated A1c b. Glucose in urine c. Hyperinsulinemia d. High fasting blood glucose levels

Insulin is absent in type 1 diabetes mellitus (c) Both types of DM can cause elevated A1c levels, glucose in urine, and hyperglycemia

A patient's wife hears the physician mention "right upper extremity lymphadenopathy". She asks the nurse what that may mean. Which of the following is an appropriate answer? a. "The patient has an infection in the right arm" b. "The patient has swollen lymph nodes in the right arm" c. "The patient has cancer in the right arm" d. "The patient has problems with the adenoids or tonsils"

Lymphadenopathy refers to a problem with lymph system. Infection and cancer can cause lymphadenopathy but there is NOT enough information to determine this status of the patient. Lymphadenopathy can occur anywhere in the lymph system, NOT just the tonsils

A nursing student learns of a case in which a patient has a genetic disorder in which adaptive immunity is disrupted. Which of the following would be reduced in this patient? a. Neutrophils b. Lymphocytes c. Eosinophils d. Basophils

Lymphocytes (T and B cells) comprise adaptive immunity. Neutrophils, eosinophils, and basophils are NOT part of adaptive immunity

A patient has a tumor that compresses the posterior pituitary. Which hormones are likely affected? Select all that apply a. Adrenocorticotropic hormone (ACTH) b. Growth hormone (GH) c. Oxytocin d. Thyroid stimulating hormone (TSH) e. Antidiuretic hormone (ADH)

Oxytocin and antidiuretic hormone (ADH) are released from the posterior pituitary. Adrenocorticotropic hormone (ACTH), growth hormone (GH), and thyroid stimulating hormone (TSH) are released from the anterior pituitary (c, e)

Which of the following response indicates that a nursing student needs further help in understanding the pain response? a. OLDCART is the mnemonic device for assessing pain b. Pain is objective c. Some patients catastrophize their pain d. Pain behaviors are consistent from person to person e. Pain rating scales are seldom helpful f. The level of pain reported may or may NOT accurately reflect the underlying pathology

Pain is subjective, vary from person to person, and pain rating scales help the clinician and patient more accurately assess the level of pain

A nurse is evaluating patients for risk of deep venous thromboembolism (DVT). Of the following patients, who you she consider MOST at risk? a. Postoperative orthopedic surgery patients have a significantly increased risk for DVTs b. Patient with mildly elevated total cholestrol c. Patient with varicose veins d. Patient suffering an ankle sprain

Postoperative orthopedic surgery patients have a significantly increased risk for deep venous thromboembolism (DVT)

Which of the following changes in pulmonary function is likely to occur in a patient with pulmonary fibrosis? a. Air trapping b. Decreased lung compliance c. Increased FVC d. Bronchiectasis

Pulmonary fibrosis alters the lung tissue, making the lungs less compliant. In pulmonary fibrosis, there is difficulty getting air into the lung, rather than trapping the air in the lungs. Increased FVC would indicate the person would be able to exhale more air and bronchiectasis is dilation of the bronchiole airways

A patient has right ventricular heart failure due to a valve disorder. Which valve disorder most likely contributed to this development? a. Pulmonic valve stenosis b. Tricuspid stenosis c. Aortic stenosis d. Mitral stenosis

Pulmonic valve stenosis would cause right ventricular (RV) failure because the RV has to work harder to eject blood

A patient exposed to elevated levels of pyrogens will likely present with which of the following? a. Fever b. Edema c. Immunosuppression d. Contracture

Pyrogens cause fevers. It is a sign of an active immune response, NOT immunosuppression and pyrogens does NOT stimulate edema

A patient is diagnosed with syphilis presents with rashes on the palms of the hands and soles of the feet. Which stage of syphilis does this nurse assess this to be? a. Primary b. Secondary c. Latent d. Tertiary

Secondary syphilis causes a rash to form over the entire body, including palms and soles. Primary syphilis presents with a lesion, latent syphilis has no signs or symptoms, and tertiary syphilis results in neurological symptoms

A patient, age 63 years, was hospitalized with a sepsis infection. He has now developed adult respiratory distress syndrome (ARDS). A nursing student wonders why this condition developed. Which of the following placed the patient at greatest risk for ARDS? a. His age b. His gender c. His hospitalization d. His sepsis infection

Sepsis increases the risk for adult respiratory distress syndrome (ARDS). Although hospitalization may increase the risk for ARDS, sepsis is a more significant risk

Which disease is a person with AIDS 20 to 40 times more likely to develop than an immunocompetent person? a. Measles b. Pneumonia c. Tuberculosis d. Common cold

Someone with AIDS is 20 to 40 times more likely to develop TB

A nurse is reviewing a patient's chart and notes familial hypercholesterolemia. Which medication does the nurse expect to see listed on the patient's chart? a. Beta blockers b. Statins c. Angiotensin-converting enzyme (ACE) inhibitors d. Corticosteroids

Statins are used to lower blood lipid levels. Beta blockers do NOT treat high blood lipid levels, ACE inhibitors treat hypertension, and corticosteroids treat inflammation

Which of the following bacterial infections is most worrisome to the clinician in regards to glomerular damage? a. Staphylococcal b. Clostridial c. Neisseria d. Streptococcal

Streptococcal pyogenes can cause glomerulonephritis

A nurse hears from a laboratory technician that a fluorescent antibody test has revealed the presence of spirochete bacteria. Which of the following disorders should the nurse plan to speak to the patient about? a. Chancroid b. Lymphogranuloma venereum c. Gonorrhea d. Syphilis

Syphilis is caused by a spirochete shaped bacterium, identified with fluorescent antibodies

A urine culture from a patient reveals a very high colony count. However, the patient denies any symptoms and no signs are apparent. What is the appropriate response? a. Recommend cystoscopy b. Begin antibiotic treatment immediately c. Recommend that the patient increase fluid intake d. Obtain another sample, on another day, and culture agian

The patient may have asymptomatic bacteriuria. Another culture should be performed

A patient's parathyroid glands were damaged. What signs and symptoms does the nurse expect? Select all that apply a. Tetany b. Chvostek's sign c. Trousseau's sign d. Low serum calcium e. Tingling sensation

Trousseau and Chvostek signs are common signs of hypocalcemia. Low parathyroid hormone (PTH) may cause a reduction in serum calcium. Low calcium may cause a tingling sensation. Tetany tends to occur with hypercalcemia (b, c, d, e)

A patient has congestive heart failure. Place in proper order the sequence of how this condition of the heart may affect urine output - Increased glomerular filtration rate (GFR) - Natriuresis - Fluid volume excess - Atrial natriuretic peptide (ANP) released - Stretch of heart

When there is fluid overload, the heart is stretched and releases ANP. ANP increases the GFR, which leads to natriuresis (increased urine output) ( Fluid volume excess - Stretch of heart - Atrial natriuretic peptide (ANP) released - Increased glomerular filtration rate (GFR) - Natriuresis)

A nursing student is studying venous disorders. She has listed venous insufficiency and varicose veins. Which other disorder is she missing? a. Arteriovenous shunting b. Deep vein thromboembolism c. Aneurysms d. Arterial vasculitis

Deep vein thromboembolism are the most severe form for venous disorders. Arteriovenous shunting is NOT a disorder and aneurysms and arterial vsculitis occur in arteries

A nurse suspects that a patient is suffering from diverticulitis. Which of the following are typical signs of diverticulitis? Select all that apply a. Left lower quadrant pain b. Pain radiating to the shoulder c. Fever d. Right upper quadrant (RUQ) pain e. Nausea

Diverticulitis may cause fever, nausea, and left lower quadrant pain. Pain radiating to the should may happen with gallbladder inflammation and right upper quadrant pain is more common with gallbladder disorders

A patient is diagnosed with an ectopic pregnancy. What is the appropriate management? a. Send the patient home with pain medications and instructions b. Schedule an appointment for follow-up care in three days to monitor any changes c. Recognize that this is a gynecological emergency d. Prepare to take blood samples every hour to monitor progression

Ectopic pregnancies are considered gynecological emergencies due to possible rupture

Which is a physical assessment finding of respiratory acidosis? a. Cyanosis b. Tachycardia c. Hypotension d. Hypertension

CYANOSIS is a symptom of respiratory acidosis

A nurse sees the words "arginine vasopressin" written on a report. What term could she substitute in its place? a. Renin b. Brain natriuretic peptide (BNP) c. Aldosterone d. Antidiuretic hormone (ADH)

Arginine vasopressin is another term for antidiuretic hormone

A patient has elevated aspartate aminotransferase (AST), alanine transaminase (ALT), and total bilirubin levels. Which of the following does the nurse expect to observe? a. Red discoloration of the skin b. Elevated heart rate and blood pressure c. Jaundice d. Decreased urine output

Elevated aspartate aminotransferase (AST), alanine transaminase (ALT), and bilirubin levels indicate liver dysfunction, which may present jaundice

A nursing student comments that a patient has a "strange breath smell, almost fruity". Which acid base disorder may the patient be experiencing? a. Respiratory acidosis b. Respiratory alkalosis c. Metabolic acidosis d. Metabolic alkalosis

Fruity smelling breath likely indicates the presence of ketones, which may form in diabetic ketoacidosis, leading to metabolic acidosis

A nursing educator is explaining that a patient has presented with granulomas in the lungs. Which of the following would show correct understanding by the students? Select all that apply a. Granulomas are histological changes in which macrophages aggregate and transform to epithelial-like cells b. The patient is demonstrating immunosuppression c. The patient may have tuberculosis d. The patient has developed a pulmonary adhesion e. The patient is demonstrating an acute inflammatory event

Granulomas are formed during wound healing, which produces epithelial like cells and granuloma in the lungs are a common finding in tuberculosis. Granuloma formulation is NOT a sign of immunosuppression, are NOT adhesions, and usually form with chronic inflammation

A patient presents in the emergency department having problems breathing and coughing up gray sputum. Which of the following is appropriate follow-up questions by the nurse? a. Have you traveled to countries where tuberculosis is present b. How long have you had pneumonia c. Do you feel like a lung has collapsed d. What is your occupation

Gray sputum may be indicative of coal dust in the lungs, so the nurse wants to ask about occupational exposure to such substances

Test results come back positive for Helicobacter plyori. Which of the following does the nurses suspect? a. Orders for a colonoscopy b. Ulcerative colitis c. Crohn's disease d. Peptic ulcer disease

Helicobacter plyori infections are known to cause ulcers. Because H plyori infections occurs in the stomach, a colonoscopy would NOT be a preferred diagnosed option. Ulcerative colitis and Crohns disease are autoimmune diseases (d)

A patient wants to know why he is receiving an antibiotic prescription for ulcers. He thought ulcers were cause by stress. What is the appropriate response? a. Antibiotics do NOT treat the cause of the ulcer. They are for preventative reasons b. All ulcers are cause by bacteria, and thus we use antibiotics for anyone diagnosed with an ulcer c. A type of bacteria, Helicobacter pylori, can destroy the lining of the stomach, and is the cause of some ulcers d. Stress can play a role in ulcer formation, and antibiotics can help with damage due to stress

Helicobacter pylori is a bacterium that may cause stomach ulcer. It is treated with antibiotics. NOT all ulcers are caused by H pylori (c)

A patient presents at the clinic with bright-red blood appearing in his stools. Which of the following may the nurse report? a. Upper gastrointestinal bleeding (UGIB) b. Esophageal varices c. Menorrhagia d. Hematochezia

Hematochezia is bleeding from the anal or rectal area. A upper gastrointestinal bleeding (UGIB) would NOT present with bright red blood in the stools. Esophageal varices may rupture and bleed but NO bright red blood in the stools. Menorrhagia is excessive menstrual bleeding

A nurse is treating a patient with hepatic encephalopathy. Which of the following is true about this condition? a. The prognosis is good for hepatic encephalopathy b. Hepatic encephalopathy is easily reversed c. Hepatic encephalopathy is often associated with elevated ammonia levels d. Hepatic encephalopathy is an early sign of liver dysfunction

Hepatic encephalopathy often develops due to elevated ammonia levels, among other factors. Hepatic encephalopathy does NOT have a good prognosis nor is it easily reversed. It occurs late in liver dysfunction

A patient has a hyperfunctioning adrenal cortex tumor. Which of the following will the nurse expect to observe.? a. Hypotension b. Suppressed adrenocorticotropic hormone (ACTH) levels c. Suppressed cortisol levels d. Weight loss

High cortisol levels from the adrenal cortex will suppress adrenocorticotropic hormone (ACTH) production. High cortisol levels causes hypertension and weight gain (b)

A patient has tested positive for hepatitis B surface antigen (HBsAg). Which of the following is the correct interpretation? a. The patient has active hepatitis B b. The patient may have been infected with hepatitis B or received the vaccination for hepatitis B c. The patient has received the vaccination for hepatitis B but has never been infected with hepatitis B d. The patient has lifelong immunity for hepatitis B

Hpatitis B surface antigen (HBsAg) indicates a current hepatitis B infection. It is the surface antigen

A patient is diagnosed with hyperglycemia hyperosmolar syndrome (HHS). Which of the following support this diagnosis? Select all that apply a. Blood glucose < 70 mg/dL b. Blood osmolarity of 340 mOsm/L c. Arterial pH of 7.21 d. Tachycardia e. Hypotension

Hyperosmolar syndrome (HHS) presents with elevated plasma osmolarity and causes tachycardia and hypotension due to fluid loss. HHS presents with elevated blood glucose levels and does NOT lower blood pH (b, d, e)

A patient is displaying increased pulmonary capillary wedge pressure due to coronary issues. What else may the nurse expect to see? a. Increased jugular vein distention b. Paroxysmal noctural dyspnea c. Decreased left ventricular size d. Hepatomegaly

Increased pulmonary capillary wedge pressure indicates left side heart failure. This may also cause paroxysmal noctural dyspnea. Jugular vein distention is right side heart failure and hepatomegaly normally occurs with right side heart failure

A nurse is working with a patient who is performing "insulin purgining" What is the correct association for the nurse to make? a. The patient is taking all her insulin in one shot per day b. The patient is restricting insulin usage to promote weight loss c. The patient is taking extra insulin to stimulate her metabolic rate d. The patient is using only one site for her insulin injections

Insulin purging is the restriction of insulin in order to promote weight loss

A patient with type 1 diabetes mellitus asks the nurse why he must inject insulin, while other diabetes patients can take pills. What is the appropriate response by the nurse? a. You don't need to use insulin. We can switch you to oral medications b. You require insulin because your body cannot make it. Insulin must normally be injected because in pill form it is broken down by the stomach c. Insulin is destroyed by your saliva, so you must inject it d. You need your insulin to work within two minutes. Ingesting insulin takes too long for it to be effective

Insulin, primarily, is injected because oral forms are NOT effective. Insulin is destroyed by the stomach acid and it is NOT required to act within two minutes (b)

In order to make studying easier, some nursing students have decided to group together items that increase during the stress response. Which of the following should be included on the student's list? Select all that apply a. Urine output b. Gastrointestinal (GI) motility c. Epinephrine d. Blood glucose e. Acute immune response f. Blood flow to the skin

Items that increase during the stress response include epinephrine, blood glucose levels, and immune system is stimulated with stress. With stress, vasoconstriction to the skin occurs, GI motility and urine output decreases.

A child presents with vasculitis following an infection. A nurse suspects which of the following? a. Kawasaki disease b. Polyarteritis nodosa c. Wegner's granulomatosis d. Raynaud's syndrome

Kawasaki disease is a disorder that more commonly appears in children and has been associated with viral infections. Polyarteritis primarily affects adults, age 45 to 65 years. Wegner's granulomatosis is a progressive disorder that occurs in adults and primarily the kidneys. Raynaud's syndrome is NOT associated with a prior viral infection, it primarily affects the hands

Because diabetes mellitus (DM) is associated with long term complications, which of the following common follow up examinations are recommended? Select all that apply a. Feet examination b. Fundoscoptic examination c. Liver function analysis d. Skin examination e. Tests for peptic ulcer disease

Examination of the feet is common in diabetes mellitus to prevent foot injuries and poor wound healing. Eye examinations are needed for DM follow up. Skin changes are common with DM. Liver function analysis is NOT a common follow up for DM. Peptic ulcer disease is NOT a common development in DM

Which statement is correct regarding the effects of exercise? a. Decreases the rate of breathing b. Enlarges the muscle fibers c. Decreases the number of mitochondrial enzymes d. Increases the risk of colon cancer

Exercise ENLARGES THE MUSCLE FIBERS. With isometric exercise, muscles can hypertrophy, which will result in increase in the size of the muscles

A patient has been diagnosed with a methicillin-resistant staphylococcus aureus (MRSA) skin infection. Which of the following is an appropriate response by the nurse? a. "This is a minor, typical infection we allow to heal on its own" b. "This is a highly contagious viral skin condition" c. "This infection will likely result in quarantine and amputation" d. "This is bacterial infection that is NOT treatable by antibiotics

MRSA infection is a methicillin resistant bacterial infection. MRSA is NOT a minor infection because it is unresponsive to some antibiotics and it does NOT require quarantine

Men who have sex with men (MSM) are at an increased risk for sexually transmitted diseases that affect which of the following areas? Select all that apply a. Penis b. Rectum c. Oropharyngeal d. Urethra e.. Scrotum

MSM have increased risk of infection of the rectum and oropharyngeal areas

A man traveling in an area endemic for malaria contracted the disease. Which category of medications should be used for treatment? a. Antibacterial b. Antiviral c. Antifungal d. Antiprotozoal

Malaria is a protozoal disease, so antiprotozoal medications are needed

A patient is assessed and found to have a left ventricle ejection fraction (LVEF) of 35%. What interpretation of this result would be correct for the nurse to make? a. The LVEF is normal b. The LVEF represents a high-functioning heart c. The LVEF is reduced compared to normal values d. The LVEF is incorrect and should NOT be expressed as a percentage

Normal values for LVEF are 50 to 55%

Over time a patient's partial pressure of carbon dioxide has remained at 55 mm Hg and partial pressure of oxygen has been at 80 mm Hg. Which following interpretations are correct? a. The values are normal b. The values indicate hypoxemia and hypocapnia c. The values indicate normal oxygen and hypercapnia d. The values indicate hypoxemia and hypercapnia

Oxygen levels are lower than normal, while carbon dioxide values are elevated

A nursing student hears an abnormal sound with auscultation during diastole. It is indicative of blood flow vibrating against a ventricular wall. Which heart sound would this be describing? a. S1 b. S2 c. S3 d. S4

(c) S3 heart sound is due to blood pouring against the ventricular wall. S1 is due to closure of atrioventricular (AV) valves, S2 is due to closure of the semilunar valves, and S4 is due to stiffness of the left ventricles

A female patient experiences urinary incontinence whenever she coughs or sneezes. Otherwise, she states she has no urinary problems/ What type of incontinence is she experiencing? a. Neurogenic b. Overactive bladder c. Overflow d. Stress

(d) Stress incontinence occurs with actions like sneezing or coughing when abdominal muscles place stress on the bladder. Neurogenic incontinence develops with disruption of neurological signals to the bladder. Overactive bladder is when the baldder cannot hold urine as expected and overflow incontinence occurs when the bladder cannot be fully emptied

A nurse is working with an obese patient. Which of the following are risk factors for this patient's condition? Select all that apply a. Female b. Advance age c. Hyperthyroidism d. Compulsive exerciser e. Non-Hispanic black

Risk factors for obesity include females, advanced age, and non Hispanic black ethnicity. Thyroid hormones stimulate metabolism, so high thyroid hormone levels will induce weight loss

A young patient wants to listen to his heart. He hears the "lub" and "dub" sound. How are these respective sounds identified clinically? a. S2 and S1 b. S3 and S4 c. S2 and S1 d. Atriovetricular (AV) and semilunar valve openings

S1 and S2 heart sounds are due to the closure of the AV and semi lunar valves, respsectively which is the lub dub sound

A patient is born with severe combined immunodeficiency disease (SCID). Which of the following cell types are disrupted by this disorder? Select all that apply a. B cells b. T cells c. Lymphocytes d. Neutrophils e. Basophils

SCID is a disorder of B and T cells, which are lymphocytes

The patient has suffered a myocardial infarction (MI). The family members wants to know how "bad" the heart attack is. Which of the following factors are primary contributors to the severity of an MI? Select all that apply a. Patient's gender b. Patient's body mass index (BMI) c. Location of occlusion d. Length of time of occlusion e. Collateral circulation

Severity of an MI is based primarily on location of occlusion, duration of occlusion, and collateral circulation

A patient has been diagnosed with a bacterial gastrointestinal infection. Which of the following may the nurse expect to see on the culture results to reveal? a. Shigella b. Malaria c. Cholera d. Giardiasis e. Diphtheria

Shigella is a bacterium that infects the gastrointestinal system. Cholera is caused by a bacterium that infects the gastrointestinal system. Diphtheria is caused by a bacterium but infects the respiratory system. Malaria and Giardiasis are caused by protozoans

Which of the following is true for both hyperlipidemia and hypertension ? a. They both present with immediate and overt symptoms b. They both induce wheezing and shortness of breath c. They cannot be managed with medications d. They may present with few signs and symptoms, yet cause significant damage

Signs and symptoms may be lacking in hypertension and hyperlipidemia, but damage may still occur. Hyperlipidemia and hypertension often causes few signs and symptoms, can be managed with medications and do NOT induce wheezing

A patient complains of severe dehydration. The nurse examines the vital signs and other signs and symptoms. Which support the patient's claim? Select all that apply a. Bradycardia and hypotension b. Decreased skin turgor c. Pale yellow urine d. Dry mucous membranes e. Complaints of thirst

Signs and symptoms of dehydration include bradycardia, hypotension, decreased skin turgor, dry mucous membranes, and complaints of thirst

A nurse sees Sjorgen's syndrome on a patient's chart. The nurse should follow up specifically with which aspects for this disease during her assessment of the patient? Select all that apply a. Swollen hands b. Dry eyes c. Tightening of the skin d. Dry mouth e. Butterfly rash across face

Sjogren's syndrome tends to present with dry eye and mouth. Sclerodema presents with tightened skin and lupus presents with a butterfly rash

A nurse reads on a patient's chart that a patient has suffered a sacral pressure ulcer, Stage II. What characteristics of the wound will the nurse expect to see? a. Slight redness and irritation in the sacral areas b. Loss of full thickness of the tissue down to the bone c. Skin loss in epidermal and partial dermal areas d. Deep ulceration into subcutaneous tissue

Skin loss of epidermis and/or dermis is a Stage II ulcer. Stage I decubitus ulcer involves slight redness, stage III involves deep ulceration into subcutaneous tissue, and stage IV involves loss of skin tissue down to the bone

A wife and daughter of a man has brought him to the emergency department believing he is suffering from a stroke. Which of the following are typical presentations of a stroke? Select all that apply a. Manic speech b. Hyperactivity c. Slurred speech d. Facial droop e. Uneven gait

Slurred speech, facial droop, and uneven gait may develop with strokes. Hyperactivity and manic speech are NOT signs of stroke (c, d, e)

Some individuals appear to suffer from stress to a greater degree than others. Which of the following may account for a more pronounced stress response? Select all that apply a. Poorer ability to adapt b. Infrequent stressors c. Shorter duration stressors d. Strong adaptations and copings

Some individuals have a reduced capacity to adapt to stressful events and individuals with a prolonged stress response are more likely to have a pronounced response to stress

A patient has been diagnosed with hepatitis B and wants to understand how he contracted the infection. Which of the following does the nurse share as a means of transmission of hepatitis B? Select all that apply a. Blood borne transmission b. Skin to skin contact c. Sexual activity d. Fecal oral route e. Needle sharing with infected person

Some ways to contract hepatitis B include through blood, sexual activity, or needle sharing

A nurse reads off a patient's arterial blood gases and includes the following: percentage of hemoglobin saturation, partial pressure of oxygen, bicarbonate, and pH. Which is she missing? a. Blood pressure b. Respiratory rate c. Hemoglobin levels d. Partial pressure of carbon dioxide

Partial pressures of carbon dioxide are part of arterial blood gas measurements. Blood pressure, respiratory rate, and hemoglobin levels are NOT part of ABG measurements

A female patient with chronic obstructive pulmonary disease (COPD) presents with a hematocrit value of 48%. What is the likely explanation? a. The patient has primary polycythemia b. The patient has aplastic anemia c. The decrease oxygenation of the blood associated with COPD has increased erythropoietin release d. The COPD has caused dehydration, increasing hematocrit values

The decrease oxygenation of the blood associated with COPD will increase erythropoietin which will increase RBC levels

Dysfunction of which of the following organs may most likely disrupt platelet levels? a. Stomach b. Spleen c. Heart d. Thymus

The spleen sequesters platelets. Alterations in functions of the spleen can affect platelet levels

Both parents are heterozygous for a recessive trait. What percentage of their offspring will be carriers for the trait?

(50%) A carrier for a recessive trait is heterozygous for the trait. In this case, a 25% chance exists for inheritance of both dominant alleles and a 25% chance exists for inheritance of both recessive alleles. A 50% chance exists for inheritance of one dominant and one recessive allele, leading to carrier status.

When asking a patient's about sexual history, which type of sexual activity is often assumed on the part of the clinician, unless he or she is informed otherwise? a. Homosexual behavior b. Heterosexual behavior c. Transsexual behavior d. Bisexual behavior

Heterosexual behavior is often assumed, but this does NOT apply to all patients

A patient with congestive heart failure is being monitored for weight gain. He has gained 3 pounds. How many liters of fluid provide a weight gain of 3 pounds?

1.5 because each gain of 2 pounds of weight equals 1 L of fluid

A patient tells you, "My mom does NOT have familial hypercholesterolemia at all. My dad has two genes for it" The patient asks, "What is my chance of getting this disease?" What is the likelihood in percentages, that the patient has inherited the disorder?

100% because the father has two copies of the autosomal dominant gene for hypercholesterolemia, each offspring will inherit the condition

A patient had to have surgery to remove a parathyroid gland tumor. What location of the body should the nurse help prepare for surgery? a. Superior abdomen b. Neck c. Mediastinum d. Near adrenal glands

The parathyroid glands are located in the neck

A physician has prescribed an anticoagulant for a patient and would like to double the clotting time for the blood. What should the international normalized ratio (INR) be to accomplish this goal, if normal INR is 1.0?

2.0. Normal INR is a given value of 1.0. Anticoagulants should increase INR, prolonging clotting time. Clotting time is twice as long as normal is given a ratio of 2.0

What is the daily potassium requirement of humans for optimal cell functioning

20 mEq

A patient presents complaining of a horrible medical diet he or she has been prescibed. The patient normally eats 2,800 calories per day and is now on a very low calorie diet. To meet the expectation of this diet, the patient will need to reduce caloric intake by how many calories per day?

2000 because a very low calories diet is less than 800 calories per day

Due to a patient's risk of clot formation, a clinician wants to prolong clotting time by a factor of 1.5. If normal clotting is 20 seconds, what is the desired clotting time in seconds?

30 because to increase clotting time by a factor of 1.5, take 20 seconds times 1.5

A 50 year old man has smoked for 30 years, 1.5 packs per day. What is the smoking pack history for this individual expressed pack/years?

45 because to calculate pack per years, multiple the number of year smoked times the packs per day

A patient is being assessed for dysphagia. What diagnostic test does the nurse expect to be most likely scheduled? a. Manometry b. Barium swallow c. Abdominal computed tomography (CT) scan d. Urea breath test

A barium swallow test can help evaluate swallowing dynamics. Manometry measures pressure in the esophagus, an abdominal CT scan would NOT be used to evaluate swallowing, and a urea breath test is for stomach ulcers

A patient has unexplained hematuria with no pain. Which diagnostic follow through may be most helpful ? a. IV pyelogram procedure b. Postvoid volume measurement c. Straining the urine d. Cystocopy

A cystoscopy could be done to determine any abnormalities in the bladder that may cause bleeding

Which condition is caused by an increase in the amount of potassium in the blood? a. Hypocapnea b. Hypotension c. Hyperkalemia d. Hypercalcemia

A potassium increase in the blood leads to HYPERKALEMIA

A patient with diabetes insipidus will likely have which of the following laboratory values? a. Elevated blood glucose b. Elevated plasma osmolarity c. Elevated urine osmolarity d. Elevated urine retention

In diabetes insipidus, fluid loss in the urine is increased. This, in turn, raises plasma osmolarity. Diabetes insipidus does NOT affect glucose levels but instead excess fluid reduces urine osmolarity and increases urine output

A nursing student reads the following in her study manual: "Cytomegalovirus (CMV) is the most common congenitally acquired infection in infants." Which of the following indicates correct understanding by the nursing student? a. CMV infection results from a sporadic genetic mutation b. CMV infection develop within 2 to 3 years after birth c. CMV infection can be transmitted to the infant from the mother d. CMV infection in the infant results from transmission from visiting family members

A congenital condition is present a birth. In this case, CMV can be transmitted from the mother

Which of the following are most likely to result in prerenal failure? a. Nephrotoxic drugs b. Kidney stones c. Failing heart with decrease in blood pressure d. Poststreptococcal glomerulonephritis

A decrease in renal perfusion brought on by a failing heart indicates prerenal failure. Nephrotoxic drugs and glomerulonephritis cause intrarenal failure. Kidney stones cause postrenal failure

A patient has a pH of 7.42, a partial pressure of carbon dioxide of 40 mm Hg, and bicarbonate values of 24 mEq/L. For which acid base disorder should the patient be treated? a. Metabolic alkalosis b. Respiratory acidosis c. Mixed acid-base disorder d. None. The patient's values are normal

A normal pH is 7.35 to 7.45, partial pressure of carbon dioxide is 35 to 45 mm Hg, and bicarbonate is 22 to 26 mEq/L

Hyperpituitarism is most commonly caused by which of the following? a. Prolactinoma b. Autoimmune disorder c. Congenital defect d. Sheehan syndrome

A prolactinoma is the most common cause of elevated pituitary activity

A clinician suspects a volvulus, which diagnostic tests confirms. What is the appropriate management strategy? a. A wait-and-see approach b. Colostomy c. Antibiotic treatment only d. Surgical correction

A volvulus is a twisting of the intestines, which must be surgically corrected. Antibiotics will NOT be the only treatment

A patient positive for HIV is to receive ART therapy. He wants to know when ART means. What is the appropriate response from the nurse for the meaning of ART? a. Anucleotide retrotherapy b. Antiretroviral therapy c. Adjuvant retroviral therapy d. Aplastic recombinant therapy

ART stands for antiretroviral therapy because HIV is a retrovirus

In primary dysmenorrhea, the nurse knows it is critical to do which of the following? a. Use a pain scale with the patient b. Ask the patient to recall pain patterns over the past several months c. Eliminate pelvic pathologies d. Prescribe oral contraceptives

Although dysmenorrhea may be common, pelvic pathologies do need to be ruled out. Analysis of pain, a pain scale, and oral contraceptives may be helpful but pelvic pathologies need to be ruled out first (c)

A patient has very low cortisol levels. Which of the following tests may be appropriate? a. Dexamethasone suppression test b. Adrenocorticotropic hormone (ACTH) stimulation test c. Parathyroid hormone (PTH) stimulation test d. Dexamethasone activation test

An adrenocorticotropic hormone (ACTH) stimulation test will provide ACTH to a patient to see if the adrenal cortex responds by producing cortisol. A dexamethasone suppression test is done if cortisol levels are elevated.

Paramedics bring in a patient who has lost consciousness. Friends state that prior to losing consciousness, the patient complained of severe pain in the chest and hearing a ripping or tearing sound. What does the nurse likely suspect? a. Myocardial infarction b. Aortic dissection c. Temporal arteritis d. Kawasaki disease

An aortic dissection can cause a ripping or tearing sound in the chest area. Myocardial infarction tends to present with angina pectoris, NOT ripping or tearing sounds. These symptoms are NOT signs and syptoms of temporal arteritis or Kawaski disease

An induration of 18 mm is noted in response to the Mantoux test. What is the likely conclusion? a. Active tuberculosis b. Quarantine needed c. Negative Mantoux result d. Chest x-ray follow up

An induration of 15 mm or more is considered positive and a follow up examination is required. The Mantoux test does NOT differentiate active tuberculosis and decision to quarantine cannot be based on the test results

Place in order the proper sequence of events for the process of sensation of hunger to satiety - Feeling of satiety - Ghrelin secreted - Cholecystokinin released - Empty stomach - Stomach distention - Hypothalamus initiates feeding behaviors

An empty stomach secretes ghrelin, which affects the hypothalamus to induce feeding behaviors. Caloric consumption results in stomach distention, which sends out a signal of cholecystokinin, which produces a sense of satiety ( Empty stomach - Ghrelin secreted - Hypothalamus initiates feeding behaviors - Stomach distention - Cholecystokinin released - Feeling of satiety)

A pregnant woman wants to know why she needs to be concerned about gestational diabetes. What of the following are possible developments the nurse could share with the patient? a. Gestational diabetes can lead to newborns presenting with signs and symptoms similar to fetal alcohol syndrome b. If you have gestational diabetes, you will be at a decreased risk for developing diabetes while NOT pregnant c. Infants of mothers with gestational diabetes may develop macrosomia d. Infants of mothers with gestational diabetes are more likely to develop vision problems

An infant born to a mother with gestational diabetes may have a larger than normal body size. Gestational DM does NOT cause signs similar to fetal alcohol syndrome. It increases the risk for type 2 DM development. Vision problems are NOT more likely with gestational diabetes

A patient is newly diagnosed with HIV. Which of the following is the best recommended treatment protocol? a. Azithromycin b. Proton pump inhibitor c. Statins d. Anti-retroviral therapy

Anti retroviral therapy is a combination of medications used as the primary treatment for HIV infections. Azithromycin drug in combination with other drugs is used in HIV treatment. Proton pump inhibitors are used to treat peptic ulcer disease and statins are used to treat high cholesterol

A patient is in the fetal position, complaining of right lower quadrant (RLQ) pain. Which of the following would be the most likely cause? a. Peptic ulcer disease b. Cholecystitis c. Appendicitis d. Crohn's disease

Appendicitis occurs with right lower quadrant (RLQ) pain. The pain is often severe so the patient assumes the fetal position. Peptic ulcer disease and Crohn's disease do NOT present with RLQ pain. Cholecystitis refers to gallbladder inflammation, which presents with right upper quadrant (RUQ) pain

Azotemia is noted on a patient's file. Serum valves will likely show which of the following? a. Increased blood urea nitrogen (BUN) b. Increased creatinine c. Increased red blood cells d. Increased inflammatory markers

Azotemia is an increase in blood urea nitrogen (BUN) levels

A patient has an inherited disorder of B cells. Which of the following would the nurse expect to be lacking on a serum immune profile? a. Immunoglobulins b. CD4 cells c. CD8 cells d. Macrophages

B cells are the cells that produce immunoglobulins. CD4 and CD8 cells are a form of T cells and macrophages are derived from monocytes

A nurse is reviewing the laboratory values and vital signs of a female patient, age 48 years. Which may be concerning to the nurse? Select all that apply a. Hematocrit 45% b. Blood pressure (BP) 148/94 mm Hg c. High density lipoprotein (HDL) 42 mg/dL d. Triglycerided (TG) 190 mg/dL e. Total cholesterol 180 mg/dL

Blood pressure and trigylceride levels are elevated. Hematocrit 45%, HDL 42 mg/dL and total cholesterol 180 mg/dL are normal

A nurse reviews a patient's HIV infection markers. Which values have led the nurse to the conclusion that the patient has AIDS? a. CD4 150 cells/microliter b. CD4 400 cells/microliter c. CD4 levels higher than CD8 levels d. Continual elevation of CD4 values between clinic visits

CD4 levels less than 200 cells/microliter is considered a criteria for AIDS diagnosis

A patient with liver cirrhosis has severe portal hypertension. Which of the following does the nurse expect to observe? Select all that apply a. Grey Turner's sign b. Caput medusa c. Retroperitoneal pain d. Esophageal varices e. Splenomegaly

Caput medusa is enlarged umbilical veins that may develop with portal hypertension. The spleen may enlarge as blood backs up from portal hypertension. Esophageal varices may develop as blood backs up in hepatic portal circulation (b, d, e) Grey Turner's sign occurs in pancreatitis and portal hypertension does NOT cause retroperitoneal pain

A nurse is working with a young woman, age 20, who has abused laxatives for three years. She is complaining of incomplete fecal evacuation. What is the diagnosis? a. Constipation b. Irritable bowel syndrome c. Inflammatory bowel disease d. Cathartic colon

Cathartic colon results from laxative abuse, as the colon is unable to move fecal matter normally

A patient's primary complaint is that he often feels abdominal pain and bloating after eating. After further questioning by the nurse, it is revealed that patient feels worse after eating items such as bread, pasta, cookies, and crackers. What disorder does the nurse expect? a. Gallbladder disease b. Celiac disease c. Pyloric stenosis d. Diverticular disease

Celiac disease is an autoimmune response to gluten. The foods giving the patient problems contain gluten. Gallbladder disease is aggravated by foods high in fat and diverticular disease is often aggravated by high fiber foods. Pyloric stenosis would present with severe pain and vomiting (b)

A woman is about to give birth. A concern exists for infectious neonatal conjunctivitis to develop. Which organisms can cause this infection? Select all that apply a. Human papillomavirus (HPV) b. Treponema pallidum c. Chlamydia trachomatis d. Neisseria gonorrhoeae e. Candida albicans

Chlamydia trachomatis and Neisseria gonorrhoeae are two organisms known to cause neonatal conjunctivitis (c, d)

A patient is seeking medical attention due to endometritis. Which of the following organisms may most likely be the cause? a. Chlamydia trachomatis b. Candida albicans c. Human papillomavirus d. Herpes simplex virus

Chlamydia trachomatis is one of the primary causes of enometritis

A patient diagnosed with cholelithiasis asks what this means. What is the appropriate response? a. A kidney stone has formed and you will need to pass it b. You will need your gallbladder removed c. You have formed stones in your gallbladder d. You have inflammation of the gallbladder

Cholelithiasis refers to gallstones. Kidney stones are referred to as urolithiasis. The presence of gallstones does NOT mean that the gallbladder must be removed. Cholelithiasis indicates the presence of gallstones, which may lead to inflammation

A patient education group is discussing sources of cholesterol. Which of the following shows current understanding? a. You get cholesterol from only your diet b. You get cholesterol from only plant foods c. You get cholesterol from your liver, making it and your diet d. You ingest some cholesterol with every food you eat

Cholesterol is found both in the diet and is synthesized by the liver. Cholesterol is NOT found in plant foods nor is it present in all foods

Which of the following are conditions associated with chronic stress? Select all that apply a. Myocardial infarction b. Peptic ulcer disease c. Hypertension d. Acute, traumatic fractures e. Migraines

Chronic stress can increase the risk of migraines, hypertension, and myocardial infarctions. Peptic ulcers have been associated with chronic stress

A nurse is explaining how problems in the liver can result in vomiting of blood. Please place the following in the proper sequence - Portal hypertension - Blood appears in vomitus - Cirrhosis of liver - Blood backs up in esophageal vein - Esophageal varices rupture

Cirrhosis of the liver can lead to portal hypertension, which causes blood to back up in the esophageal veins. The veins may rupture, causing blood to appear in the vomitus ( Cirrhosis of liver - Portal hypertension - Blood backs up in esophageal vein - Esophageal varices rupture - Blood appears in vomitus)

A client who is taking his board exams is nervous and breathing heavily in the examination hall. Which acid-base imbalance has the client developed? a. Metabolic acidosis b. Metabolic alkalosis c. Respiratory acidosis d. Respiratory alkalosis

Clients with RESPIRATORY ACIDOSIS exhibit symptoms such as anxiety, restlessness, lethargy, rapid breathing , and cough

A patient with severe pulmonary issues develops right-side heart failure due to these issues. What is an appropriate term for the nurse to use? a. Cor pulmonale b. High-output heart failure c. Pulmonary edema d. Pneumocystis

Cor pulmonale develops when pulmonary disorders cause right ventricle failure

A colonoscopy reveals a cobblestone appearance in the large intestines. These areas are separated by normal-appearing sections. Which diagnosis is likely? a. Ulcerative colitis b. Irritable bowel syndrome c. Crohn's disease d. Volvulus

Cobblestoning appears in Crohn's disease

Which of the following conditions are due to a hormone excess? Select all that apply a. Cushing's syndrome b. Grave's disease c. Addison's disease d. Giantism e. Hasimoto's thyroditis

Cushings syndrome is due to excess adrenal cortex hormone. Graves disease is due to excess thyroid hormone. Giantism is due to an excess of growth hormone (GH). Addisons disease is due to a deficient in adrenal cortex hormones and hasimotos thyroiditis causes inadequate thyroid hormone levels

A patient presents with cyanosis, pH of 7.30, hypercapnia, and tachycardia. Which acid base disorder is MOST likely? a. Respiratory acidosis b. Respiratory alkalosis c. Metabolic acidosis d. Metabolic alkalosis

Cyanosis, hypercapnia, and tachycardia with acidosis indicate a respiratory condition

Universal newborn screening reveals the presence cystic fibrosis. Which of the following comments made by the parents reflects their child's diagnosis? a. "We can expect our child to die by the age of 1, even with treatment" b. "Our child will require lifelong management of pulmonary and pancreatic issues" c. "We did NOT need the screening results. We could tell by our child's appearance at birth what the problem was" d. "We understand our child will suffer an increased risk of tumors"

Cystic fibrosis affects pulmonary and pancreatic function that requires management. Life expectancy is reduced in cystic fibrosis, death occurs within 3 years of age, NOT one year. Cystic fibrosis does NOT appear with significant overt physical changes at birth and does NOT increase tumor risk

After age of 55 years, females experience an increased risk for cardiovascular disease. Which explanation is most likely? a. Women, up to age 55 years, experience less stress than women over age of 55 years b. Women receive better cardiovascular care and testing when younger c. Women benefit from the cardioprotective effect of endogenous estrogen. After menopause, this effect dissipates d. Younger women are able to tolerate ischemic myocardial episodes better

Endogenous estrogen is though to have a cardioprotective effect, lowering the incidence of heart disease in premenopausal women

A patient is diagnosed with epiglottis. Which of the following should the nurse be prepared for? a. IV to increase fluid volume b. Use of incentive spirometer c. Epinephrine injection d. Intubation

Epiglottis is a medical emergency due to risk of airway closure. Intubation may be necessary. Epiglottis is NOT a disorder of fluid volume, an incentive spirometer would NOT be appropriate and epinephrine injections are used in anaphylactic shock

A patient suffering from prolonged, chronic inflammation is likely to display which of the following systemic effects of inflammation? Select all that apply a. Weight gain b. Fever c. Lymphadenopathy d. Anemia e. Elevated ESR

Fever, lymphadenopathy, anemia, and ESR levels elevate with inflammation. Weight loss, NOT weight gain is common in inflammation

A patient presents with hematochezia. What is the most likely cause? a. Diverticulitis b. Stomach ulcer c. Short bowel obstruction d. Hemorrhoids

Hemorrhoids may cause bleeding in the rectum, which presents in hematochezia

A client is suffering from prolonged ischemia and tissue death. What term can be used to refer to such tissue death?

INFARCTION is the death of tissue due to prolonged insufficient blood supply (ischemia)

A patient has a parathyroid gland disorder. Which supplement may be needed? a. Potassium b. Sodium c. Calcium d. Iron

If the parathyroid gland cannot produce enough PTH, then serum calcium levels may be low

A trauma patient arrives in the emergency department with a stab wound. If a blood sample were to be looked at underneath the microscope, which type of anemia would likely be present? a. Megaloblastic b. Microcytic c. Sickle cell d. Normocytic

In anemia related to blood loss, cell size is NOT changed

A nursing student sees on a patient's chart "IBS". Which shows further learning is needed? a. IBS stands for irritable bowel syndrome b. IBS and IBD are basically the same thing c. IBS presents with alternating bouts of constipation and diarrhea d. IBS presents with no pathological changes detectable in the bowels

Irritable bowel syndrome (IBS) and irritable bowel disease (IBD) are different disorders. IBS deals with motility of the gastrointestinal (GI) tract, whereas IBD refers to inflammatory disorders

A prescription of anticholineric medications would be effective for which of the following? a. Appendicitis b. Irritable bowel syndrome c. Crohn's disease d. Ulcerativie colitis

Irritable bowel syndrome may develop from overactivity of intestinal tsisue. Anticholineric agents slow down intestinal activity. Appendicitis requires surgical correction and Crohn's disease and ulcerative colitis manifests as inflammaton

Hepatic encephalopathy may begin to develop in a patient with which of the following? a. Hypernatremia b. Elevated serum ammonia levels c. Hypokalemia d. Deficient fat soluble vitamins

Liver failure can cause an increase in ammonia, which will affect neurological functioning

A clinician is asking a patient with severe liver diseases about bruising and other signs of bleeding. Which of the following explains why the clinician is concerned? a. All medications for liver failure work on thinning the blood to reduce clotting b. The liver synthesis clotting factors, which may be affected by the patient's liver disease c. The patient has likely developed an autoimmune response to clotting factors d. Liver diseases disrupt portal circulation that may appear as bruising on the extremities

Liver synthesis of clotting factors may be declining, increasing the risk for bleeding and bruising

A patient suffering from severe stress and anorexia has the following laboratory values, which are all low; gonadotropin-releasing hormone (GnRH), growth hormone-releasing hormone (GHRH), and corticotropin-releasing factor (CRF). Which tissue is likely dysfucntional? a. Adrenal cortex b. Hypothalamus c. Posterior pituitary d. Anterior pituitary

Low gonadotropin-releasing hormone (GnRH), growth hormone-releasing hormone (GHRH), and corticotropin-releasing factor (CRF) are indicated from the hypothalamus, so this tissue is likely disrupted by the patient's condition

A recent outbreak of measles is reported in the news. A patient asks the nurse how measles is spread. What is the appropriate response? a. Skin-to-skin contact b. No person-to-person transmission c. Fecal-oral d. Respiratory route

Measles is spread by droplet transmission (d)

A patient passes a kidney stone. What is the most likely composition of the stone? a. Cystine b. Uric acid c. Calcium d. Magnesium

Most kidney stones are composed of calcium

A woman is reviewing her father's medication list and comes across nitroglycerin tablets. She asks about the purpose of the tablets. Which of the following is an appropriate response by the nurse? a. The tablet helps with your father's dizziness b. The tablets help with the workload on the heart c. The tablets can help relieve chest pain d. The tablets help your father breathe easier

Nitroglycerin tablets are used to relieve angina pectoris

A patient was recently diagnosed with non-Hodgkin's lymphoma. Which cell types can be potentially involved in these lymphomas? Select all that apply a. Neutrophils b. B cells c. T cells d. NK cells e. Myoblasts

Non Hodgkins lymphoma develops from T cells, B cells, or NK cells

A nursing student is to help evaluate a patinet with pneumoconiosis. Which of the following may the student consider to be part of the patient's profile? Select all that apply a. Tension pneumothorax b. Idiopathic pulmonary fibrosis c. Scoliosis d. Asbestosis e. Silicosis

Pneumoconiosis develops due to inhalation of particles, asbestosis and scliosis are forms of it. Tension pneumothorax, idiopathic pulmonary fibrosis and scoliosis are NOT forms of pneumoconiosis

As the nurse checks the patient's medication chart, he notices medications listed, including diuretics. Which plasma ion would most likely need monitoring given this information a. Potassium b. Magnesium c. Calcium d. Phosphorus

Potassium loss can accompany diuretics, particularly those that are NOT potassium sparing

A nurse sees on a patient's notes the presence of purulent exudate. Which of the following will the nurse expect to observe? a. Blister formation b. Redness and swelling around a wound c. Pus d. Wound contracture

Purulent exudate is pus

A teenager believes she may have scoliosis. Which of the following does she point out that may be indicative of scoliosis? a. One hip and shoulder higher than the other b. Lumbar sway back c. Shoulders rotate anteriorly d. Nuchal pain

Scoliosis is curvature of the spine, which may cause one shoulder and hip to be higher than the other

A nurse is working with a patient diagnosed with postherpetic neuralgia. Which of the following terms may arise in the discussion? Select all that apply a. Varicella zoster b. Tic Douloureux c. Bacterial infection d. Shingles e. Complex regional pain syndrome (CRPS)

Shingles is sometimes called postherpetic neuralgia and varicella zoster is the cause of chickenpox and shingles. Postherpetic neuralgia is a virus infection not a bacterial infection.

Mark suffers from significant hypertension and has been given a diuretic, which is NOT potassium-sparing. A clinician is counseling Mike about the signs and symptoms of hypokalemia. Which should be included on the list? Select all that apply a. Postural hypertension b. Leg cramps c. Anorexia nervosa d. Muscle fatigue e. Chvostek's sign

Signs and symptoms include hypokalemia include leg cramps, anorexia nervosa, and muscle fatigue.

Which of the following conditions would the nurse consider to be in a state of medical emergency? a. Benign prostatic hyperplasia b. Testicular torsion c. Cryptochidism d. Erectile dysfunction

Testicular torsion is a medical emergency because blood supply can be disrupted to the testicles

A young man with very tall, lanky stature is known to have an inherited disease of the connective tissue. Which of the following statements may the young man make relative to his follow-up care? a. "Why is everyone always worried about my heart?" b. "I get my cholesterol measured constantly because it is so very high" c. "Why do I always get jaundice?" d. "I have to get a colonoscopy every year"

The connective tissue disorder, known as Marfan syndrome, can result in tall stature but also affect the connective tissue of the heart.

Please place the following stages of a malaria infection in the proper order - Splenomegaly - Female anopheles mosquito bite - Parasite release from liver - Parasite invasion of liver - Destruction of red blood cells

The initial step in a malaria infection is transmission through a mosquito bite. The infectious agent then travels to the liver and is then released from the liver to invade red blood cells. As red blood cells are damaged, they are sequestered in the spleen, which causes splenomegaly ( Female anopheles mosquito bite - Parasite invasion of liver - Parasite release from liver - Destruction of red blood cells - Splenomegaly)

A nursing educator is asking students to review the cerebral artery most commonly affected in strokes. What is the correct response? a. Basilar artery b. Anterior cerebral artery c. Middle cerebral artery d. Posterior cerebral artery

The middle cerebral artery is the most common location for stroke

A nurse is reviewing a set of laboratory values. Which result requires the immediate follow-up by the nurse? a. Plasma K + 3.0 mEq/L b. Plasma K + 4.0 mEq/L c. Plasma K + 4.5 mEq/L d. Plasma K + 5.0 mEq/L

The normal range for plasma K + is 2.5 to 5.2 mEq/L.

A nursing student is asked to point out where the nucleus raphe magnus (NRM) is located. Where is the NRM located??

The nucleus raphe magnus is located in the center of the brainstem

A patient's test results reveal hepatitis C virus (HCV)(+)RNA and antibody test. The patient wonders how he got this infection. What is the primary means of transmission the nurse should describe? a. Fecal-oral b.. Skin to skin c. Via the blood d. Sexual activity

The patient has a hepatitis C infection and the primary route is through the blood

Margaret is known to have a posterior pituitary injury, which prevents proper secretion of posterior pituitary hormones. Margaret was recently in an automobile accident. Which of the expected responses to stress may be lacking in Margaret? a. Elevated heart rate (HR) b. Decreased urine output c. Pupil dilation d. Elevated immune response

The posterior pituitary secretes antidiuretic hormone (ADH), which normally reduces urine output in times of stress. Posterior pituitary hormones do NOT directly affect HR, immune response, or dilation of the pupils

Which of the following statements by a patient may indicate unstable angina? a. My chest pain feels like it always does b. I took a nitroglycerin tablet and now feel better c. This pain is like nothing I have ever felt before d. When I laid down for a few moments, I felt better

Unstable angina presents in new or unusual pain. Unstable angina does NOT easily resolve on its own nor does it response to nitroglycerin in a way that the pain resolves completely

Within the urologic system, which disorder is the nurse most likely to encounter? a. Pyelonephritis b. Interstitial cystitis c. Urinary tract infection d. Bladder cancer

Urinary tract infections are the most common urologic disorder

Which of the following statements may alert the nurse that a patient is suffering from a urinary tract infection? a. "I feel like I have to urinate, but I can't go at all" b. "My urine keeps starting and stopping when I go" c. "I feel pain when I go the bathroom" d. "I know I should go to the bathroom more, but I never feel like I have to "

Urinary tract infections can cause dysuria. Urinary tract infections do NOT prevent urination flow completely but it does increase the urge to urinate. Interruption of urinary flow is more common in benign prostatic hyperplasia

A patient has appeared in the emergency department with diabetic ketoacidosis. What are the steps that the nurse should explain, in the proper sequence, as to why the patient's respiration is now increased? - Increased ventilation - Ketone formation - Acidosis detected by chemoreceptors - Inability to use carbohydrates for metabolism - Decrease in blood pH

When carbohydrates cannot be utilized, ketones are formed. The ketones lower pH, which activates the chemoreceptors, which increase ventilation (Inability to use carbohydrates for metabolism - Ketone formation - Decrease in blood pH - Acidosis detected by chemoreceptors - Increased ventilation)

A female patient presents with inflammation of the vulva and vaginal tract as her only symptoms. Which is a likely cause? a. Candida albicans b. Ovarian torsion c. Fibroadenoma d. Masitis

(a) Candida albicans is a common cause of inflammation of the vulve and the vaginal tract. Patients with ovarian torsion has unilateral, sudden, severe abdominal pain radiating to the pelvis, groin, and thigh. Fibroadenoma is a benign breast mass and masitis is inflammation of the breast

A nursing student is becoming frustrated with her family as they discuss the disease shingles. Which of the following statements made by family members does the nursing student want correct? Select all that apply a. Shingles is caused by measles b. Shingles is usually asymptomatic c. If you had chickenpox, you cannot get shingles d. Shingles is just another name for herpes e. Shingles develops from the reactivation of a virus in the body

(a, b, c, d) Shingles is caused by the chickenpox virus, symptoms include a rash and severe pain. Shingles is renamed the herpes zoster in adults, it is NOT the same as a herpes simplex virus infection

Of the following valve disorders, which are most likely to be heard during systole? Select all that apply a. Mitral insufficency b. Tricuspid insufficency c. Mitral stenosis d. Aortic stenosis e. Tricuspid stenosis

(a, b, d) Blood flowing backwards through the mitral valve and tricuspid valve during systole will cause a murmur. Aortic stenosis murmur will appear during systole as blood is being ejected. Mitral stenosis and tricuspid stenosis murmurs would occur during diastole

A clinician needs to be able to assess a patient's suspected renal calculi. What diagnostic tools may aid the clinician? Select all that apply a. IV pyelogram (IVP) procedure b. Cystoscopy c. Cough test d. Computed tomography (CT) scan e. Urinalysis

(a, d) An IV pyelogram (IVP) examination and a CT scan can show the presence of stones blocking fluid flow. Cystoscopy examines the bladder, a cough test if for urinary incontinence, and urinalysis does NOT show the presence of stones

A clinician is gently tapping on a patient's facial area, asking whether pain is present in the maxillary or ethmoid area. Which disorder is the clinician assessing? a. Epiglottis b. Sinusitis c. Tonsillitis d. Pharyngitis

(b) Inflammation within the sinus cavities can cause a sense of pain over these areas. Epiglottis is inflammation of the epiglottis. Tonsillitis and pharyngitis would present with throat pain

A nursing student is asking about which general signs and symptoms may be present in cases of leukemia. Which of the following are appropriate responses by the nurse educator? Select all that apply a. No overt signs or symptoms are present b. Bone pain c. General leukopenia on completed blood count (CBC) d. Splenomegaly e. Anemia

(b, d, e) Overproliferation of cells can cause pain the bone marrow, can be sequestered in the spleen, causing splenomegaly, and can result in lower than normal RBCs. Leukemia can present with signs and symptoms and leukemia is associated with leukocytosis

A nursing learns that a patient has arrived in the emergency department complaining of a kidney stone. How will the nurse likely expect the patient to appear? a. Sitting quietly, trying NOT to move b. Taking loud and slow breaths c. Pacing to decrease pain d. Lying supine

(c) A patient with a kidney stone may try to move around to lessen pain. A patient is more likely to be breathing rapidly due to pain and lying supine will likely increase pain from a kidney stone

A patient's mother worries about a vaso-occlusive crisis for her son, who desires to play a team sport. Which of the following conditions does the son likely have? a. Polycythemia b. Hemophilia c. Sickle cell anemia d. Iron deficiency anemia

(c) Sickle cell anemia alters the shape of red blood cells, which can lodge in capillaries. Polycythemia is an elevation in RBC levels, hemophilia is a condition that causes excessive bleeding, and iron deficiency anemia does NOT cause vaso occlusive crisis

A urine sample is pink and foamy. Which factors are likely present? Select all that apply a. Glucose b. Ketones c. Blood d. Protein e. Bilirubin

(c, d) Blood gives urine a pinkish color and proteins give urine a foamy color. Glucose and ketones do NOT change urine appearance. Bilirubin makes the urine dark in color

A male client is diagnosed with benign prostatic hyperplasia (BPH). Which of the following symptoms likely brought him to seek assistance? a. Intense back pain b. Discharge from penis c. Fever and feeling poorly d. Difficulty urinating

(d) Benign prostatic hyperplasia (BPH) often disrupts urine flow in men. It does NOT normally cause back pain, discharge from the penis, or fever

A nurse is being asked to evaluate a patient who believes he has pneumonia. Which of the following is a common clinical presentation of pneumonia? a. Lack of fever b. Lack of cough c. Initial hemoptysis d. Sudden onset of symptoms

(d) Pneumonia symptoms often appear suddenly. Fever and cough are normally present with pneumonia but hemoptysis is NOT

A nursing student is handed a karyotype. She scans it quickly to see if it confirms diagnosis of Down syndrome. What information is she looking for on the karyotype? a. None. The presence of Down syndrome cannot be detected on a karyotype b. Alterations in the number of sex chromosomes c. One less chromosome on the karyotype d. XXY e. Three copies chromosome 21 f. Aneuploidy g. 46 chromosomes

(e, f) Down syndrome results from 3 copies of chromosome 21 (trisomy 21) and is detectable on a karyotype and aneuploidy is abnormal chromosome number. XXY is Klinefelter syndrome, NOT Down syndrome

A patient's chart shows blood urea nitrogen (BUN) is 22 mg/dL and creatinine is 2.0 mg/dL. The nurse is asked to report the BUN-to-creatinine level. What is the correct response for the nurse to make. expressed as a ratio?

11 because the BUN to creatinine ratio is calculated by dividing BUN levels by creatinine levels

A patient has one glass of wine each Friday and Saturday night. The glasses of wine are 8 ounces each. How many grams of alcohol does the patient consume each week?

48 because each 4 ounces of wine contain 12 grams of alcohol. The patient has two glasses, each 8 ounces, for 16 ounces total each week. 16 ounces of wine will contain 48 grams of alcohol

A male patient receives a prescription for 5-alpha-reductase inhibitor. Which of the following does the nurse expect to see documented on the patient's chart? a. Erectile dysfunction b. Testicular cancer c. Benign prostate hyperplasia (BPH) d. Vasectomy

5-alpha-reductase inhibitor are common treatment for benign prostate hyperplasia (BPH)

A semen sample is to be evaluated. Which of the following is NOT typically assessed in a semen sample? a. Sperm motility b. Sperm count c. Hormone levels d. Sperm morphology

A semen sample primarily evaluates sperm, it is NOT designed to assess hormone levels. Sperm motility, count, and morphology are evaluated in a semen sample

A patient has been scheduled for a sleep study for obstructive sleep apnea. The patient wants to know what they will be looking for during the test. Which of the following is appropriate for the nurse to share? a. We will see whether you try to sleepwalk during the test b. We will be looking to see whether you enter deep sleep and for how long c. We will be looking to see whether you stop breathing, even briefly, during the night d. We will see whether you have restless-leg syndrome

A sleep study for sleep apnea will look for cessation of breathing during sleep

A patient has suffered a spinal cord injury, resulting in a compromised of urinary reflex. The badder often overfills. Which type of incontinence does the nurse recognize the patient is experiencing? a. Functional b. Neurogenic c. Overactive bladder d. Mixed

A spinal cord injury can interrupt the neurological signals involved in the urination reflex

A nurse is assessing a patient for skin changes that can occur with diabetes mellitus (DM). Which of the following will the nurse note that is likely associated with diabetes mellitus? a. Red rash on chest b. Acanthosis nigricans c. Petechiae d. Purpura

Acanthosis nigricans are skin spots that occur with diabetes mellitus. A red rash on the chest, petechiae, and purpura are NOT typical of DM

Which of the following laboratory values supports the clinician's assessment of edema related to malnutrition? a. Hypercalcemia b. Hypoalumnemia c. Hypernatremia d. Hyperkalemia

Albumin synthesis decreases with malnutrition. Loss of albumin decreases oncotic pressure leading to to edema

A nurse is educating a patient who has been given a prescription for warfarin. Which of the following should the nurse inform the patient may affect the medication's effectiveness? a. Vitamin D b. Sodium c. Vitamin K d. Potassium

As warfarin effects vitamin K clotting factors, alterations in vitamin K levels can alter the medications effectiveness. Warfarin is NOT affected by vitamin D levels, sodium levels, and potassium levels

A stroke victim has suffered damage to Broca's area. Which of the following can the nurse expect? a. The patient cannot hear b. The patient loses balance easily c. The patient has problems speaking d. The patient has problems understanding what words mean

Broca's area controls expressive speech. Damage to the temporal lobe would cause hearing problems, damage to the cerebellum would cause balance loss, and damage to Wernickes area would case problems understanding speech

A nurse educator is speaking to students about a pulmonary condition that results in chronic bronchodilation. Which of the following conditions is he speaking about? a. Asthma b. Bronchiectasis c. Pulmonary fibrosis d. Emphysema

Bronchiectasis results in development of sustained bronchodilation. Asthma results from bronchoconstriction. Pulmonary fibrosis is NOT related to bronchodilation. Emphysema is a process in which alveoli are damaged

What state likely leads to the development of clubbing of the fingertips? a. Chronic hypercapnia b. Chronic hypoxemia c. Chronic hypocapnia d. Acute hypoxemia

Chronic hypoxemia can cause clubbing o the fingertips

Please place in the proper order how atherosclerosis may lead to a myocardial infarction - Myocardial ischemia - Decreased myocardial blood flow - Coronary artery occlusion - Myocardial infarction - Decreased myocardial oxygen delivery

Coronary artery occlusion leads to decreased myocardial blood flow, leading to decreased oxygen delivery and ischemia. This may result in myocardial infarction ( Coronary artery occlusion - Decreased myocardial blood flow - Decreased myocardial oxygen delivery - Myocardial ischemia - Myocardial infarction )

At which step does Corticosteroids block prostaglandin synthesis?

Corticosteroids reduce arachidonic acid formation through inhibition of the phospholipase. NSAIDs block the cyclooxygenase enzymes directly to inhibit prostaglandins (Inflammation - phospholipase - arachidonic acid - cyclooxygenase pathways - prostaglandins)

An order for a 24 hour urine collection has appeared. Which hormone is most likely being assessed? a. Oxytocin b. Thyroid stimulating hormone c. Cortisol d. Thyroxine

Cortisol levels can be assessed with a 24 hour urine collection

A nurse notes on a patient's chart a prescription for corticosteroids. The patient has severe Crohn's disease. What is the primary purpose of the medication? a. To decrease bouts of diarrhea b. To actively repair the intestinal lining c. To aid in nutrient absorption d. To suppress inflammatory response in the gastrointestinal (GI) tract

Crohn's disease cause inflammation and corticosteroids may lessen the inflammation. Corticosteroids are NOT antidiarrheal medicaations and do NOT repair the intestinal lining or aid in nutrient absorption

A 2 year old male is diagnosed with cryptorchidism. Which treatment recommendation does the nurse expect? a. Wait-and-see approach b. Hormone replacement c. Surgical correction d. Diet and lifestyle changes

Cryptochidism must be surgically corrected

A nurse is speaking to a patient about his sexual history. The patient has a disease that result in 50% of males with the disease developing erectile dysfunction. Which disease does the patient have? a. Cirrhosis of the liver b. Diabetes mellitus c. Hypertension d. Gastroesophageal reflux disease (GERD)

Diabetes mellitus is a primary risk factor for erectile dysfucntion

A patient is to receive digitalis. Which of the following laboratory serum values should a nurse be concerned about? a. Potassium b. Magnesium c. Sodium d. Calcium

Digitalis toxicity can develop with low potassium levels

Which of the following should be on a chart that describes anginal equivalents, especially in women? Select all that apply a. Dizziness b. Crushing chest pain c. Episodic dyspnea d. Pain in back e. Levine's sign

Dizziness, episodic dyspnea, and pain in the back describes anginal equivalents especially in women. Crushing chest pain and levine's sign are typical presentation of myocardial infarction

Please place the following steps in order leading to atherosclerosis - Endothelial injury - Low density lipoprotein (LDL) deposition - Atherosclerotic plaque - Foam cell formation - Macrophages attracted to area

Endothelial injury leads to macrophages entering the area. LDL deposition results in the formation of foam cells, which may lead to plaque formation ( Endothelial injury - Macrophages attracted to area - Low density lipoprotein (LDL) deposition - Foam cell formation - Atherosclerotic plaque)

Urine specimens are cultured to determine the causative organism. Which organism does the nurse expect the specimen to be positive for? a. Escherichia coli b. Streptococcus pyogenes c. Clostridium difficile d. Treponema pallidum

Escherichia coli is the most common infectious agent in urinary tract infections

A nurse is surprised to see a young individual, age 18 years, scheduled for a colonscopy. The nurse learns that the patient has an inherited disorder requiring earlier and more frequent colonoscopies. Which disease is the nurse likely to see present in the patient's history? a. Familial hypercholesterolemia b. Huntington's disease c. Ehlers-Danlos syndrome d. Familial adenomatous polyposis

Familial adenomatous polyposis is associated with a genetic mutation causing increased polyps in the colon. The risk for colon cancer is increased, necessitating more frequent assessment

A patient has hemophilia. Which shows a correct understanding of this disease? a. Thalassemia is the result of folic acid deficiency b. Thalassemia is also known as sickle cell anemia c. Thalassemia is a result of a genetic mutation d. Thalassemia is commonly outgrown by adulthood

Hemophilia can be an inherited disorder or arise from spontaneous mutation. Thalassemia and sickle cell anemia are NOT the same condition and it is a lifelong condition

A patient is requiring a rapid-acting insulin, with peak activity in 1 hour. Which of the following should be considered? a. Humulin N b. Humalog c. Humulin U d. Humulin L

Humalog is a rapid acting insulin with peak activity at 1 hour

The acronym "LGB" refers to which of the following? a. Sexual orientation b. Gender identity c. Biological identity d. Gender transitioning

LGB refers to lesbian, gay, and bisexual, which are sexual orientations

Release of endogenous opioids is key in pain management. Please place in order the steps involved in this process - Endogenous opioid release - Nucleus raphe magnus (NRM) stimulated - Periaqueductal gray matter (PAG) stimulated - Painful stimuli - Enkephalin-releasing neurons activated - Serotonin releasing neurons activated

Pain results in stimulation of the PAG, which causes activation of enkephalin releasing neurons, which in turn activate the NRM. The NRM stimulates releasing neurons which stimulates endogenous opioid release ( Painful stimuli - Periaqueductal gray matter (PAG) stimulated - Enkephali releasing neurons activated - Nucleus raphe magnus (NRM) stimulated - Serotonin releasing neurons activated - Endogenous opioid release)

A 24-hour urine collection reveals very elevated catecholamine levels. What may account for these results? a. Parathyroid gland tumor b. Anterior pituitary tumor c. Adrenal cortex tumor d. Pheochromocytoma

Pheochromocytoma is a tumor of the adrenal medulla. The adrenal medulla produced epinephrine and norepinephrine

A clinician is explaining to a patient that the patient has been diagnosed with Pneumocytis pneumonia. Which category of organism causes this form of pneumonia? a. Bacteria b. Virus c. Fungus d. Helminth

Pneumocystic pneumonia is a form of fungal pnumonia caused by pneumocystis jiroveci

When a female patient presents with secondary amenorrhea, which is the next diagnostic step? a. Obtain height and weight b. Review diet log c. Pregnancy test d. Pelvic examination

Secondary amenorrhea first requires the clinician to rule out pregnancy, which is the most common cause of secondary amenorrhea (c)

In which section is sodium levels are the highest? a. Intracellular fluid b. Interstitial fluid c. Extracellular fluid

Sodium is the predominant ion in extracellular fluid (ECF) and is higher in the ECF than the intracellular fluid (ICF)

A patient's chart lists the presence of incontinence. With no further information provided, which type of the incontinence is the nurse most likely to encounter in the patinet? a. Overflow b. Stress c. Neurogenic d. Functional

Stress incontinence is the most common type of incontinence

Which organ is most susceptible to damage and death due to prolonged ischemia?

The BRAIN is the organ that is most sensitive to reduction in its blood supply. The brain undergoes infarction and cell death within minutes

On serum blood laboratory results, a nurse notes elevated serum amylase. Which organ is likely damaged? a. Gallbladder b. Stomach c. Pancreas d. Appendix

The pancreas produces amylase. With pancreatic injury, amylase levels may elevate in the blood

A patient's bone marrow results reveal platelet levels of 90,000 per microliter. Which of the following is the correct term to describe this condition? a. Thrombocytosis b. Erythropoiesis c. Thrombocytopenia d. Thromboxane

Thrombocytopenia is the term for a low number of platelets, less than 100,000/uL. THrombocytosis is elevated platelets, erythropoieis is the formation of new red blood cells, and thromboxane is a signal released by platelets

A nurse is reading a medical journal article about levels of preventative health care in different populations. Of the following list, which group is most likely to experience less preventative health care? a. Men who have sex with men b. Heterosexual women c. Women who have sex with women d. Heterosexual men

Women who have sex with women are incorrectly thought to be at less risk for certain infections and thus receive less care

A person has an active pulmonary tuberculosis (TB) infection. Which of the following apply to this situation? Select all that apply a. A positive Mantoux test is likely b. The person can transmit TB to others c. The clinician will be unconcerned because this disease is easily treated d. Treatment will involve oral antibiotics for 10 to 14 days e. The sputum sample is positive for M. tuberculosis

(a, b, e) A Mantoux test will show exposure to TB, active TB can be transmitted to others and active TB infections present with M tuberculosis in the sputum. TB is NOT an easy disease to treat, it requires months of antibiotic treatment. The clinician will report the infection to public health

A nurse is working with a woman suffering from fatigue. Which of the following supports the nurse's conclusion that the patient is suffering from iron-deficiency anemia? Select all that apply a. Decreased hemoglobin b. Macrocytic cells c. Hypochromic cells d. Decreased hematocrit e. Megaloblastic cell

(a, c, d) Anemia causes decreased hemaglobin levels and hematocrit. Iron deficiency anemia causes cells that are paler in color. Iron deficiency anemia causes smaller than normal cells

While speaking with a patient suffering from mild, chronic pain, you learn that the patient exercises moderately to experience less pain and to gain a sense of well-being known as the "runner's high". Which of the following signals helps to suppress the sensation of pain? Select all that apply a. Enkephalins b. Substance P c. Dynorphins d. Endorphins e. Glutamate

(a, c, d) Enkephalins, dynorphins and endorphins are forms of endogenous opioids, which are natural analgestic substances

Parents present in the emergency department with a child suffering from diabetic ketoacidosis. The parents want to known why their child is hyperventilating. Which of the following responses by the nurse are appropriate? Select all that apply a. "Your son cannot get enough air into his lungs" b. "Your son's lungs are attempting to compensate for changes in the acidity of his blood" c. "Your son is demonstrating a panic response, which is altering his breathing" d. "This is a common response of the body to diabetic ketoacidosis" e. "The diabetic ketoacidosis is blocking gas exchange in his lungs"

(a, d ) In diabetic ketoacidosis, respiration increases to compensate for the acids produced and increased acidity of the blood due to ketones. Diabetic ketoacidosis does NOT inhibit ventilation

A nursing student is reviewing medications. Which of the following applies to heparin? Select all that apply a. Clot formation is NOT prevented b. Clot degradation speeds up c. Responses are consistent in all patients d. Effects are easily reversed e. Activated partial thromboplastin time (aPTT) values are used to monitor effectiveness

(a, e) Heparin does NOT stop clots from forming, it limits clot extension. aPTT indicates clotting time, which should be increased in a patient with heparin. Heparin is NOT a clot buster, NOT quickly or easily reversed, and must be monitored because the response is variable in patients

A clinician is reviewing white blood cell (WBC) differential results. The clinician states that the patient must be suffering an acute bacterial infection. Which of the following results did the clinician observe? Select all that apply a. Increase in neutrophil levels b. Increase in lymphocyte levels c. Increase in basophil levels d. Increase in monocyte levels e. Increase in polymorphonuclear (PMN) cell levels

(a, e) Neutrophils, also known as polymorphonuclear cells, are the first responders to injury or infection. Lymphocytes elevate in viral, NOT bacterial, infections. Basophils and monocytes do NOT reflect acute infections

A patient has developed pleural effusion. What is likely to be present? a. Increased tactile fremitus over affected area b. Dullness to percussion over affected area c. Coughing up gray sputum d. Hyperresonance

(b) Fluid in the lungs makes the area sound dull to percussion. Tactile fremitus tends to decrease with fluid in the pleural space. Gray sputum is a sign of coal miner's pneumoconiosis and hyperresonance occurs with increased air in the lungs

A patient has developed atrial fibrillation due to pooling of the blood in the right atrium due to improper movement forward of blood. Which valve disorder may contribute to this development? a. Left atrioventricular valve (AV) stenosis b. Tricuspid stenosis c. Aortic stenosis d. Mitral stenosis

(b) If blood cannot exit the right atrium, then it can increase the risk of clot formation, which can travel to the brain. Aortic stenosis would reduce blood flow from the left ventricle, NOT the right atrium. The issue is blood pooling in the right atrium, the mitral valve is on the left side of the heart and the left AV stenosis indicates mitral valve stenosis

A patient is complaining of very heavy menstrual bleeding lasting eight to nine days. She has never experienced this before and is worried about her health. Which of the following is correct for the nurse to document? a. The patient is overreacting b. The patient has menorrhagia c. The patient has amenorrhea d. The patient has dysmenorrhea

(b) Menorrhagia is excessive menstrual bleeding with prolonged menstrual periods. Amenorrhea is the cessation of periods and dysmenorrhea is painful periods

Results from a patient's family history and genetic testing results reveal the patient is homozygous dominant for an autosomal dominant disease. Which of the following explanations to the patient by the nurse is correct? a. "You have nothing to worry about. You cannot pass this disease on to your offspring" b. "You have a 50/50 change of passing this disease on to your offspring" c. "You have 100% chance of passing the genetics for this disease on to your offspring and your offspring developing the condition" d. "Although you can pass this genetically on to your offspring, your offspring will never develop the disease

(c) As the father is homozygous for an autosomal dominant condition, he will pass this on to his offspring, who will express the trait

A nurse is reviewing high-output heart failure. Which of the following is the correct association? a. Most common type of heart failure b. Develops in response to poor heart filling c. Develops due to decreased venous return to the heart d. May occur in conditions such as thyrotoxicosis

(d) High output failure results from extreme demands on the heart. This can occur with hormones released in thyrotoxicosis. High output failure is rare and develops due to excessive demands on the heart

A child's x-ray reveals a steeple sign in the throat area. Which of the following are correct associations? Select all that apply a. Medical emergency b. Bronchitis c. Pulmonary edema d. Epiglottitis e. Laryngitis

A steeple sign indicates epiglottis and it is a medical emergency. Bronchitis, pulmonary edema, and laryngitis are NOT present with a steeple sign

A patient is given an angiotensin-converting enzyme (ACE) inhibitor medication. Which of the following steps does this medication work on? a. Angiotensin to aldosterone b. Renin to angiotensinogen c. Angiotensin I to angiotensin II d. Angiotensinogen to renin

ACE stands for angiotensin converting enzyme, which converts angiotensin I to angiotensin II

A nurse learns that her patient has central diabetes insipidus and does NOT produce antidiuretic hormone (ADH). As the nurse scans the laboratory report, which values are expected given the patient's condition? a. Elevated plasma osmolarity, low urine osmolarity b. Elevated plasma osmolarity, elevated urine osmolarity c. Low plasma osmolarity, low urine osmolarity d. Low plasma osmolarity, elevated urine osmolarity

ADH stimulates fluid retention. Without ADH, the plasma osmolarity increases while the urine osmolarity decreased

A patient presents with a strange complaint of "I am more tan than ever. I am seldom out in the sun, but I look so tan" Which laboratory values may be evaluated? a. Parathyroid hormone (PTH) b. Adrenocorticotropic hormone (ACTH) c. Growth hormone (GH) d. Epinephrine

Adrenocorticotropic hormone (ACTH) is synthesized with the molecule melanocyte stimulating hormone (MSH). When ACTH levels are high, MSH levels can be high, leading to a tan appearance

A nursing student is reviewing her note cards for the RAAS. She has renin, angiotensinogen, angiotensin I, and angiotensin II. Which factors should come next? a. Aldosterone b. Antidiuretic hormone (ADH) c. Angiotensin converting enzyme d. Atrial natriuretic peptide (ANP)

Angiotensin II stimulates aldosterone release from the adrenal cortex

A patient's laboratory values over the course of several weeks reveal that beta-human chorionic gonadotropin (b-HCG) levels are significantly increasing. What is the correct interpretation? a. The patient is pregnant b. The patient is in menopause c. The patient is infertile d. The patient has polycystic ovarian syndrome

Beta-human chorionic gonadotropin (b-HCG) is the hormone secreted with embryo implantation. Levels will increase as growth occur (a)

A nursing student is listing the factors measured in arterial blood gases. She states partial pressure of arterial oxygen, partial pressure of arterial carbon dioxide, and pH. Which factor is she missing? a. Hydrogen ion b. Partial pressure venous oxygen c. Bicarbonate d. A1c

Bicarbonate values are a standard arterial blood gas measurement. Hydrogen ion levels are reported as pH and A1c is a measurement for diabetes mellitus examining blood sugar and red blood cells

A child ingests a large amount of bicarbonate. Which compensations will occur in the body? Select all that apply a. Increased ventilation b. Increased H+ ion excretion by kidneys c. Decreased ventilation d. Increased H+ ion reabsorption by kidneys e. Increased bicarbonate excretion by kidneys

Decreasing ventilation will increase carbon dioxide levels to compensate for the alkalosis. In metabolic acidosis, the kidneys compensate by retaining hydrogen ion. In alkalosis, the kidneys excrete more bicarbonate (c, d, e)

A patient diagnosed with rheumatoid arthritis (RA) is talking to the nurse about treatment options. Which of the following is considered to be a standard treatment for RA? a. Antihistamines b. Disease-modifying antirheumatic drugs c. Bone marrow transplant d. Conservative wait-and-see approach

Disease modifying antirheumatic drugs are a standard treatment for RA. Antihistamines are NOT effective for autoimmune diseases and bone marrow transplants are NOT used for RA

Assessment of a patient is requiring an evaluation of the velocity of blood flow through heart structures. Which diagnostic tool will provide the best information? a. Electrocardiogram b. Transthoracic echocardiography c. Venography d. Doppler echocardiography

Doppler echocardiography provides as evaluation of blood flow in the heart

Which is the most common cause of erythropoietin deficiency? a. Renal failure b. Spleen dysfunction c. Hyperthyroidism d. Bone marrow damage

Erythropoietin is a hormone produced in the kidneys. Therefore, renal failure would affect the synthesis of the hormone. Renal failure is the most common cause of erythropoietin deficiency

Which is an example of eustress a. Job promotion b. Divorce c. Unemployment d. Death of a friend

Eustress is the stress that motivates an individual positively. A JOB PROMOTION is an example of eustress as it gives positive feelings

A patient's file reports the presence of mesothelioma. Which substance may the patient have been exposed to? a. Asbestos b. Coal dust c. Silica d. Dust

Exposure to asbestos has been linked to an increased risk for mesothelioma

In folic-acid deficiency, the laboratory clinician is looking for cells of which shape or color? a. Normocytic b. Megaloblastic c. Microcytic d. Hypochromic

Folic acid deficiency results in larger than normal cells, which are referred to as megaloblastic. Hypochromic cells occur with iron deficiency anemia, NOT folic acid deficiency anemia

A patient, age 28 years, presents with a minor hematocele. What is the appropriate treatment response? a. Emergency surgery b. Follow-up digital rectal exam c. Needle aspiration d. Watchful waiting

Hematoceles often heal on their own

A nurse is speaking to a patient who will be receiving hemodialysis. Which of the following is accurate information for the nurse to share? a. Hemodialysis is done one time per week for eight hours b. Hemodialysis is done one time per week for 45 minutes c. Hemodialysis is done daily for five hours d. Hemodialysis is done three times per week for four to six hours

Hemodialysis reigmens may vary, but typically hemodialysis is done several times per week for several hours

A nurse is taking a morbidly obese patient's vital signs. Which comment is MOST appropriate for the nurse to make? a. "Please step on the scale. You should NOT be ashamed just because you are extremely overweight" b. "It's too bad you do NOT have more willpower around food. Your weight is negatively impacting your health" c. "I will now take your vital signs. Please let me know if you have any questions" d. "Wait right here. Your arm is so large, I will need a larger blood pressure cuff"

I will now take your vital signs. Please let me know if you have any questions

As a dentist examines a patient's tooth enamel and sees significant erosion of the enamel, which of eating disorder may she be most concerned with happening? a. Binge eating b. Night-eating syndrome c. Anorexia nervosa d. Bulimia nervosa

If a bulimia nervosa patient is inducing vomiting, then the enamel of the teeth can be worn down by the stomach acid. Anorexia nervosa may affect teeth health, bulimia is more likely to specifically erode enamel

A nurse educator is preparing educational materials for sexually transmitted infections (STIs). Which group is the most likely target for this information? a. Individuals less than 15 years of age b. Individuals age 15 to 24 years c. Individuals age 35 to 45 years d. Individuals older than 50 years

Individuals age 15 to 24 years are the greatest risk for sexually transmitted infections (STIs)

A patient presents in the emergency department with diabetic ketoacidosis. Which of the following treatments will be administered first? a. Fluid restriction b. IV administration of insulin c. Bicarbonate to correct acidosis d. Beta blockers

Insulin administration will counteract the hyperglycemia and lessen ketone production

Which condition is developed due to metabolic acidosis with an elevated anion gap? a. Hypralimentation b. Ketoacidosis c. Anerobic metabolism d. Hyperaldosteronism

KETOACIDOSIS is a condition where an elevated anion gap is observed

A patient is unable to pass a renal calculi. Which of the following treatment options may the clinician consider? a. Insertion of medications in the baldder b. Lithotripsy c. Ingestion of a beverage formulation designed to break up the stone d. Immediate surgery

LIthotripsy uses sound waaves to break up renal calculi

A nurse sees on the patient form "LMP". What does "LMP" refer to ? a. Length between menstrual periods b. Last missed period c. Last menstrual period d. Length of menstrual period

LMP stands for last menstrual period

A male, age 15 years, is being assessed for delayed puberty. Which of the following may the nurse note is indicative of delayed puberty? a. Lack of pubic hair b. Elevated testosterone c. Enlarged penis and testes d. Short stature

Lack of pubic hair by age 15 years is a sign of delayed puberty. Elevated testosterone and small penis and testes are indicative of delayed puberty, short stature is NOT

An injury that compromises S1 and L5 may result in which of the following? a. Lack of sensation from the pelvic area and below b. Lack of sensation from the lateral malleolus and lateral knee c. Lack of sensation from the medial malleolus d. Lack of motor movement of the knee and ankle

Lack of sensation from the lateral malleolus and lateral knee

A nurse needs to place a patient in the best position possible to hear a mitral valve stenosis murmur. Which position should the nurse choose? a. Supine b. Supine with feet elevated c. Prone d. Left lateral decubitus

Left lateral decubitus is the best position for hearing mitral valve stenosis

A physical therapist wants to convey the benefits of exercise to a patient. The patient claims to know all the benefits of exercise and proceeds to list these benefits. Which of the following exercise benefits listed by the patient show further education is needed? Select all that apply a. Increased fat storage b. Increased bone formation c. Increased high density lipoprotein (HDL) levels d. Increased low-density lipoprotein (LDL) levels e. Increased glucose uptake f. Decrease blood vessel formation

Lipolysis, NOT fat storage, is increased with exercise. LDL levels are reduced and angiogenesis is stimulated with exercise. Exercise facilitates the movement of glucose into cells, increased HDL levels, and increased bone density/formation (a, d, f)

What causes lymphomas? a. They do NOT differentiate into mature cells b. They arise from abnormal proliferation of T lymphocytes c. They appear like blast cells d. They develop in the precursor cells of the bone marrow

Lymphoma cells specifically arise from proliferation of B or T lymphocytes. The tumor mainly begins in the lymph nodes

Tay Sachs is a disease resulting from an accumulation of lipids due to a lack of which type of enzymes a. DNA enzymes b. Lysosomal enzymes c. Ribosomal enzymes d. Nuclear enzymes

Lysosomal enzymes are used to degrade cellular products. When lysosomal enzymes re disrupted, cellular accumulations, as in Tay Sachs disease, may occur. DNA enzymes are involved in DNA replication, ribosomal enzymes are involved in protein synthesis, and enzymes in the nucleus are NOT involved with lipid accumulation

Which of the following distinguishing features of pain are mismatched with the disorder? a. Appendicitis and right lower quadrant pain b. Peripheral artery disease and intermittent claudication c. Myocardial infarction and squeezing chest pain d. Nephrolithiasis and right upper quadrant pain

Nephrolithiasis, or kidney stones, often presents with flank pain NOT right quadrant pain

A mother brings her child to seek medical attention because she has noted some pigmented areas on the child's torso. The areas of pigmentation are dark and flat. The mother looked for images of these spots while on an internet website and now is very concerned. As the mother speaks with the clinician, she expresses her worry that the spots represent which disease a. Neimann-Pick disease b. G6PD deficiency c. Neurofibromatosis d. Ehlers-Danlos syndrome

Neurofibromatosis can present with darkened, flat, pigmented areas known as cafe au lait spots. Neimann Pick disease presents with gastrointestinal problems, Ehlers Danlos syndrome causes hypermobility in the joints, and G6PD deficiency causes hemolysis

A nursing student is looking at a blood sample slide. He is told to look for granulocytes. Which of the following cells should he be looking for? Select all that apply a. Neutrophils b. Basophils c. Monocytes d. Lymphocytes e. Eosinophils

Nuetrophils, basophils, and eosinophils are granulocytes. Monocytes and lymphocytes are agranulocytes

What is the characteristic clinical manifestation of Wilson disease? a. Corneal Kayser-Fleischer rings b. Difficulty in speaking c. Tremor d. Excessive salivation

On a clinical eye examination, corneal Kayser-Fleischer rings are characteristically seen in clients who have Wilson disease

A patient is in severe pain with a renal calculi. Which of the following should be considered for pain management? a. The nurse suggests that the patient "pace the halls" because the pain can be controlled with movement b. The nurse provides a muscle relaxant c. The nurse provides an NSAID medication d. The nurse provides an opioid

Opioids are normally needed to manage the pain associated with renal calculi

A patient is to receive oxytocin infusion. What does the nurse assume to be the most likely reason for this infusion? a. Increase fluid retention to maintain blood pressure b. Stimulate uterine contractions during delivery c. Suppress elevated blood glucose levels d. Maintain open bronchiole airways

Oxytocin causes the smooth muscle of the female reproductive tract to contract. Oxytocin increases during labor and delivery

A male patient is diagnosed with gonorrhea. Which of the following is the most likely symptom that prompted the patient to seek medical help? a. Flank pain b. Dysuria and penile discharge c. Blood in urine d. Severe headache and dizziness

Pain with urination and discharge from the penis are common presentations with a gonorrhea infection

A nursing educator is discussing that urinary incontinence is often under-reported. What is the primary reason for this under-reporting that nursing students should know? a. Urinary incontinence is extremely rare b. Patients may be embarrassed to share details of incontinence c. Most patients do NOT know what urinary incontinence is d. No documentation system tracts urinary incontinence

Patients may NOT report urinary incontinence due to embarrassment. Clinicians should be aware of this hesistancy.

A patient is suspected of developing pericarditis. Which of the following may the nurse observe to support this diagnosis? a. Kussmaul's respiration b. Profound hypotensive shock c. Pericardial friction rub d. Patient writhing in pain

Pericardial friction rubs are a common sign of pericarditis due to inflammation. Kussmaul's respirations appear with diabetic ketoacidosis. Hypotensive shock is NOT a sign of pericarditis. Although pain may be present with pericarditis, the patient is unlikely to be writhing in pain

A nurse is speaking to a patient who is to be on long term corticosteroid treatment. Such a treatment may suppress the functioning of which tissues? a. Adrenal medulla b. Pancreas c. Parathyroid gland d. Adrenal cortex

Prolonged corticosteroids usage can suppress adrenal gland function (d)

A patient presents with an eating disorder. The patient is normal weight and reports no night-time eating or restrictive eating, nor does the patient feel that he eats abnormally large amounts of food at one sitting. The patient does admit to excessive exercise to control his weight. Which eating disorders meets the criteria outlined? a. Bulimia nervosa b. Anorexia nervosa c. Purging disorder d. Binge eating

Purge disorder involves engaging in excessive exercise to control weight

In which of the following conditions would a nurse expect to see lower-than-normal B and T cells? a. HIV b. Severe combined immmunodeficiency (SCID) c. Immunoglobulin A (IgA) deficiency d. DiGeorge syndrome

SCID disrupts development of both B and T cells. HIV is primarily disorder of T cells and DiGeorge syndrome affects T cell development. T cells are NOT affected in IgA deficiency

Which of the following individuals would a nurse be most concerned about with respect to the development of Sheehan's syndrome? a. 15 year old male b. Postpartum woman c. Young child with growth delay d. Postmenopausal woman

Sheehan's syndrome, which results from disruption of blood flow to the pituitary gland, most commonly happens after childbirth

A nurse is reviewing a patient's fluid status, as well as plasma values for electrolytes. The nurse knows which ion MOST significantly influences plasma volume levels? a. Calcium b. Potassium c. Magnesium d. Sodium

Sodium levels are high in the ECF. The quantity of sodium in the ECF make it the primary determinant of plasma volume. Calcium is high in the ECF but is NOT the primary determinant of plasma volume

Some antibiotics target which of the following cellular structures? a. Endoplasmic reticulum b. Lysosomes c. Ribosomes d. Golgi apparatus

Some antibiotic medications, such as erythromycin, disrupt bacterial ribsomal function

A woman suffering from interstitial cystitis is looking for advice on how to improve her condition. Which of the following foods may the nurse suggest the patient avoid? Select all that apply a. Spicy foods b. Cheese c. Coffee d. Red meat e. Tomatoes

Spicy foods, coffee, and tomatoes can worsen the symptoms of interstitial cystitis. Cheese and red meat are NOT known to worsen interstitial cystitis

A patient has been diagnosed with pancreatic cancer. The family has been doing some research on the disease. Which of the following is an idea expressed by the family that the clinician should correct? a. The exact cause of pancreatic cancer is unknown b. There may be no significant signs and symptoms early in pancreatic cancer c. Survival rate of pancreatic cancer is very high d. Pancreatic cancer is often a silent disease

Survival rate for pancreatic cancer is NOT high. Pancreatic cancer cause is NOT well understood, it may be advanced before signs and symptoms appear, and it may develop with few signs and symptoms

A new medication is developed to block inflammatory cytokines produced by macrophages. Which of the following signals are the targets of the medication? a. TNF-alpha b. C-RP c. IL-1 d. ESR e. IL-6

TNF alpha, IL 1 and IL 6 are inflammatory cytokines created by WBCs. C-RP and ESR are markers of inflammation, NOT inflammatory cytokines

A nurse explains to the parents of a young child with a severe skin injury that the best course of action is to leave the wound open for 4 to 5 days before closure. What type of the skin wound is the nurse likely describing? a. Contracture b. Primary intention c. Secondary intention d. Tertiary intention

Tertiary intention healing occurs with severe wounds, often left open for several days before closure.

A patient presents to the clinic with hives. The patient states that she has been experiencing extreme stress between home and work. Which of the following does the nurse practitioner think may be playing a role? a. Allostasis and adaptation b. Allostatic overload c. Strong, positive adaptive approach d. Decreased allostatic load

The current stressors appear to be greater than the patient's ability to cope

Which is the reservoir or Neisseria meningitidis? a. Human nasopharynx b. African fruit bat c. White-tailed deer d. Contaminated drinking water

The human nasopharynx is the reservoir for Neisseria meningitidis

Which of the following sexually transmitted infections, if left untreated, can lead to neurological disturbances? a. Chlamydia trachomatis b. Syphilis c. Chancroid d. Neisseria gonorrhoeae

The tertiary stage of syphilis infection causes neurological issues

Which area contains 40% of total body weight? a. Intracellular fluid b. Interstitial fluid c. Extracellular fluid

Total weight in the body is 60% water, with 40% in the intracellular fluid (ICF) and 20% in the extracellular fluid (ECF)

A patient has an anatomical abnormally that allows fluid in the bladder to flow up into the ureters. What should be listed on the patient's chart? a. Hydronephrosis b. Urinary incontinence c. Vesicoureteral reflux d. Pyelonephritis

Vesicoureteral reflux indicates backwards flow up the ureters. Hydronephrosis is accumulation of fluid in the renal pelvis, urinary incontinence is leakage of urine, and pyelonephritis is infection of the renal tissue

A patient has a disruption of bile function. What nutrient absorption will be affected the most? a. Electrolytes such as sodium and potassium b. Vitamin A, D, E, and K c. Vitamins B and C d. Calcium

Vitamin A, D, E, and K are fat soluble vitamins. Adequate bile will be needed for their absorption. Bile does NOT directly affect electrolyte levels, absorption of vitamins B and C, or absorption of calcium

A patient has kidney failure. How may this affect the patient's ability to compensate for acidosis? a. Kidneys will be unable to retain H+ as expected b. Kidneys will be unable to excrete bicarbonate c. Kidneys will be unable to excrete H+ as expected d. Kidneys will be unable to decrease urine output

(c) In acidosis, the kidneys need to be able to excrete hydrogen ions. In acidosis, the kidneys excrete hydrogen and retain bicarbonate. The amount of urine output is NOT a primary compensation in acid base disorders

A patient has a cardiac output of 5 liters per minute and a heart rate of 100 beats per minute. What is the stroke volume in milliliters per beat?

50 because cardiac output is calculated as heart rate (100 beats per minute) times stroke volume (50 mL per beat)

A patient has end-diastolic volume of 120 ml and end-systolic volume of 60 ml. What is the patient's stroke volume for that beat in ml/beat?

60 because stroke volume is calculated by substracting end systolic volume from end diastolic volume

Heart rate equals 100 beats per minute (bpm) and stroke volume equals 70 mL/beat. What is the patient's cardiac output in L/min?

7 because cardiac output is calculated by multiplying heart rate times stroke volume

A young patient needs to take medication in an easily absorbed mist. Which delivery method will be used? a. Nebulizer b. Nasal cannula c. Inhaler d. Sinus drop

A nebulizer can be used to deliver inhaled medications in a fine mist form

A stroke victim has a diminished gag reflex. Which lobe of the lung is most at risk for aspiration pneumonia? a. Left lower lobe b. Left upper lobe c. Right lower lobe d. Right middle lobe

Because of the secondary bronchi structure, the right middle lobe is most at risk for aspiration pneumonia

A clinician is examining colonoscopy results. Which of the following findings points to Crohn's disease rather than to ulcerative colitis? a. The disease process is ascending from the rectum b. Pseudopolyps can be seen in several locations c. A cobblestone appearance is apparant d. Only the inner lining of the intestine is damaged

Cobblestoning may occur in Crohn's disease. Crohn's disease can affect any part of the gastrointestinal (GI) tract and ulcerative colitis tends to cause polyp function

A woman has received the results of an abnormal Pap smear. A follow up assessment of cervical cells is recommended. Which of the following procedures should the nurse prepare to discuss with the patient? a. Lapartomy b. Colposcopy c. Cystoscopy d. Hysteroscopy

Colposcopy is a procedure to look at the cervix in greater detail. Lapartomy is a surgical procedure, cystoscopy is used to examine the bladder, and hysteroscopy is a procedure to examine the uterus (b)

A patient undergoes bariatric surgery. Which potential complications should the nurse take time to discuss with the patient? Select all that apply a. Dumping syndrome b. Herniation c. Lifelong inability to due abdominal exercises d. Rapid, excessive weight gain e. Vitamin B12 deficiency

Complications of bariatric surgery include dumping syndrome, herniation (hernia formation), and vitamin B12 deficiency (a, b, e)

A critical care patient is to receive continual renal replacement therapy (CRRT). What is an appropriate description to share with family members of the patient? a. The patient's blood will be purified over 24 hour b. CRRT is a form of hemodialysis and is done every three days c. CRRT is a form of plasmapheresis and removes antibodies from the plasma d. CRRT provides IV infusions due to fluid loss

Continual renal replacement therapy (CRRT) is a continual process to purify the blood. Hemodialysis is completed on a schedule every several days. CRRT is NOT a form of plasmapheresis or fluid replacement

Please place the following in order relating to how damage from a myocardial infarction may lead to an increase in stroke risk - Blood stasis - Atrial fibrillation - Damage to the conduction circuit from myocardial infarction - Reentry circuit - Increased clot formation

Damage to the conduction circuit can happen with a myocardial infarction. This can cause reentry circuits to develop, which may cause atrial fibrillation. In the atria, stasis of blood may develop leading to increased risk of clot formation ( Damage to the conduction circuit from myocardial infarction - Reentry circuit -Atrial fibrillation - Blood stasis - Increased clot formation )

Which of the following assessment results may point to liver dysfunction in a patient? Select all that apply a. Dark-colored urine b. Hepatomegaly c. Reduced prothrombin time (PT) and activated partial thromboplastin time (aPTT) d. Left lower quadrant (LLQ) pain e. Elevated serum ammonia levels

Dark colored urine indicates bilirubin in urine, enlargement of the liver can occur with liver damage, and ammonia levels may elevate with liver dysfunction (a, b, e) Clotting times are normally elevated with liver dysfunction and left lower quadrant pain is NOT present

A nurse is charged with caring for a patient with disseminated intravascular coagulation. Which of the following will the nurse likely expect? a. A patient in overall good health, presenting with some bruising b. Speaking with the patient about hydration and nutrition lifestyle choices c. Assisting in managing clotting time and bleeding developments in a severely ill patient d. Carefully charting developments but allowing current disorders to run its course

Disseminated intravascular coagulation is a severe development of clotting and bleeding disruption, it should be medically managed

A patient is prescribed a diuretic and is directed to consume more bananas and orange juice. Which electrolyte condition are the dietary recommendations designed to help manage? a. Hypocalcemia b. Hypokalemia c. Hyponatremia d. Hypermagnesemia

Diuretics may cause a loss os potassium. Orange juice and bananas are high in potassium

A patient is to be assessed for a heart murmur. Which technique should the nurse most likely plan on discussing with the patient? a. Angiography b. Electrocardiography c. Echocardiography d. Cardiac catheterization

Echocaardiography is the gold standard for heart murmur diagnosis

A patient presents with an ectopic pregnancy. Which structure does the nurse believe is most likely the location for the implantation? a. Uterine lining b. Surface of ovary c. Fallopian tubes d. Abdominal wall

Ectopic pregnancies most commonly implant in the fallopian tubes

A nursing student is directed to perform an egophony examination on a patient. Which of the following shows current understanding by the nursing student? a. Asking patient to repeat the 'e' sound b. Tapping on the chest wall c. Asking the patient to whistle d. Taking measurement of the patient's chest wall with respiration

Egophony uses changes in vowel sounds as assessment of lung disorder. Tapping on chest wall is percussion. Whistling is involved in the whisper pectoriloquy test to detect change in sound due to lung consolidation

A patient is to be assessed for the presence of human leukocyte antigen (HLA). Which type of disorder is the clinician most concerned about? a. Autoimmune diseases b. Allergies c. Delayed hypersensitivity reaction d. Contact dermatitis

Elevated HLA is common in some autoimmune diseases

A nurse explains to a male patient that they will be doing a laboratory test for the beta-human chorionic gonadotropin (b-HCG). The patient asks why that test is being done on him because he knows that b-HCG is the pregnancy hormone. Why would the clinician request this test in a male? a. Elevated levels of b-HCG are indicative of prostate cancer in men b. Elevated levels of b-HCG are associative with testicular cancer in men c. Decreased levels of b-HCG are indicative of erectile dysfunction in men d. Decreased levels of b-HCG are indicative of the success of a vasectomy

Elevated levels of b-HCG are associative with testicular cancer in men

Which of the following, in the correct order of progression, would a nurse expect in a patient with untreated syphilis? - All over rash - Infection - Neurological symptoms - Gummas - Chancre

First, the person is infected with syphilis. A chancre appears, followed by an overall rash, gummas, and neurolgoical symptoms (Infection - Chancre - All over rash - Gummas - Neurological symptoms)

A woman is considering hormone replacement therapy for night sweats and hot flashes. Which of the following shows a correct understanding? a. The woman is breastfeeding b. The woman is menopausal c. The woman is wanting to increase her chance of conceiving d. The woman is looking to prevent ovulation

Hormonal changes associated with menopause have been known to cause night sweats and hot flashes

A patient has been diagnosed with benign prostatic hyperplasia. The patient wants to known what "hyperplasia" means. Which of the following statements by the nurse correctly explains hyperplasia to the patient? a. "Your prostate is shrinking as the cells decrease in size due to lack of use" b. "The prostate gland is enlarging due to each cell growing in size" c. "Your prostate gland is experiencing cancerous growth" d. "Your prostate gland is increasing in size due to increase in cell number"

Hyperplasia is an increase in cell number. Atrophy is when cells shrink due to lack of use, hypertrophy is an increase in cell size, and neoplasia is cancerous growth, NOT hyperplasia

A patient has severe pulmonic valve stenosis. Which of the following will the nurse expect to also see? a. Jugular vein distension b. Pulmonary edema c. Exertional dyspnea d. Enlarged left ventricle

If blood cannot be ejected from the right side of the heart, it will back up into systemic veins

A nursing student is reviewing the role of glutamate in cerebral cellular damage. Please place the following events in the correct order - Increased release of degrading enzymes - Glutamate opens sodium and calcium channels on the postsynaptic membrane - Increased potassium across the cell membrane - Cell death - Increased glutamate releases from cells

In a stroke, potassium may enter the cell and increase glutamate release. Glutamate may accumulate and activate bind to the postsynaptic receptor of neurons. This causes an influx of sodium and calcium, which activates degradative enzymes, causing cell death ( Increased potassium across the cell membrane - Increased glutamate releases from cells - Glutamate opens sodium and calcium channels on the postsynaptic membrane - Increased release of degrading enzymes - Cell death)

A patient presents with the following characteristics: reduced white blood cell (WBC) levels, immunosuppression, a sense of being rundown and unable to cope, and suppressed hormone levels. Which stage of the stress response may the patient be experiencing? a. Alarm b. Resistance c. Exhaustion d. Indeterminate

In the exhaustion stage of the stress response, hormone levels decline, immunosuppression develops, adn sense of being overwhelmed may develop. Alarm stage is characterized by elevated hormone levels and a strong immune response. Resistance stage has high stress hormones. Indeterminate is NOT a stage of the stress response

A patient asks about heart disease and atherosclerosis because she is concerned about her family history. Which of the following does a nurse share as risk factors for athersclerosis? Select all that apply a. Age b. Diabetes mellitus c. Diet high in saturated fats d. Moderate active lifestyle e. Lower than normal blood lipid levels

Increasing age is a risk for heart disease. Diabetes mellitus and diets high in saturated fats increases the risk of heart disease in atherosclerosis. A moderate active lifestyle and lower than normal blood lipids decreases the risk for heart disease

A nurse is speaking to parents prior to their child's abdominal surgery. Which of the following presents the most risk for healing of the surgical wound? a. Infection b. Poor nutritional status c. Not enough ultraviolet (UV) light d. Limited activity

Infection is the number one risk for prolonging wound healing. Poor nutrition can slow wound healing, but it is NOT the number one risk factor. A lack of UV light is NOT known as a risk factor for wound healing

A patient appears for a follow up appointment. The patient was previously diagnosed with Mycoplasma pneumoniae infection. Which is an appropriate thought the nurse may have about this patient? a. The patient is likely HIV positive, as this is a common opportunistic pneumonia with HIV infections b. The patient has what is known as walking pneumonia c. The patient has a rare, fungal form of pneumonia d. The patient has a mild form of a tuberculosis infection

Infections with mycoplasma pneumoniae are often considered walking pneumonia. Pneumocysstic penumonia, NOT M pneumoniae, is an opportunistic form of pneumonia that may occur in HIV. M pneumoniae is a bacterium, NOT a fungus

A patient is taking a nephrotoxic drug. Which type of renal dysfunction is the nurse most likely concerned about developing? a. Prerenal b. Intrarenal c. Postrenal d. Juxtarenal

Intrarenal injury occurs with direct damage to the kidney, such as by nephrotoxic drugs. Prerenal injury occurs with reduced blood flow to the kidneys. Postrenal injury occurs with obstruction and juxtarenal is NOT a form of kidney injury

For which of the following would antiplatelet medications be part of the treatment strategy? a. Subarachnoid hemorrhage b. Brain bleed c. Ischemic stroke d. Hemorrhagic stroke

Ischemic strokes are due to clots. Antiplatelet medications can help prevent clot formation. Antiplatelet medication would NOT be helpful with bleeding

A nurse is reviewing urinalysis results. Which factors does she review to see whether the patient has a urinary ract infection? Select all that apply a. Glucose b. Bilirubin c. Albumin d. Leukocyte esterase e. Nitrite

Leukocyte esterase and nitrite will be high in urine sample if a urinary tract infection is present (d, e)

A patient with Hasimoto's thyroditis is being given levothyroxine. The patient wants to know how the medication works. Which of the following responses is correct for the nurse to make? a. Levothyroxine blocks excess thyroid hormones b. Levothyroxine stimulates thyroid receptors c. Levothyroxine blocks the antibodies that cause Hashimoto's thyroditis d. Levothyroxine replaces missing thyroid hormone

Levothyroxine is a medication that compensates for lacking thyroid hormones

A patient is taking a proton pump inhibitor medication. Which of the following is the most likely purpose of this medication? a. Reduce inflammation in the gastrointestinal (GI) tract b. Decrease acidity of stomach secretions c. Increase GI motility d. Decrease bouts of diarrhea

Proton pump inhibitors reduce hydrogen ion levels (acidity) in the stomach

A patient is suspected to have a hepatitis infection. Which of the following will be most helpful for the clinician's diagnosis? a. Presence of jaundice b. Presence of hepatomegaly c. Serology results d. Elevated aspartate aminotransferase (AST), alanine transaminase (ALT)

Serology results can identify the specific form of hepatitis. Jaundice and hepatomegaly are NOT specific for different forms of hepatitis. Although aspartate aminotransferase (AST), alanine transaminase (ALT) elevate with liver dysfunction, they are NOT specific for different forms of hepatitis

What is the BEST intervention to correct hyprvolemic hyponatremia in a client diagnosed with a serum sodium level of 120 mEq/L? a. Administration of alendronate b. Administration of tolvaptan c. Administration of sevelamer d. Administration of hypotoniterm-243c IV fluids

TOLVAPTAN is used to treat hypervolemic hyponatremia

A group of nursing students has created a flowchart to learn the hormones of the stress response. Which of the following show a mistake has been made on the flowchart? a. Hypothalamus - post pituitary - adrenal gland b. Posterior pituitary - antidiuretic hormone (ADH) - fluid retention c. Chronic renal failure (CRF) - Adrenocorticotropic hormone (ACTH) - cortisol d. Anterior pituitary - adrenal cortex - cortisol

The hypothalamus stimulates the anterior pituitary, which then stimulates the adrenal gland

A test for a protozoal sexually transmitted infection is ordered. Which organism is likely the reason for the test? a. Treponema pallidum b. Chlamydia trachomatis c. Trichomonas vaginalis d. Klebsiella granulomatis

Trichomonas vaginalis is a protozoal sexually transmitted infection. Treponema pallidum, chlamydia trachomatis, and klebsiella granulomatis are bacterial infections

A patient arrives at the emergency department complaining of abdominal pain. Please place the following steps in order for this assessement - Palpitation - Percussion - Inspection - Auscultation - Vital signs

Vital signs should be assessed first, followed by inspection, auscultation, percussion, and finally palpitation ( Vital signs - inspection - auscultation - percussion - palpitation)

A nursing student is reviewing genetic disorders for an upcoming exam. On her flashcards, she has listed Tay Sachs disease, Niemann-Pick disease, and Gaucher disease. Now she cannot remember why she listed these diseases together. What do these disorders have in common? a. They are types of lysosomal enzyme disease b. They are X-linked diseases c. Aneuploidy is present d. They have the same signs and symptoms

Tay Sachs disease, Niemann-Pick disease, and Gaucher disease are examples of lysosomal enzyme disorders

A patient presents at the clinic asking for a test to determine whether he is infected with HIV after having unprotected intercourse the night before. What is an appropriate response by the nurse? a. Let''s get you tested, as testing is most accurate in the first 24 hours b. Testing is best within the first 72 hours, so treatment can begin immediately c. The presence of HIV antibodies, the confirmatory test for HIV, takes several weeks after exposure to the virus d. We cannot give a confirmatory test result for HIV because the diagnosis is based on ruling out other diseases

The definitive diagnosis of HIV is the presence of antibodies, and this takes weeks to develop. A confirmatory test for HIV does exist

A patient's laboratory work reveals pneumonia due to infection with Streptococcus pneumoniae. Which type of pneumonia does the patient likely have? a. Community-acquired b. Aspiration c. Hospital-acquired d. Ventilation-acquired

Streptococcus pneumoniae is the most common cause of community acquired pneumonia

A chest X-ray is ordered for a patient with an infectious pulmonary disease. Which of the following may the nurse consider to be possible sources of the infection? Select all that apply a. Streptococcus pneumoniae b. Neisseria meningitidis c. Legionella pnemonphilia d. Bordetella pertussis e. Pneumocystis jiroveci

Streptococcus pneumoniae, legionella pneumophilia, and pneumocystic jiroveci cause pneumonia. Neisseria meningitidis causes meningitis and bordetella pertussis causes whooping cough

A nurse is analyzing an arterial blood gas report of a client with chronic obstructive pulmonary disease and respiratory acidosis. Which compensation mechanism is likely to occur? a. The lungs will excrete carbon dioxide b. The kidneys will excrete bicarbonate c. The lungs will retain carbon dioxide d. The kidneys will retain bicarbonate

The kidneys will compensate for a respiratory disorder by RETAINING BICARBONATE

A patient presents with jaundice. A nurse is reviewing a patient's laboratory values and notes the following: normal aspartate aminotransferase (AST), normal alanine transaminase (ALT), and elevated reticulocyte percentaeg. What is the correct interpretation? a. The liver is failing b. The kidneys are failing c. The rate of hemolysis is increased d. The gallbladder is obstructed

The liver function tests reveal normal values. The elevated reticulocyte percentage indicates increased red blood cells synthesis, so the likely cause of the jaundice is excessive hemolysis

A patient chart shows that neutrophil levels re 600 cells per microliter. Which of the following indicates a proper interpretation of this result? Select all that apply a. The patient's neutrophil levels are normal b. The patient is demonstrating neutropenia c. The patient's medication should be evaluated to determine whether any may cause damage to the bone marrow d. The patient has an over-reactive immune system leading to possible autoimmune disease e. The patient has a form of myelogenous leukemia

The low neutrophil levels indicate neutropenia which can be caused by certain medications

At which level of occlusion do the symptoms of peripheral arterial disease normally first begin to appear? a. Less than 10% b. 10% to 20% c. 70% d. 100%

Symptoms of peripheral artery disease normally begins to appear with approximately 70% occlusion

A young child, age 4, is hospitalized after appearing in the Emergency Department. The child is diagnosed with hemolytic uremic syndrome. What infectious agent does the nurse likely suspect? a. Escherichia coli b. Clostridium difficile c. Bordetella pertussis d. Clostridium botulinum

The most common infectious agent for hemolytic uremic syndrome is escherichia coli (E. coli)

A patient's blood glucose levels are 90 mg/dL after an eight hour fast. Which of the following is the correct assessment of these results? a. The blood glucose levels are normal b. The blood glucose levels indicate prediabetes c. The blood glucose levels diabetes d. The blood glucose levels indicate hypoglycemia

The patient's blood glucose levels are normal

What segment of the nephron does the most reabsorption occur? a. Proximal tubule b. Collecting duct c. Glomerulus d. Loop of Henle e. Distal tubule

The proximal tubule is the location on the nephron where most reabsorption occurs

A mother brings in a child, who has white, fluffy lesions on the tongue and buccal surfaces. The nurse recognized the condition. A test for which type of organism will likely come back positive in this case? a. Bacteria b. Virus c. Fungi d. Helminth

The signs point to thrush, a fungal infection of Candida albicans

A patient is given tissue plasminogen activator. Which of the following will result from this treatment? a. Increased clot formation b. Decreased clotting times c. Increased fibrinolysis d. Decreased platelet adherence

Tissue plasminogen activator breaks down clots, which is known as fibrinolysis. Tissue plasminogen activator does NOT decrease clotting time nor alter platelet adherence

A patient with an ischemic stroke is given tissue plasminogen activator. Which of the following explains the purpose of this treatment? a. Tissue plasminogen activator causes vasodilation b. Tissue plasminogen activator stimulates blood clots to form to decrease blood loss c. Tissue plasminogen activator stimulates fibrinogen to fibrin conversion d. Tissue plasminogen activator helps to dissolve clots

Tissue plasminogen activator participates in the process of firbinolysis, which breaks up clots. Tissue plasminogen activator does NOT affect vessel diameter, stimulate clot formation, nor stimulate fibrin conversion

A nurse needs to assess a patient for venous ulcers. What area should the nurse assess as the most common location for these to appear? a. Axillary area b. Popliteal area c. Ankle area d. Lateral hip area

Venous ulcers most commonly present near the medial melleolus (c)

A nurse is talking to a patient who mentions a pertussis outbreak at her child's school. She says she has never heard of the disease. What is another name for this infection that the nurse may use for which the patient may be more familiar? a. Fifth disease b. Chickenpox c. Whooping cough d. Measles

Whooping cough is a term that refers to pertussis

A nursing student is seeing a patient for the first time. He tells the nursing educator that he has noted some "strange rings" around the patient's eyes. What condition may be present? a. Prader-Willi Syndrome b. Angelman syndrome c. Wilson disease d. Neurofibromatosis

Wilson disease causes a build up of copper that may show as Kayser Fleischer rings around the eyes. Prader Willi syndrome affects the hypothalamus, Angelman syndrome has neurological effects, and neurofibromatosis leads to tumor development, none causing rings around the eyes

A patient would like to know why it is so important to take antibiotics if "strep throat" is present. Which of the following are known developments that may occur if the infection is NOT treated? Select all that apply a. The infection may progress and damage the heart valves b. The infection will progress and cause viral pneumonia c. The infection may progress and damage the kidneys d. The infection may progress and cause scarlet fever e. The infection may progress and cause influenza to develop

(a, c, d) Streptococcus pyogenes infections may cause rhuematic fever, which may damage the heart valves, glomerulonephritis, and scarlet fever. It does NOT cause viral pneumonia or influenza

A nurse is monitoring a patient for Cushing's triad. Which of the following should he note? Select all that apply a. Resting heart rate of 52 bpm b. Blood pressure of 90/40 mm Hg c. Irregular respiratory rate d. Resting heart rate (HR) of 88 bpm e. Blood pressure (BP) of 158/100 mm Hg

(a, c, e) Bradycardia, irregular respiration, and hypertension occurs in Cushing's syndrome

A clinician is educating a patient who was recently diagnosed with chronic obstructive pulmonary disease (COPD). The clinician states that COPD is a collection of disorders. Which of the following contribute to the development of COPD? Select all that apply a. Chronic bronchitis b. Pulmonary edema c. Pulmonary fibrosis d. Hypersensitive airways e. Emphysema

(a, d, e) Chronic obstructive pulmonary disease (COPD) develops from airway hypersensitivity, chronic bronchitis, and emphysema

A young patient age, 20 years, is complaining of severe menstrual pain. Which of the following options may be beneficial? a. Using some over-the-counter pain relievers, such as NSAIDs b. Refraining from using oral contraceptives c. Understanding that this is normal and a sign of good health d. No treatment because, as this is common, pelvic pathologies do NOT need to be assessed

(a) Antiprostaglandin medications, such as NSAIDs, can help with pain associated with menstrual periods. Oral contraceptive use can help with dysmenorrhea, menstrual pain should be managed, and pelvic pathologies need to be ruled out when pelvic or menstrual pain occurs

A male patient is experiencing chronic renal failure and decreased synthesis of erythropoietin. Which of the following related to red blood cells may the nurse expect? a. Hematocrit of 45% b. Low red blood cell number c. Polycythemia d. Vitamin B12 deficiency anemia

(b) Erythropoietin stimulates red blood cell formation. If erythropoietin is lacking, then red blood cell formation will be reduced. Polycythemia is an elevation in red blood cell levels. Gastric problems resulting in reduced intrinsic factor lead to vitamin B12 deficiency anemia

A patient has tested positive for gonorrhea. Which of the following does the nurse assume will be the next step? a. Antibiotic sensitivity testing b. Antiviral prescription c. Instructions to refrain from all sexual activities d. Discussion as to whether or NOT to report the infection to public health

Before treatment begins, sensitivity testing for effective medications should be conducted. Gonorrhea is a bacterial infection that must be reported. Sexual activities can continue with precautionary measures

The following values are being read by a nurse: - Neutrophils 70% - Lymphocytes 20% - Basophils 2% What report is the nurse reading? a. Complete blood count (CBC) b. White blood cell (WBC) total c. WBC differential d. Antibody panel

A WBC differential indicates different WBC types. A WBC total indicates total number of WBCs, NOT individual types. A CBC does include specific WBC types and an antibody panel does NOT indicate different forms of WBCs

A patient has a positive Mantoux test result and suggests that it may be due to a vaccine he received. What is an appropriate response by the nurse? a. A positive Mantoux test indicates active tuberculosis (TB) b. No vaccine for TB is currently in use c. Your vaccine may be causing these results d. You do NOT need any follow up

A positive response on a Mantoux test can be related to being immunized for TB. It does NOT indicate active TB and a follow chest x ray may be required. A vaccine for TB is used in various parts of the world

A nurse sees that hypermagnesemia is evident on a patient's chart. Which of the following does the nurse consider as a possible cause for this finding? a. Alcohol abuse b. Malnutrition c. Prolonged diarrhea d. Renal failure

In renal failure, magnesium may NOT be excreted adequately. Alcohol abuse, malnutrition, and prolonged diarrhea are associated with hypomagnesemia

A nurse is educating a patient about her levothyroxine prescription. Which of the following responses by the patient indicates further education is needed? a. I need to take this medication daily b. I should NOT abruptly stop taking this medication c. I am taking this medication to help my thyroid d. I will eventually be able to stop taking this medication with physician supervision

Levothyroxine is a lifelong prescription. Levothyroxine is taken daily and it replaces inadequate levels of thyroid hormone

A pulse pressure of 40 mm Hg is recorded. Systolic blood pressure equals 130 mm Hg. What is diastolic blood pressure in mm Hg?

90 because systolic blood pressure minus diastolic blood pressure equals pulse pressure (130-90=40 mm Hg)

A nurse is speaking to a patient with nephrotic syndrome. Which recommendations will be helpful for the patient? a. Ingest a high protein diet (3 to 4 g/kg/day) b. Ingest a low-sodium diet (<1500 g/day) c. Encourage overhydration d. Ingest a low-calorie/restrictive diet

A low sodium diet is recommended for patients with nephrotic syndrome

A patient is suffering an attack of diverticulitis. Which of the following should the nurse be prepared for? a. Encouraging the patient to increase dietary fiber intake b. Inserting a nasogastric tube c. Providing any foods the patient requests d. Providing larger-than-normal meals to help with colon motility

A nasogastric tube may be necessary to enable the colon to rest. High fiber foods can aggravate diverticulitis. Correct dietary choices is needed and resting the colon is sometimes needed in severe cases

A medication blocks potassium movement at the cellular membrane in response to voltage changes. Where does the medication most alter cellular response? a. Repolarization b. Depolarization c. Threshold

Altering potassium movement in response to voltage changes would affect the repolarization step

A nurse notes a prior orchiectomy on a patient's chart. Which of the following can the nurse expect? a. The patient's testis failed to descend and had to be surgically corrected b. Upon palpation of the scrotum, the nurse will note the presence of one testicle c. The patient has had a prior testicular torsion that was surgically corrected d. The patient will require testosterone supplementation

An orchiectomy is the removal of a testicle. Testosterone production is normal with one testis. Cryptorchidism is when one or both testes fail to descend

A nursing educator is helping students understand how an ovarian torsion may present in a patient. Which of the following is correct? a. An ovarian torsion may present similarly to an appendicitis b. An ovarian torsion presents with no signs and symptoms c. An ovarian torsion presents with diffuse symptoms, such as nausea d. An ovarian torsion normally presents with flank pain

An ovarian torsion may present with signs and symptoms similar to appendicitis such as pain in the abdminopelvic region and distress

A man underwent a vasectomy yesterday. Which of the follow up strategies is correct? a. As long as no bruising or swelling is apparent, no follow up is needed b. After 72 hours, a backup birth control method will be unncessary c. A semen sample, showing no sperm, must be provided after healing to make certain the vasectomy was successful d. Daily semen samples will be necessary for one to two weeks postsurgery

Analysis of semen is necessary to determine the success of the vasectomy. Daily semen samples are NOT needed

A patient is given an angiotensin receptor blocker. As the nurse with the discharge instructions, which of the following would you tell the patient is the effect of this medication? a. The medication increases heart rate b. The medication decreases pulmonary congestion c. The medication decreases the constriction and resistance of your blood vessels d. The medication increases urine output

Angiotensin receptor blockers help to prevent vasoconstriction, lowering blood pressure. Angiotensin receptor blockers do NOT increase heart rate nor alter pulmonary congestion levels. Diuretics increase urine output

What is the common usage for an Anion gap? a. Assessment of edema b. Assessment of dehydration c. Assessment of acidosis d. Assessment of alkalosis

Anion gap is used to differentiate different forms of acidosis

Parents of a 21 year old female, diagnosed with anorexia nervosa are speaking to the nurse. Which statements by the parents are helpful and correct? a. "This disease should be easy to treat. She just needs to eat more" b. "She will likely outgrow this phase" c. "We know recovery may take years and may never be finished" d. "We are worried about her overall health. We know anorexia nervosa can affect the entire body " e. "She is an adult now. She needs to be able to handle her own diet "

Anorexia is NOT easily treated, recovery may take years, and it may affect all systems of the body

A nursing student learning about pulmonary embolism would expect which of the following to be the most common presentation of a patient with this disorder? a. Immediate cyanosis b. Patient in severe distress c. Dry, hacking cough d. Few signs and symptoms apparent

Typically, a pulmonary embolism may present with few signs and symptoms

A patient diagnosed with leukemia is prescribed a tyrosine kinase inhibitor. What is the purpose of this medication? a. Decrease pain b. Prevent cachexia c. Decrease unregulated cell growth d. Decrease cell destruction due to chemotherapy

Tyrosine kinase activity increases cell proliferation, so a tyrosine kinase inhibitor would decrease unregulated cell growth

A patient's laboratory values revealed the following: CD4 values of 800 cells per microliter with CD8 cells of 400 cells per microliter. What is the correct interpretation? a. The patient has HIV b. The patient has AIDS c. The patient has leukemia d. The patient has normal laboratory values

The CD4 number and the fact CD4:CD8 ratio is 2:1 is normal. CD4 cells less than 200 per microliter would indicate AIDS

A patient with asthma completes a spirometry test and is then given a bronchodilator and a second spirometry test. What is the expected result for the second spirometry test? a. Increased forced expiratory volume (FEV) 1.0 b. Decreased FEV 1.0 c. Decreased forced vital capacity (FVC) d. Increased energy recovery ventilation (ERV)

(a) Bronchodilators, if effective for a patient with asthma, will help increase the amount of air the patient can exhale in the first second. Energy recovery ventilation (ERV) indicates the air NOT exhaled. Bronchodilation will result in more air being exhaled, lowering ERV

A client is scheduled for a heart valve replacement. Which strategic action of the primary healthcare provider enhances the client's adaptive ability and coping mechanism to reduce stress? a. Administration of hypertonic potassium chloride IV bolus b. Administration of diluted IV potassium c. Administration of 20 mEq of potassium chloride d. Slow administration of potassium supplements

Administration of hypertonic potassium chloride IV bolus

A nursing student has been studying nucleotides for an upcoming exam. Which of the statements indicate she may have made an error in her note taking? a. The nucelotide in DNA are indicated by A, T, C, G b. In both DNA and RNA, adenine always binds with cytosine c. Uracil replaces cytosine in RNA d. Cytosine and guanine are paired together in DNA e. The nucelotides in RNA are indicated by A, U, C, G

Adnine binds with thymine in DNA and uracil in RNA and in RNA, uracil replaces thymine (b, c)

Anorexia nervosa compromises many bodily systems. Which of the following laboratory values may a nurse expect to observe on the chart of a patient suffering from severe anorexia nervosa? Select all that apply a. Decreased blood urea nitrogen (BUN) b. Decreased albumin c. Increased estrogen d. Increased blood glucose e. Decreased red blood cells

Albumin levels are decreased because of reduced protein synthesis and RBC synthesis is compromised due to malnutrition. Severe anorexia nervosa also decreased blood glucose and estrogen levels and elevates BUN due to dehydration

A nurse is educating a patient about pancreatic function. Which of the following are associated with the exocrine function of the pancreas? Select all that apply a. Insulin secretion b. Amylase secretion c. Bicarbonate secretion d. Glucagon secretion e. Lipase secretion

Amylase, bicarbonate, and lipase are secreted into a duct leading to the small intestines (b, c, e) Insulin and glucagon secretion are endocrine functions

A nurse expects a patient with liver failure to have which of the following laboratory values to be increased? Select all that apply a. Aspratate aminotransferase (AST) b. Alanine transaminase (ALT) c. Prothrombin time (PT) d. Serum albumin e. Alkaline phosphatase

Aspratate aminotransferase (AST), Alanine transaminase (ALT), Prothrombin time (PT) and Alkaline phosphatase levels increase with liver damage. Liver failure will compromise the liver's ability to synthesize albumin (a, b, c, e)

A nurse is working with a patient with DiGeorge syndrome. The nurse needs to explain to the family the tissue or organ that is the source of dysfunction in this disease. Which of the following would be the correct explanation for the nurse to provide? a. Heart b. Thymus c. Thyroid d. Bone marrow

DiGeorge syndrome arises from a problem in thymus development

Which of the following diseases is MOST commonly associated with metaplasia ? a. Jaundice b. Gastroesophageal reflux disease c. Peptic ulcer d. Rheumatoid arthritis

Gastroesophageal reflux diseases occurs when gastric contents enter the esophagus. Over time, the esophageal cells undergo metaplastic changes as an adaptation to the gastric acid

When would osmoreceptors most likely signal for the release of antidiuretic hormone (ADH) ? a. Patient with edema due to congestive heart failure b. Plasma sodium level of 130 mEq/L c. Patient with compulsive drinking behaviors d. Plasma osmolarity of 305 mOsm

Plasma osmolarity of 205 mOsm (elevated) would signal the release of ADH to retain fluid to dilute the plasma concentration. Edema indicates fluid volume overload, ADH would be signaled when there is a fluid volume deficit

A 48 year old male has elevated prostate-specific antigen (PSA) levels. What is an appropriate interpretation by the clinician? a. The patient has testicular cancer b. The patient has prostate cancer c. PSA values are variable. Further evaluation is necessary d. The patient has erectile dysfunction

Prostate specific antigen (PSA) values vary between individuals for a variety of reasons. Further follow up is needed. PSA is NOT a marker for testicular cancer or erectile dysfunction. Although elevated PSA is associated with prostate cancer, it alone is NOT diagnostic for this cancer (c)

A nurse is creating a patient educational brochure to identify symptoms of benign prostatic hyperplasia (BPH). Which of the following should she include on her brochure? Select all that apply a. Severe back pain b. Yellow discharge from the penis c. Hesitancy during urination d. Straining to urinate e. Blood in urine

Prostatic hyperplasia (BPH) can block urine flow. BPH does NOT cause severe back pain or blood in the urine. Yellow discharge is a sign of infection (c. d)

A nurse is told to evaluate Ranson criteria upon a patient's admission. Which condition does the patient have? a. Gallstones b. Appendicitis c. Pancreatitis d. Peritonitis

Ranson criteria are used for pancreatitis

A nursing student is directed to assess a patient's acute pancreatitis severity. What scale should the nursing student use? a. Acute physiology, age, chronic health evaluation (APACHE) b. Glasgow Coma scale c. Ranson criteria d. Duke criteria

Ranson criteria is used to assess pancreatitis severity

A laboratory technician is looking at a lymph node biopsy sample. The appearance of which cell type would help to diagnose Hodgkin's lymphoma? a. Apoptoic cells b. Sickle cells c. Reed-Sternberg cells d. B cells

Reed Sternberg cells may appear in Hodgkins lymphoma. Apoptopic cells represent cells undergoing programmed cell death, which is a normal process. B cells are normal cells and would NOT be considered diagnostic. Sickle cells refer to red blood cells, NOT white blood cells

A nurse learns that she will be caring for a patient with a primary spontaneous pneumothorax. Which of the following patients will the nurse most likely will be caring for? a. A male, greater than 75 years of age b. Postmenopausal woman c. A male, age 18 years d. Female, age 10 years

Teenage male are those most likely to suffere from spontaneous pneumothorax

A nurse is explaining to a patient who is concerned her aging facial skin is showing too many wrinkles that the treatment available involves a toxin from bacterial organisms. Which bacteria produces this toxin? a. Clostridium botulinum b. Clostridium tetani c. Clostridium perfringes d. Clostridium difficle

The botulism toxin causes muscle paralysis, which may reduce the appearance of wrinkling of the skin. The toxin is released by clostridium botulinum

A patient is to receive a prescription for an NSAID medication. Which choices are available? a. Celecoxib b. Aspirin c. Methadone d. Nalcoxone e. Selective serotonin-reuptake inhibitors (SSRIs)

Calecoxib and aspirin are NSAID medications. Methadone helps with opioid withdrawal symptoms, Naloxone is for opioid overdose, and SSRIs are used to treat depression by keeping serotonin levels elevated

For a confirmative diagnosis of Goodpasture's disease, the nurse anticipates which of the following? a. Glomerular filtration rate (GFR) measurement decreses b. Positive immunoglobulin results c. Intravenous pyelogram (IVP) shows obstruction d. Urinalysis shows the presence of casts

Goodpasture's disease is an autoimmune disease diagnosed by the presence of antibodies (b)

A nurse is reviewing a patient's blood values and vital signs. Which should be evaluated for cardioprotective effects? a. Triglycerides b. Low-density lipoproteins (LDL) c. High-density lipoproteins (HDL) d. Blood pressure (BP)

High density lipoproteins have a cardioprotective effect. Increased triglycerides, LDL levels, and BP increases the risk for heart disease

Please place the following sequences in order as to how exercise can result in increased cardiac output - Increased compression of the veins - Increased preload - Increased stroke volume - Increased venous return - Increased skeletal muscle contraction

Increased skeletal muscle contraction causes compression of veins. This increases venous return, which increases preload and stroke volume ( Increased skeletal muscle contraction - Increased compression of the veins - Increased venous return - Increased preload - Increased stroke volume)

A patient with suspected appendicitis is found lying on his side with his knees drawn up in the fetal position. What is the appropriate response by the nurse to the patient? a. Please lie down straight on your back b. You must be doing better. If you had appendicitis, you would NOT be able to lie in this position c. This position is likely helping you feel less pain because it lessens the tension on your abdominal wall d. This position is dangerous for you to be in now because it cuts off the blood supply to vital organs

Lying in the fetal position can help alleviate tension on the abdominal wall. Lying supine will likely increase the pain (c)

Which of the following terms indicate the most severe coronary event? a. Partial coronary artery occlusion b. Myocardial infarction c. Myocardial ischemia d. Myocardial apoptosis

Myocardial infarction is the death of myocardial cells due to ischemia. A partial coronary artery occlusion, depending on the extent, may or may NOT lead to myocardial ischemia. Ischemia is reduced oxygen delivery to the tissue, if prolonged, it may lead to necrosis. Apoptosis is normal, programmed cell death

A patient, with no underlying conditions, presents in the emergency department hyperventilating because of a panic attack. Which of the following values would a clinician expect for arterial blood gases? a. pH less than 7.35 b. Oxygen saturation of 85% c. Partial pressure of carbon dioxide less than 35 mm Hg d. Partial pressure of oxygen less than 90 mm Hg

Normal partial pressure of carbon dioxide is 35 to 45 mm Hg. Hyperventilation reduces partial pressure of carbon dioxide, leading to alkalosis. A pH of less than 7.35 indicates acidosis

A nursing student is reviewing the "polys" associated with diabetes mellitus (DM). Which of the following should she include in her list? Select all that apply a. Polyphagia b. Polycythemia c. Polycystic d. Polyuria e. Polydipsia

Polyphagia which is increased eating, polyuria which is increased urination, and polydipsia which is increased drinking are occur with diabetes mellitus. Polycythemia, an elevation in red blood cells, and polycystic do NOT occur in DM

A nurse is speaking to a patient with premenstrual dysphoric disorder (PMDD). Which of the following shows correct understanding by the patient? a. PMDD and premenstrual syndrome (PMS) are basically the same thing b. PMDD is less severe than PMS c. PMDD can interfere with work and social life d. PMDD is NOT something to be concerned about

Premenstrual dysphoric disorder (PMDD) can impact a woman's work and social life due to its severity. PMDD is a more severe form of PMS and it can impact a woman's life and should be addressed

A nurse needs to record the stroke volume (SV) for a patient. Which of the following units would be the proper one for stroke volume? a. Beats per minute b. Percentage c. Liters per minute d. Milliliters per beat

Stroke volume is measured in milliliters per beat

A nurse is reviewing a patient's medications. Which of the following would be ineffective for the patient's hypertension a. Angiotensin-converting enzyme (ACE) inhibitors b. Diuretics c. Statins d. Angiotensin receptor blockers

Statins work to lower blood lipid levels. ACE, diuretics, and angiotensin receptor blockers work to lower blood pressure

A young male patient is admitted in severe pain suffering from a vaso-occlusive crisis. Which condition likely underlies this development? a. Iron-deficiency anemia b. Sickle-cell anemia c. Aplastic anemia d. Prenicious anemia

Vaso occlusive crisis is a common development in sickle cell anemia. Iron deficiency does present with vaso occlusive, aplastic anemia does NOT. Pernicious anemia presents with neurological symptoms, NOT vaso occlusive crisis

On a news broadcast, a cholera outbreak is reported in a refugee camp. What sign or symptom should responding healthcare workers to expect patients to have? a. Severe pulmonary congestion b. Hives and angioedema c. Rice water stools d. Cardiac tamponade

Cholera affects intestinal transport, resulting in very watery diarrhea (c)

A nursing student is practicing a presentation on pain management for her fellow students. Which of the following statements may her fellow students suggest she change? a. Corticosteroids and NSAIDs work on the same enzyme b. Most NSAIDs block the cyclooxygenase-1 (COX-1) and cyclooxygenase-2 (COX-2) pathways c. NSAIDs are in the opioid classification, and thus they are used sparingly d. COX-1 pathway produces prostaglandins that cause pain, edema, and fever e. Selective cyclooxygenase inhibitors may help protect stomach mucosa

Corticosteroids work on the phospholipase enzyme, while NSAIDs work on the COX enzymes and NSAIDs are NOT in the opioid classification and NSAIDs are used frequently

A patient is suffering from plyoric obstruction. What does the nurse expect to observe? a. Projectile vomiting b. Flattened stomach with no distension c. Severe diarrhea d. Inhibition of pain signals

(a) If the pyloric valve is obstructed, then the stomach cannot empty. The stomach would be distended. Severe diarrhea is unlikely because materials cannot exit the stomach. Because stomach emptying is prevented, the patient will have pain

A nurse is told that her patient has severe leukopenia. Which of the following can the nurse expect? a. Increased infection risk b. Significant elevation in all white blood cells (WBC) values c. Neutrophilia d. Normal complete blood count (CBC) values

(a) Decreased WBC numbers can increase infection risk. Leukopenia is a ddecrease in WBC number, neutrophilia is an increase in neutrophil number, and a CBC will NOT be normal with a low WBC value

A patient wants to improve her blood lipid profile and elevate her high-density lipoprotein (HDL) levels. Which of the following may meet the patient's goals? a. Regular excerise b. Medications, which are the only known way to raise HDL levels c. Restricting all fats from the diet d. Liver supplements

(a) Exercise has been shown to increase HDL levels. Fat is needed in the diet and liver supplements will NOT alter HDL levels

A heart murmur is described as holosystolic. What is the nurse prepared to hear? a. Murmur consistent throughout systole b. Murmur occurring between S2 and S1 c. Murmur that ebbs and flows during systole d. Murmur with increasing loudness

(a) Holosystolic means that the murmur is heard throughout systole. S2 to S1 indicates diastole. Crescendo indicates a murmur with increasing loudness

A patient is being assess for a pneumothorax. Which of the following may be part of the clinical presentation? a. Patient insists on using inhaler b. Patient presents with clubbing of the fingertips c. Patient presents with obvious asymmetry to the chest wall d. Patient coughs up gray-colored sputum

A pneumothorax may present with asymmetry. Use of an inhaler would NOT be indicative of pneumothorax. Clubbing of the fingers is sign of prolonged hypoxia and gray sputum is a sign of pneumoconiosis

A nursing student is assessing a patient with a Type IV hypersensitivity reaction. The nursing student knows that which of the following applies to these reactions? a. Is antibody-mediated b. Manifests as a sign of the innate immune system c. Develops due to a cytokine storm d. Is a T cell-mediated reaction

A Type IV hypersensitivity, also known as a delayed hypersensitivity, is produced by T cells. T cells do not produce antibodies, they are a part of the acquired immune system and cytokine storms are rare responses to an antigen

A patient is suspected to have a pulmonary embolism. Which would be the best diagnostic test to schedule? a. Chest x-ray b. Echocardiogram c. Ventilation-perfusion ratio d. Spirometry

A pulmonary embolism blocks blood flow, changing the ventilation perfusion ratio. A chest x ray may show no changes with pulmonary embolism, echocardiogram evaluates heart function, and spirometry measures lung volumes and capacities

Which of the following complications of bariatric surgery may result in hypotension? a. Folic acid deficiency b. Herniation c. Dumping syndrome d. Vitamin B12 deficiency

As contents move the gastrointestinal (GI) tract very rapidly, excess fluid may be lost, resulting in reduced blood volume and blood pressure

A nurse is working with a patient who finds her incontinence to be very embarrassing. She is wondering if any medications are available that could help her. What class of medications does the clinician consider? a. Angiotensin receptor blockers b. Corticosteroids c. Anticholinergic d. Antileukotrienes

Anticholinergics block parasympathetic signals, decreasing, for some patients. Angiotensin receptor blockers dilate vessels, corticosteroids reduce inflammation, and antileukotrienes are asthma treatment

The urinalysis results of a patient in the end stage of chronic renal failure is likely to show which of the following? a. Presence of white blood cells (WBCs), red blood cells (RBCs), and protein b. Elevated leukocyte esterase c. Elevated ketones d. Elevated nitrite

As glomerular damage increases, substances NOT normally passing the filter of the glomerulus will be able to pass through and appear in the urine. Elevated leukocyte esterase and nitrite levels occur with urinary tract infections. Ketones may appear in the urine when a patient has diabetes

Which of the following individuals should NOT handle 5-alpha-reductase inhibitor medications? a. Elderly males over age 70 years b. Men suffering from testicular cancer c. Pregnant women d. Prepubescent females

Because of the action on testosterone metabolism, pregnant women should NOT handle 5-alpha-reductase inhibitor medications

A male patient, age 72 years, presents with complaints of urinary outflow problems. Which of the following does the clinician consider to be the most likely cause? a. Urethral blockage b. Benign prostatic hyperplasia (BPH) c. Urethral deviation d. Kidney stone

Benign prostatic hyperplasia (BPH) can block urine outflow and is the most common cause of urinary outflow problems in elderly men

A nursing educator is lecturing on transient ischemic attacks (TIAs). When reviewing with students, which statement indicates correct understanding? a. TIAs are minor strokes that are NOT a major concern medically b. TIAs may be more noticeable by bystanders, rather than the patient c. TIAs result from microhemorrhages in the brain d. TIAs is another name for a stoke

Bystanders may notice changes brought on by a transient ischemic attacks (TIAs), which are overlooked by the person suffering the TIA. TIAs should be evaluated and managed. They are ischemic attacks and are NOT synonymous with strokes

A patient has problems swallowing after a stroke. Which of the following cranial nerves (CNs) may have been affected? a. CN III b. CN VI c. CN IX d. CN X

CN IX is the glossopharyngeal nerve. Disruption of the nerve can affect swallowing

A patient has suffered a stroke and is experiencing facial droop. What cranial nerve (CN) does the nurse expect to be affected? a. CN III b. CN IV c. CN VII d. CN IX

CN VII innervates the facial muscles

Karen arrives at the clinic seeking help for her mother, who is suffering debilitating pain from cancer. Karen is wondering why this pain is happening. Which of the following would be an appropriate response from the nurse? a. "I am surprised, as cancer pain is uncommon" b. "Your mother should probably be heavily sedated" c. "Destruction of tissue by growing cancer cells can be quite painful. Let's see what we can do to manage the pain" d. "Thankfully, we know the pain your mother is suffering is from her cancer and NOT the treatment for this cancer"

Destruction of tissue by growing cancer cells can be quite painful. Let's see what we can do to manage the pain

Digitalis is known as a positive inotropic and negative chronotropic medication. How would the nurse interpret this information? a. The medication stimulates an increase in heart rate and a decrease in contractility b. The medication decreases both heart rate and contractility c. The medication decreases heart rate and increases contractility d. The medication increases both heart rate and contractility

Digitalis increases heart contractility (inotropic) while decreasing heart rate (chronotropic)

A nursing student is present at the delivery of a child. Which of the following conditions may present with noticeable physical abnormalities at birth? a. Ehlers-Danlos syndrome b. Gaucher syndrome c. Down syndrome d. G6PD deficiency

Down syndrome causes facial abnormalities that are present at birth. Genetic testing is required for diagnosis

A patient is listing complaints such as a "burning sensation" in the abdominal area. He states that the pain is worse when he has NOT eaten for some time. The pain is somewhat alleviated with eating. Which of the following does the nurse most likely suspect? a. Gastroesophageal reflux disease b. Peptic ulcer disease c. Zollinger-Ellison syndrome d. Hiatal hernia

Due to acid levels in the stomach, peptic ulcer disease often presents with pain between meals. Gastroesophageal pain often presents after meals. Although Zollinger Ellison syndrome may present with similar signs, the condition is very rare. Hiatal hernia does NOT present with pain between meals

In the emergency department, a patient arrives complaining of dizziness, sweating, and diarrhea after he eats. HE is wondering if he has food poisoning. THe nurse sees on his chart that he has had bariatric surgery. What does the nurse suspect? a. Gastroenteritis b. Dumping syndrome c. Volvulus d. Esophageal reflux

Dumping syndrome develops after bariatric surgery when gastrointestinal contents pass through rapidly through the intestines. This can result in dizziness and diarrhea. The patient's symptoms occur after eating, so dumping syndrome is more likely than gastroenteritis. A volvulus is a twist in the intestines and esophageal reflux presents as heartburn

A nursing educator is working with students. Which of the following shows correct understanding of myocardial infarction (MI) diagnosis? a. Immediate electrocardiogram (ECG) is confirmatory b. Myocardial infarction diagnosis is based on patient self-report c. Chest x-ray and echocardiogram are diagnostic for MI d. ECG can be indicative of myocardial infarction, but ECG alone can NOT confirm diagnosis

ECG can provide signs of MI, but cannot confirm an MI. Elevation of cardiac markers, such as troponin, are diagnostic for MI

A nursing educator is determining whether a student nurse can assess the difference between hypovolemic and hypervolemic hypernatremia. Which answer indicates that the student has the correct understanding? a. Hypervolemic hypernatremia may be accompanied by edema b. Hypervolemic hypernatremia results in hypotension c. Hypervolemic and hypervolemic hypernatremia present in similar manner. The key is the hypernatremia d. Hypervolemic hypernatremia is often noted by apparent weight gain

Excess fluid retention due to elevated sodium increases the risk for edema. The excess fluid in hypervolemic hypernatremia causes increased blood pressure (NOT hypotension), weight loss, and is different in every patient

A patient believes she may be experiencing a pulmonary embolism. What is one indicator that she is likely NOT experiencing a pulmonary embolism? a. Elevated fibrin degradation products b. Normal D-dimer values c. Normal chest x-ray d. Normal vital capacity measurement with spirometry

Fibrin forms clots, and when fibrin breakdown products increase, it can be a sign of clot breakdown and formation. Pulmonary embolism patients may present with normal chest x rays and spirometry results. D dimer levels increase with clot formation and breakdown

For the most accurate results of a fecal occult blood test (FOBT), what will the nurse recommend to the patient? a. The test should be conducted two times on the same day, if possible b. The test should be conducted two times within 48 hours c. The test should be conducted three times on three different occasions d. The test should be conducted one time per month for four months

For best results, a fecal occult blood test (FOBT) should be done on three different days

A nurse reminds a patient of the acronym of FAST regarding stroke presentation. Which of the following lists shows that the patient has a correct understanding of the acronym? a. Fear, Anxiety, Stress, Tremors b. Facial droop, Arm weakness, Speech difficulties, Time c. Facial droop, Ataxia, Speech difficulties, Time d. Facial droop, Arm weakness, Sensations, Time

Facial droop, Arm weakness, Speech difficulties, Time are used to remind individuals of signs and symptoms of stroke and to seek help quickly

A nursing student suspects that a patient has a lower urinary tract infection (UTI). The nursing educator, however, believes that the patient may have pyelonephritis. Which of the following signs or symptoms did the nursing educator notice that may indicate pyelonephritis ? a. Presence of leukocyte esterase in the urinalysis results b. Level of pain the patient is self-reporting c. Presence of fever d. Patient's past medical history

Fever may be present in pyelonephritis but is often NOT present in urinary tract infections. Elevated leukocyte esterase and pain are present in both UTI and pyelonephritis. The patient's past medical history will NOT differentiate the condition

In blood typing and crossmatching, which of the following molecular type serves as antigens? a. Phospholipids b. Glycoproteins c. Steroids d. Cholesterol

Glycoproteins are on the cell membrane and serve as cell surface markers, known as antigens. On red blood cells, these antigens are used to identify blood types. Phospholipids, steroids, and cholesterol are part of the cell membrane but do NOT serve as antigens for blood typing

A nurse is reviewing laboratory values, which of the following is typically present on arterial blood gas values? a. H2CO3 b. Hct c. HCO3- d. Hb

HCO3- is the symbol for bicarbonate and it is typically reported for arterial blood gases. H2CO3 is the symbol for carbonic acid, Hct is for hematocrit, and Hb is for hemoglobin, all of which are NOT reported on arterial blood gas values

A nursing student is reviewing liver function. What is the correct sequence in the process of conjugation of bilirubin? - Biliverdin converted to unconjugated bilirubin - Porphyrin converted biliverdin - Hemoglobin broken down to heme - Unconjugated bilirubin conjugated by liver - Heme converted to porphyrin

Hemoglobin is broken down to heme, which is converted to porphyrin, which is converted to biliverdin. Biliverdin is converted to unconjugated bilirubin, which is converted to conjugated bilirubin (Hemoglobin broken down to heme - Heme converted to porphyrin - Porphyrin converted biliverdin - Biliverdin converted to unconjugated bilirubin - Unconjugated bilirubin conjugated by liver)

Parents have just been told that their child has hemophilia. Which of the following information would be correct for the nurse to share? a. Your child has a type of hemophilia that can be outgrown b. In your child, platelets do NOT work correctly. A bone marrow transplant may help c. Your child will likely suffer what is known as "vaso occlusive" crisis d. Your child has an increased risk for bleeding, so care must be taken with even small wounds

Hemophilia disrupts the clotting cascade, increasing the risk for excessive bleeding. Hemophilia is a lifelong disorder. It is NOT a platelet disorder and vaso occlusive crisis occurs in sickle cell anemia

A nurse educator is outlining the progression of a hepatitis B infection. Please place the following steps in the correct order - Inflammatory reaction of the hepatocytes - Reduction in hepatitis B virus (HBV) viral load to immune response - Inoculation of HBV - No signs or symptoms, although virus can be passed to others - No viral levels detected, high level of HBV antibodies

Hepatitis B infections is contracted. Initially, there are no signs and symptoms. Then inflammation occurs in the hepatocytes. As the immune response mounts, the HBV viral load is reduced. Eventually the viral load is undetectable and there is a high level of antibodies ( Inoculation of HBV - No signs or symptoms, although virus can be passed to others - Inflammatory reaction of the hepatocytes - Reduction in hepatitis B virus (HBV) viral load to immune response - No viral levels detected, high level of HBV antibodies )

A nurse needs to assess a patient for Homan's sign. Which of the following should be performed? a. Ask the patient to run in place for 30 seconds b. Ask the patient to stand with arms extended and eyes closed c. Ask the patient to assume a supine position while the nurse dorsiflexes the foot d. Ask the patient to assume a prone position, while the nurse plantar flexes foot

Homan's sign is assessed by having a patient lie supine while the clinician dorsiflexes the foot. If pain is elicited, then a blood clot may be present

Although many genetic disorders appear in childhood, which of the following genetic conditions are MORE likely to manifest in adulthood? Select all that apply a. Huntington's disease b. G6PD deficiency c. Type 1 Gaucher disease d. Tay Sachs disease e. Cystic fibrosis

Huntington's disease and type 1 Guacher disease are adult onset autosomal dominant conditions. Cystic fibrosis is present at birth, G6PD deficiency is identified with newborn screening and Tay Sachs disease occurs before the age of 3, resulting in death

A client has an abnormal thickening of the lining of the uterus due to an increase in estrogen levels. How can such a condition be reversed? a. Restoration of blood circulation b. Hormone therapy to counteract the effects of excessive estrogen c. Acid suppression treatment d. Surgical removal

Hyperplasia of the uterine endometrium is caused by an overproduction of estrogen. Hormone therapy to counter the effects of excessive estrogen helps reverse the condition

A patient states the following: "I am feeling fine, so I stopped taking my blood pressure medications". What is an appropriate response? a. We don't recommended you abruptly stop your hypertension medications; however; if you are feeling better, that is fine b. Because hypertension has no long term risks, it is fine if you stop taking medication for it c. Most people cycle on and off hypertension medications, therefore, you will likely begin taking them again d. Hypertension can cause "silent damage" to your tissues. It is important to manage hypertension, even if you feel fine

Hypertension may NOT present with signs and symptoms noticed by the patient but can still cause damage. Patients should NOT stop medications on their own. Hypertension do NOT cycle on and off blood pressure medication and it has long term risks and must be managed

Please place the steps in order which demonstrates how chronic obstructive pulmonary disease (COPD) can lead to secondary polycythemia - Decreased oxygen deliver to kidneys - Decreased blood oxygen levels - Bone marrow stimulation - Pulmonary dysfunction - Increased erythropoietin levels

In COPD, prolonged hypoxemia stimulates erythropoietin release from the kidneys. This in turn stimulates the bone marrow to produce more red blood cells ( Pulmonary dysfunction - Decreased blood oxygen levels - Decreased oxygen deliver to kidneys - Increased erythropoietin levels - Bone marrow stimulation)

A nursing student asks whether there is a difference between Cushing's disease versus Cushing's syndrome. Which laboratory value can help differentiate Cushing's disease from Cushing's syndrome? a. Elevated adrenocorticotropic hormone (ACTH) b. Elevated cortisol c. Decreased cortisol d. Elevated epinephrine

In Cushing's disease, the problem is in the anterior pituitary, so adrenocorticotropic hormone (ACTH) levels are elevated. Both disorders cause elevated cortisol levels and do NOT affect epinephrine levels

A patient is severely obese, and the nurse is talking to her about her "heartburn". Please place the following in the proper sequence of how the patient's weight may play a role in her heartburn and also lead to changes in esophageal cells - Poor closure of lower esophageal sphincter (LES) - Burning sensation in sternal area _ Metaplasia of esophageal cells in response to stomach acid - Stomach acid enter esophagus - Increased body weight

Increased body weight can increase the likelihood of poor lower esophageal sphincter (LES) closure, which enables stomach acid to enter the esophagus. This can cause a burning sensation in the sternal area (heartburn). The acid from the stomach can cause metaplastic changes of esophageal cells ( Increased body weight - Poor closure of lower esophageal sphincter (LES) - Stomach acid enter esophagus - Burning sensation in sternal area - Metaplasia of esophageal cells in response to stomach acid )

A nurse is monitoring an eldery bedridden patient. The nurse is concerned about the pulmonary functioning of the patient. Which of the following actions by the nurse may help the patient's pulmonary functioning? a. Help place the patient in a more upright, seated position b. Encourage rapid shallow-breathing patterns c. Have the patient lie flat with head slightly lowered d. NOT drawing attention to the patient's breathing difficulties, as it makes them worse

Individuals exert more energy to breathe in the supine position. Placing a patient in a more upright position helps. Rapid shallow breathing patterns and lying flat will increase the work of breathing

A nurse is working with a patient who has had type 2 diabetes mellitus (DM) for several years. The nurse will need to keep in mind which of the following psychological aspects? a. Patients should be encouraged to refrain from getting emotional about having DM b. So that patients with DM do NOT become discouraged, long-term complications of the disease should NOT be discussed c. Schizophrenia may develop in those with DM for 10 or more years d. People with DM are twice as likely to suffer depression

Individuals suffering from diabetes mellitus are twice as likely to suffer from depression. Emotional aspects are part of any disease, educating patients about risks and complications are necessary, and DM does NO increase the risk of schizophrenia

Which of the following would lead a nurse to believe that a patient has right heart failure rather than left heart failure? a. Increased jugular vein distention b. Pulmonary edema c. Left ventricular hypertrophy d. Increased pulmonary capillary wedge pressure

Jugular vein distention is associated with right side heart failure. Left side heart failure causes fluid to back up into the lungs, ventricular hypertrophy, and blood to back up into the pulmonary capillaries

A patient states to a nurse that he is experiencing changes in the skin and tissue in the location he used for his insulin injection. What is an appropriate response by the nurse? a. Ask the patient if he is rotating injection sites b. Suggest that the patient is having an allergic reaction c. Suggest that the patient switch insulin types d. Ask the patient to refrain from insulin injections until the site heals

Insulin injection sites should be rotated. The patient cannot refrain from insulin injections (a)

A nursing student states that jaundice means that the liver is failing. Which of the following response by the nurse educator is correct? a. The student is correct. Jaundice indicates liver failure b. The student is incorrect. Jaundice is caused by renal failure c. The student is incorrect. Gallbladder issues are the only cause of jaundice d. The student is incorrect. Jaundice can result from several factors including excessive red blood cells (RBC) breakdown and liver problems

Jaundice has several causes, including liver problems, excessive hemolysis, and biliary duct problems

A patient with left ventricular failure is completing a cardiac assessment. Which of the following results would be concerning to a clinician? a. Left ventricular ejection fraction (LVEF) of 35% b. Resting cardiac output of 5.2 liters per minute c. Low to normal central venous pressure d. Blood pressure (BP) of 130/82 mm Hg

Left ventricular ejection fraction (LVEF) should be approximately 55 to 60%. Cardiac output of 5.2 liters per minute is normal, low to normal central pressure shows fluid is NOT backing up and blood pressure of 130/82 mm Hg is normal

Which of the following white blood cell (WBC) levels would indicate that a patient has leukocytosis? a. 1200 cells per microliter b. 1800 cells per microliter c. 5000 cells per microliter d. 12,000 cells per microliter

Leukocytosis is elevated WBC count above 11,000 cells per microliter. 5000 is normal, 1800 and 1200 cells per microliter indicate leukopenia

A nurse is speaking to a patient about the patient's increased risk for clot formation. Which of the following patient characteristics would contribute to increased risk clot formation? Select all that apply a. Thrombocytopenia b. Oral contraceptive usage c. Daily aspirin usage d. Smoking e. Atrial fibrillation

Oral contraceptive, smoking, and atrial fibrillation increases the risk of clot formation. Thrombocytopenia is low platelets and increases the risk of bleeding. Aspirin has been shown to decrease adherence of platelets

Which arterial blood gas values are indicative of an adult respiratory distress syndrome? a. Partial pressure of oxygen 70 mm Hg and partial pressure of carbon dioxide 35 mm Hg b. Partial pressure of oxygen 50 mm Hg and partial pressure of carbon dioxide 50 mm Hg c. Partial pressure of oxygen 90 mm Hg and partial pressure of carbon dioxide 45 mm Hg d. Partial pressure of oxygen 100 mm Hg and partial pressure of carbon dioxide 50 mm Hg

Partial pressure of oxygen 50 mm Hg and partial pressure of carbon dioxide 50 mm Hg indicates that oxygen is falling below normal and carbon dioxide is elevating above normal, showing that the patient cannot inhale adequate oxygen nor exhale adequate carbon dioxide

A nurse is educating a patient about the stress response because the patient wants to know why the stress response is so strong. The nurse responds that several systems are the initial primary responders to stress. Which systems was the nurse speaking about? Select all that apply a. Gastrointestinal tract b. Nervous c. Endocrine d. Cardiac e. Immune f. Respiratory

Primary responders to a stress event include the nervous system, endocrine system, and immune system. The gastrointestinal response, cardiac system, and respiratory response are controlled by the nervous and endocrine systems (b, c, e)

A clinician is discussing that a patient has Prinzmetal angina. What is the defining characteristic of this type of angina? a. Early diagnosed b. Specific electrocardiogram (ECG) pattern c. Develops as the result of coronary artery vasospasm d. Most common type of angina

Prinzmetal angina is the result of coronary artery vasospasm. It is a rare form of angina that is difficult to diagnose because signs and symptoms can appear intermittently. It does NOT appear with a specific, identifiable ECG change

A clinician is palpating a patient's abdomen. The clinician places firm pressure on the abdomen. When the clinician removes her hand, the patient states that he feels pain. Which of the following should be charted based on this result? a. Acute abdomen syndrome b. Abdominal guarding c. Rebound tenderness d. Abdominal rigidity

Rebound tenderness occurs when pressure is removed

A patient has been diagnosed with a gastric ulcer. He want to discuss dietary recommendations. Which of the following may the nurse share that will be helpful? a. Increase your caffeine intake to aid in stomach motility b. Eat plenty of high-fat foods, such as ice cream c. Refrain from or severely limit alcohol intake d. The dietary choices you make do NOT directly impact healing of this disorder. Feel free to eat as you like

Restricting alcohol can have a beneficial effect on ulcers. Caffeine and high fat foods will worsen stomach ulcers

A nurse hears that a patient is demonstrating respiration changes in response to the metabolic acidosis the patient is experiencing. What does the nurse expect to see? a. Pursed-lip breathing b. Shallow, rapid breathing c. Deep, labored breathing d. Prolonged inhalation while breathing

Respirations will be deep, labored breaths

A patient is having a severe asthma attack. Which of the following may be elevated? a. Hemoglobin saturation b. Forced expiratory volume (FEV) 1.0/forced vital capacity (FVC) ratio c. Respiratory rate d. FEV 1.0

Respiratory rate will be increased in an asthma attack. Hemoglobin saturation will NOT be altered in an asthma attack. FEV 1.0/FVC ratio will be reduced in an attack

A nursing student has mixed up the flash cards she was using to study for an upcoming ? Select all that apply a. Shigella and gastrointestinal infection b. Pertussis and whooping cought c. Streptococcus pneumoniae and strept throat d. Clostridium botulinum and muscle contractions e. Varicella zoster virus and chickenpox

Streptococcus pyogenes, NOT pneumoniae, causes strept throat. An infection with C botulinum will cause muscle paralysis, NOT contractions

A patient arrives in the emergency department complaining of a kidney stone. Which of the following is the appropriate nursing intervention? a. Restrict all fluids b. Withhold pain medications until the stone is passed c. Strain urine d. Offer sedative

Straining the urine is important so that the stone can be captured. Pain medications and increasing fluid intake would be provided. Sedatives are NOT provided when a patient is attempting to pass a kidney stone

A nurse is educating a patient about the male reproductive system. What is the path the nurse would trace from semen formation to ejaculation? Place the events in the proper order - Epididymis - Urethra - Prostate - Vas deferens - Testes

Testes is where semen is first produced. The fluid travels into the epididymis, vas deferens, prostate gland, and out the urethra (Testes - epididymis - Vas deferens - prostate - Urethra)

Which of the following conditions will require surgery for treatment? Select all that apply a. Testicular torsion b. Erectile dysfunction c. Hydrocele d. Cryptorchidism e. Priapism

Testicular torsion, or twisting of the testis, and cryptochidism require surgical correction. Medications are the normal treatment for erectile dysfunction, watchful waiting is the normal treatment for hydrocele, and priapism is a prolonged erection and is NOT surgically corrected

A nurse is educating a patient about the possibility for increased ischemic stroke with some heart conditions. Please place the following steps in order relating blood clots to stroke - Atrial fibrillation - Clot formation - Thrombus travels to carotid artery - Stagnation of blood - Stroke

The heart condition, atrial fibrillation, may cause stagnant blood to form a clot. This clot may travel in the carotid artery to the brain, causing a stroke ( Atrial fibrillation - Stagnation of blood - Clot formation - Thrombus travels to carotid artery - Stroke )

A nurse is counseling a patient with anorexia nervosa. Which of the following statements show the nurse has a correct understanding of this disease? a. The patient must compulsively overeat and then purge b. The patient's appetite has disappeared, and she is no longer hungry c. The most common age of onset for anorexia nervosa is from 13 to 18 years d. The recovery from anorexia nervosa is quick e. The patient likely has perfectionist tendencies

The most common age for anorexia nervosa to first present is from 13 to 18 years and one of the traits is a sense of perfectionism. Binging and purging are signs of bulimia nervosa, appetite and hunger are still present, and recovery may require years and may never be complete

A patient is at the emergency department with a severe asthma attack, with decreasing oxygen levels, increasing dyspnea and profound wheezing. The best rescue medication option for such a severe attack is which of the following? a. Inhaled corticosteroids b. Long-acting beta-agonists c. Short-acting beta-agonists d. Short-acting beta-agonists, epinephrine and corticosteroids

The patient is displaying signs of a severe asthma attack. Therefore, bronchdilation and inflammatory suppressants are given, it is the best option

A patient presents with secondary hyperthyroidism. Which of the following values does the nurse expect to see reduced on the chart? a. Thyroid stimulating hormone (TSH) b. Thyroxine c. Thyrotropin releasing hormone (TRH) d. Triiodothyronine

Thyrotropin releasing hormone (TRH) would be reduced by the elevated thyroid stimulating hormone (TSH) levels in secondary hyperthyroidism. In secondary hyperthyroidism, TSH, thyroxine, and triiodothyronine levels would be increased

A patient is suffering from severe hepatitis B infection. Which of the following are dietary recommendations the nurse will make? Select all that apply a. Small, frequent meals b. High-fat meals c. Liquid only diet d. High calorie meals e. High protein meals

(A, D, E) Small frequent meals help NOT overload the digestive tract. High calorie meals can help prevent tissue breakdown and high protein meals can help maintain muscle tissue. High fat meals are NOT beneficial for liver function

A complete blood count (CBC) shows an elevated total white blood cell (WBC) level of 20,000 cells per microliter. Which of the following would be the simplest test that would enable a determination of which cell type is increased? a. WBC differential b. Computed tomography (CT) scan c. Lumbar puncture d. Bone marrow biopsy

(a) A WBC differential will reveal which cell type is elevated. A bone marrow biopsy can reveal information about WBC levels, however, this is NOT the simplest test

A nurse notices the asymmetry of a patient's chest wall and absent breath sounds on the affected side. Which of the following does she suspect? a. Pneumothorax b. Pneumonia c. Pulmonary embolism d. Pleural effusion

(a) A collapsed lung may cause asymmetry of the chest wall

A patient recently had orthopedic surgery on the left lower extremity. Which of the following may be prescribed to reduce the risk of deep venous thromboembolism? a. Pneumatic compression device b. Incentive spirometer c. Immobilization cast d. Complete bedrest

(a) A pneumatic compression device will present blood stasis and clot formation. Incentive spirometer works on pulmonary function and complete immobilization and complete bedrest increases risk of clots

A patient read a medical article and now feels she has hyperthyroidism. Which of the following of the patient's symptoms may be caused by hyperthyroidism? a. Sense of racing heart and tremors b. Lethargy and extreme fatigue c. Feeling cold all the time d. Gaining weight while feeling she is NOT eating more

(a) A racing heart can be cause by elevated thyroid hormone levels. Lethargy, cold intolerance, and gaining weight occur with hypothyroidism

A patient is recovering from ankle surgery that his ankle "feels stuck" when he tries to move into some positions. The nurse is worried that the continued decreased mobility may be because of which of the following developments? a. Adhesions b. Primary intention c. Keloid formation d. Fistula

(a) Adhesions are abnormal bands of internal scar tissue. If they occur in joints, then it may limit mobility. Primary intention healing may occur with a surgical incision. Keloids are a hyperepithelialiation of the skin in response to a wound. A fistula is an abnormal connection between organs, these would be unlikely in an ankle injury

A patient was recently diagnosed with Addison's disease as a result of an autoimmune condition affecting the adrenal cortex. Which laboratory values would support this diagnosis? a. Elevated adrenocorticotropic hormone (ACTH) b. Elevated cortisol c. Elevated aldosterone d. Decreased corticotropic releasing factor (CRF)

(a) Adrenocorticotropic hormone (ACTH) levels would elevate in an attempt to stimulate adrenal cortex functioning. In Addision's disease, adrenal cortex hormones cortisol and aldosterone levels are lower than expected. Corticotropic releasing factor (CRF) levels would be elevated to stimulate the adrenal cortex

Which of the following shows a patient's misunderstanding about infertility? a. About 70% of infertility cases are due to female issues b. 10% to 15% percent of couples experience infertility c. To meet the criteria for infertility, couples must have frequent, unprotected intercourse for at least a year d. Endometriosis can increase the risk for infertility

(a) Approximately 50% or slightly less of infertility cases are due to female infertility. Approximately 15% of couples experience infertility. Infertility is defined by having unprotected, frequent intercourse for at least a year with no pregnancies resulting. Endometriosis does increase the risk for infertility

If all people are assumed to have cardiac output of 5 L/min at rest, with a resting heart rate (HR) of 70 beats per minute (bpm), and an athlete has a resting HR of 58 bpm, which of the following likely applies? a. The athletes has a higher resting stroke volume b. The athlete has higher afterload c. The athlete has a significantly reduced ejection fraction d.The athlete has higher resting blood pressure

(a) Cardiac output is heart rate times stroke volume. For the same cardiac output, if HR is less, then stroke volume must be increased. Increased afterload makes the heart work more. given that the athletes stroke volume is likely high, ejection fraction will be high. No information is given relative to blood pressure

A patient is speaking about his risk of heart disease and his lipid levels. Which of the following shows further education is needed? a. I need to raise my low-density lipoprotein (LDL) levels and lower my high-density lipoprotein (HDL) levels b. Cholesterol is found in the diet and is synthesized by the liver c. HDL is the "good" cholesterol d. Some medications can lower your cholesterol

(a) HDL is considered the good cholesterol and LDL is the bad cholesterol

A patient's chest x-ray reveals nodular infiltrates. The patient is experiencing immune system mediated inflammatory response to an inhaled substance. Which condition is present? a. Hypersensitivity pneumonitis b. Pneumothorax c. Pulmonary hypertension d. Sleep disordered breathing

(a) In hypersensitivity pneumonitis, an inflammatory response is mediated. A pneumothorax is a collapsed lung, pulmonary hypertension causes vessel constriction, and sleep disordered breathing causes cessation of breathing during sleep

A nurse is working with a patient diagnosed with primary hyperparathyroidism. Which condition is the nurse concerned the patient may be at increased risk for? a. Pathological fracture b. Severe hypotension c. Weight gain and truncal obesity d. Acromegaly

(a) Increased parathyroid hormone (PTH) causes elevated bone break down, increasing the risk for fractures. Hypotension may develop with Addisons disease. Weight and truncal obesity occurs with elevated adrenal cortex hormones. Acromegaly occurs with elevated grwoth hormone (GH)

The wife of a patient with heart failure wants to known why everyone is so concerned about the patient's lungs. Which of the following is a correct explanation from the nurse? a. If the heart fails, fluid can back up into the lungs b. Disorders of the heart and lungs do NOT affect one another; we are just being extra cautious c. Heart failure is always brought on by lung problems, so we need to check the lungs regularly d. Heart failure results in inflammation of the heart, which can compromise lung function

(a) Left side heart failure results in fluid back up in the lungs. Some forms of heart failure may be brought on by lung problems, but NOT all

A patient presents with tenderness over the bones in vertebral fractures. Diagnostic results reveal excessive plasma B cells. Which secrete poorly functioning immunoglobulins. Which condition is suspected? a. Multiple myeloma b. Chronic myelogenous leukemia c. Non-Hodgkin's lymphoma d. Acute myelogenous leukemia

(a) Multiple myeloma often initially presents with bone pain, particularly in the area of the spine. Overproduction of B plasma cells develops. Non Hodgkins lymphoma are NOT associated with vertebral fractures

A clinician is assessing a patient for Cholelithiasis. Which of the following signs, if positive, would support the diagnosis of Cholelithiasis? a. Murphy's sign b. Kernig's sign c. Psoas sign d. McBurney's point

(a) Murphy's sign is used as an assessment for the gallbladder. Kernigs sign is used for meningitis, psoas sign and McBurneys point are used for appendicitis

A patient suffers a spinal cord injury. Which of the following statements by the nurse explains why recovery from this injury is likely to be limited? a. "In spinal cord injuries, neurons do NOT undergo further mitosis, which is a type of cell division. Thus, regeneration of spinal cord tissue is limited" b. "Spinal cord neurons undergo metaplasia and become columnar cells in response to injury " c. "Injured neurons are filled with intraceullar accumulations, making it impossible for the neurons to function" d. "Injured neurons form keloids, which limit the ability of spinal cord neurons to regenerate"

(a) Neurons do NOT undergo mitosis, so regeneration is limited. Injured neurons do NOT become columnar cells. In spinal cord injuries, the inability of neurons to regenerate is the major issue preventing full recovery. Keloids are formed on the surface of the skin, NOT in the spinal cord area

A patient is hemorrhaging, and cardiac output and stroke volume are falling due to decreased venous return. Which of the following describes most accurately what is happening? a. Decreasing preload b. Increasing afterload c. Increasing central venous pressure d. Decreasing heart contractility

(a) Preload is based on blood return to the heart. With hemorrhage, preload is reduced. Afterload is the resistance the heart has to work against. Central venous pressure will decrease with inadequate return. Contractility of the heart is likely elevated to compensate for poor venous return

A clinician is worried about renal failure in a patient suffering from acute pancreatitis. What should be addressed to prevent this development? a. Correct fluid imbalance to maintain renal perfusion b. Surgical reduction of pancreatic size c. Provide diuretics d. Begin hemodialysis

(a) Reduced renal perfusion can occur in pancreatitis. Maintaining adequate fluid administration is important. Surgical reduction would NOT be indicated. Diuretics would cause fluid loss and hemodialysis would NOT be an initial recommendation

When kidney functions begins to decline, which of the following may the nurse expect to see? a. Decreased serum creatinine clearance b. Increase glomerular filtration rate (GFR) c. Decreased blood urea nitrogen (BUN) d. Ketones in urine

(a) Serum creatinine levels will increase because the kidneys are unable to filter the blood effectively. Decreased GFR and increased BUN levels occur when kidney function declines. Ketones in urine occurs with diabetes mellitus

A patient with Addison's disease is at increased risk for an adrenal crisis. Which of the following does the nurse need to be aware of the patient developing? Select all that apply a. Hyponatremia b. Hyperkalemia c. Hyperglycemia d. Polycythemia e. Elevated stress response

(a, b) Addisons disease presents with reduced aldosterone, which may lower sodium levels and increase potassium levels. Addisons disease results in a reduced stress level and does NOT affect red blood cell levels

Which of the following is correct information to share with a patient whose child has been diagnosed with sickle cell anemia? Select all that apply a. Sickle cell anemia is a genetic disorder b. In certain conditions, your child's red blood cells, which carry oxygen, will undergo a shape change c. Sickle cell anemia is NOT a severe disease, and very little management will be needed for this conclusion d. We cannot be certain your child has sickle cell anemia, as there are no diagnostic tests for this disease e. Your child will require a bone marrow transplant

(a, b) Sickle cell anemia is an inherited disorder and it gets its name from the shape change that cells undergo. Sickle cell anemia requires medical management, there are diagnostic tests, and bone marrow transplant is NOT a treatment for sickle cell anemia

A patient is on anticoagulant therapy. Which factors are likely to be elevated? Select all that apply a. Activated partial thromboplastin time (aPTT) time b. Prothrombin time (PT) time c. International normal ratio (INR) d. D-dimer levels e. Fibrin degradation products

(a, b, c) Anticoagulation therapy increases clotting time indicators. D dimer indicates fibrinolysis, NOT clotting time. Firbin degradation products indicate clot breakdown, NOT clotting time

A patient presents with biliary colic. Which enzymes does the clinician expect to see elevated? Select all that apply a. Aspartate aminotransferase (AST) b. Alanine transminase (ALT) c. Bilirubin d. Troponin e. Gastrin

(a, b, c) Aspartate aminotransferase (AST), alanine transminase (ALT), and bilirubin levels are elevated with biliary colic. Troponin is found in muscle and gastrin is released from the stomach which are NOT elevaated with biliary colic

A patient has aplastic anemia with pancytopenia. Which of the following can the nurse expect? Select all that apply a. Increased risk for infection b. Anemia c. Increased risk for bleeding d. Elevated hematocrit e. Polycythemia

(a, b, c) Pancytopenia would imply a reduction in all blood cells, including white blood cells, red blood cells, and thrombocytes. Hematocrit would be reduced due to decreased RBCs and polycythemia indicates high RBC numbers

A laboratory report shows abnormal serum calcium levels. Which of the following tissues will the nurse need to evaluate, as these tissues are affected by abnormal calcium levels? a. Parathyroid gland b. Nervous system c. Skeletal muscles d. Lungs e. Cartilage

(a, b, c) Parathyroid hormone (PTH) regulates calcium levels. Abnormalities in calcium will disrupt neuronal signaling and affect muscle contraction

A nursing student is participating in patient education for parents with children who have genetic disorders. Which of the following show that nurse is well prepared? a. Cystic fibrosis will require pulmonary management b. Familial adenomatous polyposis may necessitate colonoscopies at a younger than normal age c. Down syndrome is always fatal by age 3 d. Familial hypercholesterolemia may increase risk of cardiac disease e. Turner syndrome will require testosterone for masculine development f. Huntington's disease symptoms will appear before the age of 5 g. Children with Ehlers Danlos syndrome should be encouraged to take gymnastics

(a, b, d) Cystic fibrosis damages the lungs, familial hypercholesterolemia increases serum LDL levels which may cause cardiovascular disease, and familial adenomatous polyposis increases the risk of colon cancer for which colonoscopies can be used to screen

Which signs or symptoms may the nurse be alert for in a patient with poorly controlled hypertension? Select all that apply a. Protein in urine b. S4 heart sound c. Jaundice d. Atrioventricular (AV) nicking on fundoscopic exam e. Polycythemia

(a, b, d) If hypertension damage is in the kidneys, then protein may show up in the urine. An additional heart sound may develop in hypertension. Atrioventricular (AV) nicking may be apparent in the retina. Jaundice and polycythemia are NOT related to hypertension

A clinician needs to assess the body's response to an infectious agent. Which of the following assessments are the best options? Select all that apply a. Complete blood count (CBC) b. Gram stain c. Antibody titer d. Acid-fast stain e. Culture and sensitivity

(a, c) A CBC may show elevated WBC levels in response to infection. An antibody titer is a measurement of antibodies formed in response to infection. Gram stain and culture and sensitivity evaluate the infectious agents, NOT the body's response. An acid fast test evaluates the infectious organism

A nursing student is being assessed on her knowledge of the genetic basis of disease. Which statement shows that the student needs to complete further studying? a. P53 is an oncogene b. Damage to tumor suppressor genes and proteins may increase the risk of cancer development c. Aneuploidy is another term for chromosomal translocation d. Knudson's "two hit hypothesis" states that most cancers result from damage to 1 allele e. Autosomal dominant disorders do NOT follow Knudson's "two hit hypothesis"

(a, c, d) P53 is a tumor suppressor gene, NOT an oncogene, aneuploid indicates abnormal chromosome number, and Knudsons two hit hypothesis states that damage to both alleles is needed for most forms of cancer development

In the bicarbonate buffering system, which of the following is true about carbonic acid? Select all that apply a. Carbonic acid dissociates into hydrogen and bicarbonate b. Carbonic acid is represented by HCO3- c. Carbonic acid levels decrease as carbon dioxide levels increase d. Carbonic acid is formed by carbonic anhydrase e. Carbonic acid is NOT associated with buffering pH changes

(a, d) Carbonic acid dissociates into hydrogen and bicarbonate. Carbonic anhydrase converts water and carbon dioxide into carbonic acid. HCO3- is the formula for bicarbonate, NOT carbonic acid. Carbonic acid is formed from carbon dioxide and water, so as carbon dioxide increases, carbonic acid will also increase. Carbonic acid can alter pH levels as it is converted into hydrogen ions and bicarbonate

A patient has suffered a myocardial infarction. Which factors are expected to be elevated? Select all that apply a. Cardiac troponin b. Hematocrit (Hct) c. Aspartate aminotransferase (AST) d. Creatine phosphokinase-myocardial band e. High-density lipoprotein (HDL)

(a, d) Cardiac troponin is released from damaged myocardial cells. Creatine phosphokinase myocarial band elevates with cardiac cell damage. Hematocrit and HDL levels are NOT affected by myocardial infarction. AST is a liver factor NOT affected by myocardial infarction

A nursing student is reviewing common signs and symptoms of hematologic neoplasms. Which of the following should be included in her list? Select all that apply a. Bone pain b. Thrombocytosis c. Polycythemia d. Lymphadenopathy e. Splenomegaly

(a, d, e) Bone pain often develops in leukemia due to overproduction of some cells. Abnormal lymph system function is common in leukemia. The spleen may enlarge in leukemia due to sequestration of overproduced cells. Thrombocytopenia, NOT thrombocytosis, can develop in leukemia. Anemia, NOT polycythemia, tends to develop in leukemia

A young child is diagnosed with cryptorchidism. The parents want to know what the risks are if this is NOT surgically corrected. Which of the following responses are appropriate? Select all that apply a. There is an increased risk of infertility b. There is no significant increase in health risks c. Surgical correction is for cosmetic purposes only d. Sperm development is impaired e. There is an increased risk for testicular cancer

(a, d, e) Cryptorchidism will risk infertility is left untreated, impair sperm development, and increase risk of testicular cancer. Surgical correction is necessary

Which of the following would be correct information for the nurse to share about testicular cancer? Select all that apply a. It has one of the highest cure rates for a cancer b. It is most common in men age 65 and older c. Even after treatment, fertility is maintained d. Cryptorchidism increases risk for testicular cancer e. An initial sign is often a firm nodule in one testicle

(a, d, e) Testicular cancer does have a high cure rate. Cryptorchidism increases the risk for testicular cancer. An early sign of testicular cancer is a node present in one testicle. It is more common in young men and can disrupt fertility

A nurse is working with a patient with pulmonary difficulties. Which of the following pulmonary conditions are more likely to result in respiratory alkalosis rather than acidosis? Select all that apply a. Hypoxia with hyperventilation b. Airway obstruction c. Cystic fibrosis d. Under ventilation with mechanical ventilation e. Asthma with hyperventilation

(a, e) Hyperventilation lowers carbon dioxide levels resulting in respiratory alkalosis. Airway obstruction, cystic fibrosis, and under ventilation would cause carbon dioxide retention, leading to respiratory acidosis

A ventilation-perfusion scan on a patient with a pulmonary embolism may reveal which of the following? a. Ventilation and perfusion matched b. Ventilation with perfusion disruption c. Normal perfusion with disrupted ventilation d. Fluid accumulation in alveoli

(b) A pulmonary embolism will reduce perfusion while ventilation remains unchanged. Perfusion is disrupted with a pulmonary embolism and fluid accumulation is a sign of edema

A patient has severe hypertension and enlargement and distention of the left ventricle making filling very difficult. What type of heart failure may the patient display? a. Systolic heart failure b. Diastolic heart failure c. High-output heart failure d. Cor pulmonale

(b) Diastolic heart failure manifests when the heart cannot be filled properly. Systolic heart failure occurs when the heart cannot eject blood properly. High output failure occurs when the heart cannot meet excessive demands. Cor pulmonale is a right-side heart tissue

A clinician performs tactile fremitus and percussion on a patient with pneumonia. Which of the following are expected? a. Hyperresonance b. Dullness with percussion c. Decreased fremitus d. Decreased vibration

(b) Due to fluid accumulation, dullness to percussion occurs in pneumonia. Fremitus and vibrations would be increased with pneumonia. Hyperresonance develops with additional air in the lungs

A nurse is reviewing clotting factor laboratory values. Activated partial thromboplastin time (aPTT) and international normal ratio (INR) are listed. The nurse questions why the prothrombin time (PT) value may be missing. Which of the following is an appropriate explanation? a. PT does NOT assess clotting times for blood b. INR values reflect PT clotting time c. D-dimer is a more accurate measurement than PT for blood clotting times d. aPTT values reflect PT clotting times

(b) INR is a ratio used to express PT clotting time. PT is a clotting time assessment, d dimer is indicative of clot breakdown, NOT clotting times, and aPTT and PT measure two different aspects of the clotting cascade

Results show that a patient has had an ischemic stroke rate than a hemorrhagic stroke. What is the appropriate treatment? a. Wait and see how patient stabilizes b. If proper criteria are met, initiate thrombolytic therapy c. Schedule a craniotomy to release pressure from bleeding in skull d. Administer antihypertensive medications

(b) Ischemic criteria are met, initiate thrombolytic therapy. Control of hypertension is critical in hemorrhagic strokes and ischemic stroke does NOT cause excessive bleeding

A patient suffers severe kidney damage and the juxtaglomerular cells are disrupted. Which of the following manifestations does the nurse expect? a. Elevated blood pressure (BP) b. Decreased renin secretion c. Decreased antidiuretic hormone d. Increased angiotensin

(b) Juxtaglomerular cells normally secrete renin, which would activate angiotensin resulting in decreased angiotensin. Disruption of the juxtaglomerular cells will result in a reduced ability to respond to BP decreases. Juxtaglomerular cells are NOT associated with antidiuretic hormone

A nurse is to counsel a patient recently diagnosed with mastitis. What should the nurse be prepared to focus on? a. Options for addressing a uterus that has fallen out of position b. Breastfeeding and how to reduce the risk of infections c. Recovery from breast tissue removal d. Issues surrounding infertility

(b) Mastitis which is inflammation of the breasts normally occurs with breastfeeding. A uterus that has moved downward from the normal position is referred to as a prolapse

A patient has a fluid volume overload due to congestive heart failure. Which of the following compensations will the body attempt to address the fluid volume overload? a. Increased renin b. Increased natiuretic peptide c. Increased angiotensinogen d. Increased aldosterone

(b) Natriuretic peptide is released, primarily from the atria, when there is a volume overload. Renin, angiotensinogen, and aldosterone increase with fluid volume deficit

A nursing student has a question about prothrombin (PT) and activated partial thromboplastin time (aPTT) values. Which of the following responses show that the student has correct understanding? a. PT and aPTT measure the same thing but are tests run by different companies b. PT and aPTT, when increased, increases the risk for bleeding c. PT and aPTT values should be less than 1 second d. A person with hemophilia would have a reduced PT and aPTT values

(b) Prolonged clotting times increase the risk for excessive bleeding. PT measures the extrinsic pathway, while aPTT measures the intrinsic pathway. Normal clotting times are in the range of several seconds. Hemophilia disrupts the clotting cascade, which prolongs clotting times

A patient with chronic obstructive pulmonary disorder (COPD) has now developed right ventricular failure. What is the likely explanation? a. The two events are unrelated b. Vasoconstriction in the lungs, due to hypoxia, has increased the workload on the right ventricle c. The COPD has caused a reduction in venous return to the heart d. As the right ventricle receives fluid from the lungs, it has become overworked by increased fluid volume

(b) Pulmonary vasoconstriction may occur with hypoxia which will increase the workload on the right ventricle. The right ventricle does NOT receive a large proportion of fluid from the lungs and COPD does NOT alter venous return to the heart

A sickle cell anemia crisis leads to hemolysis of red blood cells (RBCs). Which of the following may the nurse expect to see? a. Decreased serum bilirubin b. Increased reticulate percentage c. Increased hematocrit d. Decreased likelihood of jaundice

(b) Reticulocytes are immature RBCs and levels will increase to replace lost cells. Elevated hemolysis will increase bilirubin levels, decrease hematocrit levels, and increase the risk of jaundice formation

A male client has developed benign prostatic hyperplasia. He would like to know what the role of the prostate gland is before considering treatment options. Which of the following can be shared with the patient as the primary role of the prostate gland? a. Help maintain erection of the penia b. Secrete an alkaline fluid that accounts for a large percentage of ejaculate c. Secrete chemicals that aid in the sense of sexual pleasure d. The location for final maturation of sperm cells

(b) Semen is compromised of secretions and fluid from the prostate gland. The prostate gland does NOT maintain the erection of the penis. The epididymis is the place for final sperm maturation

A patient states that he has always taken one or two medications to manage his type 2 diabetes mellitus (DM). But now the physician is discussing the possibility of using insulin for treatment. The patient is worried and wants to know how this developed. What is the appropriate response? a. Your type 2 DM is turning into type 3 DM b. Type 2 DM requires a stepwise treatment plan. In some cases, the next management step is to use insulin c. The need for insulin developed because you were NOT using the medications correctly d. All of these medication options could be avoided with the proper diet and exercise

(b) Some patients with type 2 diabetes mellitus do eventually require insulin

Looking at a patient's set of results, a nurse noticed FEV 1.0 has increased from 63% to 85% after bronchodilator usage. Which of the following is a correct interpretation? a. After bronchodilator usage, FEV 1.0 should decrease, NOT increase b. The patient may be experiencing bronchial constriction which is alleviated somewhat with a bronchodilator c. The results show the patient can inhale 85% of expected maximal inhalation d. The results indicate the patient has severely compromised lung function

(b) The fact that FEV1.0 increased shows that a bronchodilator made it possible to exhale more air in the first second. FEV1.0 is the amount of air exhaled in the first second of a maximal exhalation. The results do NOT indicate severely compromised lung function

A nursing student is studying for an examination. Which of the following should she list in her notes as factors the kidneys release? Select all that apply a. Creatinine b. Erythropoietin (EPO) c. Renin d. Aspartate aminotransferase (AST) e. Plasminogen activator

(b, c) Erythropoietin (EPO) is released by the kidney to stimulate red blood cell formation. Renin is released in response to low blood pressure to the kidney. Creatinine is a waste product produced by amino acid metabolism, AST is a liver factor, and plasminogen activator is NOT produced by the kidney

A nurse states that a patient in the emergency department is presenting with Levine's sign. Which of the following responses apply? Select all that apply a. The patient needs intubation b. The patient may be experiencing angina pectoris c. The patient is presenting with a clenched fist over the sternum d. The patient is presenting with severe flank pain e. Suggest that a computed tomography (CT) scan be ordered

(b, c) Levine's sign is a clenched fist over the sternum and is indicative of chest pain. It may indicate a myocardial infarction , so an electrocardiogram is needed

A nurse is explaining a teritary intention healing to a patient. Which of the following characteristics are correct for the nurse to include in her explanation? a. Common post-surgical wound b. May require temporary packing with sterile gauze c. May be left open for 4 to 5 days d. May have extensive drainage e. Wound that forms with stitching of incisions

(b, c, d) A tertiary intention is left open and may be packed with sterile gauze, be left open for several days before closure, and involve extensive drainage. Post surgical wounds form primary intentions and stitching an incision would be an example of primary intentions

A nurse is monitoring a patient with metabolic acidosis. Which of the following may she expect to observe? Select all that apply a. An increase in pH b. Development of hyperkalemia c. Ventilation increases d. An increase in plasma hydrogen ion levels e. Potassium shifts into cells from extracellular fluid (ECF)

(b, c, d) In acidosis, H+ may shift into cells and K+ shifts out of cells, leading to hyperkalemia. To compensate for acidosis, ventilation increases to reduce carbon dioxide levels. When acidosis occurs, pH is lower than normal and H+ ions increase. In acidosis, pH decreases and H+ shifts into cells, while K+ shifts out of cells

The patient is diagnosed with chronic bronchitis. Which of the following are signs and symptoms of this disorder? Select all that apply a. Air trapping b. Cyanosis c. Clubbing of the fingers d. Wheezing e. "Pink puffer"

(b, c, d) Prolonged hypoxemia may cause cyanosis and clubbing of the fingertips. Bronchiole constriction can cause wheezing, as air moves past the constriction. Air trapping and pink puffer occurs in emphysema

A nurse is speaking to a patient with very high lipid levels. The nurse is discussing food choices with the patient. Which of the following recommendations would be helpful for the patient? Select all that apply a. Increase dietary cholesterol b. Decrease trans-fatty acid c. Increase fiber d. Increase fish oil e. Increase saturated fats

(b, c, d) Trans fatty acids in the diet at high levels have been shown to increase the risk of heart disease. Fiber may have a beneficial effect on blood lipid levels. Fish oils, or omega 3 fatty acids, may reduce heart disease risk. Dietary cholesterol and saturated fats in the diet have been shown to increase blood lipid levels

A patient's blood pressure is falling due to hemorrhage. Which of the following are appropriate compensations by the body? Select all that apply a. Decreased heart rate b. Increased antidiuretic hormone (ADH) c. Increased aldosterone d. Decreased peripheral vasoconstriction e. Increased fluid retention

(b, c, e) ADH is released to retain fluid volume. Aldosterone will increase and aid in sodium and water retention. Fluid will be retained to increase blood volume and blood pressure. Heart rate increases to compensate for falling blood pressure and vasoconstriction will increase to help maintain blood pressure

A nurse notes on a patient's chart the presence of syndrome of inappropriate antidiuretic hormone (SIADH) due to a pulmonary tumor. Which of the following can the nurse expect to observe? Select all that apply a. Dry mucous membranes b. Plasma osmolarity less than normal c. Urine output greater than normal d. Urine osmolarity higher than normal e. Decreased blood pressure

(b, d) As SIADH retains water in the body, plasma components are diluted and urine becomes more concentrated. The fluid retention in SIADH will decrease urine output and increase blood volume and blood pressure

Which of the following would alert the clinician that the patient may have acute glomerulonephritis? Select all that apply a. Pain with urination b. Protein in urine c. Oliguria d. Blood in urine e. Fluid accumulation in renal pelvis

(b, d) Glomerular damage can lead to protein in the urine and blood in the urine. Pain with urination more commonly occurs with infection. Oliguria does NOT present with acute glomerulonephritis and fluid accumulation occur with hydronephrosis

A nursing student is reviewing blood flow through the heart. She is writing all the valve names that can be used for movement of blood from the left atrium to the left ventricle. Which should she include in her list? Select all that apply a. Tricuspid valve b. Bicuspid valve c. Pulmonic valve d. Mitral valve e. Left atrioventricular valve

(b, d, e) Mitral valve and the left atrioventricular valve are between the left atrium and left ventricle and bicuspid valve is another name for the mitral valve. Tricuspid valve is between the right atrium and right ventricle. Pulmonic valve is between the right ventricle and pulmonary trunk

A dexamethasone suppression test is performed on a patient. The diagnosis is Cushing's syndrome. What was the result of the dexamethasone test? a. Cortisol levels were suppressed b. Catecholamine levels remained elevated c. Cortisol remained elevated d. Catecholamine levels were suppressed

(c) A dexamethoasone test is diagnosed to suppress cortisol levels if the adrenal cortex is functioning properly. In Cushing's syndrome, despite dexamethasone, cortisol levels remain high

A patient has a collapsed lung due to a stab wound. What type of penumothroax would this injury be classified as? a. Primary spontaneous b. Tension c. Traumatic d. Iatrogenic

(c) A stab wound would be considered a form of traumatic pneumothroax. A primary pneumothroax occurs for no apparent reason. A tension pneumothorax is a life threatening condition in which air builds up around the lungs. IAtrogenic pneumothorax occurs during surgery or some procedure

A patient has exertional dsypnea with an inability of the left ventricle to empty fully. Chest pain develops. Which valve disorder could contribute to such development? a. Pulmonic stenosis b. Tricuspid stenosis c. Aortic stenosis d. Mitral stenosis

(c) Aortic stenosis would make it difficult for the left ventricular to eject blood. Pulmonic stress and tricuspid insufficiency would NOT affect the left ventricle. Mitral insufficiency means blood flows backward from the LV to the left atrium

A patient's laboratory results reveal pancytopenia with normal-shaped cells and reduced reticulocyte percentage. Which of the following should be considered as possible causes? a. Folic acid deficiency b. Thalassemia c. Aplastic anemia d. Hemolytic anemia

(c) Aplastic anemia affects all cell types of the bone marrow. Folic acid deficiency affects red blood cells, thalassemia is a disruption in hemoglobin, and hemolytic anemia would cause rupture and loss of RBCs

A clinician needs to determine whether a myocardial infarction (MI) has occurred and how it may be progressing. Which serum marker may be helpful? a. Lactic acid b. Aspartate aminotransferase (AST) c. Cardiac troponin d. Amylase

(c) Cardiac troponin levels increase following cellular damage due to MI. The changes in serum troponin levels can be tracked over time. Lactic acid levels are NOT diagnostic for a MI. AST levels elevate with hepatic, NOT cardiac damage. Amylase levels are unrelated to MI

A female patient appears at the clinic complaining that she "looks different". She states that her face is more rounded than normal, she has abnormal hair growth, and she feels like she gained fat around her back. What condition does the clinician suspect? a. Phenochromocytoma b. Hyperthyroidism c. Cushing's syndrome d. Addison's disease

(c) Cushing syndrome results from elevated cortisol levels. This can cause fat deposition in the back and hirsutism in women. A phenochromocytoma would present with elevated heart rate, blood pressure, and nervousness. Hyperthyroidism would present with weight loss and agitation whereas Addison disease causes weight loss and hypotension

A nurse sees a prescription for an anti-histamine medication for a patient with allergies. Which of the following explanation is correct? a. An anti-histamine blocks the fever response common in allergies b. A mistake has been made on the prescription. Anti histamines are NOT prescribed for allergies c. An anti-histamine will block the congestion resulting from increased capillary permeability caused by histamine d. An anti-histamine will block prostaglandin synthesis, which will reduce congestion due to allergies

(c) Histamine increases congestions by increasing capillary permeability. Histamines are NOT involved in the generation of fevers or prostaglandin synthesis. Anti histamines are commonly prescribed for allergies to reduce congestion

A patient at a high risk for deep venous thromboembolism is placed on anticoagulant therapy. Which of the following will indicate effectiveness of this treatment? a. Reduced prothrombin time (PT) b. Reduced activated partial thromboplastin time (aPTT) c. Elevated international normalized ratio (INR) d. Patient reports feeling better

(c) INR is a measurement of clotting time. With anticoagulant therapy, it should increase. PT and aPTT indicate clotting time, which should increase with anticoagulant therapy. Anticoagulant therapy should NOT be evaluated based solely on how a patient feels

An international normal ratio (INR) value of 2.5 is reported. Which of the following is the correct interpretation? a. The patient has increased the risk of clot formation, given this INR values b. The patient likely has hemophilia c. The patient has a prolonged prothrombin time (PT) time d. The patient has reduced platelet values

(c) INR is reflective of the PT clotting measurement. A normal INR is reported as 1.0. An elevated INR indicates prolonged clotting times, hemophilia is NOT diagnosed through INR times, and a reduction in platelet values cannot be assumed from an elevated INR

A patient has a positive result for a sexually transmitted disease. Which of the following would be appropriate to state to the patient? a. The test results are likely inaccurate. Why don't you return in a few days and we will test you again? b. If you refrain from sexual activity, most sexually transmitted infections will resolve on their own c. Would you consider additional testing for other sexually transmitted diseases? d. You should keep this information private. Your sexual partners will likely seek help if they develop signs or symptoms

(c) If a patient has one sexually transmitted infections then the opportunity for further testing should be presented. Tests for STDs are highly accurate

A nursing educator is evaluating the student's understanding of electrolyte shifts with acid-base disturbances. Which of the following statements indicate that the students have correct understanding? a. If a patient has acidosis and hyperkalemia, then treatment protocols require a reduction in potassium levels b. Hyperkalemia occurs in acidosis as total body potassium is increased c. In acidosis, hyperkalemia may appear, as potassium shifts from the intracellular fluid (ICF) to the extracellular fluid (ECF) d. Hypokalemia develops in acidosis, as potassium and hydrogen are contransported into the cell

(c) In acidosis, hydrogen ions shift into cells and potassium ions shift out of cells. This increases potassium levels in the plasma

A wife accompanies her husband to a medical checkup. She states that she catches here husband eating in the middle of the night, but he denies his behavior. What response from the nurse may be appropariate? a. "Your husband is in denial. We all know when we eat food, even late at night" b. "Your husband likely has very high levels of nocturnal melatonin and leptin, driving his need to eat c. "A syndrome, known as night-eating syndrome, may be occurring" d. "This is very unusual, as night eating syndrome is known to affect only women"

(c) In night eating syndrome, patients may consume food at night and be unaware of doing so

A few days prior, a patient presented at a clinic stating he was generally NOT feeling "all that well". He now presents at the clinic with significant signs and symptoms of an infectious disease. Which stage of infection in the patient displaying presently? a. Incubation b. Prodromal c. Acute d. Convalescent

(c) In the acute phase of infection, signs and symptoms are apparent. Incubation period is from exposure to first signs and symptoms and prodromal is after exposure when initial signs and symptoms appear. The convalscent phase is recovery from the infection

A patient complains that she feels she must urinate three times an hour or more. She says urinating helps relieve the discomfort she feels. But she says this is disrupting her life. Which of the following disorders is most likely present? a. Renal calculi b. Asymptomatic bacteriuria c. Interstitial cystitis d. Pyelonephritis

(c) Interstitial cystitis causes a significant increase in frequency of urination. Renal calculi would cause a sensation of pain and asymptomatic bacteriuria does NOT present with any signs of discomfort

A patient wants to know if there is a difference between a lymphoma and leukemia. Which statement should the nurse share? a. Lymphomas develop due to a disorder or red blood cells, while leukemias are disorders of white blood cells (WBCs) b. Lymphomas are neoplasms, and leukemias are NOT c. Lymphomas develop in the lymph nodes while leukemias are present in the blood d. Lymphomas develop in adults, while leukemia develop in children

(c) Lymphomas develop in the lymph nodes while leukemias is due to a disorder of WBCs in the blood. Both lymphomas and leukemias are neoplasms and it can develop in children and adults

A nursing student is looking at a while blood cell (WBC) diffferential test and cannot find the cell type macrophage listed. Another nurse provides which of the following correct explanation? a. These are improper results. Macrophages should be listed b. Macrophages are transitory cells that exist for only several minutes c. Macrophages are derived from monocytes, a type of cell that is reported on a WBC differential test d. Macrophages are an advanced, mature type of neutrophil, which is reported on the laboratory results

(c) Macrophages are derived from monocytes which are reported on a differential WBC. Macrophages are NOT normally listed on a WBC differential and has a life span beyond several minutes

A nursing student is learning about diabetes mellitus (DM). Which statement indicates correct understanding on her part? a. The different types of DM are all treated the same way b. The difference types of DM have the same etiology destruction of the beta cells of the pancreas c. Treatment of patients with DM involves creating an individual treatment plan d. Patients with DM should be given a choice as to whether to use insulin injections or oral medications for diabetes management

(c) Patients with diabetes mellitus require individual treatment plans. The choice to use insulin or NOT is NOT possible for all patients with DM

A patient is experiencing a decrease in blood pressure with myocardial infarction. What type of heart failure is exemplified by this scenario? a. Diastolic heart failure b. High output heart failure c. Systolic heart failure d. Cor pulmonale

(c) The patient is unable to maintain blood pressure due to the heart's inability to contract effectively. The heart can fill properly but cannot eject blood effectively, no excessive demands are being placed on the heart, and cor pulmonale is a right side heart issue due to pulmonary problems

A patient with sickle cell anemia, having been hospitalized with a vaso-occlusive crisis, is being discharged. Which instructions will be helpful for the patient? a. The number one recommendation is to increase your activity level b. Do NOT overreact. Sickle cell anemia is NOT a severe disease and should NOT require hospitalization c. Make certain you have adequate hydration and rest, and avoid stressors when possible d. Use caution because your blood has limited ability to clot in a small cut may result in significant bleeding

(c) To reduce the risk of vaso occlusive crisis, patients should maintain adequate hydration, get adequate rest, and try to avoid them. The risk of vaso occlusive increases if activity is increased beyond normal. Sickle cell anemia and particularly vaso occlusive crisis are severe medical conditions but do NOT lead to significant bleeding issues

A man has a severe valve disorder. He asks the nurse why he should be worried because he feels fine at rest. What is the appropriate response? a. As long as you feel fine at rest, there is nothing to worry about b. You should NOT worry because valve disorders are easily corrected with valve replacements c. Your valve disorder could increase the risks of clots d. Valve disorders normally lead to myocardial infarction

(c) Valve disorders may increase the stasis of the blood, leading to increased clot formation. A valve disorder, even without signs and symptoms, can cause complications. Valve disorders are NOT easily corrected. It can contribute to myocardial infarction, but NOT always

A nursing student is reviewing the circulatory system by comparing the different structures. Which of the following is correct in regard to arteries and veins? a. The venous system is a higher-pressure system b. Less blood volumes is in the veins c. Veins have valves and arteries do NOT d. Veins always carry deoxygenated blood while arteries carry oxygenated blood

(c) Veins have valves and arteries do NOT. The venous system has lower pressure than the arterial system. The veins hold approximately 70% of the blood volume. Pulmonary veins do NOT carry deoxygenated blood, although the majority of veins do carry deoxygenated blood

You have been assigned to your hospital's "Opioid Management" committee. Which of the following can you likely expect? Select all that apply a. A small committee that meets infrequently, as opioids are a well-tolerated class of medications b. Instructions on the use of naloxone to manage opioid withdrawal c. Discussions about screening patients for risk of opioid abuse d. Information about symptoms of opioid withdrawal e. Discussions on increasing patient compliance to complete all opioid medication

(c, d) Assessment and screen of patients at risk for opioid abuse is a critical step in the management strategy. It is important that everyone be well informed about opioid withdrawal symptoms so that the symptoms can be recognized. Opioid abuse is a critical issue, thus the committee would most likely meet frequently. Naloxone is used to treat opioid overdose, NOT withdrawal

A nurse is to speak to a patient about his recent vitamin B12 deficiency. Which recommendations would benefit the patient? Select all that apply a. Eat a vegetarian diet b. Restrict folic acid intake c. Take a supplement of vitamin B12 d. Food choices should include meat and eggs e. Get plenty of sunshine as ultraviolet (UV) light activates vitamin B12

(c, d) Supplementation of vitamin B12 may be helpful and vitamin B12 is found in high levels in meat and eggs. Folic acid restriction will NOT aid in vitamin B12 levels and vitamin D, NOT vitamin B12, is activated by UV light

Which of the following may require increased vigilance for bleeding disorders? Select all that apply a. Woman using oral contraceptives b. Individual who smokes c. Cancer is bone marrow d. Hemophilia e. von Willebrand disease

(c, d, e) Cancer in the bone marrow may disrupt platelet function. Hemophilia effects the clotting cascade and Von Willebrand disease increasing the risk for bleeding. Oral contraceptive and smoking increases the risk for clot formation

A patient needs a valve replacement. Which of the following characteristics will apply if a mechanical valve replacement is used? Select all that apply a. It is commonly made from cadaer, porcine, or bovine materials b. It is referred to as a valvotomy c. It can last a patient's lifetime d. Anticoagulation therapy is necessary e. Risk for thromboembolism is increased

(c, d, e) Mechanical valves can last a lifetime and encourage clot formation so anticoagulant therapy will be needed. Mechanical valves are NOT constructed from biological tissues. Valvotomy is a valve repair process, NOT a valve replacement

Laboratory values reveal the following: hemoglobin of 19 grams per deciliter and hematocrit of 53%. Which of the following correctly interprets these values? Select all that apply a. Red blood cell (RBC) loss is increased b. Hemoglobin is normal while hematocrit is less than normal c. Percentage of blood volume that is plasma is reduced d. Anemia has developed e. Both hemoglobin and hematocrit levels are elevated

(c, e) If hematocrit percentage is elevated, then plasma percentage is likely reduced. Both hemoglobin and hematocrit are higher than normal which indicates increased RBC

Immediate wound protection with sterile saline-moistened dressings and prompt surgical evaluation is part of a treatment strategy for which of the following wound types? Select all that apply a. Keloid b. Contracture c. Wound evisceration d. Adhesions e. Wound dehiscence

(c, e) Wound evisceration is when organs protrude through a wound opening and wound dehiscence is rupturing of a wound. Keloids are due to hyperplasia of the epithelial tissue, contracture is the inflexible shrinkage of a wound, and adhesions are internal bands of scar tissue

A patient states that she was diagnosed with mitral valve prolapse and wants to know more about this disorder. Which of the following should the nurse include in her description? a. This type of valve disorder is very rare b. A valve in your heart is being destroyed by infection c. A valve in your heart does NOT open correctly d. A valve in your heart does NOT close correctly

(d) A mitral valve prolapse indicates that the valve does NOT close correctly. Mitral valve prolapse is a common valve disorder. Infective endocarditis is an infection that damages heart valves

A nurse notices a pulsing mass in the abdomen of a thing elderly gentleman. What is an appropriate response? a. Place firm, hard pressure on the mass until pulsation stops b. Schedule immediate abdominal surgery c. Prepare for intubation d. Recognize that this is likely an abdominal aortic aneurysm, which will need to be monitored

(d) A pulsing mass in the abdomen likely indicates an abdominal aortic aneurysm, which often requires monitoring. Firm pressure should NOT be place on an abdominal anuerysm. Surgery is NOT always indicated and intubation would NOT be needed for an abdominal aortic aneurysm

A patient with liver failure has dilated veins over the umbilical area. Which of the following should the nurse chart? a. Ascites b. Petechiae c. Angioedema d. Caput medusa

(d) Caput medusa is when the vessels in the umbilical area dilate, most likely due to portal hypertension. Ascites is edema in the abdominal cavity, petechaie results from capillary breakdown, and angioedema occurs in allergic reactions

A nurse is assigned to treat a patient with chronic lymphocytic leukemia (CLL). The nurse has never met the patient before. Which of the following most closely fits the typical patient profile for this disease? a. Newborn infant b. Child, age 10 year c. Female, age 30 to 50 years d. Elderly male, age 65 to 70 years

(d) Chronic lymphocytic leukemia (CLL) is most common in elderly persons. Acute lymphocytic leukemia is more common in children

An elderly patient's laboratory results reveal lymphocytosis with WBC count greater than 20,000 cells per microliter. Which leukemia type is most likely? a. Acute myelogenous leukemia b. Chronic myelogenous leukemia c. Acute lymphocytic leukemia (ALL) d. Chronic lymphocytic leukemia

(d) Chronic lymphocytic leukemia often presents with very high lympocyte levels in an elderly individual. Elevated lymphocytes, NOT myelogenous cells, are present in the blood. ALL is more common in children

A chest x-ray reveal hyperinflation of the lungs, increase anterior-posterior chest wall dimensions, and flattened diaphragm. Which of the following conditions does a nurse suspect? a. Tuberculosis b. Asthma c. Pulmonary fibrosis d. Chronic obstructive pulmonary disease (COPD)

(d) Chronic obstructive pulmonary disease (COPD) can cause air trapping, resulting in increased anterior posterior dimensions. Tuberculosis shows tubercles in the lungs. Asthma and pulmonary fibrosis do NOT result in hyperinflation of the lungs

A patient suffers a burn wound and is now requiring surgery on the healed area to increase mobility in the knee. Which has likely occurred? a. Wound dehiscence b. Wound evisceration c. Keloid formation d. Contracture

(d) Contractures are inflexible shrinkage of a wound. Wound dehiscence is a rupture of a wound, wound evisceration is when organs or tissue protrude from a wound, and keloid formation results from hyperplasia of epithelial cells

A patient presents with a white blood cell (WBC) count of 20,000 cells per microliter. Which of the following interpretations would be correct for the nurse to make? a. The WBC count is so high that the patient will never have to worry about an infection b. The patient's results are diagnostic for allergies or autoimmune disorders c. The patient likely has anemia, and the elevated WBC level is attempting to compensate d. Although elevated, the WBCs may be poorly functioning, increasing a patient's risk for infection

(d) Elevated WBCs ma be poorly functioning. A WBC value of 20,000 cells in beyond the normal range for autoimmune and allergy disorders. WBCs do NOT compensate for red blood cells

A nurse sees in a patient's file the diagnosis of phenochromocytoma. No treatment have begun. Which of the following symptoms and signs may the nurse expect to observe in the patient? a. Severe hypotension b. Sluggish responses and lethargy c. Weight gain d. Tachycardia

(d) Pheochromocytoma is a tumor of the adrenal medulla, causing excess epinephrine and norepinephrine. These signals increase heart rate (HR) and blood pressure as well as stimulate activity and weight loss

On a fundoscopic examination of a patient with diabetes mellitus (DM), proliferative retinopathy is noted. Which of the following is the correct interpretation? a. Positive sign of retinal recovery b. Loss of retinal blood vessels c. Occurrence of retinal detachment d. Damage to microvasculature of the retina

(d) Proliferative retinopathy often develops in diabetes mellitus. Although new vessels are formed, the vessels are fragile. Retinal detachment does NOT show proliferation of retinal vessels

Despite being vaccinated, a young child, age 3, is chronically ill. Immunoglobulin levels are measured and found to be low in response to vaccination. What is the correct interpretation a nurse may make? a. Low immunoglobulin levels are the expected normal response b. The child has abnormally functioning T cells c. Both the innate and adaptive immune systems of the child are compromised d. Hypogammaglobulinemia may need to be investigated in this child

(d) The abnormal immunoglobulin response, as well as the infection, require further follow through. Hypogammaglobulinemia may be a possible diagnosis. Immunoglobulins are antibodies and antibody levels should increase after a vaccine. Immunoglobulins are produced by B cells, NOT cells, and they are a part of the adaptive immune system, NOT the innate

A patient with a clotting disorder is scheduled for removing of the body's plasma, followed by replacement of fresh, frozen plasma. Which of the following conditions are treated by this method? a. Hemophilia b. Essential thrombocytosis c. Drug-induced thrombocytopenia d. THrombotic thrombocytopenia purpura

(d) The etiology of thrombotic thrombocytopenia purpura is unknown. Removal of the plasma and replacement with fresh plasma has been shown to help. Hemophilia causes excessive bleeding and can be treated with blood transfusion. Essential thrombocytosis is treated with aspirin

A male patient is worried about the changes that will result from his vasectomy. Which of the following may the nurse correctly share? a. You will notice a significant reduction in the amount of your ejaculate b. You will NOT be able to ejaculate as frequently as before c. You will require testosterone supplementation d. You will ejaculate, however, your ejaculate will NOT contain sperm

(d) The purpose of a vasectomy is to prevent sperm from entering the ejaculate. You will notice a signficant reduction in the amount of your ejaculate. Ejaculation frequency is NOT altered and testosterone production is NOT disrupted by a vasectomy

A patient has orthopnea and difficult sleeping at night. Which of the following would be considered in relation to this disorder? a. Cor pulmonale b. Backward effects right side heart failure c. Forward effects left side heart failure d. Backward effects left side heart failure

(d) When the left side of the heart fails, fluid backs up into the lungs, causing breathing difficulties. Cor pulmonale is right side heart failure due to pulmonary issues. When the right ventricle fails, fluid backs up in the systemic veins. Forward effects of left side heart failure are decreased ejection fraction and blood pressure

A patient's right ankle systolic blood pressure is 90 mm Hg and right arm systolic blood pressure is 120 mm Hg. What is the ankle brachial index (ABI)

0.75 because the ankle brachial index is calculated by dividing ankle systolic blood pressure from arm systolic blood pressure on the same side of the body

A patient with congestive heart failure is given a diuretic to reduce excess fluid volume. That patient has lost 2 pounds. How many liters of fluid have been lost?

1 because 2 pounds of weight equal 1 L of fluid

The kidneys receive 20% of the cardiac output at rest. If cardiac output is 5 liters per minute, how much blood flow do the kidneys receive in liters per minute?

1 because to determine cardiac output to the kidney, multiply 5 lites/minute by 0.20

A man with type 1 diabetes mellitus is complaining of hunger, dizziness, and agitation. He thinks that he may be suffering from hypoglycemia. He has been counseled as to how to counteract hypoglycemia. Which of the following recommendations should he follow? a. Ingest 1 gram of fast-acting carbohydrates b. Inject rapid-acting insulin c. Ingest 15 grams of simple carbohydrates d.Ingest 30 to 35 grams of simple carbohydrates

15 grams is the recommended amount of carbohydrates initially for hypoglycemia. Insulin would worsen the hypoglycemia condition

The following values are on a laboratory report: - Sodium: 140 mEq/L - Chloride: 100 mEq/L - Bicarbonate: 22 mEq/L Please calculate the anion gap

18 because Anion gap is measure cations minus measured anions. Sodium 140 subtracts bicarbonate 22 plus chloride 100 which equals 18

A heart valve opens normally to 10 cm. A stenotic valve has reduced the opening by 80%. How open is the heart valve in centimeters?

2 because a 10 cm opening with an 80% reduction means an 8 cm reduction, which leaves a 2 cm opening

Which of the following would cause concern on a karyotype? a. 23 pairs of chromosomes b. 46 XX c. 47 XXY d. 46 XY e. 2 pair sex chromosomes f. 45 X0

47 XXY indicates an extra X chromosome, a human karyotype contains 1 pair of sex chromosome, and 45 X0 indicates a missing chromosome (c, e, f)

A patient has a blood pressure reading of 130/80 mm Hg. What is the pulse pressure?

50 because pulse pressure is calculated as systolic blood pressure minus diastolic blood pressure

On a patient's fluid intake and output sheet, the following fluid intakes for the day are listed: 8 ox tea and 10 oz water. How many milliliters (mL) of fluid did the patient consume on that day?

540 mL. Each ounce of fluid equals 30 mL. 18 ounces multiplied by 30 mL per ounce equals 540 mL

A patient weighs 120 pounds with 25% body fat. What is the patient's lean body mass in pounds?

90, lean body mass is calculated by multiplying body weight by percentage of body fat

A severely overweight individual is requesting to be considered for bariatric surgery. Which factors are considered MOST important in determining whether the patient is a good candidate for the surgery? a. Patient is female b. Patient has a body mass index (BMI) of 42 c. Patient read an article on the surgery last week and feels ready for the procedure d. Patient suffers from binge eating

A BMI of 40 or greater is one guideline for considering bariatric surgery. The surgery requires a thorough assessment of the patient before proceeding

A clinician is working with a patient with type 2 diabetes mellitus, and the clinician wants to evaluate the patient's insulin production. What test can be used? a. A1c b. C-peptide test c. Islet antibody test d. Oral glucose tolerance test

A C peptide test measures insulin production. A1c measures the glycation of red blood cells over time. An islet antibody test evaluates destruction of beta cells. An oral glucose tolerance test does NOT measure insulin levels directly

A male who engages in high-risk sexual practices, such as having anal intercourse with men, has been exposed several times to HIV but has NOT developed an infection. He believes that he has the "right genes" to avoid infection. Which genetic test would support this conclusion? a. Increased expression of CCR5 gene b. Increased reverse transcriptase expression c. Deletion of the HLA gene d. Lack of gene for CCR5 receptor

A CCR5 receptor is needed for HIV to infect a cell. Increased expression of CCR5 gene allows HIV to infect more cells. Reverse transcriptase is a protein produced from HIV genes and the HLA gene is NOT involved with direct HIV infection of cells

A nurse is discussing with a patient the placement of a Greenfield filter. Which is likely the patient's issue? a. Pulmonary embolism near the heart b. Lower extremity deep vein thromboembolism c. Varicose veins d. Abdominal constriction

A Greenfield filter prevents a clot from traveling through the circulatory system, usually a clot formed in the lower extremities

A clinician needs to know whether an infection is viral and wants to see whether lymphocyte levels are elevated. Which test would provide the information the clinician needs? a. Complete blood count (CBC) b. White blood count (WBC) total c. WBC differential d. Prothrombin time (PT)

A WBC differential will indicate changes in specific cell types, such as lymphocytes. A CBC and WBC total do NOT differentiate different types of WBCs. PT tests measure clotting time, NOT response to infection

A Well's criteria score of less than 2 and which of the following would rule out a deep vein thrombosis (DVT) in a patient the nurse is treating? a. Normal respirations b. No pain and swelling in the legs c. Elevated D-dimer levels d. Normal D-dimer levels

A Well's criteria of less than 2 and normal D dimer levels indicating no clot breakdown rule out a deep vein thrombosis (DVT). Normal respirations and a lack of pain and swelling in the legs do NOT rule out a DVT. Elevated D dimer levels would indicate formation and breakdown

A patient is complaining that she saw on her chart that she was listed as "obese". She disagrees with this assessment. Which of the following values of the patient do NOT place her in the obese category? a. Body mass index (BMI) of 32 b. Waist circumference of 38 inches c. 35% over ideal body weight d. Body fat percentage of 25%

A body fat percentage of 25% places the woman in the normal category. Women are considered obese if their body weight is 30% over ideal, waist circumference of over 35 inches, and a BMI of at least 30

Which of the following would be a possible treatment for the development of bronchoconstriction inflammation from asthma a. Bronchoconstriction agent b. Beta-2 receptor antagonist c. Bronchodilator agent d. Beta-blocker

A bronchodilators would reverse the constriction apparent in asthma. A beta 2 agonists, NOT antagonist, works for asthma. Beta blockers are NOT used to treat asthma

A nurse sees on a patient's chart the following: "cholecystectomy". Which of the following is the correct association for the nurse to make? a. The patient has gallstones b. The patient has gallbladder cancer c. The patient had his gallbladder removed d. The patient had bariatric surgery

A cholecystectomy indicates gallbladder removal

A patient is to receive a quadruple coronary artery bypass graft. What is the result of the procedure? a. Create a bypass around the four chambers of the heart b. Reconnect vessels to connect the heart chambers differently c. Use harvested vessels to circumvent four arterial blockages d. Place four stents in the heart arteries

A coronary bypass reroutes blood flow around a coronary artery occlusion

Which of the following has the greatest risk of fatal complications? a. Varicose veins b. Venous insufficiency c. Stasis dermatitis d. Deep venous thromboembolism

A deep vein thromboembolism can block blood flow to critical tissues

A patient has been diagnosed with a deep vein thrombosis (DVT). He would like to know what exactly this is. Which of the following responses by the nurse is correct? a. A DVT is a blood clot in the lungs b. A DVT is a blood clot which forms in a vein, normally in the lower extremities c. A DVT first develops in a major artery d. A DVT is nothing to worry about

A deep vein thrombosis (DVT) is a blood clot in a vein and usually occurs in the lower extremities. A DVT may travel to the lungs but is NOT necesarily in the lungs. It can also travel in the circulatory system and block blood flow

A volunteer at the hospital is required to complete a tuberculosis test (Mantoux test), which works on a Type IV hypersensitivity reaction. The person wants to know when to expect the resutls. What is an appropriate response ? a. Once we do the test, we will have your results immediately b. Your results will be available 20 minutes after the test c. The results are read in stages, from 30 minutes to 1 to 2 hours d. We will complete the test and review the results after 48 hours has passed

A delayed hypersensitivity reaction usually takes 24 hours or more to develop

A discharge nurse is speaking to a patient who received a prescription for a diuretic for heart failure. Which of the following would be appropriate for the nurse to state? a. This drug will help your heart beat more forcefully b. This drug will make you have to go to bathroom more frequently c. Do NOT take this drug if you're planning to drive d. This drug will help your body to retain fluid

A diuretic increases urine output, reducing fluid load on the heart. Diuretics do NOT affect one's ability to drive but it will lessen fluid retention

A patient states that, ever since he has had heart failure and was given a medication he has to "pee more". Which of the following is a correct explanation from the nurse? a. The medication increases your urine output to reduce the fluid load on your heart b. The heart failure must have damaged your kidneys c. The medication must NOT be working. With heart failure, you want to retain as much fluid as possible d. Heart failure causes constant thirst, so you are drinking and urinating more

A diuretic will increase urination because it reduces fluid overload on the heart

A patient's rapid streptococcal test is negative. However, the nurse informs the patient that they will culture a sample and call later to confirm the results. What is the following reason for this process? a. The rapid streptococcal test has a high percentage of false positivies b. We never give confirmatory results on any test until at least 48 hours has transpired c. The rapid streptococcal test requires a 48 hour waiting period d. False negative can occur with the rapid streptococcal test

A false negative is when a test result is negative but is incorrect. By culturing the throat sample, the rapid test results can be confirmed. The rapid streptococcal test has a high percentage of false negative. Streptococcal rapid test can be provided within the office visit, it does NOT require 48 hours for results

A complication of Crohn's disease, a disorder of the gastrointestinal tract, a fistula formation. Because of this complication, what development may the nurse expect? a. A narrowing of the colon lumen b. An abnormal connection between the bowl and other tissues c. A growth adhering two portions of the colon together d. Rupture and prolapse of the colon

A fistula is an abnormal connection between organs. A narrowing of the colon would be a stricture. An adhesion is when two portions of tissues adhere to one another

A patient with severe hypertension is undergoing a fundoscopic examination. Which tissues are being evaluated? a. Retina b. Glomeruli c. Coronary arteries d. Extremity circulation

A fundoscopic examination evaluates the retina

A nurse is concerned about the poor glycemic control of a patient and the complaints of blurred vision by the patient. Which of the following is recommended? a. Fundoscopic examination b. Change in corrective eye wear c. LASIK surgery d. Cataract surgery

A fundoscopic examination will enable an examination of any retinal changes. The eye should be examined first before any treatment begins

A clinician has ordered an assessment of alpha-antitrypsin (AAT)-1 levels. The values came back much lower than normal. What condition may the patient have increased risk of developing? a. Pulmonary fibrosis b. Chronic obstructive pulmonary disease (COPD) c. Asthma d. Pulmonary hypertension

A genetic disorder that lowers alpha-antitrypsin (AAT)-1 levels can result in increased risk for chronic obstructive pulmonary disease (COPD)

A child is noted to have failure to thrive and frequent infections despite routine vaccinations. The clinician suspects hypogammaglobulinemia. Which diagnostic result would support this diagnosis? a. Lack of umunoglobulin As (IgAs) b. Normal IgG levels, with reduced IgM levels c. Lack of B and T cell development d. IgG levels below the fifth percentile for age

A hallmark of hypogammaglobulinemia is very low IgG levels. In hypogammaglobulinemia, T cells and IgAs are present and IgG levels are low

A nurse is working with a child who has been diagnosed with celiac disease. The child will be attending a birthday party. What is an appropriate food choice for the child? a. Hamburger patty without the bun b. Ham sandwich on sourdough bread c. Pasta salad d. Cake and/or cookies as long as frosting is removed

A hamburger without the bun is an acceptable choice because it does NOT contain gluten. Sourdough bread, pasta salad, cakes and cookies all likely contain flour, which contain gluten

A patient mentions that bladder cancer runs in his family. Which signs and symptoms may appear in this cancer? a. Blood in the urine with severe pain upon urination b. Increased urge to urinate c. No pain with urination, but blood appears in urine d. Urinating up to 50 times per day

A key sign of bladder cancer is the appearance of blood in urine without pain

A patient is scheduled for a manometry test. How would the nurse explain the purpose of this procedure to his patient? a. The procedure measures pressure at the junction between the stomach and esophagus b. The procedure measures pressure at the base of your stomach c. The procedure measures stomach acid levels d. The procedure measures bacterial levels in the stomach

A manometry test measures the pressure at the end of the esophagus and beginning on the stomach. It can be used in assessment of gastroesophageal reflux disease (GERD)

A patient is asking a nurse whether the human body has unlimited ability to store fat. The nurse replies that adipocytes undergo hyperplasia and hypertrophy to accommodate excess calorie consumption. Which of the following is the BEST way for the nurse to have conveyed this information at an appropriate level for a patient? a. Adipocytes store excess fuel in the form of triglycerides b. Fat cells undergo increase in number and size in order to store excess fuel c. Fat cells increase in number, NOT size, over time d. Fat cells only increase in size, NOT number

A more appropriate way to convey the idea of hyperplasia and hypertrophy to discuss them in terms of increase in cell number and size

A patient has suffered a massive myocardial infarction. What are some common complications of postmyocardial infarction that may develop? Select all that apply a. Papillary muscle rupture b. Conduction disturbances c. Pneumonia d. Skin rash e. Heart murmur

A myocardial infarction may disrupt papillary muscle function and electrical signaling in the heart. A heart murmur may develop because the valve structures are disrupted. Pneumonia and skin rashes are NOT common complications after a myocardial infarction (a, b, e)

Nurse is reviewing a patient's medications to see whether a negative chronotropic agent has been prescribed. Which of the following will the nurse be looking for based on this information? a. Reduced heart rate b. Increased respiratory rate c. Increased blood pressure d. Increased edema

A negative chronotropic agent reduces heart rate

A patient's glomerular filtration rate (GFR) is 100 ml/min. Which of the following interpretations is correct? a. Renal function is normal b. The value is too high; the kidneys are NOT concentrating the urine c. This value is too low; the patient is retaining fluids d. The value is too low due to decreased renal perfusion

A normal renal function occurs with a GFR greater than 90 ml/min, so the patient's value would enable normal renal function

A nurse is working with a patient diagnosed with metabolic alkalosis. Which of the following values support this diagnosis? a. pH of 7.2, bicarbonate of 22 mEq/L b. pH of 7.6, bicarbonate of 46 mEq/L c. pH of 7.2, bicarbonate of 46 mEq/L d. pH of 7.6, bicarbonate of 22 mEq/L

A pH of 7.6 indicates alkalosis due to an elevated bicarbonate level. A pH of 7.2 is acidic

A patient is to receive a partial bone marrow ablation. How should the nurse explain this procedure? a. All your bone marrow will be physically removed and replaced with donor marrow b. Chemotherapy will destroy all of your bone marrow c. Your bone marrow will be removed while activation agents are given to stimulate new bone marrow d. A portion of your bone marrow will be destroyed and replaced with healthy bone marrow stem cells

A partial ablation destroys only a portion of the cells and these cells are replaced by healthy stem cells from a donor

A patient arrives in emergency department complaining of severe abdominal pain and demanding immediate pain medication. What is the appropriate response by the nurse? a. Ask the patient whether he has ever had pain medications before and whether he is allergic to any pain medications b. Ask the patient which type of pain medications he prefers c. Tell the patient that for proper assessment, pain medications may be withheld until a diagnosis can be made d. Tell the patient to stop asking. The clinician will prescribe pain medications when he or she feels it is necessary

A patient's report of pain can provide important information for a clinician. Withholding pain medications until assessment is complete may be appropriate

Which of the following hormones are NOT altered in pheochromocytoma? a. Cortisol b. Aldosterone c. Epinephrine d. Norepinephrine e. Adrenal androgens

A phenochromocytoma is a tumor of the adrenal medulla. Cortisol, aldosterone, and adrenal androgens are made by the adrenal cortex. Epinephrine and norepinephrine are affected by pheochromocytoma

A patient with a traumatic brain tumor suffered a pituitary apoplexy. Which of the following does the nurse expect to encounter? a. Patient with limited signs and symptoms b. Blood laboratory values revealing elevated growth hormone-releasing hormone (GHRH), thyrotropic releasing hormone (TRH), and corticotropin-releasing factor (CRF) c. Hyperfunctioning of thyroid and adrenal gland d. Increased adrenocorticotropic hormone ACTH) and thyroid-stimulating hormone (TSH) blood levels

A pituitary apoplexy results in pituitary dysfunction. Hypothalamic hormones, growth hormone-releasing hormone (GHRH), thyrotropic releasing hormone (TRH), and corticotropin-releasing factor (CRF), would increase in an attempt to stimulate the pituitary gland Pituitary apoplexy would disrupt stimulatory signals to thyroid and adrenal gland. THe pituitary gland would NOT be able to synthesize adrenocorticotropic hormone ACTH) and thyroid stimulating hormone (TSH)

Please place the following in order leading to red blood cell formation - Erythroblast - Reticulocyte - Normoblast - Mature erythrocyte - Pluripotent stem cell

A pluripotent stem cell in the bone marrow becomes and erythroblast. An erythroblast becomes a normoblast which then becomes a reticulocyte, immature red blood cells, and then a mature erythrocyte 77

A patient has suffered a pneumothorax. His parents wonders what this means. What terms may the nurse use to explain the condition that the parents may be familiar with? a. This condition is similar to asthma b. It means he suffered a collapsed lung c. It means he developed an inflammatory autoimmune disease in the lungs d. This is a severe form of pneumonia

A pneumothorax is a collapsed lung

A nurse receives the results of a potassium hydroxide saline-wet mount smear. The results are positive. Which of the following is a correct association for the nurse to make? a. The patient is suffering from a Trichomonas vaginalis infection b. The patient is suffering from a syphilis infection c. The patient is suffering from a Candida albicans infection d. The patient has endometriosis

A potassium hydroxide saline wet mount is a test for fungal infections. Candida albicans is a fungal infection

A single PVC appears on a patient ECG. Which is the appropriate interpretation by the nurse? a. State that the patient is entering cardiac arrest b. Recognize that the premature ventricular contraction (PVC) represents a premature contraction by the ventricles c. The problem is in the sinoatrial (SA) node of the heart d. Realize that atrial fibrillation is likely

A premature ventricular contraction (PVC) develops when the ventricles contract out of sequence. A single PVC does NOT indicate cardiac arrest. PVC develops from ventricular activity, NOT the SA node

In a ventilation-perfusion scan, a decreased perfusion mismatch will most likely be seen with which of the following conditions? a. Pulmonary embolism b. Pulmonary hypertension c. Idiopathic pulmonary fibrosis d. Pneumoconiosis

A pulmonary embolism can block arterial flow, disrupting ventilation perfusion matching. Ventilation perfusion is NOT normally disrupted by pulmonary hypertension, idiopathic pulmonary fibrosis, or pneumoconiosis

Which of the following is indicative of a pyrogenic response? a. Body temperature of 35.0 Celsius b. Body temperature of 37.0 Celsius c. Body temperature of 37.5 Celsius d. Body temperature of 39.0 Celsius

A pyrogenic response is a fever which would be indicated by a body temperature of greater than 37.0 Celsius

Arterial blood gases initially reveal pH of 7.50. Over time, the pH is reduced to 7.44. Which of the following would describe the pulmonary compensation that developed? a. Ventilation slowed, enabling acidosis to develop b. Ventilation slowed, enabling pH to return to normal c. Ventilation increased, enabling pH levels to drop d. Ventilation increased, enabling acidosis to develop

A reduced ventilation resulted in carbon dioxide retention, lowering pH to normal. Acidois did NOT develop and an increase in ventilation increases pH

A report reads "Increased PMNs, shift to the left". Which of the following would be proper interpretations by the nurse? a. A chronic autoimmune condition is present b. Basophil levels are elevated c. Immature neutrophil levels are elevated d. An increase in monocyte conversion to macrophages e. Acute infection

A shift to the left indicates immature neutrophils responding to an acute event. PMNs are neutrophils and elevate during acute inflammation

A nurse overhears a laboratory technician speaking about a patient's WBCs, showing a shift to the left. What is the best way to correctly interpret these results? a. The results show an increase in neutrophils b. The total number of WBCs has increased c. The results show an increase in immature neutrophils, indicating an acute infecttion d. The results show an increase in mature neutrophils, indicating a chronic infection

A shift to the left indicates increased immature neutrophils, a common sign for acute infection

A surgery is scheduled to repair a bile duct stricture. Which of the following would be an appropriate explanation for a nurse to make to the patient? a. "This surgery will remove gallstones" b. "This surgical procedure will help stop internal bleeding" c. "The surgical procedure will help widen an abnormal narrowing" d. "This surgical procedure will remove an abnormal connection that has developed between the gallbladder and liver "

A stricture is an abnormal narrowing of a structure. No internal bleeding is mentioned and an abnormal connection between tissues is a fistula

A patient presents in the emergency department with a thyrotoxic crisis. Which of the following are correct associations and actions for the nurse to make in this situation? a. Work to calm the patient down. Thyroid hormone levels will decrease with time b. Have the patient more vigorously. Thyroid hormone levels are too low c. Expect to find a calm, complacent patient who does NOT recognize the severity of the situation d. Recognize that this a medical emergency and treatment is needed immediately

A thyrotoxic crisis is a medical emergency that is caused by elevated thyroid hormones increasing heart rate and blood pressure. Management of the elevated thyroid hormone levels is needed immediately. In a thyrotoxic crisis, thyroid hormones are elevated and it will place the patient in a state of distress

A client reports persistent cough. The nurse suspects the client may have developed tuberculosis. Which diagnostic test does the nurse expect the primary health care provider to prescribe to confirm this condition? a. Electrocardiogram b. X-ray of the lungs c. Histological examination d. Both X-ray and histological examination are correct

A tubercle is a chronic inflammatory granuloma where the region of infection is infiltrated with macrophages. It can be identified through histological examination. X-ray can also be used as a diagnostic test

A urea breath test is being conducted on a patient with a gastric ulcer. The results are positive. Which organism is present? a. Clostridium difficile b. Escherichia coli c. Helicobacter plyori d. Candida albicans

A urea breath test is for the presence of Helicobacter pylori because it contains a urease enzyme

A patient is recommended for a urodynamic study. Which of the following pieces of information can be learned from this study? Select all that apply a. Functioning of the bladder b. Postvoid volume c. Pyelonephritis d. Stones in the ureters e. Bladder filling and emptying

A urodynamic study can assess how well the bladder functions and empties and the amount of urine remaining in the bladder after voiding (a, b, e)

A nurse is reviewing how a urologic obstruction may affect kidney function. Please place the events in the proper sequence - Increased pressure in nephrons - Decreased glomerular filtration rate (GFR) - Increased pressure in Bowman's capsule - Urological obstruction - Increased pressure in ureter

A urological obstruction will increase fluid pressure in the ureter. This can increase pressure in the nephron, which will increase pressure in Bowman's capsule and decrease GFR ( Urological obstruction - Increased pressure in ureter - Increased pressure in nephrons - Increased pressure in Bowman's capsule - Decreased glomerular filtration rate (GFR))

If normal plasma volume is 290 mOsm, which of the following patient osmolarity values may indicate dehydration? a. 270 mOsm b. 280 mOsm c. 290 mOsm d. 300 mOsm

A value of 300 mOsm shows greater concentration of the plasma versus normal, indicating dehydration. A value of 290 mOsm would be isotonic, anything lower would be considered hypotonic

A clinician is palpating a male patient's scrotum and notices a "bag of worms" appearance. Which of the following is most likely? a. Cryptorchidism b. Hematocele c. Hydrocele d. Varicocele

A varicocele is enlarged veins in the scrotum. They can appear worm like. Cryptorchidim is failure of one or both testes to descend into the scrotum. Hematocele is a collection of blood in the scrotal sac. Hydrocele is collection of fluid in the scrotal sac

A patient has a genetic defect that has resulted in malformed A-delta nociceptors. Which of the following is the patient likely to experience? a. Reduced ability to detect acute, painful stimuli b. Reduced ability to detect dull, persistent, chronic pain c. Inhibition of the spinothalamic signal d. Disruption of the nucleus raphe magnus (NRM)

A-delta nociceptors detect acute pain. A malformation in these receptors would limit the ability. Type C, NOT A-delta, detects dull chronic pain

A patient with diabetes states that he feels fine. A fasting blood glucose level reveals a slightly higher than normal value. The clinician wants to evaluate glycemic control over the preceding months. Which test will be most helpful? a. Oral glucose tolerance test b. Islet cell antibody test c. C-peptide test d. A1c test

A1c levels evaluate glycemic control over a three month period. An oral glucose tolerance test is used to evaluate glycemic control following ingestion of glucose. An islet cell antibody test is used to evaluate antibody attack of B cells. A C peptide test is used to measure insulin levels (d)

Hypoxia may limit a cell's ability to generate adenosine triphosphate (ATP). Which of the following cellular disruptions may occur when ATP levels fall? a. Decreased protein synthesis b. Decreased lactic acid levels c. Increased plasma membrane integrity d. Increased endoplasmic reticulum activity

ATP is necessary for protein synthesis. Without adequate ATP, the plasma membrane may be compromised and the cell must use anaerobic metabolism that leads to increased lactic acid production. Endoplasmic reticulum activity is reduced when ATP levels are decreased

Which of the following does a clinician know to be untrue about acalculous cholecystitis? a. Acalculous cholecystitis is much rarer than calculous cholecystitis b. Acalculous cholecystitis is due to stones in the gallbladder c. Acalculous cholecystitis is best treated by a wait-and-see approach because it normally resolves on its own d. Acalculous cholecystitis caused by infection is more serious than calculous cholecystitis

Acalculous cholecystitis requires treatment

Reviewing a patient's chart, a nurse notes the presence of acromegaly. Which of the following is likely to be present when the nurse assesses the patient? a. Increased stature b. Cold intolerance c. Overgrowth of jaw, hands d. Galactorrhea

Acromegaly is increased growth hormone in adulthood. Increase growth hormone in childhood can result in increased stature, low thyroid hormone levels can result in cold intolerance, and elevated GH does NOT cause galactorrhea

For which of the following disease is the causative agent likely to be a virus? Select all that apply a. Tuberculosis b. Bronchitis c. Strep throat d. Acute pharyngitis e. Rhinitis

Acute bronchitis, acute pharyngitis, and rhinitis are often caused by viruses. Tuberculosis and strep throat are caused by bacterias (b, d, e)

A patient is diagnosed with acute bronchitis. Which of the following is the most common cause of this disorder? a. Genetic defect b. Virus c. Bacteria d. Fungus

Acute broncitis is most commonly caused by a viral infection

A nursing student is studying the progression of acute kidney injury. What is the proper order for the progression of acute kidney injury? - Diuresis with reduced concentrating ability by kidney - Initial insult or injury to the kidney - Recovery with return to normal urine concentration - Kidney function decreases - Oliguria develops

Acute kidney injury begins with some type of insult or injury, leading to decreased kidney function. This causes oliguria, at first. This is then followed by a time of diuresis, and finally, a return of the kidney's ability to concentrate urine ( Initial insult or injury to the kidney - Kidney function decreases - Oliguria develops - Diuresis with reduced concentrating ability by kidney - Recovery with return to normal urine concentration)

A nurse learns that a young child, under age 8, has been diagnosed with leukemia. Which type of leukemia is a child most likely to have been diagnosed with? a. Acute myelogenous leukemia b. Chronic lymphocytic leukemia c. Acute lymphocytic leukemia d. Chronic myelogenous leukemia

Acute lymphocytic leukemia is more common in children. Chronic lymphocytic leukemia is more common in elderly individuals. The median age of diagnosis for chronic myelogenous leukemia is 50 years

A patient has short bowel syndrome. What are three phases of this disorder in order? a. Adaptation, acute, maintenance b. Maintenance, adaptation, acute c. Acute, adaptation, maintenance d. Adaptation, maintenance, acute

Acute, adaptation, and maintenance are three phases, in order, of short bowel syndrome

A patient presents with hyperkalemia. When a nursing student is asked the possible causes of elevated potassium, which of his responses are correct? a. Addison's disease b. Diuretic therapy c. Metabolic acidosis d. Laxative abuse e. Renal failure

Addison's disease causes reduced aldosterone levels which in turn results in reduced potassium secretion. In metabolic acidosis, H+ ions shift into the cells and K+ out of the cells into the plasma. When the kidneys fail, potassium secretion declines, leading to elevated potassium in the serum

Which value decreases in cases of metabolic acidosis caused by addition of acidic substances, such as ketones a. Repolarization b. Depolarization c. Threshold

Addition of acids will be buffered by bicarbonate, lowering bicarbonate levels and increasing the anion gap (c)

A patient is given an adrenocorticotropic hormone (ACTH) stimulation test. Which of the following results will confirm hypofunctioning of the adrenal gland? a. Elevated ACTH b. Suppressed corticotropin releaseing factor c. Cortisol levels less than expected d. Suppressed thyroid hormone levels

Adrenocorticotropic hormone (ACTH) should increase cortisol levels. If cortisol does NOT increase with ACTH administration, then the adrenal glans are underperforming. Administration will increase ACTH levels and suppress corticotropin releasing factor levels but it does NOT provide information about adrenal cortex functioning. Thyroid hormones are NOT altered by ACTH

Which of the following patients will a nurse most likely suspect to be most at risk for gallstone formation? a. A child under the age of 5 years b. Overweight female, age of 48 years c. Female, age 25 d. Elderly male, age of 70 years

Age over 40 years, being female, and being overweight increases the risk for gallstones

Which of the following choices show the correct sequence for Han Selye's stress response? a. Exhaustion, alarm, resistance b. Resistance, alarm, exhaustion c. Alarm, resistance, exhaustion d. Alarm, exhaustion, resistance

Alarm, resistance, exhaustion

Which of the following in a patient with type 2 diabetes mellitus (DM) may alert the nurse to diabetic nephropathy? a. Increased need to urinate b. Microalbuminuria c. Pain with urination d. Lack of pain sensation in legs

Albumin in the urine can indicate glomerular damage due to diabetic nephropathy. Polyuria can present with DM, however it does NOT indicate nephropathy. Pain with urination and is NOT a sign of diabetic nephropathy. Lack of pain sensation is a sign of neuropathy, NOT nephropathy

A patient with type 2 diabetes mellitus (DM) receives urinalysis results that are positive for albumin. What does the nurse suspect? a. The patient is dehydrated b. The patient has entered renal failure c. The patient may be developing diabetic nephropathy d. The patient has a kidney stone

Albumin in the urine may indicate glomerular damage, which occurs with diabetic nephropathy. Dehydration and kidney stones do NOT cause albumin to appear in the urine. Albumin in the urine does NOT indicate renal failure

To accurately assess calcium levels, which additional laboratory value will the nurse also evaluate ? a. Potassium b. Sodium c. pH d. Albumin

Albumin is a plasma protein. Because calcium is highly protein bound, changes in albumin levels can alter calcium levels

A patient has developed chronic pancreatitis after several bouts of acute pancreatitis. Which of the following place the patient at greatest risk for this development? a. Age b. Low-fiber diet c. Alcohol abuse d. High-fat diet

Alcohol abuse is a primary risk factor for pancreatitis

A nurse is counseling a patient with pancreatitis about lifestyle changes. Which changes should the patient consider? a. Reduce or eliminate alcohol intake b. Increase high fat foods in diet for caloric reasons c. Restrict fluid intake so as NOT to overhydrate d. Eliminate carbohydrate intakes as much as possible

Alcohol intake should be severely restricted in cases of pancreatitis. High fat food would NOT be beneficial for pancreatic functioning, fluid restriction should NOT be utilized, and carbohydrates should NOT be eliminated from this diet

Brian is to undergo major abdominal surgery. As his nurse, you are worried because Brian has a tumor of the adrenal cortex, which reduces his ability to respond to stress. Which hormones are directly affected by Brian's tumor? a. Antidiuretic hormone (ADH) b. Norepinephrine c. Cortisol d. Aldosterone e. Epinephrine

Aldosterone and cortisol are secreted from the adrenal cortex. Epinephrine and norepinephrine are secreted by the adrenal medulla and ADH is secreted by the posterior pituitary

A mother is wondering how her son developed asthma, as everyone in the family is in "good health". Which of the following would be correct for the nurse to state is the number one cause of asthma? a. Viral infection b. Improper fetal lung development c. Genetic defect d. Allergies

Allergies are the number one cause of asthma. Fetal lung development can contribute to asthma, but is NOT the number one cause. Genetics can play a role in allergies, but a specific genetic defect has NOT been detected

Please place the allergic response steps in the proper order - B cell activated - Immunoglobulin Es (IgEs) produced - Inflammatory mediators released - Exposure to allergen - Mast cells degranulate

Allergies begin with exposure to the allergen, which activates B cells, which causes IgEs to be produced. Mast cells then degranulate and release inflammatory mediators ( Exposure to allergen - B cell activated - Immunoglobulin Es (IgEs) produced - Mast cells degranulated - Inflammatory mediators released)

Please place the allergic response steps in the proper order - Exposure to allergen - Inflammatory mediators released - Mast cells degranulate - B cells activated - Immunoglobulin Es (IgEs) produced

Allergies begin with exposure to the allergens, which activates B cells, which causes IgEs to be produced. Most cells then degranulate and release inflammatory mediators ( Exposure to allergen - B cells activated - Immunoglobulin Es (IgEs) produced - Mast cells degranulate - Inflammatory mediators released)

While taking a patient's history, the nursing student learns that the patient has been under extreme stress at school and has suffered repeated infections of the common cold. The nursing student knows that prolonged stress may produce wear and tear on the body' systems. What term describes the nursing student's understanding? a. Alarm stage of stress b. Eustress c. Allostasis d. Allostatic load

Allostatic load is the wear and tear on the body due to repeated stressors. Allostasis is a dynamic state of balance, the patient's constant colds indicate that this state of balance is lacking

A nurse is completing a quick scan of a patient's laboratory results. Which of the following would be most indicative of acute pancreatitis? Select all that apply a. Increased serum albumin b. Increased serum amylase c. Increased serum lipase d. Increased aspartate aminotransferase (AST) e. Increased bilirubin

Amylase and lipase are digestive enzymes made by the pancreas, which will elevate with pancreatitis. Albumin levels would NOT be a primary indicator of pancreatitis. Increased aspartate aminotransferase (AST), or bilirubin levels are indicative of liver dysfunction

A nurse sees an anergy panel set of results revealing no reaction. She knows she will have to speak to the patient about which of the following scenarios? a. Elevated antibiotic response indicating autoimmune disease b. Severe allergies c. Increased immunocompetence d. Increased risk of infection

An NRG panel measures antibody levels. With inadequate antibody production, there is an increased risk for infection. An energy panel does NOT show antibody production nor does it set evaluate allergies. If antibody production is NOT adequate, immunodeficiency is present

A nurse educator hears a S3 gallop sound in a patient. She asks a nursing student to listen for the sound. What sounds will the nursing student expect to hear? a. Systolic murmur b. Vibration of ventricle wall during filling c. Murmur increasing in pitch and grade during systole d. Murmur with crescendo shape

An S3 gallop develops when blood fills the ventricular wall, causing detectable vibrations (b)

A clinician needs to assess whether a patient has active tuberculosis. Which test would confirm this diagnosis? a. Positive Mantoux test b. Positive purified protein derivative test c. Tubercle on an x-ray d. Positive acid fast stain of sputum

An acid fast test would indicate organisms in the sputum. A positive Mantoux test and a positive purified protein derivative test, and tubercles on an x raydoes NOT indicate active infection or NOT

In a patient suspected of having an endocrine disorder, which of the following diagnostic options may be considered first? a. Immunoassay of blood hormones levels b. Computed tomography (CT) scan c. Magnetic resonance imaging (MRI) d. Antibody testing

An immunoassay of blood hormone levels is usually the first step in endocrine assessment. A CT scan and an MRI can be used as an assessment in some endocrine disorders, but a blood test is normally done first. Antibody testing, if needed, is usually done after testing blood hormone levels

Please place in the proper order the processes leading to the development of jaundice - Increased bilirubin levels - Increased biliverdin levels - Increased red blood cell (RBC) breakdown - Increased porphyrin levels - Accumulation of bilirubin in skin

An increase in RBC breakdown will increase porphyrin levels, which are converted to biliverdin. Biliverdin is then converted to bilirubin, which may lead to jaundice, the accumulation of bilirubin in the skin

An infection of hepatitis D virus requires which of the following for replication and expression a. hepatitis A virus b. hepatitis B virus c. hepatitis C virus d. hepatitis E virus

An infection with hepatitis B is required for a hepatitis D infection

Which is an inherited connective-tissue disorder? a. Familial hypercholesterolemia b. Marfan syndrome c. Familial adenomatous polyposis d. Neurofibromatosis

An inherited connective tissue disorder transmitted as an autosomal dominant trait is known as MARFAN SYNDROME

A trauma patient is experiencing a decrease in urine output. The patient has experienced significant blood loss. Which is one factor that accounts for the decrease in urine output? a. Decreased aldosterone secretion b. Suppression of the renin-angiotensin-aldosterone system (RAAS) c. Decreased sodium and water reabsorption d. Increased antidiuretic hormone (ADH)

Antidiuretic hormone (ADH) is released in response to decreases in blood volume. With blood loss, there is increased aldosterone, the RAAS is activated, and sodium and water reabsorption increases

A patient needs a screening of anterior pituitary hormone functioning. Which hormone should NOT be included in the evaluation? a. Antidiuretic hormone b. Thyroid stimulating hormone (TSH) c. Growth hormone d. Follicle-stimulating hormone

Antidiuretic hormone is produced by the posterior pituitary. Thyroid stimulating hormone, growth hormone, and follicle stimulating hormone are produced by the anterior pituitary

A nurse sees that an emergency surgery has been scheduled for an arterial disorder. Which of the following may most likely require this intervention? a. Peripheral arterial disease b. Temporal arteritis c. Aortic dissection d. Intact abdominal aortic aneurysm

Aortic dissection is a medical emergency. Peripheral arterial disease and abdominal aortic aneurysm are NOT medical emergencies in most cases. Temporal arteritis is NOT a medical emergency

A nurse learns that an improperly functioning heart valve has led to a patient's pulmonary edema. Which valve disorder does she suspect? a. Pulmonic valve insufficiency b. Pulmonic valve stenosis c. Tricuspid stenosis d. Aortic stenosis

Aortic stenosis would cause fluid backup on the left side of the heart and pulmonary vessels. The pulmonic valve and the tricuspid valve is on the right side of the heart and the patient is developing left side heart issues

While reading a medical journal article, you learn that HIV infection increases the rate of apoptosis in CD4 cells. What can you likely expect to find in laboratory values a. An increase in CD4 levels with patient infected by HIV b. An increase in CD4 cell size c. Lower than normal CD4 levels will be present with HIV infection d. CD4 mitotic rate will be elevated

Apoptosis is programmed cell death. If apoptosis rates are increased, CD4 levels will decrease

A fundoscopic exam is being conducted on a patient with poorly controlled hypertension. Which of the following is the clinician expecting to find? a. Cotton wool spots b. Retinal detachment c. Arteriovenous nicking d. Blood vessel hyperproliferation

Arteriovenous nicking may develop in retinal vessels in cases of hypertension. Cotton wool spots and blood vessel hyper profliferation tend to present in diabetic retinopathy

A patient is vomiting blood. Which of the following values would support the nurse's concerns about an upper gastrointestinal (GI) bleed (UGIB)? a. Hypertension b. Lower than normal heart rate c. Decreased hematocrit d. Elevated white blood cell (WBC) count

As a patient vomits blood, hematocrit levels may drop due to cell loss. Hypotension and elevated heart rate more commonly occurs with vomiting and bleeding. Elevated WBC would NOT support the conclusion of an upper gastrointestinal (GI) bleed (UGIB)

A young child is rushed to the emergency department having ingested a large amount of bicarbonate. Which acid-based imbalance does the nurse expect to be present? a. Respiratory acidosis b. Respiratory alkalosis c. Metabolic acidosis d, Metabolic alkalosis

As bicarbonate is a base, ingestion of large amounts will cause metabolic alkalosis

A patient with type 1 diabetes mellitus is going to begin an exercise program. Which of the following advice from the nurse is appropriate for the patient to avoid hypoglycemia? a. Make sure to take your insulin right before you begin to exercise b. Restrict carbohydrates before and during exercise c. If possible, work out with a partner who can recognize the signs off hypoglycemia d. If your workout is over an hour, be prepared to give yourself insulin half-way through the workout

As exercise moves glucose into cells, hypoglycemia may develop. working out with a partner who recognizes the signs and symptoms of hypoglycemia can help. Insulin plus exercise may move glucose into the cells too rapidly and carbohydrates should be available

Statins are a class of medications that lower cholesterol. A patient wants to stop taking this medication, but you inform him that this may NOT be possible because of his genetic disease. For which of the following conditions may statins be standard treatment? a. Familial adenomatous polyposis b. Cystic fibrosis c. Familial hypercholesterolemia d. Klinefelter syndrome

As familial hypercholesterolemia causes elevated low density lipoprotein (LDL) levels, statins are often a treatment option. Familial adenomatous polypossi involves colon polyps, cystic fibrosis affects the lungs and pancreas, and Klinefelter syndrome is a chromosomal abnormality of XXY, leading to problems with testosterone production, none involving cholesterol levels

Following the development of left ventricular failure, a patient develops pulmonary edema. Which of the following factors accounts for the formation of this edema? a. Increased hydrostatic pressure in the pulmonary capillaries b. Increased oncotic pressure in the pulmonary capillaries c. Decreased hydrostatic pressure in the pulmonary capillaries d. Hopelessness r/t perceived lack of support and ability to change the present situation e. Explains to visitors everything is okay, he really was fortunate it wasn't worse than it could have been, "he will be fine", tells everyone to go home (doesn't want visitors now)

As fluid backs up due to heart failure, the hydrostatic pressure in the capillaries increase. When the left heart fails, fluid back up into the pulmonary capillaries. Oncotic pressure is due to albumin levels in the blood

Naloxone is being used for a patient. Which of the following acute manifestations should the clinical team be aware may develop? a. Stomach ulcer formation b. Opioid withdrawal symptoms c. Rapid decrease in heart rate (HR) and blood pressure (BP) d. Patient develops sense of euphoria and hallucinations

As naloxone blocks opioid receptors, it will induce opioid withdrawal symptoms. Naloxone does NOT cause acute stomach ulcers, decrease heart rate or blood pressure, nor induce a state of euphoria

Which of the following is true regarding pancreatic cancer? a. Identified with alpha-fetoprotein (AFP) marker b. High survival rate c. Jaundice is often the first sign d. Present with severe acute symptoms

As pancreatic cancer presents with few signs and symptoms early on jaundice may be the first sign. Alpha fetoprotein (AFP) is a marker for hepatocellular cancer. Pancreatic cancer often develops without signs and symptoms and it does NOT have a high survival rate.

A female patient is worried about developing a deep vein thrombosis (DVT), as she recently witnessed her mother suffer from a DVT. As the clinician, which of the following recommendations may reduce the patient's risk for DVT? a. Take a prescription oral contraceptive b. Stop smoking c. Increase the amount of sitting at work d. Refrain from use of anticoagulants

As smoking increases the risk of clots, smoking cessation is good advice. Oral contraceptives and lack of activity increases the risk for clot formation. Anticoagulants would decrease clot formation

For which of the following conditions may a patient need digestive enzyme replacement? a. Diverticulitis b. Pyloric stenosis c. Large bowel obstruction d. Chronic pancreatitis

As the pancreas synthesizes digestive enzymes, pancreatic damage may result in the need for replacement enzymes

What would be a correct statement by a nurse working with parents who have given birth to a child with an inheritable disease and ask the following: "We, and two of our children, are fine. so why does our other son have this disease?" a. "You both have an autosomal dominant form of the disease" b. "The inherited disease requires one copy of the defective gene" c. "Your son with the illness inherited two copies of the gene for this disease" d. "Your children who do NOT have the disease both inherited two normal copies of the gene"

As this is a recssive disease, the affected son has inherited two copies. As the parents do NOT have the disease, it is NOT autosomal dominant

A nurse learns that a colleague was recently diagnosed with trigeminal neuralgia. When the nurse sees her colleague, which of the following behaviors can the nurse expect in her friend? a. A slow, halting gait b. Difficulty rising from a chair c. Caution when eating, chewing, and talking d. Complaints by her friend about all of her "tender points"

As trigeminal neuralgia causes facial pain, sufferers may use caution when doing activities that require facial muscle involvement. Trigeminal neuralgia does not affect gait or the lower extremities. Tender points are found in fibromyalgia, NOT trigeminal neuralgia

A nurse is questioning a patient about alcohol intake. Which of the following is appropriate? a. The nurse should assume such self reports are accurate b. The nurse should confront the patient if it is believed that the patient is NOT being honest c. The nurse should present such questioning without judgment to facilitate open discussion d. The nurse should assume that the patient is NOT telling the truth and stop asking further questions

Asking patients about alcohol intake should be handled without judgment

To reduce the risk of a coronary event, a patient informs the nurse that he is taking an aspirin daily. By taking this aspirin, which of the following processes are altered? a. Blood vessel dilation b. Coagulation factor synthesis c. Fibrinolysis d. Platelet adherence

Aspirin decreases platelet adherence. Aspirin does NOT cause vessel dilation nor alter clotting factor synthesis. Fibrinolysis is clot breakdown and aspirin does NOT alter this process

A patient asks about an article he read referring to taking a baby aspirin daily. Which of the following shows correct information by the patient? a. A daily aspirin helps to break up clots that form b. A daily aspirin uses up clotting factors, decreasing the risk of clot formation c. A daily aspirin dilutes the blood, making it thinner, so clots do NOT form d. A daily aspirin decreases platelet activity and platelets are involved in clot formation

Aspirin does reduce platelet activity. Aspirin prevents platelets from sticking together and does NOT alter clotting factors or dilute the blood

A patient is told to refrain from taking his daily aspirin prior to surgery. Which of the following provides the correct explanation for this recommendation? a. Aspirin increases clot formation b. Aspirin lowers blood pH c. Aspirin increases clotting times d. Aspirin stimulates disseminated intravascular coagulation

Aspirins do increase clotting times, and thus may be detrimental before surgery. Aspirins do NOT increase clot formation, cause an acidic pH, nor has it been associated with disseminated intravascular coagulation

A nursing student moves from sea level to mile-high elevation. Please place in the proper order the sequence of events that likely occurred for her blood laboratory values - Increased erythropoietin release - Increased Hct - Bone-marrow stimulation - Decreased arterial oxygen - Increase red blood cell (RBC) synthesis

At elevation, lower oxygen is available. This stimulates erythropoietin release, which stimulates the bone marrow to produce RBCs leading to an increase in hematocrit ( Decreased arterial oxygen - Increased erythropoietin release - Bone-marrow stimulation - Increase red blood cell (RBC) synthesis - Increased Hct)

A clinician suspects nonalcoholic fatty liver disease (NAFLD) in a patient. Which of the following would confirm this diagnosis? a. Blood test b. Endoscopy c. Abdominal computed tomography (CT) scan d. No single diagnostic test exists for NAFLD

At this time, there is no confirmatory test for nonalcoholic fatty liver disease (NAFLD)

A nurse is reviewing a patient's health history. Which of the following is of greatest concern to the nurse with regard to the patient's risk for stroke? a. Osteoporosis b. Atrial fibrillation c. Hepatitis d. Chronic obstructive pulmonary disease (COPD)

Atrial fibrillation can cause stagnation of blood, resulting in clot formation. Osteoporosis, hepatitis, and chronic obstructive pulmonary disease (COPD) do NOT increase stroke risk

A patient has atrial fibrillation. Which stroke type does the patient have increased risk of developing? a. Hemorrhagic b. Ischemic c. Hypocapnic d. Asymptomatic

Atrial fibrillation causes stasis of the blood, increasing the risk for clots and ischemic stroke

A pregnant woman and her husband have come to talk to the nurse. They are worried they will pass on a genetic disease to their offspring. Upon reviewing their history, the nurse relieves an autosomal dominant genetic disease is present, as the father has the condition. Which of the following diseases may be present? Select all that apply a. Cystic fibrosis b. Huntington's disease c. Marfan syndrome d. Down syndrome e. Familial hypercholesterolemia

Autosomal dominant conditions include Huntington's disease, Marfan syndrome, and familial hypercholesterolemia. Cystic fibrosis is an autosomal recessive condition and down syndrome is due to an extra copy of chromosome 21

The level of brain natiuretic peptide (BNP) is often measure in states of congestive heart failure. A patient asks a nurse why this factor is released during congestive heart failure. What is the appropriate association for the nurse to make? a. BNP increases heart rate b. BNP helps to elevate blood pressure c. BNP increases the heart's ability to contract d. BNP helps to alleviate the fluid overload on the heart

BNP increases urine output, reducing fluid load on the heart. BNP does NOT influence heart rate, help maintain blood pressure, nor alter heart contractility

A hormone panel was done on a patient with congestive heart failure and fluid volume overload. Which elevated hormone on the patient's chart is indicative of the body's attempt to reduce the fluid overload? a. Antidiuretic hormone (ADH) b. Brain natiuretic peptide (BNP) c. Aldosterone d. Renin

BNP is released when fluid volume excess is present. ADH increases fluid volume, aldosterone increases fluid retention, and renin is released when blood pressure is low

A patient with heart failure is likely to display which of the following? a. A robust, healthy appearance b. Edema, either peripheral or pulmonary c. Adequate heart rate and blood pressure response to stressors d. Frequent dizziness and loss of consciousness early in the disease

Because of the fluid retention, heart failure often results in edema. In heart failure, the heart is incapable of meeting demands and in the early stages, the patient may have only minor signs and symptoms

Which of the following applies to a patient with infective endocarditis? a. IV drug users may be at increased risk b. The infection often resolves on its own due to increased immunity of the heart c. The Harvard criteria is used to classify patients d. The infective endocarditis is usually diagnosed with a single blood test

Because of venous access to the heart, IV drug users are at an increased risk for infective endocarditis. Infective endocarditis usually requires antibiotic treatment, it is assessed using the Duke criteria, and multiple blood cultures may be necessary to identify agent in infective endocarditis

A nurse is talking to a patient who has hypertension. The nurse states that it is NOT only the heart that is involved in maintaining blood pressure. What other organ is the nurse most likely referring to ? a. Liver b. Stomach c. Spleen d. Kidney

Because the kidney regulates the RAAS, it plays a key role in blood pressure regulation

A patient is prescribed a beta-2 agonist for pulmonary issues. Which of the following is the purpose of the prescription? a. Decrease mucus production b. Bronchodilation c. Decrease pulmonary edema d. Prevent atelectasis

Beta 2 agonists are bronchodilators

A patient is taking a medication for asthma, which is a beta-2 agonist. The patient wants to know the purpose of the medication. Which of the following is the appropriate response by the nurse? a. It decreases mucus production b. It dilates your airways c. It decreases your coughing spells d. It facilitates gas exchange in the lungs

Beta 2 agonists cause bronchiole dilation

A patient is wanting to know whether she is pregnant. What hormone level can be measured to provide this answer? a. Gonadtropin-releasing hormone (GnRH) b. Estrogen c. Beta-human chorionic gonadotropin (b-HCG) d. Progesterone

Beta-human chorionic gonadotropin (b-HCG) is the hormone secreted with embryo implantation

What is the molecule that serves to buffer acids? a. CO2 b. H2O c. H2CO3 d. H+ e. HCO3-

Bicarbonate (HCO3-) is a base that can buffer acids

A student working in a rural health clinic is told to evaluate a patient's obesity status. The student has basic office medical equipment on hand. Which of the following would NOT be reasonable measurements for the students to conduct? Select all that apply a. Waist circumference b. Bioelectrical impedance c. Waist-hip ratio d. Hydrodensitometry e. Body mass index (BMI)

Bioelectrial impedance is used primarily for research purposes and hydrodensitometry is a technique to assess body composition and BOTH are NOT found in a standard clinic office

The nurse is to assess a patient with Cushing's syndrome. Which of the following will the nurse NOT expect to see? a. Truncal obestiy b. Moon facies c. Decreased blood pressure d. Striae

Blood pressure is normally elevated in Cushing syndrome. Cushing syndrome often results in fat accumulation in the area of the trunk, stretch mark formation (striae) and a more rounded facial appearance

A nurse is asked to look at the laboratory values to see whether a patient's kidney function has changed. Which of the following will the nurse examine? a. Aspartate aminotransferase (AST) and alanine aminotransferase (ALT) levels b. Creatinine phosphokinase (CPK) and troponin levels c. Blood urea nitrogen (BUN) and creatinine (Cr) levels d. Red blood cell (RBC) and white blood cell (WBC) levels

Blood urea nitrogen (BUN) and creatinine (Cr) levels are commonly used to assess kidney function

Brian has suffered chronic pain over the past several years due to spinal nerve radiculopathy. He is now hesitant to go on a family vacation, worried he will suffer pain, despite reassurances otherwise. What process may Brian be undergoing relative to his pain? a. Modulating b. Catastrophizing c. Denial d. Inventing

Brian may be catastrophizing his pain, making it seem worse than it is

A patient is brought to the emergency department by his spouse. The patient is vomiting blood. The nurse asks about the appearance of the blood in the vomit, and the patient reports that it i bright red. Which of the following is the correct interpretation? a. Blood has been mixed with the stomach acid b. Blood in vomitus is partially digested c. There is a current bleed from a ruptured vessel d. The protein is likely to resolve on its own

Bright red blood in vomitus indicates undigested blood from a current bleed. Blood mixed with stomach acid that is partially digested has a coffee ground appearance. Bleeding should be managed

A stroke patient has been diagnosed with expressive aphasia. Which area of the brain has been damaged? a. Cerebellum b. Occipital lobe c. Broca's area d. Wenicke's area

Broca's area controls the expression of speech. Cerebellum works on balance and posture, occipital lobe processes vision, and Wernicke area controls the reception and understanding of speech

A patient with chronic renal failure reports that he is feeling like his skin is "itching" . What is an appropriate response by the nurse? a. Suggest that the patient switch to a different cream or lotion b. Suggest that the patient switch laundry detergents c. Evaluate laboratory values to assess toxin levels in the blood d. Suggest the patient may be developing a sensitivity to the bed linen

Buildup of toxins in the blood, which can occur with renal failure, may lead to a sensation of skin itching

A patient has developed a tumor of the hypothalamus disrupting hypothalamic hormone production. As the nurse scans the laboratory report, which finding most directly supports this conclusion? a. Elevated cortisol b. Reduced aldosterone c. Elevated antidiuretic hormone (ADH) d. Reduced corticotropin releasing factor (CRF)

CRF is produced by the hypothalamus. If the hypothalamus is damaged, CRF production will be reduced

"CVA pain" is written on a patient's file? What does the nurse expect? a. The patient has pain in the neck area b. The patient has severe ankle pain c. The patient has pain in the back near the lower ribs d. The patient has a severe headache

CVA stands for costovertebral angle pain, which appears in the back area near the lower ribs

Which of the following is true about aortic stenosis? a. It develops due to calcification of the valve b. A murmur is heard during diastole c. Left ventricular (LV) ejection is increased d. It rarely causes any problems, signs, or symptoms

Calcification of the valve is a common cause of aortic stenosis. Aortic stenosis is heard during systole, makes ejecting blood difficult, and can compromise left ventricular functioning

A nursing student craves carbohydrates while studying for examinations because she feels it lowers her stress. What is the proper sequence explaining why the nursing student may desire carbohydrates while studying? - Increased sense of swelling - Tryptophan uptake by the brain - Carbohydrate ingestion - Increased serotonin - Insulin release

Carbohydrate ingestion leads to tryptophan uptake by the brain and formation of serotonin, which is neurotransmitter that produces a sense of well being ( Carbohydrate ingestion - Insulin release - Tryptophan uptake by the brain - Increased serotonin - Increased sense of swelling)

As pancreatic cancer may be diagnosed late in the disease course due to lack of early signs and symptoms. The nurse is researching cancer markers that can be monitored for pancreatic cancer. Which of the following should he consider? a. Alpha-fetoprotein (AFP) b. Cancer antigen (CA)-125 c. Carcinoembryonic antigen (CEA) d. Prostate-specific antigen (PSA)

Carcinoembryonic antigen (CEA) can be used to help detect pancreatic cancer. Alpha-fetoprotein (AFP) is used to detect liver cancer, cancer antigen (CA)-125 is used to detect ovarian cancer, and prostate specific antigen (PSA) is used to detect prostate cancer

A nurse is given a patient's heart rate and stroke volume. How would she calculate the cardiac output? a. Heart rate times stroke volume b. Heart rate divided by stroke volume c. Heart rate plus stroke volume d. Stroke volume divided by heart rate

Cardiac output is calculated as heart rate times stroke volume

A nursing student is wondering why hospitalized patients are at an increased risk for urinary tract infections (UTIs). Which of the following is the proper explanation? a. Fluid restriction is common in the hospital b. Hospitalized patients are more immobile c. Hospitalized patients have increased catheter usage d. Hospital food increases risk for UTIs

Catheter usage is the number one risk factor for urinary tract infections in hospitalized patinets

A woman is stating that she feels she has celiac disease. Which diagnostic test may be indicated? a. Endoscopic examination b. Colonoscopy c. Serology panel d. Barium swallow test

Celiac disease is considered an autoimmune condition, so antibody testing such as serology panel would be recommended

A clinician is concerned about restoring perfusion to an area damaged by ischemic stroke. Which of the following shows correct understanding? a. Reperfusion should be withheld until the extent of damage and recovery can be assessed b. Reperfusion is important because cells in the ischemic pneumbra can be recovered c. Reperfusion is NOT time dependent because brain cells can rely on anaerobic metabolism indefinitiely d. Reperfusion has been shown to be an ineffective strategy in stroke treatment

Cells in the ischemic pneumbra suffer reversible damage. Reperfusion should be restored as soon as possible and it is effective. Brain cells cannot rely solely on anaerobic metabolism

Pain results from a complex set of signals. Place in the proper order the sequence of events leading to pain signal production - Prostaglandin production - Phospholipase activation - Arachidonic acid formed - Pain and edema - Cyclooxygenase enzymes activated

Cellular injury causes phospholipase activation, which form arachidonic acid. Arachidonic acid is converted by cyclooxygenase enzymes to prostaglandins. Prostaglandings can cause pain and edema ( Phospholipase activation - Arachidonic acid formed - Cyclooxygenase enzymes activated - Prostaglandin production - Pain and edema)

A patient is treated for central diabetes insipidus. Which statement by the patient indicates that the treatment is working? a. I am going to the bathroom more now than ever b. I don't feel as thirsty and I am NOT drinking as much c. I drink 1 L of water several times and day, and still it is NOT enough d. I feel dehydrated all the time

Central diabetes insipidus causes too much water to be lost in the urine. Correcting the issue should result in more fluid retention and less thirst. Treatment should cause less water loss in the urine

Familial hypercholesterolemia is a genetic disorder characterized by high cholesterol level. Which statement is true regarding the condition? a. Untreated men develop symptoms by the fifth decade of life b. Children are at the risk for early acute myocardial infarction c. Homozygous familial hypercholesterolemia is less severe than the heterozygous form d. Untreated women develop symptoms by the fourth decade of life

Children are at the risk for early acute myocardial infarction

A nursing student is reviewing Chlamydia trachhomatis infections. Which of the following statements show correct understanding? a. Chlamydia is easily detected by the overt physical signs and symptoms b. Chlamydia infections can lead to pelvic inflammatory disease in women c. Chlamydia is the most common viral sexually transmitted infection d. Because Chlamydia infections are so common, they are NOT reportable to public health

Chlamydia infections can lead to pelvic inflammatory diseae. Chlamydia infections are often asymptomatic, bacterial reportable infections

A patient states, "Cholesterol is so bad for you. You need to eliminate it from your body by good food choices" Which response is appropriate for the nurse to make? a. Cholesterol is bad only for adults. High levels are needed for growing children b. Cholesterol can be eliminated from the body by taking certain medications c. You are correct. Cholesterol results in severe cellular damage and should be eliminated d. Cholesterol is needed by the body, and is found in both dietary sources and liver synthesis

Cholesterol is taken in by the diet and formed by the liver. Cholesterol is needed for normal body function in all individuals, it cannot be eliminated from the body, and it is beneficial

An elderly male patient has the common type of leukemia in the United States. Which factor listed is associated with this disease? a. Survivability is almost 95% b. A single enlarged lymph node is usually the first sign noted c. Extremely elevated neutrophil levels are diagnostic for this condition d. B-cell abnormalities are present

Chronic lymphocytic leukemia is often identified by abnormal B cells. Survivability of chronic lymphocytic leukemia is NOT 95%. It does NOT normally present with a single, enlarged lymph nodes. Neutrophils are NOT lymphocyte cells

In cases of liver cirrhosis, which of the following does the nurse know to be true? a. Cirrhosis has an abrupt onset b. Cirrhosis causes macroscopic changes in the liver c. Cirrhosis is reversible d. Cirrhosis causes minor interruption to liver function

Cirrhosis results from significant changes in the liver tissue. Cirrhosis is progressive, NOT reversible, and causes significant alterations in liver function

A nurse is reviewing the prescriptions for four clients in the hospital. Compare the effects of the medications prescribed to the clients and identify the client who is most likely being treated for systemic lupus erythematous (SLE) a. Client 1: Amoxicillin, vancomycin, imodium b. Client 2: Hydrocortisone, hydroxychloroquine, methotrexate c. Client 3: Cyclophosphamide, mycophenolate, infliximab d. Client 4: Codeine, ibuprofen, naproxen

Client 2 is being treated for SLE. Hydrocortisone is a corticosteroid used in the treatment of SLE. Hydroxycholoroquine is an antimalarial drug used in the treatment of SLE. Methotrexate is an immunosuppressant used to treat autoimmune diseases such as SLE

An infectious disease specialist is explaining a recent outbreak of a specific infectious agent. The infectious agent released toxins resulting in pseudomembranes in the colon, is transmitted between patients, and appears within patients with long-term antibiotic use. Which of the following agents is suspected? a. Shigella b. Salmonella c. Clostridium difficile d. Clotridium perfringens

Clostridium difficile can form pseudomembranes and is often an opportunistic infection due to antibiotic usage. Shigella and Salmonella do NOT form pseudomembranes in the colon. Clostridium perfringes causes gas gangrene

The following assessment results for a patient are reviewed: urine culture revealed a high level of colony-forming units. Urine was cloudy in appearance and the patient reports pain with urination. No fever or other symptoms are present. What conclusion may the clinician make? a. Interstitial cystitis b. Asymptomatic bacteriuria c. Urinary tract infections (UTI) d. Bladder cancer

Cloudy urine with pain on urination indicates a UTI if no fever is present (c) Interstitial cystitis presents with frequent need to urinate. Asymptomatic bacteriuria presents without symptoms. Bladder cancer may present with blood in the urine

A clinician suspects that a patient has Crohn's disease. Which diagnostic tool would be most accurate? a. Diagnosis by ruling out other conditions b. Colonoscopy with biopsy c. Barium swallow d. Patient tracking of signs and symptoms

Colonoscopy with biopsy is most accurate for Crohn's disease. It will show changes distinct to Crohn's disease. Barium swallow is used to diagnosis dysphagia

Cerebral edema is occurring in a patient. The patient is losing consciousness and vital signs, such as heart rate and blood pressure, are abnormal. What portion of the brain is compromised? a. Frontal lobe b. Brainstem c. Cerebellum d. Temporal lobe

Compromise of brainstem function with disrupt heart rate and respiration

A patient believes the amount of concern around opioid prescriptions is ridiculous. He states that a person should be able to stop taking opioids at any time. Which of the following conditions contribute to opioid abuse? Select all that apply a. Physical dependence b. Lack of willpower c. Withdrawal symptoms d. Apathy e. Physiological tolerance

Conditions that contribute to opioid abuse include physical dependence, withdrawal symptoms, and physiological tolerance

Which of the following actions may a nurse take to reduce the risk of decubitus ulcers? Select all that apply a. Change positions every 4 to 6 hours b. Have patients use TED stockings c. Check skin pressure points regularly d. Maintain supine position for patient e. Seat the patient in a chair when possible

Consistent evaluation of skin health is a critical step in preventing decubitus ulcer formation. TED stockings help facilitate blood return from lower extremities and positional changes help to reduce the risk of decubitus ulcer formation. Positional changes should be every 2 hours and the patient should periodically be turned onto the side

In which of the following conditions may laxative use be beneficial? a. Cathartic colon b. Large bowel obstruction c. Bout of constipation d. Volvulus

Constipation may be helped with laxative usage. Cathartic colon develops with laxative abuse, volvulus must be surgically corrected, and laxatives are contraindicated for large bowel obstruction

Which of the following medications should NOT be used as a treatment for hypertension? a. Corticosteroids b. Angiotensin-converting enzyme (ACE) inhibitors c. Diuretics d. Angiotensin receptor blockers

Corticosteroids are a class of medications that can results in elevated blood pressure. ACE inhibitors block angiotension formation, diuretics reduce fluid volume, and angiotensin receptor blockers decrease vasoconstriction which all lower blood pressure

A patient has crackles in the lungs. Which of the following shows correct understanding by the clinician? a. Crackles appear only in pneumonia b. Crackles develop only in left ventricular failure c. Because crackles can develop in pneumonia and heart failure, always treat for both d. Crackles plus fever and signs of infection likely indicate pneumonia rather than heart failure

Crackles can appear in pneumonia and heart failure in cases of pneumonia, though, fever will also be present/ Crackles can develop with left ventricle failure and pneumonia

A patient states that she heard on TV show that a certain type of juice can help with urinary tract infections. What juice does the nurse think the patient is referring to ? a. Apple b. Orange c. Grape d. Cranberry

Cranberry juice has been shown to lessen the adherence of bacteria in the urinary tract

A nurse is counseling a patient diagnosed with Crohn's disease. The patient asks which prat of her intestines can be affected by this disease. Which of the following is the correct response? a. The entire length of the gastrointestinal (GI) tract from mouth to anus can be affected by Crohn's disease b. Crohn's disease affects only your ascending colon c. Beginning in the rectum, Crohn's disease will ascend up the colon d. Crohn's disease affects the transverse and descending colon, including the rectum

Crohn's disease can affect the entire gastrointestinal (GI) tract

A diagnosis of Crohn's disease rather than ulcerative colitis has been made. Which of the following would indicate the presence of Crohn's disease rather than ulcerative colitis? Select all that apply a. Cobblestone appearance b. Skip lesions c. Beginning in the rectum and moving upwards d. Pseudopolyps e. Involvement of gastrointestinal (GI) tract from mouth to anus

Crohn's disease can cause intestinal damage that can cause the interior of the intestines to take on a cobblestone appearance. Skip lesions are areas of normal appearance of the colon. Crohn's disease can affect the entire GI tract. Ulcerative colitis begins in the rectum and ascends into the colon and pseudopolyps may appear

A patient with a gastrointestinal disease presents with a fistula. Which underlying disease is the patient most likely to have? a. Ulcerative colitis b. Peptic ulcer disease c. Diverticulosis d.Crohn's disease

Crohn's disease may cause fistula formation

Which of the following are enzymes that produce prostaglandins from arachidonic acid? a. Cyclooxygenase 1 b. Lipoxygenase c. TNF-alpha d. Interleukin-1 e. Cyclooxygenase 2

Cyclooxygenase enzymes convert arachidonic acid into prostaglandins. Lipoxygenase convert arachidonic acid into leukotrienes. TNF alpha and interleukin 1 are inflammatory cytokines, NOT enzymes

Please place the following sequence in order to describe how iodine deficiency may lead to a goiter - Decreased negative feedback on anterior pituitary - Increased thyroid growth - Decreased iodine in diet - Increased thyroid stimulating hormone (TSH) secretion - Decreased thyroid hormone synthesis

Decreased iodine results in reduced ability to synthesize thyroid hormone. Reduced thyroid hormones cause decreased negative feedback on the anterior pituitary. Anterior pituitary secretion of TSH increases, which causes the thyroid gland to grow ( Decreased iodine in diet - Decreased thyroid hormone synthesis - Decreased negative feedback on anterior pituitary - Increased thyroid stimulating hormone (TSH) secretion - Increased thyroid growth)

Which condition causes pseudohypocalcemia? a. Hyponatremia b. Hypoalbuminemia c. Hypophosphatemia d. Hyperkalemia

Decreased serum albumin levels or HYPOALBUMINEMIA can cause the appearance of low serum calcium levels called pseudohypocalcemia

Clinicians are discussing a patient who they believe may benefit from a thymus transplant. Which of the following disorders may the patient have? a. DiGeorge syndrome b. HIV infection c. Wiskott-Aldrich d. Severe combined immunodeficiency disorder (SCID)

DiGeorge syndrome is a result of maldevelopment of the thymus gland. The thymus gland is NOT directly affected by HIV infection. Wiskott Aldrich system is a genetic defect of T cells and SCID is a genetic defect disrupting B and T cell development

A patient has recently been diagnosed with type 2 diabetes mellitus. He wants to know whether his disease can lead to any long term problems. Which of the following may the nurse share as a long term complication of diabetes mellitus? a. Hypoglycemia b. Blindness c. Diabetic ketoacidosis d. Dawn syndrome

Diabetes mellitus damages the retinal vessels and can cause blindness. DM is associated with hyperglycemia long term, although episodes of hypoglycemia may develop short term. Diabetic ketoacidosis and dawn syndrome are short term complications of DM

A nurse is evaluating a patient's risk for renal disorders. Which of the following increase the risk for renal dysfunction? Select all that apply a. A patient with diabetes mellitus b. A patient with severe hypertension c. A patient with systemic lupus erythematosus d. A patient with cirrhosis e. A patient with peptic ulcer disease

Diabetes mellitus increases the risk for renal dysfunction. Hypertension and lupus can damage the kidneys. Cirrhosis is a disorder of the liver and peptic ulcer disease does NOT affect renal function (a, b, c)

A nursing student knows the certain risk factors contribute to nephrotic syndrome. Which patient listed below has the greatest risk of developing nephrotic syndrome? a. Patient with diabetes mellitus b. Patient with amyloidosis c. Patient with systemic lupus erythematosus d. Patient with kidney stones

Diabetes mellitus presents the greatest risk for nephrotic syndrome

A clinician is reviewing laboratory values, signs, and symptoms. Which of the following is more indicative of hyperglycemia hyperosmolar syndrome (HHS) rather than diabetic ketoacidosis (DKA)? a. Hyperglycemia b. Polydipsia c. pH > 7.3 d. Increased plasma osmolarity

Diabetic ketoacidosis (DKA) lowers serum pH, whereas hyperosmolar syndrome (HHS) does NOT. Hyperglycemia, polydipsia, and osmolarity is increased in both DKA and HHS

A patient states that he has been diagnosed with diverticulitis. Which of the following statements by the patient indicates that he best understand this disorder? a. I am producing too much stomach acid b. I have small outpouching in my intestines c. I have small outpouchings in my intestines that are now inflamed d. I have an intestinal hernia, which is inflamed

Diverticulitis is inflammation of intestinal outpouchings

A patient post-myocardial infarction develops pericarditis, pleuritis, and pneumonitis as a hypersensitivity reaction to tissue necrosis of the myocardial infarction. Which of the following does the nurse aspect? a. Dressler's syndrome b. Ruptured ventricular aneurysm c. Sign of papillary muscle rupture d. Common sign of heart failure

Dressler syndrome occurs after myocardial infarction. Pericarditis, pleuritis, and pneumonitis develop. A ruptured ventricular aneurysm and heart failure would NOT present with pleuritis or hypersensitivity reaction. A papillary muscle rupture would present as a heart murmur

A patient has been diagnosed with Dressler's syndrome. What disorder likely preceded this development? a. Pneumonia b. Heart valve replacement surgery c. Pulmonary congestion d. Myocardial infarction

Dressler's syndrome may develop following a myocardial infarction

A patient is receiving a prescription to treat his erectile dysfunction. The prescription is for a phosphodiesterase inhibitor. Which information does the nurse share with the patient? a. Treatments for erectile dysfunction has been linked to increase testicular cancer risk b. This medication cannot be combined with nitrate medications c. This medication may prevent priapism d. This medication activates the sympathetic nervous system causing an increase in heart rate and blood pressure

Due to a significant drop in blood pressure, phosphodiesterase inhibitors cannot be combined with nitrate medications, which are vasodilators

As a woman enters menopause, for which of the following health conditions does her risk increase? Select all that apply a. Osteoporosis b. Vaginal atrophy c. Infection d. Heart disease e. Stroke

Due to estrogen decerase, resorption of bone increases (osteoporosis), vaginal atrophy occurs, risk of heart diseases and stroke increases. Infection risk is NOT influenced by menopause (a, b, d, e)

A common treatment recommendation for acute myocarditis includes which of the following? a. Placement of stents in the coronary arteries b. Prescription antibiotics c. Restriction of activity for six months or more d. Pericardial fluid removal

Due to heart damage, restriction of activity is recommended for myocarditis. Stents are used for occlusions, NOT myocardial infarctions. Myocarditis is normally of viral origin, NOT bacterial, and it does NOT necessarily cause pericardial effusion

A student nurse is reviewing electrocardiogram (ECGs). Which of the following does she recognize as true about ECGs and myocardial infarction (MIs)? a. ECGs alone are NOT diagnostic for MIs b. With MIs, P-R elevation is expected c. Most Mis are preceded by development of prolonged P-R intervals d. Most MIs show no changes on ECG

ECG analysis may determine abnormalities related to MIs, but an ECG cannot diagnose an MI. S-T elevation may occur in an MI whereas prolonged P-R intervals indicate a heart block, NOT an MI. MIs may cause changes on ECGs

A physician wants to further evaluate and assess the presentation of a heart murmur heard with auscultation. Which diagnostic test is the likely best choice? a. Electrocardiography b. Echocardiography c. Coronary artery bypass graft d. Cardiac angiography

Echocardiography is the gold stanard for diagnosis of heart murmur

A nursing student examines a patient and notes the following: poor skin turgor, edema, oliguria, dry mucous membranes, and hypertension. Which signs and symptoms do NOT support the student's diagnosis of dehydration? Select all that apply a. Poor skin turgor b. Edema c. Oliguria d. Dry mucous membrane e. Hypertension

Edema and hypertension are signs of fluid volume overload. Poor skin turgor, oliguria, and dry mucous membranes are signs of dehydration

A patient believe she has lupus. Which of the following diagnostic tests, if positive, would contribute to the diagnosis of lupus? Select all that apply a. Rheumatoid factor b. Elevated antinuclear antibody (ANA) levels c. Anti-ds DNA antibodies d. Mantoux test e. Leukocytosis

Elevated ANA levels, which are antibodies, and anti ds DNA antibodies are elevated in lupus. Rheumatoid factor indicates rheumatoid arthritis and the Mantoux test is for tuberculosis. Leukocytosis is elevation in WBCs and is NOT diagnostic, alone, for lupus

A patient's laboratory results reveal elevated adrenocorticotropic hormone (ACTH) and cortisol with lower than normal corticotropic releasing factor (CRF). Which tissue is likely the cause of the patient's endocrine disorder? a. Hypothalamus b.. Adrenal cortex c. Posterior pituitary d. Anterior pituitary

Elevated adrenocorticotropic hormone (ACTH) and cortisol, with reduced corticotropic releasing factor (CRF), would indicate a problem in the anterior pituitary. If CRF were also elevated, then the problem would be in the hypothalamus. If ACTH levels were reduced, then the problem would be in the adrenal cortex. The posterior pituitary is NOT involved in cortisol production

A patient is wondering how elevated blood pressure may lead to a heart attack. Please place the following steps in order - Myocardial ischemia - Increased workload on heart - Left ventricular hypertrophy - Elevated blood pressure - Inadequate blood supply to left ventricle

Elevated blood pressure will increase the workload on the heart, causing ventricular hypertrophy. The heart muscle may have inadequate circulation leading to myocardial ischemia ( Elevated blood pressure - Increased workload on heart - Left ventricular hypertrophy - Inadequate blood supply to left ventricle - Myocardial ischemia)

Assessment of a patient reveals markedly increased anterior posterior dimension of the chest wall. Which of the following disorders is most likely to contribute to this alteration? a. Chronic bronchitis b. Pneumonia c. Pulmonary fibrosis d. Emphysema

Emphysema causes air trapping, this can increase the diameter of the chest wall. Chronic bronchitis, pneumonia, and pulmonary fibrosis do NOT alter the diameter/dimension of the chest wall

A patient with emphysema is being assessed. Which of the following is an expected finding? a. Cyanosis b. Hyperresonance on percussion of the lungs c. Decreased chest volume d. Bronchial constriction

Emphysema traps air in the lungs, resulting in hyperresonance. Cyanosis tends to develop with bronchitis, less so with emphysema. As air is trapped in the lungs with emphysema, chest volume increases. Emphysema is NOT a disorder of bronchiole constriction

A nurse is working in a gastrologist's office. What is the most accurate method of diagnosing upper GI tract disorders? a. Computed tomography (CT) scan b. Abdominal x-ray c. Colonoscopy d. Endoscopy

Endoscopy is the most accurate method of diagnosis for upper gastrointestinal tact disorders

Please place the following in order leading to platelet plug formation - Platelets adhere to the site of injury - Platelets release thromboxane A2 - Collagen surface exposed - Injured endothelium - Additional platelets attracted to form plug

Endothelial injury exposes collagen surface, which attract platelets that adhere to the site of injury. Platelets release a signal to attract more platelets, leading to plug formation ( Injured endothelium - Collagen surface exposed - Platelets adhere to the site of injury - Platelets release thromboxane A2 - Additional platelets attracted to form plug )

A patient arrives at the emergency department having come from the scene of a motor vehicle accident. Initial blood laboratory values indicate elevated blood glucose levels. Which of the following are likely explanation? Select all that apply a. Patient must have diabetes mellitus b. Increased epinephrine c. Inaccurate specimen as blood glucose tests require fasting d. Adrenal gland suppression e. Increased cortisol release

Epinephrine and cortisol increase blood glucose levels. Although diabetes mellitus is associated with high blood glucose levels, but it can also elevate in response to stress. The adrenal gland is activated, NOT suppressed during stress because the adrenal gland hormones elevate blood glucose (b, e)

Parents have brought their child to the emergency department. The child has a severe peanut allergy and ingested peanut particles in a cookie. The parent's administered an EpiPen en route to the hospital. Which of the following can the nurse expect to result from an epinephrine injection? Select all that apply a. Peripheral vasodilation b. Increased heart rate c. Increased blood pressure d. Pale skin e. Pupil constriction

Epinephrine increases heart rate, increases blood pressure, and causes peripheral vasoconstriction resulting in pale skin appearance. Also epinephrine dilates the pupils

Because diabetes mellitus can damage the autonomic nervous system, the nurse wants to explain some signs and symptoms that the patient may experience. Which should she include on her list for a patient with disrupted autonomic nervous system functioning? a. Erectile dysfunction b. Skin appearance changes c. Sense of sustained muscle contraction d. Loss of sensation from the extremities

Erectile dysfunction can develop when the autonomic nervous system is compromised. Skin changes would NOT be linked to the autonomic nervous system disruption. Sustained muscle contractions and loss of sensation from the extremities are a part of the peripheral nervous system

An older male is speaking with embarrassment about his impotence. The nurse recognizes that he is referring to which condition? a. Prolonged erection b. Abnormally small testicles c. Failure to get or maintain an erection d. Curvature of the penis with erection

Erectile dysfunction is sometimes referred to as impotence

Which of the following patient is most at risk for erythroblastosis fetalis? a. Rh-positive mother with her third pregnancy b. Rh-negative mother with her first pregnancy c. Rh-negative mother with her third pregnancy d. Rh-positive mother with her first pregnancy

Erthroblastosis is most likely to develop in an Rh negative mother with more than one pregnancy. The mothers antibodies can cross the placenta

A young child is brought to the pediatricians, and his mother says, he has a rash on his cheeks. She wonders if it is the new detergent she has brought. The nurse notices a "slapped cheek" appearance on the child and suspects which disorder? a. Erythema infectiosum b. Influenza virus c. Pseudomonas aeruginosa d. Neisseria meningitidis

Erythema infectiosum often presents with a slapped cheek appearance

A patient has sarcoidosis. Which of the following signs and symptoms for this disease should the nurse look for? a. Dry eyes and mouth b. Tightening of the skin c. Enlarged and inflamed joints of the hand d. Erythema nodosum

Erythema nodosum, a skin rash, may develop in sarcodosis. Dry eyes and mouths are common in Sjogrens, tightening of the skin is common in sclerodema and inflamed joints of the hands are more common in rheumatoid arthritis

A patient is able to produce only limited amounts of erythropoietin. What does the nurse expect to observe? a. Polycythemia b. Increased hemolysis c. Hyperbilirubinemia d. Reduced hematocrit

Erythropoietin does NOT play a role in hemolysis. With reduced erythropoietin, polycythemia is unlikely, reduced bilirubin in the blood, and may cause a reduction in hematocrit levels

A nurse is monitoring a patient with irritable bowel syndrome for "alarm" symptoms. Which of the following are such symptoms? Select all that apply a. Fever b. Family history of colon cancer c. Weight loss d. Iron deficiency anemia e. Rectal bleeding

Family history of colon cancer, significant weight loss, iron deficiency anemia indicating blood loss, and rectal bleeding are considered alarm symptoms

A patient is worried about her risk of developing a stroke because her other has a stroke in her early 60s. Which of the following could the nurse convey as factors that increase a person's risk for stroke? Select all that apply a. Family history of stroke b. Tobacco usage c. Hypotension d. Active lifestyle e. African American

Family history, tobacco usage, and African American ethnicity increases the risk for stroke

A patient presents with right upper quadrant pain. The patient has several risk factors for gallstones. Which of the following are considered risks by the nurse? Select all that apply a. Female sex b. Oral contraceptive usage c. Low body mass index (BMI) d. Age greater than 40 years e. Low-calorie, low-fat diet

Female sex, oral contraceptive usage, and age of greater than 40 years increase the risk for gallstones. Normal body weight and a low calorie and low fat diet reduces gallstone risk

A patient has genital warts. Which of the following comments by the patient shows correct understanding? a. My genital warts are caused by a bacterial infection b. The infection that causes genital warts is easily cured c. Although the warts may be treated, the infection will remain in my body d. I cannot transmit this infection to others

Genital warts can be treated but NOT cured. They are caused by viral infections that can be transmitted to others

During a class discussion on obesity, a student raises a hand and asks whether ghrelin can be used for a treatment for obesity. What explanation may the teacher give to answer the student? a. Ghrelin is a medication given to suppress metabolic rate and thus would NOT be suitable for obesity treatment b. Ghrelin would be an excellent choice for obesity treatment because it speeds up metabolism and reduces hunger c. Ghrelin increases hunger and food consumption and thus would NOT be suitable for obesity treatment d. Ghrelin is a neurotransmitter in the skeletal muscle system and has nothing to do with obesity

Ghrelin increases hunger and food consumption and thus would NOT be suitable for obesity treatment. Ghrelin stimulates hunger, it does NOT suppress metabolic rate or speed up metabolism

A college student presents to the Emergency Department complaining of photophobia, neck pain, and positive Kernig sign. Which diagnostic test should be most important given the patient's signs and symptoms? a. Lumbar puncture b. Chest x-ray c. Spirometry d. Abdominal computed tomography (CT) scan

Given these signs and symptoms, the patient may have meningitis. and a lumbar puncture can confirm this diagnosis

Which of the following findings may encourage a clinician to begin considering discussing the possibility of a need for a kidney transplant? a. Damage to 50% of the nephrons b. Presence of periorbital edema c. Glomerular filtration rate (GFR) < 12 ml/min d. Protein in urine

Glomerular filtration rate (GFR) should be above 90 ml/min for normal renal function. GFR less than 12 ml/min shows highly compromised renal function. Damage to 90 to 95% of nephrons is needed for end stage renal disease. Periorbital edema can occur with kidney dysfunction, but it alone does NOT indicate renal failure. Protein in the urine can occur for many reasons but it is NOT necessarily a sign of end stage renal disease

Insulin helps move glucose into cells. Which other hormones are counter-regulatory to insulin and work to raise blood glucose? Select all that apply a. Glucagon b. Antidiuretic hormone (ADH) c. Thyroxine d. Cortisol e. Epinephrine

Glucagon, released from the alpha cells of the pancreas, cortisol, and epinephrine elevate blood glucose. Antidiuretic hormone (ADH) and thyroxine are NOT involved in blood glucose regulation

A patient's urinalysis reveals the presence of glucose. Which of the following should be considered first? a. The patient is eating too many carbohydrates b. Decreased glomerular filtration rate (GFR) is concentrating the urine too much c. The patient may have diabetes mellitus d. The patient has kidney failure

Glucose in the urine is often a sign of diabetes mellitus, NOT a sign of renal problems. Eating too many carbohydrates does NOT normally cause carbohydrates to appear in the urine

Which substance is transported to the cells by facilitated transport?

Glucose is transported into the cells by the carrier protein insulin through a process known as facilitated transport. It involves the passing of certain molecules through the plasma membrane with assistance from carrier proteins

A nurse is explaining to a young male patient the development of secondary sex characteristics and the tissues and hormones involved. Please place the following events in the correct sequence - Luteinizing hormone released - Gonadotropic releasing hormone (GnRH) released from hypothalamus - Anterior pituitary stimulated - Testosterone produced - Leydig cells stimulated

Gonadotropic releasing hormone (GnRH) from the hypothalamus stimulates the anterior pituitary. Luteinizing hormone is released, which stimulates the Leydig cells to produce testosterone ( Gonadotropic releasing hormone (GnRH) released from hypothalamus - Anterior pituitary stimulated - Luteinizing hormone released - Leydig cells stimulated - Testosterone produced)

A clinician orders a gram stain test that reveals gram-positive organisms. Which of the following organisms may be in the sample? Select all that apply a. Streptococcus pneumoniae b. Staphylococcus aureus c. Streptococcus pyogenes d. Escherichia coli e. Shigella

Gram positive organisms include streptococcus pneumoniae, staphylococcus aureus, and streptococcus pygoenes. Escherichia coli and shigella are gram negative organisms

An antibody test is ordered for an endocrine disorder. Which of the following conditions is commonly diagnosed by antibody test results? a. Multiple endocrine neoplasia b. Grave's disease c. Pheochromocytoma d. Cushing's syndrome

Grave's disease is an autoimmune disorder of the thyroid gland and can be detected with antibody testing. Multiple endocrine neoplasia causes multiple tumors, pheochromocytoma is a tumor of the adrenal medulla, and Cushings syndrome is due to elevated adrenal cortex hormones

A patient needs to be evaluated for specific signs of acute pancreatitis. Which of the following should be considered? Select all that apply a. Grey Turner's sign b. Cullen's sign c. Murphy's sign d. McBurney's sign e. Kernig's sign

Grey Turners sign is bruising along the flanks with pancreatitis and Cullens sign is bruising in the umbilical area with pancreatitis. Murphys sign is for gallbladder issues, McBurnerys sign is for appendicitis, and Kernigs sign is for menigitis

For which of the following would antibiotics be appropriate as the primary treatment? Select all that apply a. Group A beta-hemolytic streptococci (GABHS) glomerulonephritis b. Nephrotic syndrome c. Pyelonephritis d. Goodpasture's syndrome e. Polycystic kidney disease

Group A beta hemolytic streptococci (GABHS) glomerulonephritis is a disorder caused by streptococcal bacteria. Pyelonephritis is an infection of the renal pelvis. Nephrotic syndrome is NOT due to a bacterial infection, Goodpastures syndrome is an autoimmune disorder and polycystic kidney disease is a genetic disorder

A clinician needs to determine whether a patient has hepatitis C. Which result would confirm this diagnosis? a. Positive HBeAg b. Positive HBV RNA assay c. Positive HAV RNA assay d. Positive HCV RNA assay

HCV RNA assay test will confirm the presence of hepatitis C RNA

A patient appears to have a primary immunodeficiency disorder. Based on this information, which disorder can the clinician rule out? a. DiGeorge syndrome b. HIV c. Severe combined immunodeficiency disorder (SCID) d. Wiskott-Aldrich syndrome

HIV is acquired through an infection and is an acquired immunodeficiency. DiGeorge syndrome and SCID is present at birth and is a primary immunodeficiency. Wiskott Aldrich syndrome is a genetic disorder and a primary immunodeficiency

A patient has come to the clinic seeking information about a prophylactic medication for HIV. Which of the following is correct information for the patient? a. This medication is recommended for all sexually active people b. This medication is recommended for preteens before a first sexual intercourse has occurred c. This medication is recommended for those at a high risk of contracting HIV. It must be taken daily d. This medication is administered through 3 injections spaced 3 months apart

HIV prophylactic medication, in its current form, must be taken daily, and it is for those at high risk. It is NOT recommended for preteens

Which of the following cell types are directly targeted by HIV? Select all that apply a. T cells b. B cells c. CD4 cells d. Red blood cells e. Some lymphocyte cells

HIV targets T cells, CD4 cells (a form of T cells). T cells are lymphocytes. HIV does NOT target B cells or red blood cells

A hospitalized patient has a heart murmur and has asked to see it on the electrocardiogram (ECG) recording. What is the appropriate response by the nurse? a. Heart murmurs appear on the larger peak of the ECG b. Heart murmurs do NOT appear on an ECG. They are heard with a stethoscope c. Heart murmurs only appear on the most sensitive of ECGs. Your ECG is NOT sensitive enough to detect such a murmur d. Heart murmurs change with every heartbeat, so they are difficult to capture on an ECG

Heart murmurs are NOT electrical events detected on ECG. They are detected with auscultation

A woman believes that she has been diagnosed with anemia. Which lab value does NOT support this diagnosis? a. Hematocrit of 38% b. Hemoglobin levels of 18 grams per deciliter c. Red blood cell total count of 2.6 million per microliter d. The woman's hematocrit, hemoglobin, and red blood cell count support the diagnosis of anemia

Hemoglobin levels of 18 grams per deciliter is elevated and does NOT indicate anemia. Hematocrit of 38% is low and RBC count of 2.6 million per microliter is low which support the diagnosis of anemia

A nursing student is reviewing strokes. Which of the following indicates correct understanding? a. Initial clinicial presentation does NOT distinguish between hemorrhagic and ischemic stroke b. Treatment of stroke is the same regardless of whether it is hemorrhagic or ischemic c. No diagnostic test can be used to determine ischemic versus hemorrhagic stroke d. The risk factors for ischemic and hemorrhagic stroke differ

Hemorrhagic and ischemic strokes tend to present similarly. Treatment and risk factors for hemorrhagic and ischemic stroke differ. A computed tomography (CT) scan can reveal hemorrhagic versus ischemic stroke

A nurse is educating patients on the transmission of hepatitis. Which of the following forms of hepatitis virus are transmitted by the fecal-oral route? Select all that apply a. Hepatitis A b. Hepatitis B c. Hepatitis C d. Hepatitis D e. Hepatitis E

Hepatitis A and E are transmitted through the fecal oral route

A nurse is reviewing the progression of a Hepatitis B infection in a patient. The patient is concerned about this infection can result in cirrhosis. Please place the following events in sequence from infection with Hepatitis B to cirrhosis - Inflammatory response to infection within hepatocytes - Cirrhosis develops - Prolonged immune response results irreversible liver damage - Immune response to infection with increasing antibody production - Inoculation with Hepatitis B

Hepatitis B would begin with exposure to the virus, which leads to an inflammatory response in hepatocytes. This stimulates an immune response. A prolonged immune response can cause irreversible damage, leading to cirrhosis ( Inoculation with Hepatitis B - Inflammatory response to infection within hepatocytes - Immune response to infection with increasing antibody production - Prolonged immune response results irreversible liver damage - Cirrhosis develops)

A patient with liver disease is instructed to review how he will mange his dietary needs. Which of the following indicates correct understanding by the patient? a. I will eat one large meal per day b. I will eat small, high-calorie, high-protein meals c. I will restrict calories while my liver heals d. I will maintain a diet heavy in fats

High calories and high protein levels help maintain body function. Small frequent meals are best. Caloric restriction and high lipid intake do NOT aid liver healing

A patient is suffering from cholecystitis. Which of the following food choices by the patient may the nurse need to discuss further given his condition? a. Dessert of pie and ice cream b. Lunch of spaghetti with an apple for dessert c. Dinner of steamed rice and salmon d. Snacking on gummy candies

High fat food intake such as pie and ice cream can worsen gallbladder conditions. Carbohydrate and protein rich meals are beneficial for the gallbladder issues

A laboratory result for parathyroid hormone (PTH) is NOT available; however, the nurse suspects hyperparathyroidism. Which of the following would support the nurse's conclusion? a. The patient presents with Chvostek's sign b. The patient presents with edema c. The patient presents with muscle flaccidity and weakness d. The patient presents with paresthesia

High levels of PTH would raise plasma calcium levels, which causes muscle flaccidity and weakness. Elevation in calcium levels do NOT cause edema. Chovstek's sign is caused by and paresthesia is associated with hypocalcemia

Which of the following is true regarding congenital syphilis? a. It always causes a miscarriage b. Newborns may present with Hutchinson teeth and saddle nose c. Syphilis infections do NOT cross the placenta d. Signs and symptoms develop after several years

Hutchinson teeth are a distinguishing characteristic of congenital syphilis. Congenital syphilis does NOT always result in miscarriage. Congenital syphilis is present at birth and it can cross the placenta

A patient is to undergo renal ultrasound. Which of the following is the test most likely used to assess? a. Glomerular filtration rate (GFR) b. Hydronephrosis c. Casts d. Azotemia

Hydrophrosis is a collection of fluid in the kidney and can be detected with ultrasound. GFR is NOT assessed with IVP, casts are measured in urine samples, and azotemia is determined by urea levels in the blood

A nurse is explaining to a patient how uncontrolled diabetes mellitus may eventually lead to amputation. What is the proper sequence of steps leading to this development? - Nerve damage - Poor wound healing and increased infection - Hyperglycemia - Decreased sensation - Increased risk for wounds

Hyperglycemia may lead to nerve damage, which causes decreased sensation. This increases the risk for wounds and poor wound healing may lead to infection ( Hyperglycemia - Nerve damage - Decreased sensation - Increased risk for wounds - Poor wound healing and increased infection)

A client has increased serum phosphorus levels. Which condition related to hyperphosphatemia would the nurse expect to find in the client? a. Hyperkalemia b. Hypernatremia c. Hypocalcemia d. Hypomagnesaemia

Hyperphosphatemia is generally accompanied by low serum calcium levels, which is also known as HYPOCALCEMIA

A patient wants to know why it is so important to control blood pressure. Which end organ damage may develop due to hypertension? a. Peptic ulcer disease b. Stroke c. Cirrhosis d. Splenomegaly

Hypertension increases the risk for stroke. Hypertension does NOT affect the development of peptic ulcer disease nor it is a risk factor for cirrhosis and splenomegaly

A nurse educator is teaching students about the risk for stroke. Which of the following would place the patient most at risk for hemorrhagic stroke? a. Presence of atrial fibrillation b. Use of oral contraceptives c. Blood pressure of 160/100 mm Hg d. Resting heart rate (HR) of 78 bpm

Hypertension is the number one risk factor for hemorrhagic stroke. Atrial fibrillation and oral contraception use increases the risk for ischemic stroke. An elevated resting heart rate does NOT substantially increase the risk for hemorrhagic stroke

Which of the following risk factors should be controlled in order the reduce the risk of hemorrhagic stroke? a. Blood clotting risk b. Hypertension c. Elevated resting heart rate (HR) d. Atrial fibrillation

Hypertension is the number one risk factor for hemorrhagic stroke. Formation of blood clots and atrial fibrillation increases the risk for ischemic stroke. Elevated heart rate does NOT increase stroke risk

A nurse is educating a group of patients about the common long-term effects of hypertension. Which of the following should she include? Select all that apply a. Number one cause of amputation b. Retinal damage c. Enlargement of the heart d. Kidney damage e. Pulmonary congestion

Hypertension may cause damage to the vessels in the retina, enlargement of the heart because it has to work harder, and kidney damage. Hypertension does NOT commonly result in amputation nor cause pulmonary congestion, unless the heart has failed

A patient has several medical conditions. Which of the following most likely plays a role in the patient's elevated blood lipid levels? a. Hypothyroidism b. Peptic ulcer disease c. Vitamin D deficiency d. Osteoarthritis

Hyperthyroidism may cause elevated lipid levels. Peptic ulcer disease, vitamin D deficiency, and osteoarthritis are NOT related to high lipid levels

A nursing student is reviewing signs and symptoms of hyperthyroidism. Which of the following should she include on her list? Select all that apply a. Weight loss b. Tachycardia c. Lethargy d. Tremors e. Cold intolerance

Hyperthyroidism stimulates metabolism, causing weight loss, stimulates heart rate, and the nervous system causing tremors. Hypothyroidism stimulates activity. Hyperthyroidism causes heat tolerance, NOT cold tolerance (a, b, d)

As long as lung function is normal, hyperventilation will cause which of the following? Select all that apply a. Increased pH b. Decreased carbon dioxide c. Decreased hydrogen ions d. Acidosis e. Increased hydrogen ions

Hyperventilation will cause increased pH, decreased carbon dioxide, and decreased hydrogen ions (a, b, c)

A nurse learns that a patient has presented with hypoglycemia. Which signs and symptoms does the nurse expect to observe? Select all that apply a. Dizziness b. Hunger c. Sweating d. "Fruity" smelling breath e. Kussmaul's respirations

Hypoglycemia can cause dizziness, hunger, and sweating. Ketoacidosis may cause a fruity smelling breath and kussmauls respirations (a, b, c)

An IV drug user has developed infective endocarditis leading to a valve disorder. A heart murmur is heard. Which valve disorder likely developed? a. Mitral valve insufficiency b. Tricuspid stenosis c. Aortic stenosis d. Aortic insufficiency

IV injection of drugs provides access to the right side of the heart. This may cause problems with the tricuspid valve

A patient complains that his stomach hurts after taking ibuprofen for his sore knee. What response is appropriate for the nurse to provide? a. " It is NOT possible for ibuprofen to hurt your stomach, ibuprofen affects the brain only" b. "Ibuprofen causes stomach pain because i blocks an enzyme involved in the formation of protective stomach mucus" c. "Ibuprofen increases metabolism, and what you may be feeling is stomach hunger pains" d. "Ibuprofen may increase the risk of infective gastroenteritis, so you may need an antibiotic prescription"

Ibuprofen causes stomach pain because i blocks an enzyme involved in the formation of protective stomach mucus. Ibuprofen blocks cyclooxgenase enzymes, which reduces prostaglandins. Prostaglandins are a signal needed for stomach mucus production. Ibuprofen does NOT increase the risk for infective gastroenteritis or affect metabolism

A patient with suspected idiopathic pulmonary fibrosis is scheduled for a chest x-ray. If this condition is present, what will appear on the chest x-ray? a. Flattened diaphragm b. Enlarged blood vessels around the heart c. "Ground glass" appearance d. Mediastinal shift of lung

Idiopathic pulmonary fibrosis tends to present with a ground glass appearance on a chest x ray

A patient presents with hyperthyroidism. Which of the following values enables the clinician to rule out secondary hyperthyroidism? a. Low thyroid-stimulating hormone (TSH) b. Elevated thyroxine c. Low thyrotropic releasing hormone (TRH) d. Elevated T3 levels

If the hyperthyroidism were secondary, then thyroid stimulating hormone (TSH) levels would be high, indicating a problem with the anterior pituitary. Thyroxine and T3 levels will be high, but it is NOT enough to differentiate between primary and secondary hyperthyroidism. Thyrotropic releasing hormone (TRH) will be low, but it is NOT enough information to rule out secondary hyperthyroidism

In left ventricular failure, which factors may be reduced? Select all that apply a. Ejection fraction b. Heart rate c. Blood pressure d. Fluid volume e. Pulmonary capillary wedge pressure (PCWP)

If the left ventricles is failing, then ejection fraction is reduced and it cannot maintain blood pressure. Heart rate will be increase to compensate for reduced blood pressure, fluid volume increases to compensate for failing heart, and pulmonary capillary wedge pressure increases when the left ventricle fails

A patient is suffering from liver failure. Which of the following does the nurse expect to see elevated? Select all that apply a. Clotting times b. Plasma albumin c. Aspartate aminotransferase (AST) levels d. Risk for jaundice e. Total plasma bilirubin

If the liver fails, then clotting factors are NOT synthesized at the needed level, and clotting times increased. Aspartate aminotransferase (AST) is released with hepatocellular damage. As the liver fails, bilirubin accumulates leading to jaundice (a, c, d, e)

On a laboratory write up, an immunoglobulin E (IgE) mediated response resulting in urticaria is noted. How can these results be interpreted for the patient? a. You have a severe autoimmune disease b. You have a delayed hypersensitivity reactionn c. Your immune system has stopped functioning d. You have an allergic response

IgEs and urticaria are common in allergies. Urticaria is NOT always present in autoimmmune diseases. T cells, NOT IgEs, are involved in delayed hypersensitivity. The production of IgEs indicates some immune function

A clinician suspects that the patient has immune thrombocytopenia purpura. How is this disease diagnosed? a. Genetic analysis b. Diagnosis by exclusion c. Bone marrow biopsy d. Assessing for bacterial infection

Immune thrombocytopenic purpura is diagnosed by excluding other conditions

Place in order the sequence of events leading to oxygen titration in COPD patients - Prolonged hypercapnia - Oxygen titration levels kept low to stimulate ventilation - Oxygen, levels, rather than carbon dioxide, serves as signal for ventilation - Poor pulmonary gas exchange - Resetting of chemoreceptors to higher carbon dioxide levels

In COPD there is poor gas exchange which leads to prolonged hypercapnia. This prolonged hypercapnia resets the chemoreceptors to higher carbon dioxide levels. Oxygen levels serve as signal for ventilation, so oxygen levels titrated at lower level ( Poor pulmonary gas exchange - Prolonged hypercapnia - Resetting of chemoreceptors to higher carbon dioxide levels - Oxygen, levels, rather than carbon dioxide, serves as signal for ventilation - Oxygen titration levels kept low to stimulate ventilation)

The nurse sees on the patient's chart the diagnosis of immunoglobulin A (IgA) deficiency. Which laboratory values support this diagnosis? Select all that apply a. Normal IgG levels b. Normal IgM levels c. Low or absent IgA levels d. Low or absent T cells e. Declining CD4 cells

In IgA deficiency, IgA levels may be low, but other Ig levels may be normal (a, b, c) IgA deficiency does NOT impact T cells

A morbidity obese patient is to receive a procedure in which an upper portion of the stomach is attached the small bowel. Which of the following terms describe this procedure? a. Gastric bypass b. Gastric sleeve c. Liposuction d. Gastric banding

In a gastric bypass, the lower stomach and duodenum are bypassed as the upper stomach is attached to the small bowel. In gastric sleeve, a large portion of the stomach is removed, liposuction removes subcutaneous fat, and in gastric banding, a band is placed around a portion of the stomach, limiting space available

A patient presents with metabolic alkalosis with following values: pH of 7.2 and bicarbonate of 50 mEq/L. Which would the nurse expect to find as signs of compensation? a. Decreased ventilation b. Hypocapnia c. Increased red blood cell (RBC) synthesis d. Decreased partial pressure oxygen

In alkalosis, ventilation is suppressed in order to retain carbon dioxide to lower pH. Hypercapnia, NOT hypercapnia, is a compensation for alkalosis. RBCs levels and partial pressure do NOT change in compensation for alkalosis

A patient is wanting to learn how aortic stenosis may increase his risk for myocardial infarction. Please place the following events in the proper sequence - Increased pressure developed by left ventricle (LV) - Myocardial ischemia - Aortic valve does NOT open properly - Left ventricular hypertrophy - Inadequate circulation to enlarged left ventricle tissue

In aortic stenosis, the aortic valve does NOT open properly. This causes the left ventricular to increase pressure and hypertension in response. The increase in LV size may NOT have adequate circulation. This may lead to myocardial ischemia ( Aortic valve does NOT open properly - Increased pressure developed by left ventricle (LV) - Left ventricular hypertrophy- Inadequate circulation to enlarged left ventricle tissue - Myocardial ischemia)

A patient suffering for diabetes insipidus is given antidiuretic hormone (ADH).. The result is increased fluid retention and decreased urine output. Which of the following conditions does the patient have? a. Nephrogenic diabetes insipidus b. Central diabetes insipidus c. Diabetes mellitus d. Syndrome of inappropriate diabetes insipidus (SIADH)

In central diabetes insipidus, the kidneys can respond to antidiuretic hormone (ADH). Giving ADH and seeing fluid retention increase show that the kidneys can respond to ADH when its present. The problem is a lack of ADH. In nephrogenic diabetes insipidus, the kidneys cannot respond to ADH, so giving ADH will NOT increase fluid retention. Diabetes mellitus is NOT associated with issues of ADH. In SIADH, ADH levels are already elevated

A nurse learns one of her patients has a diagnosis of hemolytic uremic syndrome. The patient is a child. Which of the following is the most likely cause? a. A result of metastatic cancer treatment b. Escherichia coli infection c. Severe trauma d. Medication trauma

In children infections with E coli is the most common cause of hemolytic uremic syndrome

A nurse is trying to explain to a patient who has been diagnosed with endometriosis what this means. Which of the following would be correct for the nurse to share? a. You are suffering anovulatory cycles leading to infertility b. Your uterus is growing larger than normal, impinging on other tissues c. Your uterine lining has become infected d. Portions of your uterine lining are growing outside the uterus

In endometriosis, portions of the uterine lining implant outside of the uterus

A 52 year old woman believes she is experiencing menopause. Which of the following would support this woman's assessment? a. Laboratory values reveal elevated follicle-stimulating hormone (FSH), luteinizing hormone (LH) and estrogen b. Menstrual periods are occurring every 33 days, rather than 28 days c. Laboratory values reveal elevated FSH and lower than expected estrogen levels d. The woman's age determines menopause

In menopause, the ovaries cease to respond to normal signals. Follicle stimulating hormone (FSH) is high, while the ovarian response, estrogen production, remains low. Increasing length between periods does NOT define menopause. The age of menopause were among women (c)

A nurse notes paraphimosis on a patient's chart. Which of the following can she expect to observe? a. Only one testicle in the scrotal sac b. Urethral opening improperly located c. Constricted foreskin d. Foreskin in retracted position

In paraphimosis, the foreskin remains in a retracted position. It does NOT affect the testes. Hypospadias is when the urethral opening is improperly positioned on the penis

A patient wants to know how her polycystic ovarian syndrome (PCOS) is linked to her fertility problems. Which of the following is the correct explanation? a. In PCOS, the eggs of the ovaries do NOT develop and remain immature b. In PCOS, the corpus luteum degenerates too quickly c. In PCOS, no eggs are present in the ovaries d. In PCOS, eggs begin to develop, but ovulation does NOT occur

In polycystic ovarian syndrome (PCOS) ovulation is disrupted

Laboratory values for a patient reveal elevated parathyroid hormone (PTH) and elevated serum calcium levels. What condition is most likely? a. Primary hypoparathyroidism b. Primary hyperparathyroidism c. Secondary hyperparathyroidism d. Primary hyperthyroidism

In primary hyperparathyroidism, elevated parathyroid hormone (PTH) causes an elevation in serum calcium levels

Which of the following substances may be needed as a supplement for a patient suffering renal osteodystrophy? a. Iron b. Vitamin D c. Glucosamine d. Vitamin E

In renal osteodystrophy, a patient suffers bone breakdown due to low calcium retention by the kidneys. Vitamin D helps with calcium absorption

A group of nursing students are discussing the role of cortisol in the stress response. Which nursing student needs to study more? a. Student 1: Cortisol is NOT beneficial for a stress response because it suppresses the immune system b. Student 2: Long-term cortisol suppresses the immune system c. Student 3: Cortisol mobilizes glucose and fats for fuel d. Student 4: Cortisol is released from the adrenal cortex

In the acute response of stress, cortisol stimulates the immune system and is beneficial in acute situations

A nurse is reviewing the ovulation cycle with a patient suffering from infertility. Please place the following steps in the correct sequence for this cycle - Estrogen levels increase - Increased follicle-stimulating hormone (FSH) - Luteinizing hormone (LH)/FSH surge - Egg development - Ovulation

In the ovarian cycle, follicle-stimulating hormone (FSH) levels increase, which stimulate egg development. This causes an increase in estrogen secretion. Midway through the cycle a surge in FSH/LH causes ovulation ( Increased follicle-stimulating hormone (FSH) - Egg development - Estrogen levels increase - Luteinizing hormone (LH)/FSH surge - Ovulation )

Place the following sequence of events in the proper order showing how glucose may show up in the urine of a type 1 diabetes mellitus (DM) patient? - Glucose appears in urine - Blood glucose levels remain elevated - Antibodies destroy pancreatic beta cells - Transport maximum of kidney exceeded - No insulin produced

In type 1 diabetes mellitus, antibodies destroy the beta cells. No insulin is produced, which cause blood glucose levels to elevate. The elevated glucose exceeds the transport maximum of the kidney, which results in glucose appearing in the urine ( Antibodies destroy pancreatic beta cells - No insulin produced - Blood glucose levels remain elevated - Transport maximum of kidney exceeded - Glucose appears in urine)

A morbidity obese patient is part of a clinical study examining adipokines, specifically adiponectin. Place in order the following steps that explain the relationship between obesity, adiponectin, and disease - Increased obesity - Increased blood glucose levels - Reduced adiponectin - Increased risk for diabetes mellitus - Decreased cell sensitivity to insulin

Increased obesity leads to reduced adiponectin levels. When adiponectin levels are reduced, there is decreased sensitivity to insulin. The decreased sensitivity to insulin raises blood glucose levels, increasing the risk for diabetes mellitus ( Increased obesity- Reduced adiponectin - Decreased cell sensitivity to insulin - Increased blood glucose levels - Increased risk for diabetes mellitus)

A patient wants to know how he can prevent future kidney stones. Which of the following is an appropriate recommendation? a. Drink at least 3/L per day b. Take calcium supplments c. Maintain current dietary choices d. Take a daily prophylactic antibiotic

Increasing hydration can help to prevent stone formation. Most kidney stones are comprised of calcium, so increasing calcium intake may increase the risk of stone formation. A prophylactic antibiotic would NOT be helpful

A friend states that her daughter has been diagnosed with bulimia nervosa. Which characteristics may you expect the daughter to display? Select all that apply a. Very thin, pale appearance b. Age 35 to 40 years c. A normal body mass index (BMI) d. Very restricted eating and excessive exercise e. Cycles of binging and purging

Individuals with bulimia nervosa often have a normal BMI and is marked with cycles of binging and purging at least twice weekly for three months or longer. The average onset of bulimia nervosa is 19 years

A nursing educator is working with students. Which of the following indicate correct understanding by the students in regards to hyperglycemia hyperosmolar syndrome (HHS)? a. Ketones will be elevated b. The condition will have a sudden, rapid onset c. The patient will likely present with dizziness and hypoglycemia d. The risk of developing this condition increases with infection or severe stress

Infections or illness can increase the risk for hyperglycemia hyperosmolar syndrome (HHS). HHS does NOT form ketones. It normally develops slowly and involves hyperglycemia, NOT hypoglycemia

A patient is diagnosed with infective endocaarditis. What is an appropriate response by the nurse? a. Inform the family of this highly contagious infection and that quarantine will be needed b. Restrict fluid and food intake c. Prepare patient for immediate surgeryy d. Prepare for IV administration of antibiotics

Infective endocarditis is normally treated with IV antibiotics. Infective endocarditis is NOT contagious. Fluid and food restriction are NOT needed with infective endocarditis. It is NOT treated with emergency surgery

Which component enables the blood vessels to dilate and become more permeable during the vascular phase of inflammation? a. Blood platelets b. Inflammatory mediators c. White blood corpuscles d. C-reactive proteins

Inflammatory mediators histamine enables the blood vessels to dilate and become more permeable at the site of inflammation during vascular phase III

A patient with rheumatoid arthritis, an autoimmune condition, is upset because she has been prescribed corticosteroids to suppress the immune system. The patient feels this is wrong, as she has learned that cortisol activates the immune response. Which of the following would be appropriate response by the nurse? a. Support the patient's views, as patients seem resistant to correction b. Inform the patient of the different effects of cortisol on the immune system, whether the cortisol is present for a short duration or long term c. Offer to change the prescription d. Tell the patient she is misinformed. Corticosteroids do NOT work on immune system. They were prescribed to help her bones grow

Inform the patient of the different effects of cortisol on the immune system, whether the cortisol is present for a short duration or long term. Corticosteroids do affect the immune system and do NOT promote bone growht

Nursing students are creating a flow chart describing the immune response and how white blood cells move to areas of injury and infection. Please place the steps of the immune response in the proper order - Increased capillary permeability - Injury occurs - Phagocytosis of foreign substances - Histamine released from mast cells - WBC's attracted by chemostatic signals

Injury occurs, followed by histamine being released from mast cells. Capillary permeability increases followed by WBCs being attracted by chemostatic signals. The final step is phagocytosis of foreign substances ( Injury occurs - Histamine released from mast cells - Increased capillary permeability - WBCs attracted by chemostatic signals - Phagocytosis of foreign substances )

Which of the following comments by a patient shows understanding of the dietary changes that may help in interstitial cystitis? a. "I should probably give up or limit my morning cup of coffee" b. "I need to become a vegetarian" c. "I need to drink less water each day in order to urinate less" d. "Spicy food may help alleviate my symptoms"

Interstitial cystitis can be aggravated by certain food choices, such as caffeine and alcohol

Which of the following are backward effects of right ventricular failure? Select all that apply a. Pulmonary edema b. Jugular vein distention c. Decreased arterial blood pressure d. Hepatomegaly e. Paroxysmal nocturnal dyspnea

Jugular vein distention occurs in right side heart failure as fluid backs up into the veins. If the right side of the heart fails, then fluid may back up in the liver, causing hepatomegaly. Pulmonary edema, decreased arterial blood pressure, and paroxysmal nocturnal dyspnea are more common with left side heart failure

A child is diagnosed with Kawasaki disease. Which category would the nurse place this disorder when explaining it to the parents? a. Vasculitis b. Aneurysm c. Aortic dissection d. Peripheral arterial disease

Kawasaki disease is a form of vasculitis

For which disorder may a nurse suggest Kegel exercise? a. Stress incontinence b. Asymptomatic bacteriuria c. Urinary tract infection (UTI) d. Overactive bladder

Kegel exercises may be used to increase the strength of the pelvic floor muscles decreasing incontinence

Which of the following results is more indicative of type 1 rather than type 2 diabetes mellitus? a. Hyperglycemia b. Polyuria c. Ketones in urine d. Glucose in urine

Ketone formation more commonly occurs in type 1 diabetes mellitus due to lack of insulin. Hyperglycemia, polyuria, and glucosuria occur in BOTH types of diabetes

Which of the following would indicate to a nurse that a patient has type 1 diabetes mellitus (DM) rather than type 2 DM? a. Glucose in urine b. Ketones in urine c. Blood in urine d. Albumin in urine

Ketones in the urine are more common in type 1 diabetes mellitus. Blood in the urine is uncommon in both types of diabetes. Glucose and albumin (due to kidney damage) in the urine may develop with both types of diabetes

A patient with severe renal failure may develop osteoporosis. Which factor may play a role in this development? a. Kidney are unable to activate vitamin D b. Kidney are unable to activate vitamin C c. Renal patients must avoid the sunlight d. Kidney are unable to synthesize erythropoietin

Kidneys play a role in activating vitamin D, which helps with calcium absorption

A karyotype shows one extra chromosome. What condition may be present? Select all that apply a. Cystic fibrosis b. Turner syndrome c. Down syndrome d. Neurofibromatosis e. Klinefelter syndrome

Klinefelter syndrome is 47 XXY and Down syndrome is due to three copies of chromosome 21. Cystic fibrosis and neurofibromatosis are due to a genetic mutation and does NOT appear as an extra chromosome. Turner syndrome is 45 X0 and would appear with one less chromosome than normal

A computed tomography (CT) scan shows small areas of infarction. The patient has no signs and symptoms of neurological changes. What is the best description of this presentation? a. Lacunar infarct b. Transient ischemic attack (TIA) c. Subarachnoid hemorrhage d. Ischemic stroke

Lacunar infarcts are small areas of ischemia. Transient ischemic attack (TIA) present with signs and symptoms, a subarachnoid hemorrhage is an area of bleeding, and ischemic strokes presents with symptoms

A woman states that endometriosis runs in her family. She would like to know whether she has the condition or NOT. What tool is used to definitively diagnose endometriosis? a. Analysis of blood values of follicle-stimulating hormone (FSH) and luteinizing hormone (LH) b. Laparoscopy c. Physical examination d. Diagnosis only by ruling out other disorders

Laparoscopy is the diagnostic tool for endometriosis

A nursing student wants to review how a left ventricular myocardial infarction may cause pulmonary edema. Please place the following events in the proper sequence - Left ventricular is damaged - Blood backs up into the pulmonary veins - Blood pools in the left atrium - There is increased hydrostatic pressure in the pulmonary capillaries - Blood pools in the left ventricle

Left ventricular damage will cause poor ejection and blood pooling in the left ventricle. The blood will back up into the right atrium and into the pulmonary veins, increasing fluid pressure in the lungs, leading to edema (Left ventricular is damaged - Blood pools in the left ventricle - Blood pools in the left atrium - Blood backs up into the pulmonary veins - There is increased hydrostatic pressure in the pulmonary capillaries)

A patient presents with the following complaint: "This one gland in my groin area is so swollen. It bothers me a little bit, but I am NOT sure if i should be worried". What is an appropriate response by the nurse? a. You likely have a severe infection in the groin area b. Do NOT worry. A single lymph node swelling is common and shouldn't concern you c. Surgery is an option for the removal of the lymph node d. Let us discuss further signs and symptoms and what the possibilities are for examining the lymph node

Let us discuss further signs and symptoms and what the possibilities are for examining the lymph node. Further evaluation is needed before a clinician provides a diagnosis and a single swollen lymph node can be indicative of a serious disease

A nurse is reviewing laboratory values. Which of the following would be indicative of a suppressed immune response in a patient? a. Elevated erythrocyte sedimentation rate (ESR) b. Leukocytes c. Elevated C-reactive protein (C-RP) d. Leukopenia

Leukopenia has decreased WBC levels. Elevated ESR and C-RP are elevated during inflammation. Leukocytosis is an increase in WBC number

A mother has been given a medication for her son's illness. When she researches the medication, she finds it blocks leukotrienes. What is the most likely illness her son has? a. A viral infection b. Asthma c. Peptic ulcer disease d. Rheumatoid arthritis

Leukotrienes cause bronchospasm. Thus, a medication that blocks leukotrienes may be helpful for asthma.

A nurse is told to look for Levine's sign. Which of the following should she look for? a. Patient with vertigo b. Patient with tense jaw c. Patient with fist over sternum d. Patient with pain radiating down arm

Levine's sign is common with myocardial infarction and involves a patient clutching their fist over the sternum

A patient presents with consistently low hemoglobin saturation. This may result in which of the following appearing? a. Swelling in the facial area b. Cyanosis c. Reduced stimulus for respiration d. Peripheral edema

Low oxygen levels resulting in low hemoglobin saturation may induce the appearance of cyanosis. Low hemoglobin saturation would NOT cause swelling in the facial area, respiration reduction, nor peripheral edema

A nursing student knows that patients may NOT have a good understanding of medical acronyms, so he always states the entire name rather than the acronym. Which of the following should he state for MRSA? a. Methicillin-resistant streptococcus aureus b. Methicillin-resistant staphylococcus aureus c. Methicillin-resistant streptococcus antibodies d. M=ycobaterium-resistant streptococcus aureus

MRSA stands for methicillin resistant staphylococcus aureus

A nursing student is looking at a white blood cell (WBC) differential and is confused, as the cell type "macrophages" is NOT listed. Which of the following choices correctly explains what has happened? a. A mistake has been made. Macrophages should be listed on a WBC differential b. Monocytes are listed on a WBC differential, and some monocytes later transform into macrophages c. Macrophages are common immune cells, but they are found on the lymph nodes, NOT blood samples d. Macrophages cannot be measured because they are transient cells, appearing for a moment, then disappearing

Macrophages arise from monocytes which are measured in a WBC (b)

Bernice was recently diagnosed with malignant melanoma. Which of the following statements show that the clinician working with Bernice understands the risk involved with a malignant neoplasm? a. "A malignant neoplasm has an increased risk for metaplasia" b. "Bernice will likely NOT require any follow-up appointment" c. "Bernice's results indicate a tumor with well differentiated cells" d. "Bernice is NOT suffering from a neoplasm"

Malignant neoplasms are MORE likely to metastasize than benign neoplasms

A skin biopsy reveals a malignant neoplasm. Which statement supports this conclusion? a. The biopsy revealed well-differentiated cells b. The biopsy contained normal-appearing cells c. The biopsy revealed cells with disorganized arrangement d. The biopsy revealed clear borders of neoplastic growth

Malignant neoplasms contain cells that are growing in a disorganized, random pattern. Malignant neoplasms are abnormal cells, have poorly differentiated cells, and have poorly defined borders

A patient has acute pharayngitis and is requesting antibiotics for strep throat. What is the appropriate response? a. Ask the patient which type of antibiotics he prefers b. Inform the patient that most cases of acute pharyngitis are viral, but further testing is needed to rule out a streptococcal infection c. Ask the patient to open his mouth and examine the throat area. Based on findings, decide whether or NOT antibiotics are needed d. Ask the patient to wait three to five days. If his throat is still store, he should return

Many cases of acute pharyngitis are due to viral infections. To rule out streptococcal infection, which would require antibiotic treatment, further testing would be needed. Further testing and evaluation such as rapid strep test should be performed before antibiotics are prescribed

A young child, age 12 years, wants to play competitive sports. The nurse working with the child is aware that some genetic disorders can impact a child's ability to participate in sports. Which of the following genetic disorders may the nurse consider being a risk for a child participating in competitive sports? Select all that apply a. Familial adenomatous polyposis b. Marfan syndrome c. Familial hypercholesterolemia d. Ehlers-Danlos syndrome e. Cystic fibrosis

Marfan syndrome affects the connective tissue, especially of the heart, which increases the risk of cardiac difficulties during intense competition. Ehlers Danlos syndrome affects collagen synthesis and makes the joints hypermobile, leading to increase risk for injury. Cystic fibrosis affects pulmonary function and may make sports participation difficult. Familial hypercholesterolemia increases LDL levels so exercise may be beneficial in helping to mitigate the risk for cardiac disease

A patient arrives at the clinic for follow up for recently diagnosed folic acid deficiency. Which of the following likely contributed to this diagnosis? a. Microcytic anemia b. Elevated mean corpuscular volume (MCV) c. Hypochromic anemia d. Glossitis

Mean corpuscular volume (MCV) indicates the volume of red blood cells (RBCs). In folic acid deficiency, the cells are enlarged. Folic acid deficiency does alter the color of the cells and vitamin B12 deficiency causes glossitis, NOT folic acid deficiency

Which of the following is correct regarding benign prostatic hyperplasia (BPH)? a. BPH is a form of cancer b. Drug treatment for BPH may take 6 to 12 months for full benefit c. Surgical procedures are NOT used to correct BPH d. BPH is NOT treated with medications

Medication treatment for benign prostatic hyperplasia (BPH) may take six months or more for full efficacy. It is NOT a form of cancer. Can be treated with surgery and medications

A nurse notices a medication for erectile dysfunction on a patient's chart. Which class of medications should the nurse check to see that the patient is also taking? a. Nitrates b. Statins c. Proton pump inhibitors d. Beta-adrenergic agonists

Medications for erectile dysfunction can result in hypotension if taken with nitrate medications. Statins lower cholesterol levels, proton pump inhibitors lower stomach acid production and beta adrenergic agonists are used to dilate bronchioles

A teenager presents with the signs and symptoms of meningococcus meningitis. Which of the following does the nurse know is appropriate? a. Wait and see if the infection resolves on its own b. Wait at least 24 hours for culture and antibiotic sensitivity testing results c. Begin IV fluid administration d. Begin antibiotic treatment immediately

Meningitis can be fatal, treatment must be started immediately. Wait and see approach and waiting for culture results is too long in meningitis. IV fluids are NOT adequate treatment for meningitis

A nurse sees a patient with hyperkalemia due to acid-base disturbance. Which acid-base disturbance most likely caused by hyperkalemia? a. Respiratory acidosis b. Respiratory alkalosis c. Metabolic acidosis d. Metabolic alkalosis

Metabolic acidosis causes potassium to shift out of the cells, leading to hyperkalemia. Alkalosis causes potassium to shift into the cells, leading to hypokalemia

A patient presents with metabolic alkalosis. Please place the following events in the proper sequence as to how the pulmonary system will attempt to compensate for the change in pH - pH returning to normal - Increased serum pH - Increased hydrogen ion concentration - Increased carbon dioxide levels - Decreased ventilation

Metabolic alkalosis is marked by increased serum pH, which causes a suppression of ventilation. Carbon dioxide levels elevate, which increases H+ ion concentration, which reduces pH towards normal ( Increased serum pH - Decreased ventilation - Increased carbon dioxide levels - Increased hydrogen ion concentration - pH returning to normal )

Endoscopic results reveal metaplastic changes in the duodenum. Which of the following statements describes these cellular changes? a. Metaplasia indicates an increase in cell size b. Metaplasia is indicative of cancer c. Metaplasia is the replacement of one cell type by another d. Metaplasia and dysplasia are synonymous

Metaplsia is a cellular adaption in which a cell type is changed to better accommodate environmental stressors. Dysplasia is abnormal cells

A child frequently forgets to take his asthma medications and frequently has mild asthma attacks. Which of the following is an appropriate response? a. As long as the attacks are mild, it is OK that the medication is NOT being used b. Rescue medications are the most important treatments in asthma. If he does NOT require use of rescue medications, then he is fine c. Prevention of mild asthma attacks is important because even mild attacks can cause changes in the pulmonary structures d. The child should be trained in breathing exercises. If effective, these can replace the use of medications

Mild asthma attacks may still cause restructuring of the bronchioles. Prevention of attacks is important

On auscultation, a nurse hears an opening snap, diastolic murmur at the 5th intercoastal space. Which valve disorder is most likely? a. Mitral valve insufficiency b. Aortic insufficiency c. Mitral stenosis d. Aortic stenosis

Mitral stenosis would be heard at the 5th intercoastal space and is a diastolic murmur. Mitral valve insufficiency would be a systolic murmur, aortic insufficiency would be heard at the 2nd intercoastal space, and aortic stenosis is heard at the second intercoastal space

A patient has developed mitral valve insufficiency due to damage from a myocardial infarction. Which chamber of the heart is likely to experience an increased volume of blood due to this injury? a. Right atrium b. Right ventricle c. Left atrium d. Left ventricle

Mitral valve insufficiency will cause blood from the left ventricle to enter the left atrium

Place the following pain management options in order from management techniques that are for the most severe pain to the least severe pain - No medication - Nonopioid plus adjuvant - Codeine - Morphine - NSAID

Morphine is an opioid used for severe pain. Codeine is a milder opioid, nonopioid medications with an adjuvant can be used or a medication such as NSAIDs for the mildest pain ( Morphine - Codeine - Nonopioid plus adjuvant, - NSAID - No medication )

A patient has hypertension with no known etiology. Which of the following is a correct interpretation? a. Further diagnostic tests are needed because all forms of hypertension have a known cause b. Hypertension of unknown origin is very rare c. If hypertension results from unknown causes, then hypertension causes less damage in the body d. A number of risk factors may contribute to hypertension, even if the direct cause is NOT known

Most cases of hypertension have no known etiology, however several risk factors have been identified. Approximately 905 of patients with hypertension have no known cause for the disease

Laboratory analysis reveals that a patient's gallstones were primarily made of cholesterol. Which of the following statements is correct? a. The results surprising because less than 10% of gallstones are cholesterol based b. The results are expected because more than 80% of gallstones re cholesterol based c. Typically, there is a 50-50 chance that a gallstone will be cholesterol based d. The laboratory result is incorrect. Gallstones are made of bile salts, NOT cholesterol

Most gallstones are composed of cholesterol

A patient is suffering from chronic mucocutanous candidiasis. As the nurse discusses this condition with the patient, which immune process does the nurse indicate to be functioning abnormally? a. T cells only b. B cells only c. Immunoglobin (Ig) synthesis d. Both B and T cells

Mucocutaneous candidasis is a problem with T cell function. Mucocutaneous candidasis develops when the T cells cannot respond appropriately to a Candida infection

A woman is diagnosed with stress incontinence. Which of the following are risk factors for this disorder? Select all that apply a. Multiple childbirths b. Low estrogen levels c. Female sex d. Young age e. Postmenopausal

Multiple childbirths can lessen the strength of the pelvic floor muscles. Low estrogen levels, as occur with menopause, can cause atrophy of some structures. Females are at greater risk of developing stress incontinence. Postmenopausal women have lower estrogen, which can result in increased urinary incontinence risk (a, b, c, e)

Which of the following leukemias or lymphomas may involve abnormalities in B cells or T cells ? Select all that apply a. Hodgkin's lymphoma b. Multiple myeloma c. Non-Hodgkin's lymphoma d. Acute myelogenous leukemia e. Acute lymphocytic leukemia

Multiple myeloma involves abnormal plasma B cells. Hodgkins lymphoma is due to B cell abnormality. Non hodgkins lymphoma involves B cells, T cells, and NK cells. Acute lymphocytic leukemia involves B cells or T cells (a, b, c, e)

A nurse needs to assess a patient to Murphy's sign. What instructions does she give the patient? a. Stand with your eyes closed and feet together b. Pull your knees to your chest and then extend one leg at a time c. Lie supine and take an inspiration. Pressure will be placed upon your abdomen and then released d. Begin hyperventilating for three seconds and then hold your breath while your abdomen is palpated

Murphys sign is assessed while the patient lies supine and pressure is placed on the abdomen

Anemia is present in a patient with laboratory results revealing (NCNC) anemia. Which of the following does the nurse think is the most likely cause? a. Vitamin B12 deficiency b. Folic acid deficiency c. Iron deficiency d. Blood loss

NCNC anemia indicates normal cell color and normal cell size. This type of anemia tends to develop with blood loss. Vitamin B12 and folic acid deficiency results in megaloblastic cells. In iron deficiency anemia, the cells are hypochromic and micocytic

A nursing student is working on the difference between NSTEMI and STEMI. Which of the following is associated with NSTEMI? a. S-T segment evaluation b. Partial coronary artery occlusion c. Transmural damage d. Complete coronary artery occlusion

NSTEMI indicates a partial occlusion. S-T segment elevation and transmural damage tends to occur in STEMI. STEMI indicates complete coronary occlusion

A nurse notices that a patient's temperature is elevated three days after the patient has suffered a massive myocardial infarction (MI). What possible explanation, related the myocardial infarction, is likely? a. Inflammatory response due to necrotic tissue b. Hyperthyroidism c. Disrupted blood flow to the hypothalamus d. Elevated metabolic rate

Necrotic tissue from an MI can cause an inflammatory reaction. It is the MOST likely cause of MI with disrupted blood flow to the hypothalamus being another cause. Hyperthyroidism would NOT be related to an MI. Basal metabolic rate does NOT change substantially after an MI

A mother hears a nurse mention that her son, who has been hospitalized for some time due to an ongoing infection, has negative nitrogen balance. Which of the following explains what is occurring in the woman's son? a. The son is consuming too many protein-rich foods b. The son is suffering muscle breakdown and amino acids are being lost c. The son is demonstrating an abnormal ventilation pattern d. The son is putting on excess weight

Negative nitrogen balance refers to the loss of amino acids due to muscle breakdown.

A nurse is counseling a patient who has been diagnosed with a sexually transmitted infection. Which of the following is a reportable disease? a. Human papillomavirus b. Neisseria gonorrhoeae c. Herpes simplex virus d. Pelvic inflammatory disease

Neisseria gonorrhoeae is a reportaable infection. Human papillomavirus, herpes simplex virus, and pelvic inflammatory disease is NOT a required reportable infection

A patient is suffering declining kidney function due to an autoimmune disease. The patient asks the nurse how much of his kidney must suffer damage to lead to end stage renal disease (ESRD). Which of the following is the appropriate response by the nurse? a. Usually ESRD develops when less than 10% of the nephrons are damaged b. Usually ESRD develops when 10% to 20% of the nephrons are damaged c. Usually ESRD develops when 65% to 75% of the nephrons are damaged d. Usually ESRD develops when 90% to 95% of the nephrons are damaged

Nephron damage of approximately 90% to 95% will place a patient in end stage renal failure

With a quick perusal of a white blood cell (WBC) differential, a nurse would expect which of the following in a patient with normal value? a. Equal distribution of all five cell types b. Macrophages accounting for 60% to 70% of the cells c. Neutrophils are the most prevalent cell types d. Agranulocyte percentage exceeds granulocyte percentage

Neutrophils account for approximately 65% of WBCs. Normal values indicate different percentages for the various WBC types (c)

A patient presents with acute inflammation. While reviewing laboratory values, which values will the nurse expect to see elevated? Select all that apply a. RBCs b. Neutrophils c. C-RP d. ESR e. Platelets

Neutrophils are first responders and elevate with inflammation. C-RP and ESR are markers of inflammation. RBCs and platelets are NOT altered by acute inflammation

Which cell type are also known as "polymorphonuclear" cells? a. Basophils b. Neutrophils c. Monocytes/Macrophages d. Esoinophils e. Lymphocytes

Neutrophils are often referred to as polymorphonuclear cells (PMNs)

A nurse is looking at a microscope slide of stained white blood cells. Which cells will show granules with staining? a. Neutrophil b. Monocyte c. Macrophage d. Eosinophil e. Basophil

Neutrophils, eosinophils, and basophils are types of granulocytes. Monocytes and macrophages are NOT granulocytes

A patient has consistent chest pain with exertion due to partial occlusion of a coronary artery. The patient is given a prescription for nitroglycerin tablets. Which of the following processes is the purpose of the prescription? a. Pain blockage b. Vasoconstriction c. Clot buster d. Vasodilation

Nitroglycerin dilates coronary arteries

A nursing educator tells students that endothelial tissue is an active tissue secreting many factors, one being nitric oxide. Based on this information, the nurse educator asks students to relate why nitroglycerine may be used to treat angina pectoris. Which of the following is the correct response? a. In angina pectoris, coronary arteries are occluded. Nitroglycerin serves to dilate these arteries b. Nitroglycerin will stimulate clot dissolution, reducing chest pain c. Nitroglycerin is used to block pain sensations arising in angina pectoris d. Nitroglycerin will stimulate clot formation in response to endothelial damage, reducing blood loss

Nitroglycerin is a vasodilating agent. Nitroglycerin does NOT dissolve clots, block pain signals, nor cause clot formation

A patient is worried about his risk for hepatitis C and is requesting to receive the vaccine. Which of the following is the correct response by the nurse? a. At this time, there is no vaccine for hepatitis C b. Hepatitis B and C vaccines are normally given together. Do you want both? c. The hepatitis C vaccine requires three doses d. The hepatitis C vaccine is given a single immunization

No vaccine is currently available for hepatitis C

A patient was recently diagnosed with lymphoma. In the words of the patient, the lymphoma is the "more common" type. Which type of lymphoma does the patient have if his or her information is correct? a. Hodgkin's lymphoma b. Non-Hodgkin's lymphoma c. Multiple myeloma d. Myelodysplastic syndrome

Non Hodgkins lymphoma accounts for 80% of lymphomas. Multiple myeloma and myelodysplastic syndrome are NOT forms of lymphoma

A patient wants to know the difference between Hodgkin's lymphoma and non-Hodgkin's lymphoma. Which of the following would be correct in formation to share with the patient? Select all that apply a. These types of lymphomas develop in different lymph nodes b. Hodgkin's lymphomas occur only in children c. Non-Hodgkin's lymphoma is the more common type of lymphoma d. Non-Hodgkin's lymphoma has only one type, while there are 20 types of Hodgkin's lymphomas e. Non-Hodgkin's lymphoma develops from several cell types, while Hodgkin's lymphoma develops from one cell type

Non-Hodgkin's lymphoma account for 80% of lymphomas and it develops from B, T, and NK cells, whereas Hodgkins lymphoma develops from B cells

Glomerular filtration rate (GFR) is recorded as 12 ml/min. What is the correct interpretation by the nurse? a. Renal function has returned to normal. The patient is ready to be discharged b. Renal function is NOT normal, as the patient is in stage 3 chronic renal failure (CRF) c. Renal function is above normal levels d. Renal function indicates stage 5 CRF

Normal renal function requires a GFR of 90 ml/min. The patient's level is severely reduced, placing him in stage 5 of renal disease

A nurse is treating a patient with kidney dysfunction. She notices that serum albumin levels are 2.8 g/dl. Which of the following is the appropriate interpretation by the nurse? a. Albumin levels are normal b. Albumin levels are increased. Glomerular filtration rate (GFR) must be reduced c. Albumin levels are decreased, due to loss in urine d. Albumin levels are unaffected by renal function; thus, this value is NOT helpful

Normal serum albumin levels are around 3.5 to 5.4 mg/gL. Albumin can be lost in the urine with kidney damage

A patient needs a blood transfusion. The patient has O+ blood. Which of the following blood types can the patient receive in a transfusion? Select all that apply a. A+ b. AB+ c. O+ d. O- e. AB-

O+ blood can be received as a transfusion. O- blood can be received as a transfusion because it lacks the A and B antigens. O+ blood has A and B antibodies, therefore, no blood type that has A or B antigen can be received as transfusion

A nurse sees on a patient's chart a body mass index (BMI) of 32. When the nurse walks in to see the patient, which will she expect to see? a. A patient with normal weight b. A patient who is obese c. A patient who is morbidly obese d. A patient who is underweight

Obesity is associated with a BMI of at least 30 and morbid obesity is a BMI of 40 or more. Normal BMI is between 19 to 25

A patient has oliguria. Which of the following interpretations by the nurse is correct? a. The patient has increased urine output compared to normal b. The patient has a urinary stone c. The patient's urine output is less than 400 ml/day d. The patient's glomerular filtration rate (GFR) must be elevated

Oliguria is decreased urine output, less than 400 ml/day> Oliguria would be associated with a decrease in GFR

A patient is confused about all the types of dietary fats. She has heard that one type is better than others for reducing triglyceride levels. Which type may the nurse discuss for this purpose? a. Omega 3 fatty acids b. Solid fats, such as lard c. Cholesterol-rich fats d. Fats from cheese and dairy

Omega 3 fatty acid intake has been related to improved blood lipid profiles. Saturated fat intake such as cheese and dairy at high levels and cholesterol rich fats have a negatively effect on the blood lipid profile

On the condition listed, which are associated with pathological calcification? Select all that apply a. Jaundice b. Microcalcification in breast lesion c. Arteriosclerosis d. Xantoma e. Aortic sclerosis

On mammography, areas of calcification can be observed when a malignant tumor is present, aortic sclerosis develops due to calcium deposition on the aortic valve, and a step in arteriosclerosis is the calcification of a plaque within the vessel. Xanthoma form due to cholesterol deposition and jaundice is due to a bilirubin buildup

A nurse is counseling a patient with several risk factors for deep vein thrombosis (DVT). Which of the following are increasing the patient's risk for DVT? Select all that apply a. Oral contraceptive usage b. No recent surgeries c. Smoking d. Moderately active lifestyle e. Obesity

Oral contraception, smoking, and obesity increases the risk for clot formation (a, c, e)

Given the prevalence in the United States, if a patient states, "I have diabetes mellitus" and provides no further information, which is the most likely type? a. Type 1 diabetes mellitus b. Type 2 diabetes mellitus c. Gestational diabetes mellitus d. Drug induced diabetes mellitus

Over 90% of cases of diabetes mellitus in the United States are type 2

The nurse reviews a patient's chart. The patient is to have gallbladder surgery and will require a prescription for pain management at home. The nurse notes, however, that the patient has a history of substance abuse. Which of the following possible prescriptions will the nurse be most concerned about? a. Aspirin b. Celecoxib c. Naproxen sodium d. Oxycodone

Oxycodone is an opioid and some people are known to develop addiction to opioid. Aspirin, celecoxib, and naproxen sodium are NSAIDs and NSAID addiction is uncommon

A nurse is worried about an ischemic area of the heart receiving too much oxygen, too rapidly. Why is the nurse concerned? a. The nurse is worried about increased free radical formation due to oxygen metabolism b. The nurse knows that oxygen should never be given to patients in cardiac distress c. With additional oxygen, the ischemic area of heart will produce even more lactic acid d. The nurse should NOT be concerned. Oxygen is always beneficial

Oxygen metabolism can result in free radical formation, which may damage cells further, if the cells are unable to compensate for the free radical formation. Oxygen supplementation is common in patients with myocardial distress, lactic acid is produced in the absence of oxygen, and too much oxygen may cause ischemic reperfusion injury

A client, who is on prolonged bed rest, has developed kidney stones. Which factor could have caused kidney stones? a. Change in the levels of growth hormone b. Change in the levels of glucose c. Change in the levels of parathyroid hormone d. Change in the levels of thyroid hormone

PARATHYROID HORMONE levels are increased due to immobility. This stimulates osteoclastic activity and bone degeneration, which leads to increased amount of calcium in blood stream. Excess calcium which reaches kidney precipitates and forms kidney stones

While studying, a nursing student realizes her flash cards for the sequence of steps in the Gate Control theory of pain have been mixed up. Please place the following steps of the Gate Control theory of pain in the correct order - Transduction - Perception - Painful stimuli - Modulation - Transmission

Pain begins with a painful stimulus that must be transduced into a neuronal signal that is then transmitted to the spinal cord. The pain signal may be modulated before pain is perceived (Painful stimuli, transduction, transmission, modulation, perception)

A clinician is admitting a patient with suspected acute pancreatitis. Which signs and symptoms would support this suspicion? Select all that apply a. Severe abdominal pain radiating to back b. Cullen's sign c. Hypertension d. Diminished bowel sounds e. Petechiae

Pancreatitis may present with severe pain radiating to the back, Cullens sign of bruising in the umbilical are,a, and diminished bowel sounds. Patients with pancreatitis are more at risk for hypotension. Petechiae does NOT present with acute pancreatitis (a, b, d)

A nurse believes that she has detected a heart murmur that a patient developed following a myocardial infarction. Which structures may have been damaged, resulting in this murmur? a. Papillary muscles b. Sinoatrial (SA) node c. Aorta d. Pericardium

Papillar muscles hold the AV valves in place. Damage to these muscle may result in abnormal closure of the valve, causing a murmur. SA node sets the heart rate, aortic damage would NOT occur as a heart murmur, and the pericardium may develop inflammation after a myocardial infarction, NOT producing a heart murmur

A patient has a hyperfunctioning tumor of the parathyroid gland. Which of the following conditions may the patient be most at risk for? a. Coronary event b. Osteoporosis c. Severe weight loss d. Acromegaly

Parathyroid hormone (PTH) causes bone breakdown (b)

Which of the following terms would be used to indicate a patient is suffering from neuropathic pain? a. Sensation b. Paresthesia c. Nociception d. Enkephalin

Paresthesia refers to a pins and needles sensation due to neuropathic pain. Sensation refers to perception of a stimuli, nociception refers to general pain detection, and enkephalins are endogenous opioids

A nurse is speaking to a patient diagnosed with pelvic inflammatory disease. Which of the following is correct? a. The patient is most likely a woman of childbearing age b. The nurse should assure the patient that this is a mild, easily treatable condition c. The nurse should assure the patient that pelvic inflammatory disease does NOT increase the risk of infertility d. The patient likely presented with acute symptoms, which resolved on their own

Pelvic inflammatory disease is most likely in woman of childbearing age. It is a serious disorder, that increases the risk of infertility, and may NOT present with obvious, acute symptoms (a)

A nurse is caring for a patient with periotonitis. Which of the following is appropriate management? Select all that apply a. Watchful waiting b. Peritoneal lavage c. IV fluids and electrolytes d. Antibiotics e. Proton pump inhibitors

Peritoneal lavage can be used to reduce bacterial load. IV fluids and electrolytes will help counteract any losses. Peritonitis involves a bacterial infection, so antibiotics are needed. Peritonitis is a severe condition that needs medical management but proton pump inhibitors are NOT a primary treatment

A client comes to the hospital with petechiae. The complete blood count reports shows a decrease in the platelet count. What should the nurse include in the plan of care based on the information? a. Discourage the use of stool softeners b. Avoid invasive procedures c. Encourage the use of a hard bristle toothbrush d. Assess the temperature every 6 hours

Petechiae is the development of small red or purple spots on the skin. Petechiae happen when there is a decrease in the platelet count. The nurse should avoid invasive procedures because a low platelet count increases susceptibility to bleeding

The nurse is talking with a husband whose wife served in the military. His wife suffered a below- the knee-amputation. The husband states that his wife is constantly complaining of her amputated leg hurting. He is confused as to why the leg can hurt when it is no longer even present. Which of the following ideas explain what is happening? Select all that apply a. Referred pain b. Visceral pain c. Gate-control theory d. Phantom pain e. Neuromatrix pain theory

Phantom pain is a sensation of pain originating in an amputated part of the body. The neuromatrix theory describes a neural network that accompanies the body. Even in situations such as amputations, this neural network can continue to impact signals perceived as pain. Visceral pain is pain that originates in the deeper tissue

For which of the following abnormalities will the nurse expect to inspect the penis? Select all that apply a. Cryptorchidism b. Hydrocele c. Phimosis d. Paraphimosis e. Hypospadias

Phimosis occurs when the foreskin cannot be retracted. Paraphimosis is when the foreskin remains in a retracted position. Hypospadias is improper position of the urethral opening of the penis. Cryptorchidim is undescended testicles and hydrocele is an accumulation of fluid in the scrotal sac (b, c, e)

A patient is displaying signs and symptoms of an endocrine disorder. The patient also complains of headaches and visual disturbances. Which of the following tissues will likely require further evaluation? a. Parathyroid b. Thyroid c. Pituitary d. Adrenal cortex

Pituitary tumors can cause headaches and disturbances in vision, due to the location of the pituitary gland. The parathyroid glands, thyroid gland are in the neck and would NOT cause visual disturbances. Adrenal cortex tumors would NOT cause visual disturbances

Plasmaphresis would be a treatment option for a patient with which of the following conditions? a. Acute kidney injury b. End-stage renal disease c. Pyelonephritis d. Goodpasture's syndrome

Plasmapheresis would remove the antibodies that cause damage to Goodpasture's syndrome

A nursing students is outlining the process of clot dissolution and the appearance on laboratory values. Please place the following in the correct sequence? - Plasmin stimulates fibrinolysis - Plasminogen converted plasmin - Increased fibrin degradation - Plasminogen activator - Increased D dimer levels

Plasminogen activator stimulates the conversion of plasminogen to plasmin. Plasmin stimulates fibrinolysis, which causes fibrin degradation and increased D dimer levels ( Plasminogen activator - Plasminogen converted plasmin - Plasmin stimulates fibrinolysis - Increased fibrin degradation - Increased D dimer levels )

A patient is to be treated for pneumonia. Which is an incorrect statement made by the nurse in regards to this treatment? a. An expectorant may help with mucus congestion b. Humidified oxygen may help with the patient's gas exchange c. Antipyretics may help reduce the patient's fever d. Pneumonia is always bacterial, so the mainstay of treatment is antibacterial medication

Pneumonia may be caused by a virus, bacteria, or fungal infection

A patient has polycystic kidney disease. The nurse correctly determine which of the following is the cause? a. Bacterial infection b. Autoimmune disorder c. Genetic disorder d. Too much calcium in the diet

Polycystic kidney disease is a genetic disorder

A patient states that she has been reading about polycystic ovarian syndrome (PCOS) and would like to discuss this disorder with the nurse. Which of the following statements by the patient shows correct understanding of the disorder? a. PCOS may make it more difficult to conceive b. PCOS is very rare c. PCOS results in ovulation of multiple eggs each month d. PCOS is a disorder of the ovaries only

Polycystic ovarian syndrome (PCOS) can contribute to infertility. PCOS is common, often contributes to anovulation, and is a complex disorder involving systemic complications

A nurse is scheduled to meet with a patient diagnosed with polycystic ovarian syndrome (PCOS). Which of the following expectations are accurate? Select all that apply a. There is an increased likelihood that the patient suffers from obesity b. If the patient wants information, then the issue of infertility relative to PCOS may be discussed c. Laboratory values may reveal hyperinsulinemia d. The patient may show signs of androgen excess e. The patient may display delayed breast development

Polycystic ovarian syndrome (PCOS) has been associated with obesity, infertility, insulin resistance, and excess androgen levels. Breast development delay is NOT a sign or symptom of PCOS (a, b, c, d)

A mother comes into the clinic wanting her child to be evaluated for diabetes mellitus, as she feels the child has some of the signs and symptoms. Which of the following would be signs and symptoms the child may display with diabetes mellitus? a. Hyperactivity and a lack of focus on activities b. Thirsty all the time, weight loss, and getting up in the night to urinate c. Lack of interest in activities, lethargy, and weight gain d. Nauseated, stomach bloating, and constipation

Polyuria and polydipsia are common signs of diabetes mellitus. Type 1 diabetes is associated with weight loss. Hyperactivity, nausea, and stomach bloating are NOT common signs of diabetes

A nursing student knows that changes in plasma pH can alter electrolyte levels, as well. On the laboratory value sheet, which ions will the nursing student most likely monitor in cases of pH changes? Select all that apply a. Sodium b. Magnesium c. Potassium d. Calcium e. Phosphorus

Potassium and calcium are the ions most influenced by changes in pH

A nursing student was asking about the causes of secondary amenorrhea. Which of the following would be considered causes? Select all that apply a. Lack of menses by the age of 15 years b. Pregnancy c. Drastic weight loss d. Genetic disorder e. Excessive exercise

Pregnancy and excessive exercise can cause secondary amneorrhea. Drastic weight loss have been associated with amneorrhea. Lack of menses by age 15 years and a genetic disorder are likely causes of primary amenorrhea (b,c, e)

A nursing student writes in her notes, "When the heart is filled with more blood, the heart is able to eject more blood". What term would best fits this description? a. Cardiac output b. Afterload c. Preload d. Stroke volume

Preload refers to the amount of blood the heart is filled with before contracting. Cardiac output is the ejection of the heart in liters per minute, afterload is the workload the heart is contracting against, and stroke volume is the amount of blood ejected per beat

A patient with primary hyperparathyroidism may be at increased risk for which of the following? a. Frequent urinary tract infections (UTIs) b. Renal calculi c. Interstitial cystitis d. Oxalic acid stone formation

Primary hyperparathyroidism increases calcium levels in the blood. This can increase the risk of kidney stone formation

A patient presents with right lower quadrant pain that is worsening, along with a fever. Which of the following assessments may the clinician perform? a. Psoas sign b. Romberg test c. Brudizinki sign d. Homan sign

Psoas sign is for appendicitis. Romberg test is for balance, Brudizinski sign is for meningitis, and Homan sign is for peripheral arterial disease

A patient is receiving phenazopyridine (Pyridium) for a urinary tract infection. Which of the following should the nurse note when speaking to the patient? a. Your urine may turn a red-orange color b. Your urine may smell different, more "fruity" or "sweet" c. Your urine may have a more greenish appearance in color d. You will NOT urinate as frequently on this medication. It suppresses the urination reflex

Pyridium can cause the urine to have an orange red color

A patient states that she once took a medication that turned her urine red. What medication does the nurse think the patient is referring to ? a. Pyridium b. Fluconazole c. Methicillin d. Lasix

Pyridium is a medication to treat the pain of urinary tract infections (UTIs), which will turn the urine a red color

A patient is given an antipyrogenic medication. Which of the following explains the purpose of the medication? a. Reduce edema b. Block pain signals c. Decrease inflammation d. Reduces fever

Pyrogen induces a fever, so an antipyrogenic medication reduces fever

Pyrogenic signals caused a fever in a patient and the fever broke 2 days later. Please place in the proper order the sequence of events for the patient's fever response - Hypothalamus temperature set to lower, normal temperature - Sweating to reduce body temperature - Hypothalamus temperature to increase body temperature - Pyrogens released - Decrease in pyrogens - Chills and shivering to increase body temperature

Pyrogens stimulate a reset of the hypothalamic temperature setting. The higher setting causes chills and shivering to increase body temperature. Once the fever breaks, pyrogen levels decrease. The hypothalamic set point returns to normal. Sweating occurs to reduce body temperature ( Pyrogens released - Hypothalamus temperature to increase body temperature - Chills and shivering to increase body temperature - Decrease in pyrogens - Hypothalamus temperature set to lower, normal temperature - Sweating to reduce body temperature)

A woman working in an office in which the summer air conditioning keeps the temperature very low mentions that she has Raynaud's phenomenon. What manifestations does the woman experience because of this disorder? a. Severe headaches b. Red discoloration of her cheeks c. Blanching and coolness at the fingertips d. Severe nausea

Raynauds phenomenon causes vasoconstriction of the vessels of the hands and fingers, leading to blanching and coolness of the fingertips

A renal patient presents with edema. What may be the primary cause of this edema? a. Urinary stone causing backup b. Decreased sesrum albumin due to renal loss c. Increased gluconeogenesis by the kidney d. Decreased erythropoietin synthesis

Renal dysfunction can lead to albumin loss in the urine, reducing oncotic pressure. Urinary stones may cause fluid retention in the renal system. Increased gluconeogenesis and decreased erythropoietin would NOT cause edema

A nursing educator is discussing with a nursing student that damaged heart muscle is susceptible to further injury by rapid, high-oxygenation issues. The nursing student states that she has heard of this development before. Which of the following terms is the nursing student referring to? a. Reentry circuits b. Repolarization disturbances c. Reperfusion injuries d. Recalibration injuries

Reperfusion injuries may occur with rapid reperfusion of ischemic heart tissue

Which of the following adipokines may increase the risk for diabetes mellitus? a. Leptin b. Resistin c. Adiponectin d. Ghrelin

Resistin increases insulin resistance, which may play a role in development of diabetes mellitus. Leptin and adiponectin help with insulin sensitivity whereas ghrelin is NOT an adipokine

A nurse is explaining to a colleague that HIV is a retrovirus. Which enzyme is unique to HIV and thus serves as a target for treatment? a. DNA polymerase b. RNA polymerase c. Reverse transcriptase d. Pyruvate dehydrogenase

Reverse transcriptase is part of HIV. The enzyme converts DNA from RNA and serves as a target for retroviral therapy. DNA and RNA polymerase are common enzymes found in cells. Pyruvate dehydrogenase are common enzymes, NOT present in HIV

A patient presents with aortic insufficiency. A nursing student notes this in the patient's file, and the nurse educator asks her which follow up question may be needed for this patient? a. Do you have allergies? b. Have you ever been diagnosed with rheumatic fever? c. Do you work around environmental toxins? d. Have you ever had pneumonia?

Rheumatic fever is a common cause of aortic stenosis

A nurse hears a heart murmur on a male patient. When she asks about his health history as a young child, he reports a severe streptococcal infection. Because he was living abroad at the time, he did NOT immediately receive antibiotics. What does the nurse suspect? a. Rheumatic heart disease b. Glomerulonephritis c. Myocarditis d. Kawasaki disease

Rheumatic heart disease can develop as a result of a streptococcal infection. Streptococcal infections can cause glomerulonephritis, however, the patient presents with a heart murmur. Myocarditis is usually due to a viral infection, and infects the heart muscle. Kawasaki disease is a form of vasculitis

A physician is to see a patient with rheumatoid arthritis. Which area of the body does the physician expect to show the most signs and symptoms of the disease? a. Eyes and mouths b. Lungs c. Hands and fingers d. Skin

Rheumatoid arthritis frequently effects the joints of the hands, it also may affect many other tissues

Which of the following assessments would most likely be expected in a patient with acute bronchitis? a. Crackles b. Rhonchi c. Positive egophony d. Dullness on percussion

Rhonchi are snore like sounds that occur with bronchial inflammation. Crackles indicate fluid in lungs. Positive egophony and dullness on percussion can indicate pneumonia

A nurse is listening to a patient's breath sounds and hears a snore like sound. Which term should the nurse place in her report? a. Wheezes b. Crackles c. Rhonchi d. Rales

Rhonchi are snore like sounds. Wheezes are high pitched sounds, crackles are a clicking or rattling sound in the lungs, and rales are another name for crackles

Which of the following patient presentations may increase the risk for edema? Select all that apply a. A patient suffering from protein malnutrition b. A patient suffering from liver failure c. A patient suffering from heart failure d. A patient suffering from dehydration e. A patient with hyperalbuminemia

Risk factors for edema include patients with protein malnutrition, liver failure, and heart failure. High levels of albumin would increase the oncotic pressure in the vascular system, decreasing the likelihood of edema

A nursing student is working on classifying immunodeficiencies. Which of the following are primary immunodeficiencies? Select all that apply a. HIV b. Severe combined immunodeficiency disorder (SCID) c. Wiskott-Aldrich syndrome d. DiGeorge syndrome e. Immunosuppression due to radiation treatment

SCID and Wiskott Aldrich are genetic disorders and primary immunodeficiencies. DiGeorge syndrome occurs due to incorrect development of the thymus and it is a primary immunodeficiency. HIV and radiation immunosuppression are types of acquired immunodeficiency

A nursing student is creating examples for presentation on Type III hypersensitivity reactions. Which of the following should he include in his presentation? Select all that apply a. Mantoux testing b. Hay fever c. Systemic lupus erythematosus (SLE) d. Rheumatoid arthritis (RA) e. Allergic reaction to shellfish

SLE and RA are type III hypersensitivity. Mantoux testing works on Type IV hypersensitivity reaction. Hay fever and allergic reaction to shellfish are type I hypersensitivity

Sarah is a 72 year old thin woman who has suffered a hip fracture. She states that she stepped off the stairs and fell. Sarah leads an inactive lifestyle and suffers from malnutrition due to issues with dysphagia. What term BEST describe Sarah's fracture? a. Open fracture b. Pathological fracture c. Pseudofracture d. Avulsion fracture

Sarah's bones are likely weakened due to her inactivity and malnutrition. The bones can fracture, in such as cases, without external trauma. Avulsion fractures develop due to extreme force from tendon or muscle. Open fracture breaks through the skin and a pseudofracture tends to occur in metabolic bone diseases

A patient with a family history of autoimmune disease presents at the clinic with what she feels is a very strange symptom. She states that she feels she has fewer winkles and her skin feels tighter, and she feels she looks younger. She does NOT understand what may be happening, as she has NOT used any new cosmetic products. What disorder does the nurse suscept? a. Raynaud's syndrome b. Sjorgen's syndrome c. Sarcoidosis d. Sclerodema

Sclerodema is an autoimmune disorder affecting collagen which causes tightening of the skin. Raynauds syndrome affects the hands, Sjogrens syndrome causes dry eyes and mouths and Sarcoidosis affects many tissues including the lungs

A nursing student believes that her patient has hypervolemic hypernatremia. Which of the patient's signs and symptoms would support this diagnosis? Select all that apply a. Tachycardia b. Plasma sodium concentration of 150 mEq/L c. Edema d. Hypertension e. High serum antidiuretic hormone (ADH)

Signs and symptoms of hypervolemic hypernatremia include edema, hypertension, and elevated plasma sodium concentration. Normal plasma sodium levels are 135 to 146 mEq/L. Tachycardia manifests in fluid volume deficit and ADH retains fluid which would dilute plasma sodium levels

Based on what would you justify that rubella causes teratogenic side effects? a. Vision impairment in the fetus b. Microcephaly in the fetus c. Spontaneous abortion of the fetus d. Retinitis in the fetus

Since rubella affects the embryo by causing spontaneous abortion, it is said to have teratogenic effects

A patient wants to know what "apnea" means, as he has been diagnosed with sleep apnea. Which of the following response by the nurse is appropriate? a. You have loss of consciousness from holding your breath at night b. Your swallowing reflex disrupts your breathing c. Apnea is cessation of breathing, which you experience during your sleep for brief moments d. Apnea is when you hyperventilate because you cannot catch your breath. This is happening while you sleep

Sleep apnea causes a brief cessation of breathing

A patient is suffering from gastroesophageal reflux disease (GERD). Which of the following are lifestyle management suggestions that may be beneficial to the patient? a. Lifestyle changes will have limited effectiveness because this is a progressive disease b. Eat one large meal a day to maintain caloric needs c. Lay down after eating to reduce stress on the stomach d. Plan small, frequent meals so as to NOT over distend the stomach

Smaller meals are more beneficial to gastroesophageal reflux disease (GERD). GERD can be improved with appropriate lifestyle management choices. Lying down increases the movement of stomach contents into the esophagus

A nursing student sees the normal values for plasma sodium and potassium. She asks why the normal values are so different. Which of the following responses by a fellow nursing student is correct? a. "Potassium is higher in the plasma than sodium due to sodium being actively pumped into cells" b. "Plasma potassium and plasma sodium levels are normally equal to one another. An error occurred in the reported laboratory normal" c. "Sodium is high in the extracellular fluid (ECF) while potassium is high in the intracellular fluid (ICF)" d. "Potassium levels are higher in the plasma than sodium levels because sodium enters cells during depolarization" e. "Potassium levels are higher in the plasma than sodium levels because sodium enters cells during depolarization"

Sodium is high in the plasma as it is actively pumped out of cells. Potassium is high inside of cells as it is actively pumped into the cells. Plasma sodium is much higher than plasma potassium

At discharge, a nurse is speaking to a patient about a pain medication prescription. Which statement indicates that the patient has the correct understanding for use of the medication? a. "I will make certain to take this medication at the first sign of pain" b. "I know I should finish off the entire bottle of oxycodone. If I need more, I should call the office" c. "Before taking the medication, my pain should be an 8 out of a possible 10" d. "If I feel any pain, I should double the dosage I am taking"

Some pain medications are most effective if taken during the first awareness of pain (a). Finishing off the entire prescription is NOT the intended purpose. Medication should be taken as prescribed, do NOT double the dosage

A patient is being evaluated for carotid stenosis. The results reveal significant stenosis. For which of the following may the patient be at increased risk? a. Ischemic stroke b. Epidural hematoma c. Hemorrhagic stroke d. Hypertension

Stenosis of the carotid artery increases the risk for clots and ischemia

A patient appears with complaints related to excessive bleeding relative to use of low molecular weight heparin. Which of the following will counteract the excessive bleeding? a. Protamine sulfate b. Plasmapheresis c. Blood transfusions d. Stop the medication

Stopping usage of low molecular weight heparin will prevent excessive bleeding. Protamine sulfate stops excessive bleeding due to standard heparin use. Plasmaphresis involves antibody removal from plasma and would NOT counteract excessive bleeding and blood transfusions should NOT be necessary (d)

A nurse is educating a patient that is very important to treat strep throat infections, because if it progresses, it may lead to which of the following conditions? Select all that apply a. Peptic ulcer disease b. Rheumatic heart disease c. Intestinal abscess d. Scarlet fever e. Glomerulonephritis

Streptococcus pyogenes infections can lead to rheumatic heart disease, scarlet fever, or glomerulonephritis

A patient is speaking to a nurse about the patient's stressful life. Which of the following recommendations may the nurse make? a. Up your caffeine intake so you have more energy b. Stop all activity and exercise and rest as much as possible c. If you sleep less, you can get more done d. Eat a balanced, nutritious diet

Stress can disrupt eating patterns. Eating a balanced diet can help restore the body during times of stress. Too much caffeine can increase the stress response. Exercise and restorative sleep are necessary to reduce stress

A patient is diagnosed with syndrome of inappropriate antidiuretic hormone (SIADH). Which of the following is the correct approach by the nurse? a. Frequently encourage the patient to drink water b. Restrict fluid intake c. Wait to see whether the patient stabilizes before any interventions d. Plan to administer an IV for fluid restoration

Syndrome of inappropriate antidiuretic hormone (SIADH) causes fluid retention so restriction of fluid may be necessary. Additional ingestion of fluids and IV administration are NOT needed

A patient is diagnosed with syndrome of inappropriate antidiuretic hormone (SIADH). Which of the following would support this diagnosis? Select all that apply a. Increased plasma osmolarity b. Increased plasma ADH levels c. Increased urine concentration d. Dehydration and fluid volume deficit e. Dilutional hyponatremia

Syndrome of inappropriate antidiuretic hormone (SIADH) indicates higher than needed ADH levels. Urine will become more concentrated, as fluid is retained. Dilutional hyponatremia may occur as excess fluid is retained (b, c, e) Plasma osmolarity will be decreased as excess fluid is retained

A blood test reveals very high lymphoblastic cell count. Which of the following are correct associations for the nurse to make regarding this information? Select all that apply a. Mature lymphocyte numbers are increased b. The number of macrophages has increased c. Myelogenous cell types are increased d. T and/or B-cell levels are elevated e. The cellular elevation shows an increase in immature, precursor cells

T and B cells are forms of lymphocytes. Blast cells are immature cells. Macrophages and myelogenous cells are NOT lymphocyte cells

A child is born with a defective thymus gland. Which of the following functions does the nurse expect to observe? a. Increased immunocompetence b. Neutropenia c. Decreased B-cell development d. Decreased T-cell maturation

T cell development is affected by DiGeorge syndrome which results from thymus dysfunction, leading to immunodeficiency

A nurse is speaking to a male patient about testicular cancer. Which of the nurse's statements are correct? a. Elderly men over the age of 70 are most at risk for testicular cancer b. Testicular cancer has a cure rate of at least 85% c. Nodules in both testicles is the most common initial sign d. Testicular cancer results in erectile dysfunction

Testicular cancer has one of the highest cure rates. Young males are most at risk for testicular cancer. A nodule in one testicle is usually the first sign. Testicular cancer does NOT necessarily result in erectile dysfunction

A registered nurse is teaching nursing students about CD4 cells. Which statements made by a student nurse indicate effective teaching? Select all that apply a. The CD4 cells are T cells that take part in cell-mediated immunity b. The CD4 cells mature into plasma cells c. The CD4 cells directly attack antigens d. The CD4 cells produce immunoglobulins e. The human immunodeficient virus attacks CD4 cells

The CD4 cells are T cells that take part in cell-mediated immunity and the human immunodeficient virus attacks CD4 cells

A nurse is reviewing cerebral circulation. What is the correct location for her to place the Circle of Willis? a. Lateral to the cerebellum b. Anterior the frontal lobe c. Inferior on base of brain d. Superior to cerebral cortex

The Circle of Willis is located at the base of the brain

A nurse is giving a patient the DTaP immunization, and the patient wants to know what it is for. Which of the following is the correct information for the nurse to share? Select all that apply a. Whooping cough b. Toxoplasmosis c. Strep throat d. Diphtheria e. Tetanus

The DTaP vaccine protects against diphtheria, tetanus, and pertussis (whooping cough)

A patient is being given instructions for an EpiPen. The patient has severe allergies to bee stings. Which of the following would be correct information for the nurse to provide? a. If you are stung, just use your EpiPen, and everything should be OK b. An EpiPen will stop the allergen from the bee sting from entering your system c. If you are stung, do NOT use the EpiPen immediately. Wait to see how the body responds first d. The EpiPen is to be used immediately until you can receive additional needed medical care

The EpiPen helps to counteract some of the changes brought on by a severe allergic reaction, additional medical help will be needed. An EpiPen does NOT block the allergen from entering the body, but rather counteracts the allergic response and death can happen within minutes with severe allergic reactions

A patient is suffering from dyspnea. Which of the following positions may the nurse suggest to ease breathing? a. Fowler's b. Left lateral decubitus c. Prone d. Supine with head below heart

The Fowler's position which is a more upright position will help breathing

A pregnant woman has not received the MMR vaccine at any time in her life. Which disease, prevented by this vaccine, does the clinician need to counsel the woman about, as it may cause damage to her unborn child ? a. Measles b. Mumps c. Rubella d. Rotavirus

The MMR vaccine protects against measles, mumps, and rubella. Rubella can cross the placenta and cause fetal damage

Which of the following is assessed by the National Institutes of Health Stroke Scale (NIHSS)? Select all that apply a. Ataxia b. Speech c. Language d. Sensation e. Motor function

The National Institutes of Health Stroke Scale (NIHSS) assesses ataxia, speech, language, sensation, and motor function

A nurse hears a murmur immediately after S2, which ends abruptly. What is the most likely explanation? a. A murmur at this point is normal b. Systolic murmur c. Diastolic murmur d. Holosystolic murmur

The S2 heart sound indicates the start of diastole. A murmur is NOT a normal heart sound and a holosystolic murmur is heard through systole

A patient appears with a substantial wound to the spinal cord, disrupting the signals from the sympathetic nervous system. Which of the following can be the result of sympathetic nervous system dysfunction? Select all that apply a. Hypotension b. Lower than expected heart rate c. Pupil dilation d. Increased gastrointestinal (GI) motility e. Bronchiole dilation

The SNS increases heart rate, raises blood pressure, decreases GI motility, bronchodilation, and pupil dilation

A patient is experiencing the Somogyi effect. Which of the following is the correct interpretation by the clinician? a. Increase bedtime insulin dosage b. Encourage nighttime snacking c. Restrict all insulin after noon d. After insulin so peak does NOT occur during sleep

The Somogyi effect is due to hypoglycemia activating counter-regulatory responses while sleeping. Altering the insulin dosage to prevent nocturnal hypoglycemia will help. Increasing insulin before bedtime will further induce hypoglycemia, nighttime snacking will elevate blood glucose further, and insulin is still required after noon

A nursing educator overhears some students speaking about how amazing the adrenal glands are for the stress response. Which of the following comments indicate the students need help? a. The adrenal cortex produces epinephrine and norepinephrine b. Adrenal cortex hormones help maintain blood volume and blood sugar c. The hypothalamus and the anterior pituitary regulate adrenal cortex hormone production d. Several hormones - cortisol, aldosterone, epinephrine - are produced by the adrenal gland

The adrenal medulla produces epinephrine and norepineprine, NOT the adrenal cortex

Which of the following choices could be used to rule out peripheral vascular disease? a. Left arm systolic blood pressure equals right arm systolic blood pressure b. Left arm systolic blood pressure equals right arm systolic blood pressure c. Right arm systolic blood pressure equals right arm systolic blood pressure d. Right arm systolic blood pressure is 15 mm Hg less than right arm blood pressure

The ankle brachial index for peripheral vascular disease is done by comparing systolic blood pressure in the arm and ankle on the same side of the body. To rule out peripheral vascular disease, these values should be approximately the same

A patient has suffered a stroke due to disruption of flow in the basilar artery. Which of the following is the most likely presentation in the patient? a. Loss of speech b. Loss of language interpretation c. Balance of problems and vertigo d. Hemiparalysis of face

The basilar artery serves the cerebellum, which plays a primary role in balance and coordination

In order to diagnose Crohn's disease, which option does the clinician consider most useful? a. Patient self-report b. Fecal occult blood test c. Gastric biopsy d. Colonoscopy

The best diagnostic tool for Crohn's disease is colonoscopy

A nurse working in the neonatal unit is informed about an infant who has presented with botulism. Which characteristics will the nurse expect to observe? a. Extreme muscle contraction b. Muscle paralysis c. Deep persistent cough d. Superficial rash

The botulism toxin causes muscle paralysis. Botulism suppresses respiration and does NOT present with a rash. Extreme muscle contraction is caused by tetanus, NOT botulism

A patient with type 2 diabetes mellitus, who is NOT on insulin, is experiencing morning hyperglycemia. Which of the following should be considered? a. The patient is obviously eating too many carbohydrates in the evening b. The patient is developing early signs of type 1 diabetes mellitus c. The patient's blood glucose falls overnight, and compensatory hormones elevate blood glucose in the morning d.The patient should be on insulin

The dawn phenomenon causes hyperglycemia in response to lack of glucose over the nighttime hours. Counterregulatory hormones work to keep blood glucose levels elevated

A patient is prescribed a medication that causes more rapid opening of sodium channels in electrically active tissue. Which component of cellular signaling is most likely modified by this medication? a. Polarization b. Hyperpolarization c. Resting membrane potential d. Depolarization

The depolarization phase of cellular signaling is due to sodium entry. A medication that alters sodium channel opening would change the rate of depolarization. Hyperpolarization and repolarization are primarily due to potassium reflux. Although the resting membrane potential may be influenced by this medication, changing sodium channel opening has a more profound effect on depolarization

A nursing student is reviewing risk factors for pancreatitis and notes that gallbladder issues are listed. She wonders why this may be. What is the proper explanation? a. The gallbladder and pancreas are located on the same side of the body b. Both the gallbladder and pancreas store bile c. The ducts from the gallbladder and pancreas enter the small intestine in the same location d. Digestive enzymes from the gallbladder can disrupt pancreatic functioning

The ducts for the gallbladder and pancreas enter into the same area. The gallbladder and pancreas are located on opposite sides of the body. Gallbladder stores bile but the pancreas does NOT and the gallbladder does NOT produce digestive enzymes

A patient receives a prescription for long-acting insulin. Which of the following information about duration of action should be provided to the patient? a. Duration of action is one to two hours b. Duration of action is two to four hours c. Duration of action is four to six hours d. Duration of action is 10 to 20 hours

The duration of long acting insulin is 10 to 20 hours

A nurse is questioning a family member who appears to be suffering from an ischemic stroke. Recombinant-tissue plasminogen activator (rt-PA) may be given. The nurse knows timing is important for which of the following reasons? a. Thrombolysis is most effective in the first 60 minutes b. rt-PA is best if given 24 hours after ischemic stroke c. rt-PA will help increase clot formation to reduce hemorrhage but must be given within 15 minutes of onset of symptoms d. rt-PA should be withheld until 72 hours after a stroke to see if the body will stabilize on its own

The earlier disruption of a clot can occur, the higher the chance of a better outcome. Recombinant tissue plasminogen activator (rt-PA) is a clot buster and it must be given within the first hours of a stroke

A nurse hears that a patient has a ST elevation myocardial infarction (STEMI) with elevated cardiac serum markers. Which of the following is a correct interpretation of this information? a. The patient has a severe systemic infection b. The patient is experiencing a mild coronary occlusion c. The patient is experiencing mild angina d. The patient is experiencing a myocardial infarction (MI)

The electrocardiogram (ECG) results and evaluation of serum cardiac markers indicate an MI

A child, following a viral infection, presents with thrombocytopenia. Which is the most likely type of thrombocytopenia? a. Hemolytic uremic syndrome b. Immune thrombocytopenic purpura c. Disseminated intravascular coagulation d. Thrombotic thrombocytopenic purpura

The fact the child had a viral infection prior to thrombocyopenia may indicate an immune response directed towards platelets. Hemolytic uremic syndrome tends to develop following E coli infections. Disseminated intravascular coagulation is a bleeding and clotting disorder and thrombotic thrombocytopenic purpura is uncommon in children and may develop following bacterial infection

A young patient says she had her first menstrual period at the age of 12. How may the nurse record this information? a. Age at menarche b. Age at menopause c. Age at telarche d. Age at menses

The first menstrual period is marked a menarche. Menopause is the cessation off periods that occurs with age, thelarche is breast development, and menses is the menstrual period (a)

A patient with many risk factors for atherosclerosis is talking to a nurse. The nurse states that although atherosclerosis has many causes, the process of atherosclerosis is initiated with which of the following events? a. Endothelial damage b. Foam cell formation c. Plaque formation d. Marcrophage recruitment

The first step in atherosclerosis is endothelial damageg. Foam cell formation and plaque formation are later steps in endothelial damage. Macrophage recruitment is a later step in atherosclerosis

When is a patient with gallbladder dysfunction most likely to note pain? a. When lying down after eating b. After eating fatty foods c. When the stomach is empty d. When they have to walk long distances

The gallbladder releases bile in response to fats in the diet. Gastroesophageal reflux disease (GERD) may cause pain when lying down after eating. Peptic ulcer disease tends to present with pain when the stomach is empty. Gallbladder pain does NOT present with physical activity

A female's blood analysis returns with the following: hematocrit at 44% and hemoglobin at 14 grams per deciliter. Which of the following should the nurse discuss with the patient? a. Nothing in regards to hematocrit and hemoglobin. These are normal values b. The patient may want to consider iron supplementation c. The patient may be experiencing excessive menstrual blood loss d. The patient needs better hydration as hematocrit is too high

The hematocrit and hemoglobin levels fall within the normal range for females. Female normal hematocrit range is 36 to 48% and normal hemoglobin range is 12 to 15.5 grams per deciliter

A patient is asking about the immunization for human papillomavirus (HPV) for her 12-year old daughter and 13 year old son. Which of the following is the appropriate response by the nurse? a. You need to wait for this immunization. It should be given after the individual is sexually active b. Your daughter can receive this vaccine, but your son will NOT be able to do so c. This vaccine has demonstrated nearly 100% efficacy is preventing precancerous lesions of the cervix and vagina d. The vaccine will require multiple doses over a 5- to 10-year period

The immunization for human papillomavirus (HPV) has been shown to be nearly 100% effective. Immunization for HPV should occur before the individual is sexually active, available or women and men, and it does NOT need to be paced out over 5 to 10 years

Please place the following in order for how an infection progresses - Acute - Prodromal - Resolution - Convalescent - Incubation

The incubation period is the time of exposure until the first signs and symptoms. Then a general feeling of illness may happen in the prodromal phase, followed by acute infection in which signs and symptoms are apparent. Convalescent is the recovery phase, followed by resolution ( Incubation - Prodromal - Acute - Convalescent - Resolution )

A tissue injury has activated the intrinsic coagulations cascade. Please place the following in the correct order - Factor XII - Fibrinogen - Thrombin - Prothrombin - Factor X

The intrinsic factor pathway begins with activation of factor XII, which activated factor X, which activates prothrombin. Thrombin is formed leading to fibrinogen activation ( Factor XII, Factor X, Prothrombin, Thrombin, Fibrinogen)

A patient has severe systemic lupus erythematosus (SLE). What organ damage is the clinician most likely to be concerned about? a. Skin b. Lungs c. Kidney d. Intestines

The kidneys are especially prone to damage to lupus

A lymph node biopsy of a lymphoma reveals no Reed-Sternberg cells. Which of the following is most likely? a. Hodgkin's lymphoma b. Non-Hodgkin's lymphoma c. Multiple myeloma d. Myelogenous leukemia

The lack of Reed Sternberg cells increases the likelihood that this is a non hodgkins lymphoma. Reed Sternberg cells appear in Hodgkins lymphoma, multiple myeloma does NOT affect the lymph nodes, and myelogenous leukemia is NOT a lymphoma

A patient's chart reveals that she has peripheral arterial disease. Which area of the body does the nurse most closely inspect, as this ist the most common location for this disease? a. Upper arm b. Head and neck c. Leg d. Right upper quadrantt

The leg is the most common location for peripheral arterial disease

Nursing students are learning about the renin-angiotensin-aldosterone system. They are amazed at its complexity, comment on the number of organs involved, and decide to make a list of these organs. Which of the following organs make their list? a. Liver b. Kidney c. Spleen d. Lungs e. Pituitary

The liver produces angiotensinogen, kidney produces renin, and lungs contain angiotensin converting enzyme.

The liver plays an important role in carbohydrate metabolism. Which of the following are processes the liver engages with related to carbohydrate metabolism? Select all that apply a. Glycogen storage b. Glycogenolyssi c. Gluconeogensis d. Glucagon synthesis e. Glycolysis

The liver stores glycogen, breaks down glycogen stores (glycogenolysis), forms glucose molecules (gluconeogensis), and carries out glycolysis. The pancreas synthesizes glucagon (a, b, c, e)

A patient with advanced liver failure has presented with edema. Which of the following likely played a role in this development? a. Excess clotting factors b. Decreased capillary permeability c. Hypoalbuminemia d. Hypovolemic hypernatremia

The liver synthesizes albumin, if levels decrease, edema is likely. Excess clotting factors would NOT occur with liver failure. Decreased capillary permeability and hypernatremia would decrease the risk of edema

Please place the following events in the proper sequence as related to enterohepatic recycling of bile salts, beginning with bile salt formation - Gallbladder releases bile in response to fat in diet - Liver synthesizes bile - Bile salts enter portal veins - Bile is stored in the gallbladder - Bile slats in ileum are reabsorbed

The liver synthesizes bile, which is stored in the gallbladder. The gallbladder releases bile in response to fat in the diet. Bile salts in the ileum are reabsorbed and enter portal veins ( Liver synthesizes bile - Bile is stored in the gallbladder - Gallbladder releases bile in response to fat in diet - Bile slats in ileum are reabsorbed - Bile salts enter portal veins)

Nursing students are studying for an examination. Which of the following statements show correct understanding of compensations for acid-base imbalances? Select all that apply a. The lungs compensate more quickly than the kidneys b. The kidneys maintain acid base compensation longer than the lungs c. The kidneys can compensate for metabolic acid-base disorders but NOT pulmonary disorders d. The lungs compensate primarily for respiratory acid-base imbalances e. The lungs, kidneys and spleen are the primary organs for compensation for acid-base imbalances

The lungs reach maximum compensation before the kidneys. Although it may take the kidneys longer to reach maximal compensation levels, the kidneys can maintain the compensation longer. The kidneys compensate for BOTH respiratory and metabolic acid base imbalances. In respiratory acid base imbalances, the lungs re the problem and are unable to compensate. The spleen is NOT involved in compensation for acid base imbalances (a, b)

A mother is wondering how her child, who was recently diagnosed with type 1 diabetes mellitus, developed the disease. The nurse explains that which of the following is the number one cause of type 1 diabetes mellitus? a. Genetic defect b. Autoimmune disorder c. Obesity and sedentary lifestyle d. 95% of cases are of unknown origin

The majority of type 1 diabete mellitus cases are due to autoimmune attack of the B cells. Obesity and sedentary lifestyle are more associated with type 2 diabetes mellitus. Most cases of type 1 DM have a known cause

A nurse educator is teaching about the initial history and assessment of a patient in pain. Which acronym does the educator suggest the students learn? a. APACHE b. REBOA c. OLDCART d. PAINFUL

The mnemonic to assess pain is OLDCART which is onset, location, duration, characteristics, aggravating factors, relieving factors, and treatment

A patient's neutrophil number is 900 cells per microliter. Which of the following would be correct associations for the nurse to make? Select all that apply a. The patient has an increased risk of infection b. The patient is suffering from anemia c. The patient has neutrophilia d. The patient has normal neutrophil levels e. The patient may benefit from Neupogen A

The neutrophil number is very low, which would increase the patient's risk for infection. Neupogen helps stimulate neutrophil formation. Normal neutrophil levels are approximately 2000 cells per microliter

A patient has provided a sputum sample. Which of the following tests will be the fastest for determining whether the patient has tuberculosis? a. Culture b. Acid-fast staining c. Nucleic acid amplification test (NAAT) d. Gram staining

The nucleic acid amplification test (NAAT) is the quickest assessment to determine tuberculosis. Culture requires a lengthy time to determine a mycobacterium infection. Acid fast staining can indicate mycobacterium tuberculosis but it is a slower procedure. Gram staining does NOT differentiate mycobacterium tuberculosis

Which of the following strategies reduces the risk of infection through the number one portal of entry? a. Respiratory mask b. Handwashing c. Bandaging open wounds d. Proper food preparation

The number one portal of entry for infection is through the respiratory tract, so a mask may help

Which values below would lead a nurse to suspect that a patient has respiratory acidosis? a. pH of 7.45, partial pressure of carbon dioxide of 35 mm Hg b. pH of 7.31 partial pressure of carbon dioxide of 55 mm Hg c. pH of 7.31, partial pressure of carbon dioxide of 30 mm Hg d. pH of 7.40, partial pressure of carbon dioxide of 55 mm Hg

The pH is acidic and the high carbon dioxide levels are contributing to the acidic pH, indicating a respiratory problem. A pH of 7.45 indicates alkalosis and PCO2 of 30 mm Hg is lower than normal, thus the condition is NOT respiratory in nature

A nursing educator is helping a nursing student to recognize the signs and symptoms of cholelithiasis in a patient. Which of the following should the student nurse be aware of? a. Projectile vomiting b. Severe abdominal pain between meals c. Burning sensation in sternal area d. Right upper quadrant (RUQ) pain that radiates to shoulder

The pain from gallstones can begin in the right upper quadrant and radiate to the shoulder. Projectile vomiting is NOT present with most cases of gallstones. Pain from gallstones tends to occur after eating and burning sensation in sternal area is often due to gastroesophageal reflux disease (GERD)

A patient with lower right quadrant pain, which is increasing, test positive for McBurney's point. What does the nurse expect to occur next? a. A prescription for immunosuppressants will be given b. An appendectomy will be scheduled c. Suggest the patient rest until pain passes d. Provide beverages and food as requested by patient

The patient has signs and symptoms for appendicitis, so an appendectomy is likely

A patient presents in the emergency department complaining of right upper quadrant pain radiating to the shoulder. The clinician wants to assess for Murphy's sign. What should the patient be directed to do? a. Lay on the left side and pull the right knee to chest b. Lay supine and take a deep inspiration c. Lay supine with arms overhead and knees pulled towards the chest d. Lay prone and arch the back upwards

The patient is asked to lie supine and take a deep inspiration while the clinician places pressure on gallbladder area. Pain elicits a positive Murphy's sign

A patient presents with prolonged, poorly managed chronic obstructive pulmonary disease (COPD). Which of the following laboratory values will the nurse expect? Select all that apply a. Elevated partial pressure of oxygen b. Elevated partial pressure of carbon dioxide c. Acidemia d. Increased anion gap e. Decreased pH

The patient is likely retaining carbon dioxide, reducing blood pH causing respiratory acidosis (b, c, e). Partial pressure of oxygen is likely reduced and anion gap does NOT normally elevate with respiratory acidosis

A nurse takes a patient's blood pressure while the patient is supine. The blood pressure is 112/72. When the patient stands, she complains of dizziness, her heart racing, and her blood pressure reading is 100/66. Which of the following explanations BEST describe this situation? a. Ataxia b. Arterial hypertension c. Orthostatic hypotension d. Negative Romberg test

The patient is suffering for a decrease in blood pressure that may happen with position changes. Ataxia is a lack of coordinated voluntary muscle movements, and the Romberg test assesses balance and is NOT related to blood pressure

An athlete competed in a long duration endurance event lasting more than 8 hours. Results from laboratory values obtained in the Emergency Department show that he lost a significant amount of fluid and electrolytes. What phrase may be most applicable to his current status? a. Hypervolemic hyperkalemia b. Hypovolemic hyponatremia c. Hypervolemic hypernaremia d. Hypovolemic hypernatremia

The patient lost both fluid and electrolytes, most likely due to excessive sweating. In this situation, both water and sodium are depleted.

A patient insists that his blood glucose control has been excellent. Which of the following laboratory results would support this conclusion? a. A1c of 4.9% b. C-peptide results are normal c. No glucose in urine d. Patient's self reported postprandial blood glucose results

The patient's A1c is normal, indicating glycemic control. A C peptide test is for insulin levels, NOT glucose. Hyperglycemia can be present even if glucosuria has NOT developed and a patient's self report may NOT be accurate (a)

A patient with type 1 diabetes mellitus has glycated hemoglobin levels of 7.8%. The patient states his blood glucose levels have been perfect over the past several weeks. Which of the following is an appropriate response by the nurse? a. I don't think you are testing the truth about your blood glucose levels b. All of your blood glucose values appear to be excellent c. Although you may feel your blood glucose levels have been perfect, these additional test results reveal glycemic control needs to be improved d. Your insulin regiment is apparently NOT working

The patient's A1c level is elevated, indicating poor glycemic control over the past three months. a variety of factors may result in elevated A1c levels. The nurse should NOT jump to the conclusion that their insulin regimen is apparently NOT working (c)

A critically ill elderly patient with a sepsis infection has the following laboratory results and signs: - Elevated D dimer - Increased prothrombin (PT) - Activated partial thromboplastin time (aPTT) - Petechiae The clinician most likely suspects which disorder? a. Von Willebrand (vWF) factor deficiency b. Hemolytic uremic syndrome c. Essential thrombocytosis d. Disseminated intravascular coagulation

The patient's lab results indicate increased clot breakdown along with prolonged clotting time. These are hallmarks of disseminated intravascular coagulation. vWF deficiency lead to increased risk of bleeding but NOT fibrinolysis and essential thrombocytosis is elevated platelet levels (excessive bleeding)

A patient's laboratory values reveal with bone marrow aspiration immature lymphoblast count greater than 20%. Which of the following conclusions may the nurse correcly make? a. The patient is demonstrating severe immunosuppression b. The patient has laboratory values associated with acute lymphocytic leukemia. Further diagnostic testing will be required c. The patient has a normal percentage of lymphoblastic cells d. The patient is likely suffering from an acute bacterial infection

The patient's laboratory values are consistent with acute lymphocytic leukemia, but further evaluation is needed. A conclusion of severe immunosuppression cannot be made based on these results. The percentage of immature lymphoblasts is higher than normal. Neutrophils, NOT lymphocytes, increase with bacterial infections

An elderly gentleman who is obese and living a sedentary lifestyle has presented at the clinic complaining that after running after his grandchildren, he felt cramping pain in his calf area. When he stopped and rested, the pain went away. What type of pain is the patient likely experiencing? a. Colicky b. Osteoporotic c. Rebound tenderness d. Intermittent claudication

The patient, based on symptoms, likely has peripheral arterial disease, which presets with intermittent claudication. This pain presents with activity and dissipates with rest. The patients pain is NOT presenting in waves, which would indicate colicky pain

A paramedic assesses a trauma patient and observes the following: - alert and conscious - heart rate (HR) of 92 beats per minute (bpm) - blood pressure 128/80 - Elevated respiratory rate Based on this information, what percentage of blood volume does the paramedic believe the patient has lost? a. < 15% b. 15% to 30% c. 30% to 40% d. > 40%

The patients heart rate and BP are NOT altered significantly. Thus, the loss is likely less than 15%. With 15 to 30% loss, HR elevates to compensate for falling BP. HR is normally above 120 bpm to maintain BP with 30 to 40%. With 40% or more loss, HR will be elevated to 140 bpm or more as BP falls

A patient has a platelet count of 15,000 cell per microliter. Which of the following is likely? a. The patient requires anticoagulant therapy b. The patient needs to take a daily aspirin c. The patient is at increased risk for bruising and petechiae d. The patient will likely require tissue plasminogen activator

The patients platelet levels are low, this would increase the risk of bleeding. The low platelet, there is no need for anticogulant therapy or tissue plasminogen activators which break up clots

A patient is diagnosed with pericarditis. How would the nurse explain this to the family members? a. The patient has a heart murmur b. The sac surrounding the patient's heart has inflammation c. The patient has an occlusion in his coronary artery d. The patient has fluid in his lungs

The pericardium surrounds the heart and pericarditis is inflammation of this sac

A patient presents with a plasma potassium level of 7.2 mEq/L. Which of the following is the appropriate management strategy? a. Slow, steady infusion of potassium b. Rapid infusion of dextrose and insulin c. Nothing. The potassium level is within the normal range d. IV solution with mannitol

The potassium level is dangerously high and a rapid response is needed. Infusion of dextrose and insulin will help move the potassium into the cells. Infusion of potassium will make the situation worse and IV solutions with mannitol are used to treat cellular edema, NOT hyperkalemia

A nurse is explaining to a male patient how HIV is transmitted. Which of the following are correct means for transmission of HIV that the nurse should share? Select all that apply. a. Sexual contact b. Skin-to-skin contact c. Fecal-oral d. Blood e. Respiratory droplets

The primary means of HIV transmission are sexual contact and through the blood

A nurse is reviewing movement of blood through the heart. Beginning with the vessels that bring blood to the heart from the lungs, place the following in the proper order - Aorta - Left atrium - Left ventricle - Pulmonary veins - Mitral valve

The pulmonary veins return blood from the lungs to the left atrium. Blood travels through the mitral valve to the left ventricle and is ejected into the aorta ( Pulmonary veins - left atrium - mitral valve - left ventricle - Aorta)

A drug is taken that blocks the sodium-potassium pump. Which of the following events is likely to occur? a. Increased intracellular potassium b. Decreased cellular swelling c. Decreased extracellular potassium d. Increased intracellular sodium

The sodium potassium pump expels three sodium ions. If the pump is blocked, sodium will accumulate in the cells. The pump normally moves potassium into cells, if the pump is blocked then potassium may elevate in the extracellular space

A nurse rushes up to a trauma victim. Which of the following signs and symptoms is the nurse expecting to encounter? Select all that apply a. Pupil dilation b. Increased blood pressure (BP) c. Slow, deep breathing d. Decreased heart rate (HR) e. Cool, clammy extremities

The trauma victim's stress response may involve increased blood pressure, dilated pupils, and peripheral vasoconstriction causing coolness of the extremities. Heart rate and respiratory rate is increased

How would a nurse educator instruct students to listen for a tricuspid valve murmur? a. Left lower sternal border b. 5th intercoastal space, midclavicular c. 2nd intercoastal space, right sternal border d. 2nd intercoastal space, left sternal border

The tricuspid valve is best heard at the base of the sternum on the left side (a)

A man is considering a vasectomy and wants to know which structure is surgically altered. What is the correct response by the clinician? a. Epididmysis b. Vas deferens c. Semineferous tubules d. Prostate gland

The vas deferens is cute in a vasectomy procedure

Individuals with diabetes mellitus (DM) are at an increased risk for chronic infections with which of the following infectious agents? a. Clostridium difficle b. Escherichia coli c. Candida albicans d. Neisseria meningitidis

Those with diabetes mellitus are at an increased risk for candida albicans infections

Which of the following may be present in essential thrombocytopenia ? Select all that apply a. Platelets < 150,000 per microliter b. Platelets > 600,000 per microliter c. Splenomegaly d. Ischemia and extremity clots e. Gastrointestinal bleeding

Thrombocytopenia is a low platelet number (< 150,000 per microliter). Reduced platelets may increase the risk of gastrointestinal (GI) bleeding. Thrombocytopenia decreases clot formation. Thrombocytosis causes splenomegaly

A patient with no known history presents with thrombocytopenia. Which is a likely next step? a. Genetic testing b. Bone marrow aspiration c. Computed tomography (CT) scan d. Checklist of medications

Thrombocytopenia may develop in response to medications. With no known history, genetic testing is unlikely to be needed. A bone marrow aspiration may be needed at some point, but it would NOT likely be the next step

A patient is to be given thrombolytic therapy for an ischemic stroke. Which of the following would prevent this therapy from being used? a. No use of anticogulant therapy b. Uncontrolled hypertension c. No signs of intracranial bleeding d. Absence of recent head trauma

Thrombolytic agents should NOT be used in patients with severe hypertension. Anticoagulant therapy and recent head trauma may prevent thrombolytic agent usage. If bleeding were present, a thrombolytic agent would NOT be used

Which of the following show that a nursing student has a correct understanding of thrombolytic treatment for stroke? a. Thromobolytic treatment is standard for all stroke victims b. Thromobolytics are best with hemorrhagic stroke c. Thromobolytic agents are most effective if used early in stroke course d. Thromobolytic agents should be used on patients taking anticoagulants

Thromobolytic therapy works best if used early. Thromobolytic treatment is NOT standard nor used for hemorrhagic stroke, and anticoagulants are contraindications

A nursing educator is speaking to a student about a patient with obesity. The nursing educator emphasizes that obesity is NOT simply a disease of overeating, but rather it may develop secondary to other diseases. Which of the following may the educator include in her list of diseases that may contribute to obesity? Select all that apply a. Anorexia nervosa b. Hypothyroidism c. Polycystic ovarian syndrome (PCOS) d. Addision's disease e. Hyperthyroidism f. Cushing's syndrome

Thyroid hormones are responsible for metabolism. A reduction in these hormones will result in weight gain. PCOS is an endocrine disorder and Cushing's syndrome is a disorder of adrenal cortex hormones which lead to weight gain

A nursing student notices elevated thyroid stimulating hormone (TSH) on laboratory values, along with low thyroxine levels. What is the proper interpretation of the results? a. The patient has secondary hypothyroidism b. The patient has primary hypothyroidism c. The patient has hyperfunctioning of the anterior pituitary d. The patient has Grave's disease

Thyroid stimulating hormone (TSH) levels are elevated, but the thyroid gland is NOT responding with additional thyroid hormone. If it were secondary hyperthyroidism or hyperfunctioning of the anterior pituitary, TSH and thyroid hormone levels would be high. Grave's disease is a hyperthyroid condition

A patient is working towards better glycemic control. She has become frustrated though, saying how variable her measurements are. The nurse decides to review all the factors that influence her blood glucose levels. Which of the following should be included? Select all that apply a. Timing and type of nutrient intake b. Physical activity c. Lipoprotein levels d. Status of insulin resistance e. Illness and stressors

Timing and type of nutrient intake, physical activity, insulin resistance, and illness and stressors can alter blood glucose levels. Lipoprotein levels do NOT directly affect blood glucose levels

Upon being discharged, a patient with chronic pancreatitis is reviewing the dietary recommendations. Which of the following indicates that further instruction is needed? a. The best option is to eat one large meal per day, with smaller snacks when I am hungry b. I should refrain from caffeine c. A meal of a cheeseburger and fries is NOT my best option d. I should choose easily digestible food and eat frequent meals

To help heal pancreatitis, reducing stress on the pancreas is needed. Large meals place additional digestive burden on the pancreas. Limiting caffeine, easily digestible foods, and small meals are recommended. Large, high fat meals are NOT recommended

To lose weight, a patient who consume 2,500 calories per day has been prescribed a diet that will reduce caloric intake by 7,000 calories per week. What is the patient's caloric intake per day on the prescribed diet?

To reduce caloric intake by 7000 calories per week, one has to reduce caloric intake by 1000 calories per day. As the patient currently consumes 2,500 calories per day, on the diet they would reduce this by 1000 calories per day resulting in a caloric intak eof 1500 calories per day

In iron deficiency anemia, which serum factor may be high? a. Serum iron b. Hematocrit c. Hemoglobin d. Total iron binding capacity

Total iron binding capacity increases when iron levels are low, indicating that there is increased space available for iron to bind. Serum iron and hemoglobin would be low.. Hematocrit levels will be low because RBC cannot be synthesized at the normal level

A pregnant woman is worried about possible infections being transmitted to her child in utero. She explains to the nurse that she had read somewhere that cats can carry infections that may affect pregnancies. Which infectious organism does the nurse believe the patient is referencing? a. West Nile virus b. Toxoplasma gondii c. Giardiasis d. Candidiasis

Toxoplasmosis is found in cat feces and may be transmitted to the fetus. West Nile virus is transmitted by mosquitoes, Giardiasis is transmitted through contaminated water and Candidaisis is a fungal infection

A patient's spouse reports to the emergency department physician that her husband has suffered a "brain attack" or "mini-stroke", but he seems better now. Which of the following likely occurred? a. Subarachnoid hemorrhage b. Epidural hematoma c. Transient ischemic attack d. Brain herniation

Transient ischemic attack are short duration in blood flow that often resolve. A subarachnoid hemorrhage is bleeding in the brain, an epidural hematoma is a bleeding near the skull, and a brain herniation is displacement of brain tissue

Place in proper order the steps of formation of the Philadelphia chromosome leading to hematological neoplasm treatment - Uncontrolled cell proliferation - Translocation between chromosomes 9 and 20 - Tyrosine-kinase-inhibitor treatment - Tyrosine kinase protein produced - BCR-ACL oncogene formed

Translocation between chromosomes 9 and 20 can occur. This can lead to an oncogene, which results in a tyrosine kinase protein being produced. This leads to uncontrolled cell proliferation in the treatment is a tyrosine kinase inhibitor ( Translocation between chromosomes 9 and 20 - BCR-ACL oncogene formed - Tyrosine kinase protein produced - Uncontrolled cell proliferation - Tyrosine-kinase-inhibitor treatment)

Which of the following infections would NOT be treated with an antibacterial agent? a. Mycoplasma genitalium b. Treponema pallidum c. Trichomonas vaginalis d. Klebisella granulomatis

Trichomonas vaginalis is a protozoan, so an antiprotozoal medication is needed. Mycoplasma genitalium, Treponema pallidum, and Klebisella granulomatis can be treated with antibiotics

A patient presents with blood backing up directly into the right atrium. Which valve disorders may contribute to this development? Select all that apply a. Tricuspid stenosis b. Pulmonic insufficiency c. Tricuspid insufficiency d. Right atrioventricular (AV) stenosis e. Mitral insufficiency

Tricuspid stenosis, tricuspid insufficiency, and right atrioventricular (AV) stenosis would cause blood to back up into the right atrium. Pulmonic insufficiency would cause backflow of blood into right ventricle and the mitral valve is on the left side of the heart

A nursing student is reviewing how exposure to tuberculosis may develop into a transmissible disease. Please place the following events in the proper sequence - Infectious bacteria emerge from tubercles - Lung tubercles form - There is exposure to M. tuberculosis - Infectious bacteria walled off in lungs - M. tuberculosis infects lungs

Tuberculosis is caused by the bacteria, M tuberculosis. This bacteria can infect the lungs where it is normally walled off. Lung tubercles may form. However, the tubercles can break down, enabling the infectious bacteria to emerge from the lungs ( There is exposure to M. tuberculosis - M. tuberculosis infects lungs - Infectious bacteria walled off in lungs - Lung tubercles form - Infectious bacteria emerge from tubercles)

An antibody test is ordered for a patient with newly diagnosed diabetes mellitus (DM). The antibody test comes back positive. Which form of diabetes is most likely? a. Type 1 diabetes mellitus b. Type 2 diabetes mellitus c. Gestational diabetes mellitus d. Drug induced diabetes mellitus

Type 1 diabetes mellitus is a form of autoimmune disorders. An antibody test would reveal this. Type 2 DM, gestational DM, and drug induced DM, are NOT autoimmune disorders

A patient has high lipid levels. The nurse decides to discuss some of the patient's risk factors. Which of the following should be included? Select all that apply a. Diet with high level of omega 3 fatty acids b. Type 2 diabetes mellitus (DM) c. Hyperthyroidism d. Sedentary lifestyle e. Familial hypercholesterolemia

Type 2 DM, sedentary lifestyle, and familial hypercholesterolemia will increase blood lipid levels. Omega 3 fatty acids positively effect lipid levels. Hypothyroidism elevates lipid levels

A young male, age 19 years, presents with a urinary tract infection (UTI). Which of the following shows correct understanding by the urine? a. A UTI is uncommon in young male. Further evaluation is needed b. The young male must be dehydrated c. The young male likely has benign prostatic hyperplasia d. The best approach is "wait and see" because this is likely to resolve on its own

UTIs are uncommon in young males and further evaluation is required

For which of the following conditions may immunosuppressants be a therapeutic option? Select all that apply a. Ulcerative colitis b. Irritable bowel syndrome c. Appendicitis d. Crohn's disease e. Diverticulosis

Ulcerative colitis and Crohn's disease are inflammatory conditions of the colon, so immunosuppressants may be helpful. Irritable bowel syndrome and diverticulosis are NOT treated with immunosuppressants. Appendicitis is surgically corrected (a, d)

A patient is suffering a severe attack of ulcerative colitis. Which may be of greatest concern? a. Necrosis of colon tissue due to ischemia b. Erosion of the esophageal lining due to vomiting c. Bowel obstruction due to sphincter contraction d. Fluid and electrolyte loss due to severe diarrhea

Ulcerative colitis can cause severe bouts of diarrhea, disrupting fluid and electrolytes. Ulcerative colitis would NOT cause necrosis due to ischemia, esophageal erosion, or bowel obstruction

For which of the following conditions would the clinician need to recommend straining of the urine? a. Urinary tract infection (UTI) b. Asymptomatic bacteriuria c. Urosepsis d. Urolithiasis

Urolithiasis is the presence of stone. Straining of the urine helps to assess whether a stone has been passed and enables a stone to then be analyzed. Culture and sensitivity would be the assessment completed for a UTI. Asymptomatic bacteriuria is elevated bacteria levels on urinalysis without symptoms

A nurse suspects urosepsis in a patient under her care. Which signs and symptoms is she observing that may indicate urosepsis? Select all that apply a. Confusion b. Fever c. Flank pain d. Disorientation e. Severe pain with urination

Urosepsis may present with confusion, fever, and disorientation. Urosepsis does NOT tend to present with flank pain and urinary tract signs and symptoms (a, b, d)

Which of the following patients under a nurse's care is most at risk for urosepsis? a. Newborn infant b. Young child being toilet-trained c. Young man postappendicitis surgery d. Elderly woman with indwelling catheter

Urospesis tends to develop in elderly patients

In a patient's file, it is noted, "Use of accessory muscles to breath". Which of the following is a correct interpretation? a. The breathing dynamics of the patient is normal b. The patient is using additional muscles with each breath c. The patient is making a snoring sound with each breath d. The patient has a protruding diaphragm muscle

Use of accessory muscles indicates that additional muscles are being used to breath

A clinician treating a patient with ischemic stoke is considering thrombolytic therapy. Which of the following would be a contraindication for this treatment? Select all that apply a. Anticogulant therapy b. Age less than 50 years c. Female sex d. Bleeding problems e. Recent head injury

Use of anticoagulant therapy would increase the risk for bleeding. Bleeding problems and recent head injury are contraindications. Age less than 50 years and female sex are NOT contraindications

A nurse has received a series of laboratory values and vital signs for a patient who recently experienced a traumatic accident. The laboratory values were taken when the patient was in the alarm stage of the stress response. Which of the following laboratory values and vital signs does the nurse expect to see? Select all that apply a. Decreased respiratory rate b. Elevated white blood cells (WBCs) c. Decreased antidiuretic hormone (ADH) d. Increased blood pressure (BP) e. Increased heart rate (HR)

Values and vital signs that increase in acute stress response include blood pressure, heart rate, white blood cells, respiratory rate, and ADH levels (b, d, e)

Which values are elevated in respiratory acidosis? Select all that apply a. pH b. Carbon dioxide c. Hydrogen ion levels d. Renal excretion of H+ e. Renal reabsorption of bicarbonate

Values that are elevated in respiratory acidosis include carbon dioxide, hydrogen ion levels, renal excretion of H+, and renal reabsorption of bicarbonate. In acidosis, pH is reduced`

A nurse is speaking to a patient about erectile dysfunction. She notes that which of the following is often the primary factor? a. Hormones b. Psychological factors c. Vascular issues d. Neurological issues

Vascular issues are the primary underlying cause of erectile dysfunction

The nursing student is directed to monitor urine output in a patient that has undergone a motor vehicle accident. She notes that urine output seems to be significantly reduced. Which of the following may account for decreased urine output during times of stress? Select all that apply a. Decreased antidiuretic hormone (ADH) secretion b. Increased vasopressin c. Increased aldosterone d. Increased thyroid-stimulating hormone (TSH) e. Decreased adrenocorticotropic hormone (ACTH)

Vasopressin is another name for ADH, which reduces urine output. Aldosterone increases sodium and water reabsorption. ADH reduces urine output, so a decrease in ADH will increase urine output. ACTH stimulates the release of aldosterone, which decreases urine output (b, c)

A patient was reading an article about venous ablation and wants to know more. Which of the following is correct information? a. Venous ablation must be performed before varicose veins form b. Venous ablation results in removal of most major veins in the area c. Venous ablation is a noninvasive treatment strategy d. Venous ablation is used to treat abnormal vein structures contributing to varicose veins

Venous ablation helps to correct vein abnormalities. Venous ablation does NOT need to be performed before varicose veins form nor does it remove major veins. It is an invasive procedure

A patient is suffering from gastroesophageal reflux disease (GERD) is reviewing some of the instructions she has received to help manage her condition. Which of the following statements show correct understanding by the patient? a. I should wait for two to three hours after eating before I lie down b. Larger, more filling meals will help my digestion c. Smoking does NOT make my condition worse d. Eating small, frequent meals makes my GERD worse

Waiting to lie down after eating can help prevent reflux into the esophagus. Smaller meals should be eaten to prevent overdistention of the stomach and smoking worsens gastroesophageal reflux disease (GERD)

A patient is place on warfarin, which he refers to as a "blood thinner", which makes my blood more watery. Which of the following responses by the nurse is appropriate? a. You are correct. Your blood will be more dilute b. The warfarin helps to break down clots, making the blood appear "thinner" c. Warfarin does NOT dilute the blood but rather decreases the risk for clot formation d. Warfarin retains water in the bloodstream, diluting clotting factors

Warfarin decreases the risk for clot formation. Blood thinners increase clotting time, they do NOT dilute the plasma or break down clots

A patient is to undergo outpatient knee surgery. A nurse is talking to the patient, as the patient is concerned about pain management. Which of the following statements is a clinician likely to make? a. "We begin with the most potent medications such as morphine, and then slowly taper the dosage over several weeks" b. "Pain is an important signal for the body; thus, we do NOT want to block it with any medications" c. "We will begin with moderate medications for pain, and assess your pain with a pain rating scale to make adjustments as needed" d. "Before taking any pain medications, make certain you absolutely need the medication. Your pain should be severe before medicating"

We will begin with moderate medications for pain, and assess your pain with a pain rating scale to make adjustments as needed

A nurse is informed to schedule a patient for a polysomnography. What is the most appropriate way for the nurse to convey this information to a patient? a. "We will need to schedule for a polysomnography examination, which is a sleep study" b. "You are going to be scheduled for a polysomnography examination in 3 days" c. "A polysomnography will be scheduled for you. Which day works best?" d. "You will be undergoing a polysomnography examination, which measures blood flow in your legs"

We will need to schedule for a polysomnography examination, which is a sleep study

A nurse is reviewing how heart failure may develop due to pulmonary issues. Please place the following events in the proper order - Poor pulmonary function - Pulmonary vasoconstriction - Increased workload on the right ventricle - Decreased pulmonary oxygen levels - Right ventricular hypertrophy

When function is poor in the lungs, decreased oxygen levels result. The decreased oxygen levels cause pulmonary vasoconstriction, which in turn, increases the workload on the right ventricle. This can then cause right ventricular hypertrophy and potential heart failure ( Poor pulmonary function - Decreased pulmonary oxygen levels - Pulmonary vasoconstriction - Increased workload on the right ventricle - Right ventricular hypertrophy )

A person has hypotension and the RAAS is activated. Place the steps of the sequence in the proper order - Angiotensin II - Renin - Angiotension converting enzyme - Angiotensinogen - Angiotensin I

When renin is released, it converts angiotensinogen to angiotensin I. Angiotensin I is converted by angiotensin converting enzyme into angiotensin II ( Renin - Angiotensinogen - Angiotensin converting enzyme - Angiotensin I - Angiotensin II)

Please place in the proper order the sequence of events related to how the RAAS activation causes an increase in blood pressure - Increased blood pressure - Increased blood volume - Low blood pressure detected by the kidneys - Aldosterone released - Increased sodium and water reabsorption by the kidneys - RAAS activated

When the kidneys detect low blood pressure, they release renin, activating the RAAS. Activation results in aldosterone release which stimulates sodium and water uptake and an increase in blood volume, which in turn increases blood pressure ( Low blood pressure detected by the kidneys - RAAS activated - Aldosterone released - Increased sodium and water reabsorption by the kidneys - Increased blood volume - Increased blood pressure)

A client's pulse oximetry reading is 100%, however, the hemoglobin level is 8.5 g/dL. The client's spouse asks the nurse what activates the client can safely carry out. Which activity would the nurse recommend? a. Trekking b. Skiing c. Rock climbing d. Road trip

When the oxygen saturation of blood is 100% and hemoglobin is low, an inadequate oxygen supply to cells will result. Therefore, the nurse should NOT recommend that the client do vigorous exercises or activities that demand an oxygen supply. The nurse can recommend that the client engage in activities, that are less strenuous, such as taking a road trip on a cross country vacation

Jugular vein distention often develops in response to a failing heart. Please place the following events in order that described as development - Blood pools in the right ventricle - Blood backs up into the jugular vein - Blood backs up into the superior vena cava - Right ventricular failure - Blood backs up into the right atrium

When the right ventricle fails, blood pools in the right ventricle and backs up into the right atrium. Blood then backs up into the superior vena cava and finally the jugular vein ( Right ventricular failure - Blood pools in the right ventricle - Blood backs up into the right atrium - Blood backs up into the superior vena cava - Blood backs up into the jugular vein)

A patient is admitted for acute pancreatitis. Which of the following are assessed as Ranson criteria initially upon admission? Select all that apply a. White blood cell (WBC) count b. Glucose c. Hematocrit d. Calcium e. Aspartate aminotransferase (AST)

White blood cell count, glucose, and aspartate aminotransferase (AST) are initial Ranson criterias. Hematocrit and calcium are NOT initial criteria

A patient has a complete large bowel obstruction. Which of the following are signs and symptoms present? Select all that apply a. Inability to pass gas or produce a bowel movement b. Flattened, nondistended abdomen c. No feces in rectum d. No pain as nerves are disrupted e. Absent bowel sounds

With a complete obstruction, nothing can be passed in the intestines and enter the feces, and bowel sounds may be absent

A patient with a diverticulitis flareup is relating her recent food choices to the nurse. Which of the following indicates further instruction for the patient is needed? a. Breakfast of bran cereal, whole wheat toast, and strawberries b. Lunch of boiled noodles with butter and a snack of applesauce c. Dinner of broiled salmon and instant mashed potatoes d. Snack of hard boiled egg

With a diverticulitis flareup, high fiber food should be avoided. Noodles, applesauce, salmon, mashed potatoes, and eggs are acceptable choices with low fiber content

A nurse is assigned to help a patient diagnosed with tumor lysis syndrome. Which of the following can the nurse expect? Select all that apply a. Hyperuricemia b. Hyperkalemia c. Hypernatremia d. Hypocalcemia e. Hypophophatemia

With cell lysis due to cancer treatment, uric acid levels and potassium levels are increased. Calcium levels are often decreased in tumor lysis syndrome. (a, b, d) Phosphate levels are normally high in tumor lysis syndrome and sodium levels are NOT primarily altered in tumor lysis syndrome

A nurse sees on a patient's chart that the area of the nucleus raphe magnus (NRM) and periaqueductal gray matter (PAG) has been damaged. What following characteristics may the nurse expect in the patient? a. No nociception b. Low pain threshold c. Elevated serotonin levels d. Higher than normal endorphin levels

With damage to the NRM and PAG pathway and subsequent reduced endorphin levels, the patient may present with a reduced pain threshold. Nociception is the detection of pain stimuli, NRM and PAG are NOT involved with initial pain detection. Damage to the NRM and PAG will also result in decreased serotonin levels

A nurse is reviewing how decreased perfusion to the kidneys can result in increased blood volume. Please place the following steps in order - Juxtaglomerular cells secrete renin - Sodium and water reabsorption is increased - Angiotensin I is converted to angiotensin II - Renin activates angiotensinogen - Aldosterone is released

With decreased renal perfusion, the juxtaglomerular cells will secrete renin. Renin will stimulate angiotensinogen. Angiotensin I will be converted to angiotensin II, which will stimulate aldosterone release. Aldosterone will increase sodium and water reabsorption, increasing blood volume and blood pressure ( Juxtaglomerular cells secrete renin - Renin activates angiotensinogen - Angiotensin I is converted to angiotensin II - Aldosterone is released - Sodium and water reabsorption is increased)

A bedridden patient is diagnosed with a disruption of blood flow in the lungs. Please place the following steps in the order that leads to this event - Thrombus travels to lungs - Increased risk of clot formation - Pulmonary Embolism (PE) - Venous stasis - Deep vein thrombosis (DVT)

With immobility, venous stasis develops, increasing the risk of clot formation. A DVT may form and travel to the lungs. The clot may lodge in the lungs causing PE ( Venous stasis - Increased risk of clot formation - Deep vein thrombosis (DVT) - Thrombus travels to lungs - Pulmonary Embolism (PE))

Of the following patients a nurse is charged for treating, who is most likely to suffer a urinary tract infection? a. Adult woman of childbearing age b. Child less than 5 years of age c. Young adult male, age 25 years d. Adult male, age 68 years

Women of childbearing age are at an increased risk for urinary tract infections

A young patient has been diagnosed with severe combined immunodeficiency disorder (SCID). Which of the following is appropriate for the nurse to say to the parents? a. Good nutrition and a healthy lifestyle will help reduce the damage by this disorder b. Your child has a form of leukemia. A bone marrow transplant is needed c. Your child has a genetic illness that effects the functioning of a part of the immune system d. Your child will primarily suffer from cognitive defects and delayed development. Otherwise, his overall health will be generally good

Your child has a genetic illness that effects the functioning of a part of the immune system because SCID affects both B and T cell development. SCID results in immunosuppression and is NOT a form of leukemia

Which of the following responses by a nurse is correct when asked by a patient why his glands swell when he is sick? a. "Your glands are also known as lymph nodes. These can swell during infections, as lymph nodes contain cells that fight infections" b "The word 'gland' is NOT correct. These structures are known as lymph nodes " c. "Swollen glands, even during illness, much be assessed as they usually indicate the presence of cancer" d. "Your glands swell when you are ill due to systemic edema"

Your glands are also known as lymph nodes. These can swell during infections, as lymph nodes contain cells that fight infections. Do NOT correct patients when they do NOT use medical terminology. Swollen glands occur for many reasons, NOT just cancer

A report includes a description of an infection that is vector-born and transmitted by mosquitoes. Which of the following may the report be describing? Select all that apply a. Zika virus b. Malaria c. Ebola virus d. West Nile virus e. Herpes simplex virus Methicillin-resistant

Zika virus and west nile virus are transmitted by mosquitoes. Malraia is a protozoal disease, NOT a viral disease. Ebola and Herpes simplex virus are NOT transmitted by mosquitoes

Laboratory values reveal gastrin levels 15 times higher than normal. Which condition does the nurse suspect? a. Gastroesophageal reflux disease (GERD) b. Pyloric stenosis c. Zollinger-Ellison syndrome d. Celiac disease

Zollinger Ellison syndrome is due to a gastrin secreting tumor. Gastroesophageal reflux disease (GERD) affects the esophageal lining. Pyloric stenosis prevents stomach emptying and celiac disease is due to an autoimmune response to gluten

A nurse is reviewing cases of liver cirrhosis. Which of the following risk factors concern her the most, as it is the leading cause of liver cirrhosis in the United States? a. Alcohol abuse b. Illegal drug use c. hepatitis B infection d. hepatitis C infection

hepatitis C infection is the primary cause of liver cirrhosis in the United States

A nurse is explaining to a patient that, although he feels "fine", glycemic control is important as chronic hyperglycemia has been associated with increased risk for which of the following? a. Osteoporosis b. Amputation c. Thyroid gland suppression d. Liver failure

hyperglycemia can damage the microvasculature and blood vessels. Eventually enough damage may occur so that amputation is necessary

A client presents with less strength, high elastic skin, and joints that are hypermobile. Which condition can be diagnosed in the client based on these characteristics? a. Klinefelter syndrome b. Turner syndrome c. Down syndrome d. Ehlers-Danlos syndrome

A group of disorders that involve diminished strength and integrity of the skin, joints, and other connective tissues is known as Ehlers Danlos syndrome

A bedridden client has frequent gastrointestinal reflux. Which condition might the client develop due to gastrointestinal reflux? a. Aspiration pneumonia b. Hypotension c. Hyperthermia d. Constipation

A bedridden client might be at a high risk of gastroesophageal reflux, which can cause gastric contents to enter into the lungs and result in ASPIRATION PNEUMONIA

While studying with a friend, a comment is made that changing one base pair in the human genome cannot cause any serious effect in a person. What is an accurate response to this friend? a. "A single change is a base pair that can have profound effects. This is known as Clustered Regularly Interspaced Short Palindromic Repeats (CRSPR)" b. "You are correct. The human genome is so large and complex that changing one base pair never has a serious effect" c. " You are incorrect. A change in one base pair, in some cases, can cause disease and damage. This change is known as a single nucleotide polymorphism (SNP)" d. "You are correct. Base pair changes are immediately corrected by enzymes, so no detrimental effect occurs'

A change in one base pair, in some cases, can cause disease and damage. This change is known as a single nucleotide polymorphism (SNP). In some cases, such as sickle cell anemia, change in a single base pair can cause damage

Mannitol is an osmotically active substance that attracts waters. In which of the following situations would an IV containing a mannitol solution be beneficial a. Cellular edema b. Cellular hypoxia c. Acidosis d. Cellular dehydration

A mannitol IV solution would attract water from the intracellular space to the extracellular space, reducing cellular edema. As mannitol attracts water from the cells, it would cause further cellular dehydration. Mannitol would NOT counteract acidosis and cellular hypoxia

As a registered nurse is teaching a student how to analyze the electrocardiogram of a hypokalemic client. Which statements made by the student nurse indicate effective teaching? Select all that apply. a. Widened P waves will be observed b. A reduced PR interval will be observed c. A flattened T wave will be observed d. A prolonged PR interval will be observed e. A prominent U wave will be observed

A prolonged PR interval, a flattened T wave and a prominent U wave will be observed

Which is a test primarily used diagnose fetal chromosome problems? a. Cordocentesis b. Maternal serum screening c. Chorionic villus sampling d. Amniocentesis

AMNIOCENTESIS is a test primarily used to diagnose fetal chromosome problems. It detects about 99% of fetal chromosome abnormalities

A karyotype is the overall picture of - a. All the normal chromosomal pairs in an individual b. All the chromosomal pairs in an individual c. All the chromosomal pairs of a species d. All the abnormal chromosomal pairs in an individual

All the chromosomal pairs in an individual. Each individual has a fixed number of chromosomes and gene patterns

A nurse educator is working with a nursing student and discussing prenatal testing. The nurse educator asks the nursing students to list some disorders detectable with amniocentesis. Which of the following would be correct responses by the nursing student? a. Down syndrome b. Turner syndrome c. Heart structural defects d. Klinefelter syndrome e. Cleft lip

Aminocentesis can detect chromosomal abnormalities like the extra chromosome in Down syndrome, the missing chromosome in Turner syndrome, as well s Klinefelter syndromes

A nurse is speaking with a poorly-educated individual. On the patient's chart, the nurse reads "benign neoplasm". Which of the following would BEST enable the nurse to correctly convey this information to the client in a way that could be understood? a. "You have a severe form of cancer" b. "You have tumor. It is NOT likely this tumor will spread to other parts of your body" c. "You have an infection that is affecting all the systems of the body" d. "You have a cancerous growth with well-differentiated cells and well-defined borders"

Another term for neoplasm is tumor. As the tumor is benign, it is unlikely to spread to other body regions. Answer d is correct but it does NOT meet the needs of the patient, as it is NOT likely to be understood by the patient

A nursing student is conducting a quick review of cellular pathology terminology. Which of the following matches indicates an INCORRECT understanding on the part of the student? Select all that apply a. Hyperplasia - increase cell number b. Hypertrophy - increase cell size c. Atrophy - programmed cell death d. Dysplasia - one cell type converted to another cell type e. Neoplasia - uncontrolled cell growth

Apoptosis, NOT atrophy, is programmed cell death. Metaplasia, NOT dysplasia, is conversion of cell type to another. Both are incorrect

A nurse is working with a young child who recently fell off his bike and must have his leg in a cast for 6 weeks. He has overheard a conversation in which the phrase "muscle atrophy" was used. How should the nurse explain what this means? a. The nurse should tell the child he can expect his leg to look larger after this cast is removed b.The nurse should tell the child that when the cast is removed, his leg may appear smaller than normal c. The nurse should tell the child that atrophy results in cellular changes that may cause discoloration of his leg d. The nurse should tell the child that atrophy is another term for cancer

Atrophy is a loss of tissue, so when the cast is removed, the leg may appear smaller than normal due to muscle loss. Atrophy is NOT directly associated with changes in color

What is the physiological change that occurs in stage 5 of the sleep cycle? a. Heart rate decreases b. Breathing becomes regular c. Breathing becomes rapid d. None of the above

BREATHING BECOMES MORE RAPID and shallow in stage 5 of the sleep cycle

While conducting an endoscopic examination on a client, it is found that the client has acid reflex. Which condition is associated with the gastroesophageal reflux disease (GERD) in which cell injury is reversible? a. Myocardial infarction b. Barrett's esophagus c. Parkinson's disease d. Hashimoto's thyroiditis

Barrett's esophagus is a serious complication of gastroesophageal reflux disease. In GERD, the lower esophageal squamous epithelial cells can undergo a metaplastic change into columnar stomach like cells

A client comes in with persistently uncontrolled hypertension. The nurse informs the client's spouse that one of the consequences of prolonged raised blood pressure is a weakened area in the wall of the cerebral artery, located on the Circle of Willis. What is this condition known as? a. Berry aneurysm b. Ischemia c. Infarction d. Xanthelasma

Berry aneurysm is a small berry like bulge that is caused by a weakened area in the wall of the cerebral artery at or near the Circle of Willis in the brain

A patient suffering from a cardiac event is given a cardiac glycoside medication. Which cellular component is directly affected by the medication ? a. Plasma membrane b. Sodium-potassium adenosine triphosphate (ATP) pump c. Golgi apparatus d. Mitochondria

Cardiac glycosides target the sodium potassium potassium ATP pump, which alters cardiac contractility

What is the result of decreased cardiac output in a client, who is on prolonged bed rest? a. Increase in heart rate b. Increase in stroke volume c. Increase in diastolic volume d. Increase in venous returns

Cardiac output decreases due to prolonged bed rest. To compensate for this, the HEART RATE INCREASES. The heart beats harder and faster to eject sufficient ventricular blood supply to the organs with adequate circulation

A culture of a foot wound is sent to the laboratory. The results indicate the organism is Clostridium perfringens. What is the appropriate response by the nurse? a. To apply a strong antifungal topical medication b. Begin immediate treatment with a high dose antibiotic c. Elevate and massage the limb d. To apply a mild, topical antibiotic

Clostridium perfringens cause a serious bacterial infection. The course of action is immediate treatment with strong antibiotic. It causes gas gangrene a serious bacterial, NOT fungal, infection

Arteriosclerosis is the thickening and hardening of arterial walls. Which condition acts as an initiator of arteriosclerosis? a. Endothelial cell injury b. Necrosis c. Apoptosis d. Infarction

ENDOTHELIAL CELL INJURY acts as an initiator of arteriosclerosis. The most significant injurious agents of the endothelial cells are hypertension, diabetic hyperglycemia, free radicals, persistent secretion of angiotensin II, and low density lipoprotein cholesterol

Which of the following statements indicates further education of a patient is needed? a. "Cervical dysplasia can be a precursor to cancer of the cervix" b. "I have cervical dysplasia, so that means I have cancer" c. "My Pap smear results showed cervical dysplasia, which means that some of the cells appear abnormal" d. "Because cervical dysplasia was diagnosed, I will likely require follow up examinations"

Dysplasia does NOT mean cancer. Dyslasia means the cells look abnormal (b)

A client was hospitalized due to immobility. The blood report after one month of hospitalization shows that the client has developed septicemia. Which factor could have caused septicemia? a. Excessive exercise b. Erythema c. Decubitus ulcer d. Antibacterial drugs

DECUBITUS ULCERS are vulnerable to bacterial contamination and infection. These infections can enter the blood stream and cause septicemia

A young patient suffering from severe starvation presents with swollen abdomen. Place the following sequence of events in the proper order to explain this patient's development - Decreased albumin synthesis - Increased edema - Decreased protein intake - Shift of fluid from the plasma to the extracellular space

Decreased protein intake may result in decreased albumin synthesis. As albumin is a plasma protein, loss of albumin causes fluid to leak into the extracellular space. As fluid accumulates in the extracellular space, edema develops (Decreased protein intake - Decreased albumin synthesis - Shift of fluid from the plasma to the extracellular space - Increased edema)

A client is on digitalis and the lab results show that the client is hypokalemic. What effect could hypokalemia have on the drug administered ? a. It might NOT have any effect on digitalis b. It might cause pricarditis c. It can cause digitalis toxicity d. It can increase the effectiveness of digitalis

Digitalis binds to the open binding sites of potassium and CAUSE DIGITALIS TOXICITY when the drug is administered to hypokalemic clients

In a myocardial infarction, cardiac protein levels elevate in the blood. Damage to which cellular structure enables this increase in serum cardiac protein levels? a. Endoplasmic reticulum b. Mitochondria c. Ribosome d. Plasma membrane

Disruption of the plasma membrane of cardiac cells during myocardial infarction allow the leakage of cardiac proteins into the blood. Although distruption of endoplasmic reticulum may occur with myocardial infarction, this does NOT directly lead to an elevation of cardiac proteins in the blood. Ribosomal dysfunction may affect protein synthesis levels but does NOT directly affect protein leakage into the blood

An electrocardiogram of a patient reveals displacement of point of maximal impact (PMI) of the heart. Which cellular change likely accounts for this clinical finding? a. Intracellular accumulations b. Disruption of the sodium-potassium pump c. Destruction of plasma membrane integrity d. Hypertrophy

Enlargement of an area of the heart would be the MOST likely cause to shift the PMI. Destruction of plasma membrane integrity would increase serum cardiac markers, NOT displace the PMI

Before meeting a patient, a physical therapist reads in the patient's chart that significant atrophy is present in the right upper extremity. Until further information is obtained, which of the following conditions must the physical therapist consider as possible contributors to this atrophy? Select all that apply a. Well-balanced and nutritious diet b. Casting c. Bilateral weight training d. Paralysis e. Neurodegenerative disease f. Psychological inhibition reducing extremity usage g. Ischemia in affecte limb

Factors that may result in atrophy include inadequate neural stimulation (paralysis), lack of use due to casting of a limb (casting), lack of muscle neural stimulation (neurodegenerative disease), reduced workload on the limb (Psycholoical inhibition reducing extremity usage), and inadequate blood supply will cause loss of tissue (ischemia in affected limbs)

A nurse is charged with educating patients about prenatal genetic testing and screening. Which of the following criteria should the nurse consider in deciding who needs to receive such education? Select all that apply a. Woman's age at pregnancy b. Abnormal ultrasound findings c. Presence of family history of inherited conditions for which there is a screening test d. No criteria are needed. Standard care requires that all patients receive prenatal genetic testing and screening for all diseases e. Any time a woman has had a prior miscarriage

Factors used to consider prenatal genetic testing and screening include women 35 years or older at pregnancy, abnormal ultrasound findings, and a history of inherited disorders. MULTIPLE miscarriages or unexplained miscarriages is a considerable factor for prenatal genetic testing and screening

A father has homozygous familial adenomatous polyposis. He wants to know why they are recommending a colonoscopy for his son, age 14 years. The father feels the son is "way too young" to have such a procedure. Which of the following should be conveyed to the father? a. Because the father is homozygous for an autosomal dominant disease, he passed the genes for this disease on to his offspring. Colonoscopies help assess the disease status in his son b. The son should NOT need colonoscopies because this disease is only passed on to female offspring c. Familial adenomatous polyposis is a recessive disease. The son likely did NOT inherit it d. Colonscopies are NOT necessary for this condition, thus, the test should be cancelled

Familial adenomatous polyposis causes colon polyps. Colonscopies are recommended for those age 12 and above who have the disease (a)

Similar to sickle cell anemia, which other genetic disorder offers protection against malaria? a. Tay Sachs disease b. G6PD deficiency c. Wilson disease d. Turner syndrome

G6PD deficiency, like sickle cell anemia, confers protection against malaria

A nurse is counseling a family with a history of G6PD deficiency. The family wants to know why it seems like all the boy children have the disease, but NOT the girls. What is the appropriate response by the nurse? a. This disease is inherited on the Y chromosome from the father, thus only boys are affected b. This disease is found on the X chromosome. As the boys inherit only one X-chromosome and girls inherit two, the boys are at greater risk for this disease, as the single gene may be abnormal c. Girls cannot inherit this disease due to their biology d. Boys inherit two copies of this gene and girls inherit one copy. Thus, the boys have twice the change of developing this disease

G6PD is an X linked disease. Girls must inherit two copies of this gene in order to express the disease whereas boys only need one copy

A nurse is working with a couple whose child has been diagnosed with neurofibromatosis. The nurse is attempting to explain that neurofibromatosis is a disease of variable expressivity. Which of the following statements should the nurse make? a. "Another way to say that the expressivity of neurofibromatosis is variable is to use the word penetrance in place of expressivity" b. "Some people with neurofibromatosis have very severe symptoms. Other individuals have relatively minor developments" c. "Neurofibromatosis always follows a predictable pattern in each individual" d. "An error has occurred. Neurofibromatosis is a genetic disease. Expressivity does NOT factor into genetic disease severity

Genetic expressivity refers to the severity of symptoms of the disease (b)

An adult client is diagnosed with Marfan syndrome after presenting with joint pain and weakness or numbness in the legs. What other clinical finding would be a treatment priority for this client? a. Scoliosis b. Pectus excavatum c. Heart valve problems d. Pneumothorax

HEART VALVE PROBLEMS are a priority treatment for clients with Marfan syndrome. Preventive procedures such as valve replacement and medications are necessary for cardiovascular disease

Mark recently suffered a myocardial infarction. He has suffered from hypertension for the past 10 years. What is the proper sequence of events showing how Mark's hypertension may have played a role in his myocardial infarction - Left ventricular hypertrophy - Myocardial infarction - Increased arterial blood pressure - Myocardial ischemia - Increase workload on the heart

High blood pressure can lead to an increased workload on the heart. To compensate for the increase in workload, the heart muscle may undergo hypertrophy. However, hypoxia and ischemia may develop due to inadequate blood flow to the increased muscle tissue. Prolonged ischemia can lead to infarction (tissue death) (Increased arterial blood pressure - Increase workload on the heart - left ventricular hypertrophy - myocardial ischemia - myocardial infarction)

Which of the following may increase the risk of endothelial damage? Select all that apply a. Hypertension b. Reduced low-density lipoprotein (LDL) cholesterol c. Diabetes mellitus d. Hypoglycemia e. Increased angiotensin II

High blood pressure increases the shearing force on the endothelium. Angiotensin II is a potent vasoconstrictor that increases the risk for hypertension and endothelial damage. Diabetes mellitus is related to high blood glucose levels that may damage the endothelium. High levels of LDL cholesterol levels, NOT reduced levels and hyperglycemia, NOT hypoglycemia increase the risk for endothelial damage

A clinician reviews the recent results of a patient's annual check-up. Which result has the clinician concerned about the potential for atherosclerosis in the patient? a. Elevated low-density lipoprotein (LDL) cholesterol b. Elevated high-density lipoprotein (HDL) cholesterol c. Hypoglycemia d. Hypotension

High levels of LDL have been associated with atherosclerotic plaque formation. HDL cholesterol reduces the risk for atherosclerosis. Hyperglycemia, NOT hypoglycemia, is related to endothelial damage and atherosclerosis and hypertension, NOT hypotension, increases the risk of atherosclerosis

An echocardiogram of a patient with long-term hypertension related to a sedentary lifestyle and poor diet may reveal which of the following changes? a. Pathological hypertrophy b. Physiological hypertrophy c. Benign neoplasia d. Cardiac apoptosis

Hypertension can result in hypertrophy of the heart. This would be considered pathological as it is in response to a disease state, and support for the enlarged heart, increased cardiac circulation, may be lacking

Frank has developed atherosclerosis resulting in significant occlusion of one of his coronary arteries. The nurse explains that his hypertension may have played a role in this atherosclerotic process. Place in the proper order the sequence of events by which Frank's hypertension contributes to atherosclerosis - White blood cells attracted to area of injury - Atherosclerotic plaque forms - Elevated blood pressure - Increased shearing force on the endothelium - Low-density lipoprotein (LDL) deposits form - Endothelial injury and inflammation

Hypertension may place increased shearing forces on the endothelial wall of vessels. Elevated shearing forces may damage the endothelium, leading to inflammation. White blood cells are attracted to areas of injury. These areas serve as places for LDL deposition and atherosclerotic plaque formation ( Elevated blood pressure - Increased shearing force on the endothelium - Endothelial injury and inflammation - White blood cells attracted to area of injury - Low-density lipoprotein (LDL) deposits form - Atherosclerotic plaque forms)

What change occurs as a result of hypoventilation? a. Partial pressure of carbon dioxide increases. b. Lungs expel carbon dioxide. c. Respiratory alkalosis occurs. d. None of the above

Hypoventilation INCREASES THE PARTIAL PRESSURE OF CARBON DIOXIDE as the lungs retain carbon dioxide

A genetically programmed cell death is a process that can destroy cells that are no longer needed. What is the best method to do this? a. Necrosis b. Apoptosis c. Gangrene d. Infarction

In multicellular organisms, cells that are no longer needed or are a threat to the organism are destroyed by a programmed cell death called APOPTOSIS

A nurse is talking to patients who have a child recently diagnosed with Klinefelter syndrome. Which of the following can the nurse expect to discuss with the patient? a. The child will reach puberty at a much earlier age, approximately age 8 years b. The child will require testosterone therapy c. The child is likely to have overdeveloped muscles and a thickening of body d. The child is biologically a female, despite a more male appearance

Klinefelter syndrome causes lack of development of testes, requiring testosterone therapy Klinefelter syndrome is denoted by XXY, affecting normal male development so puberty will NOT occur early and the child is likely to develop a lanky body appearance with underdeveloped muscles

What organelle has its own DNA a. Mitochondria b. Golgi apparatus c. Endoplasmic reticulum d. Lysosome

Mitochondria are the only cell organelles with their own distinct DNA. This DNA was likely obtained from bacteria like organisms

A 19-year-old male client presents with rheumatoid arthritis. The client has a tall, lanky body, sparse facial hair. He complains of decreased physical endurance. How could a diagnosis of Klinefelter syndrome be confirmed in the client? a. Evaluate muscle weakness b. Assess distribution of adipose tissue c. Perform genetic testing and hormone analysis d. Confirm absence of pubertal changes

Only genetic testing for an extra X chromosome and associated reduction in testosterone can confirm Klinefelter syndrome

How does oxidative stress occur? a. When estrogen stimulation results in mitotic division of breast gland cells b. When a cell's environment cannot support its metabolic requirements c. When a blood clot that obstructs a coronary artery causes cardiac muscle ischemia d. In cells that undergo transient ischemia and subsequent resumption of circulation

Oxidative stress is a form of cell injury that occurs when free radical generation exceeds the mechanism of removal. Oxidative stress commonly occurs in cells that undergo transient ischemia and subsequent resumption of circulation

A clinician is looking at a specimen under a microscope. Based on the histological appearance of the specimen, the clinician feels confident in the diagnosis of "peptic ulcer diseases". Which of the following terms best represents the clinician's process of diagnosis based on specific appearances? a. Pathognomonic changes b. Pathological changes c. Histological changes d. Biological changes

Pathognomonic changes are histological results related to a specific diagnoses. Pathological and biological changes are too general to be diagnostic. Histological changes are any changes in the tissues, which may or may NOT be diagnostic

A woman, age 39 years, is pregnant. She has type II diabetes and has had two prior miscarriages. She met the "man of her dreams" during her job of traveling around the world for hotel management. She and her husband have no known inheritable diseases. At this time, the ultrasound results are normal. Which factors in the patient scenario may indicate prenatal genetic screening and testing may be beneficial ? a. The woman's age b. The woman has had a normal ultrasound c. The woman has had multiple miscarriages d. The couple are NOT a close blood relation e. The woman has diabetes mellitus

Patients that may benefit from prenatal screening include woman over 35 years at pregnancy, disease states such as diabetes, and multiple miscarriages. Couples who are close blood relation may also benefit from prenatal screening

Which of the following statements are correct about RNA and DNA? Select all that apply a. RNA is single stranded b. The pentose sugar in RNA is ribose c. DNA, NOT RNA, is composed of nucleotides d. A pyrimidine base in RNA is thymine e. Both DNA and RNA are found in the cytoplasm of eukaryotic cells

RNA molecules are single stranded while DNA molecules are double stranded. RNA is composed of ribose, whereas DNA is composed of deoxyribose. Both DNA and RNA are made of nucleotides, RNA contains uracil in place of thymine, and DNA is found in the nucleus, NOT the cytoplasm

Which body part contains pluripotent stem cells? a. Lymph nodes b. Cardiac muscle c. Umbilical cord d. Skeletal muscle

Stem cells obtained from the UMBILICAL CORD during the birthing process are capable of developing into other cell types and are, therefore, referred to as pluripotent stem cells

What is oncotic pressure? a. The force exerted by albumin in the bloodstream b. The pushing force exerted by water in the bloodstream c. The pressure exerted by electrolytes in the bloodstream d. The pressure that drives a liquid to flow in narrow spaces without the assistance of gravity

THE FORCE EXERTED BY ALBUMIN IN THE BLOODSTREAM is known as oncotic pressure. Albumin attracts water and helps retain water inside the blood vessels

Which is the most prevalent method to replace permanently injured tissues and organs? a. Therapeutic cloning b. Reproductive cloning c. Transplantation d. Stem cell restoration

TRANSPLANTATION is the most prevalent method to replace permanently injured tissues or organs, such as kidneys

A client's serum phosphorus level is 5.5 mg/dL. Which clinical manifestations would the clients exhibit? a. Tetany and muscle weakness b. Ataxia and hypertension c. Muscle weakness and tremor d. Tetany and hypotension

Tetany and hypotension

Which arterial blood gas value is necessary to identify if a condition is compensated or uncompensated? a. Blood pH b. Blood bicarbonate c. Oxygen saturation d. Partial pressure of oxygen

The BLOOD pH value helps in differentiating between a compensated and uncompensated condition

A comatose client's blood pH is 7.1, partial pressure of carbon dioxide is 16 mm Hg, and bicarbonate concentration is 5 mEq/L. Which acid-base imbalance has the client developed? a. Metabolic acidosis b. Metabolic alkalosis c. Respiratory acidosis d. None of the above

The client has developed METABOLIC ACIDOSIS as indicated by low blood pH and low bicarbonate levels

Place the following sequence of events in the proper order - Peptide chain formed - mRNA transcript formed - DNA molecule uncoils and separates - tRNA delivers amino acid to ribosomes

The first step in nucleotide transcription is for the DNA molecule to uncoil and separate. Next an mRNA transcript is formed, followed by tRNA interacting with the ribosome and mRNA transcript to provide the proper amino acid. Finally, the amino acids are linked into a peptide chain (DNA molecule uncoils and separates - mRNA transcript formed - tRNA delivers amino acid to ribosomes - Peptide chain formed)

A nurse is providing care for a client, who is on bed rest due to immobility. Which measure does the nurse adopt in order to counteract the ill effects of immobility? a. Assess the client's skin for erythema b. Place the client in prone position c. Avoid seating the client on a chair d. Change the position of the client every 4 hours

The nurse should ASSESS THE CLIENT'S SKIN FOR AREAS OF ERYTHEMA and irritation periodically. This can help in preventing the development of decubitus ulcer

The BRCA1 gene has 85% penetrance for developing cancer. A female patient, age 24 years, is shown to be homozygous for BRCA1. Which of the following responses is accurate by the nurse? a. "This gene hardly ever causes cancer. You have nothing to worry about" b. "You need a double mastectomy immediately" c. " The presence of the BRCA1 gene indicates a higher probability of cancer development. Let's talk about some options for you to consider" d. "You will never be able to have children"

The presence of the BRCA1 gene indicates a higher probability of cancer development. Let's talk about some options for you to consider. 85% penetrance indicates a high likelihood of expression of the disease

A client is scheduled for a heart valve replacement. Which strategic action of the primary healthcare provider enhances the client's adaptive ability and coping mechanism to reduce stress? a. Administering epinephrine to reduce stress b. Encouraging a family member to stay with the client in the pre-operative setting c. Permitting a family member to stay in the operation theater d. Restricting the family members from talking to the client in the pre-operative setting

The primary healthcare provider can improve the client's adaptive ability and coping mechanism by ENCOURAGING THE CLIENT'S FAMILY MEMBERS TO STAY WITH THE CLIENT IN THE PRE OPERATIVE SETTING and also during the recovering phase

What defines translation in protein synthesis? a. Process of RNA synthesis b. Process of DNA synthesis c. Process of ribosome formation d. Process of protein synthesis from RNA

The process of protein synthesis from RNA at ribosomes is called translation

Which section of the chromosome indicates the p arm?

The shorter arm of a chromosome is known as the p arm and the longer arm is known as the q arm

Place the following sequences of steps in the proper order - Hormone packaged in secretory granule - Hormone processed by the endoplasmic reticulum - Hormone sent to Golgi apparatus - Peptide hormone synthesized in the ribosome

This sequence shows protein hormone proceessing after formation on the peptide at the ribosome. The first step is protein synthesis of the hormone, then it is sent to the endoplasmic reticulum for further processing. Next, the hormone is processed by the Golgi apparatus and then the hormone is packed for secretion (Peptide hormone synthesized in the ribosome - Hormone processed by the endoplasmic reticulum - Hormone sent to Golgi apparatus - Hormone packed in secretory granules )

Brian, age 48 years, was diagnosed with type 2 diabetes mellitus 3 years ago. A nursing student is educating Brian on the importance of glycemic control in order to decrease his risk of atherosclerosis. What is the proper sequence of events for atherosclerosis development due to hyperglycemia that should be explained to Brian? - Atherosclerotic plaque formation - Advanced glycation end products - Hyperglycemia - Endothelial injury and inflammation

Uncontrolled diabetes mellitus may lead to hyperglycemia. The high blood glucose levels may cause formation of advanced glycation end products in the endothelium. The endothelial injury may lead to inflammation, which may induce plaque formation (Hyperglycemia - Advanced glycation end products - endothelial injury and inflammation - atherosclerotic plaque formation)

Growing neoplasms require an adequate blood supply. A new cancer treatment inhibits this angiogenesis. Which of the following factors does the new treatment MOST likely inhibit a. Nitric Oxide (NO) b. Vascular endothelial growth factor (VEGF) c. Endothelin d. Tumor necrosis factor (TNF)-alpha

VEGF promotes angiiogenesis so a medication which inhibits this factor may reduce blood supply to a tumor. NO is a vasodilator, endothelin is a vasoconstrictor, and TNF-alpha is an inflammatory mediator, NOT a stimulate for angiogenesis

A physician reading about a genetic defect comes across the following: "8p12.1". In the proper order, from right to left, what does this terminology indicate? - Chromosome number - Band - Arm of chromosome - Region

When naming a gene locus, the first number indicates the chromosome number. The letter p or q indicates the short or long arm of the chromosome. Next is the region and the band (Chromosome number, arm of chromosome, region, and band)

A blood clot is occluding blood flow to a region of the lower extremity. Which of the following would a clinician expect to be occurring? a. Increased aerobic metabolism b. Increased lactic acid formation c. Increased mitochondrial oxygen uptake d. Increased adenosine triphosphate (ATP) synthesis per glucose molecule

Without oxygen, glucose is metabolized by anaerobic metabolism to form lactic acid. Aerobic metabolism requires oxygen, so decreased oxygen levels would lessen aerobic metabolism. Mitochondrial oxygen uptake decreases during hypoxia. Aerobic metabolism provides more ATP than anaerobic metabolism

A patient is diagnosed with familial hypercholesterolemia. Which development would you NOT be surprised to observe as a clinican given this information? a. Keloid b. Berry aneurysm c. Xanthoma d. Jaundice

Xanthomas are deposits of cholesterol that may appear anywhere on the body. As familial hypercholeterolemia increases circulating cholesterol levels, xanthomas may develop. Keloid develops due to hyperplasia of the skin and berry aneurysm and jaundice are NOT caused by high cholesterol levels


Ensembles d'études connexes

PROOF FORMATS: THE PLAN OF THE PROOF

View Set

Psychology 101 - Chapter 7 - Memory

View Set

EMT Ch 30 Abdominal and Genitourinary Injuries

View Set